problemas selectos

233
1 PROBLEMAS SELECCIONADOS DE FISICA Para entrenamiento de olimpiadas en Escuela Media Superior y para profundización en el nivel básico de Ciencias e Ingenierías Dr. Raúl Portuondo Duany UPRM 2010

Upload: walter-olivas-saldana

Post on 16-Apr-2015

508 views

Category:

Documents


29 download

TRANSCRIPT

Page 1: problemas selectos

1

PROBLEMAS

SELECCIONADOS

DE FISICA Para entrenamiento de olimpiadas en Escuela Media Superior y

para profundización en el nivel básico de Ciencias e Ingenierías

Dr. Raúl Portuondo Duany

UPRM

2010

Page 2: problemas selectos

2

Problemas Seleccionados de Física

Ed.D. Raúl Portuondo Duany

UPRM, Mayagüez, 2010, ©

A mi Profesor de Física,

Medel Pérez Quintana,

maestro, colega y amigo.

Page 3: problemas selectos

3

INDICE Página

Prólogo 5

Primera parte: Física Fragmentada 9

1a-Medidas y estimaciones. Problemas. 11

1b-Medidas y estimaciones. Soluciones. 12

2a-Cinemática. Problemas. 14

2b-Cinemática. Soluciones. 16

3a-Fuerzas. Problemas. 22

3b-Fuerzas. Soluciones. 25

4a-Conservación y variación de cantidades mecánicas. Problemas. 30

4b-Conservación y variación de cantidades mecánicas. Soluciones. 33

5a-Sólido Rígido. Problemas. 44

5b-Sólido Rígido. Soluciones. 47

6a-Hidrostática. Problemas. 57

6b-Hidrostática. Soluciones. 58

7a-Termodinámica I. Problemas. 60

7b-Termodinámica I. Soluciones. 63

8a-Física Molecular. Problemas. 70

8b-Física Molecular. Soluciones. 72

9a-Termodinámica II. Problemas. 81

9b-Termodinámica II. Soluciones. 83

10a-Oscilaciones. Problemas. 88

10b-Oscilaciones. Soluciones. 89

11a-Campo eléctrico. Problemas. 95

11b-Campo eléctrico. Soluciones. 98

Page 4: problemas selectos

4

12a-Circuitos. Problemas. 100

12b-Circuitos. Soluciones. 103

13a-Magnetismo. Problemas. 114

13b-Magnetismo. Soluciones. 115

14a-Optica. Problemas. 121

14b-Optica. Soluciones. 125

15a-Física Moderna. Problemas. 137

15b-Física Moderna. Soluciones. 139

Segunda parte: Física Integrada 146

16a-Problemas de Resortes y Algo Más. 148

16b-Problemas de Resortes y Algo Más. Soluciones. 151

17a-Problemas con Análisis y Cálculos Gráficos. 159

17b-Problemas con Análisis y Cálculos Gráficos. Soluciones. 164

18a-Problemas con Análisis de Simetría. 178

18b-Problemas con Análisis de Simetría. Soluciones. 180

19a-Problemas con Cálculo Diferencial o Integral. 188

19b-Problemas con Cálculo Diferencial o Integral. Soluciones. 193

Apéndice: Cantidades físicas fundamentales. 217

I-Magnitudes físicas 217

II-Definiciones y leyes 225

Tablas de definiciones y leyes 226

Page 5: problemas selectos

5

PRÓLOGO

El presente libro es una colección de problemas resueltos de Física, dividido en dos partes:

I-Problemas de diferentes secciones de la Física, como suele dividirse la Física en los cursos

corrientes. Esta primera parte (Física Fragmentada) es de nivel algebraico y trigonométrico.

II-Problemas que integran diferentes secciones del curso (Física Integrada), o emplean un ins-

trumento específico en cada capítulo que se usa de manera similar en todas las secciones de la

Física: análisis de simetría, análisis numérico de gráficos, análisis que requieren de derivadas e

integrales y de sus propiedades. En cada capítulo de esta parte pueden aparecer problemas de

mecánica, o de calor, o de electromagnetismo, o de óptica, o mezclas de estas secciones.

Los problemas del libro son una selección de los muchos usados por el autor durante más de

veinte años para entrenar alumnos de enseñanza media superior para Olimpiadas de Física Inter-

nacionales (IPhO) e Iberoamericanas (OIbF). Unos pocos responden a estándares de cursos clá-

sicos de enseñanza media, y la inmensa mayoría requiere de un pensamiento más profundo que

el estándar (y algunos requieren de Cálculo Diferencial e Integral, en la segunda parte).

Este no es un libro para estudiar desde cero los conceptos y leyes de los cursos regulares de

Física, sino para ofrecer una ejercitación en el proceso de solucionar problemas de alguna com-

plejidad. El libro es más bien un complemento para reforzar el estudio de la Física que se ofrece

en los cursos regulares.

El libro está destinado a:

a)Estudiantes que terminan el nivel medio superior y se preparan para entrar en carreras de cien-

cias e ingenierías de universidades de alto nivel.

b)Estudiantes de enseñanza media superior que comienzan a prepararse para participar en Olim-

piadas de Física. (Las Olimpiadas Internacionales de Física para el nivel de grado 12 requieren

de una ejercitación aún un poco más profunda que la ofrecida aquí).

c)Profesores de enseñanza media superior que desean enriquecer su arsenal de problemas resuel-

tos y profundizar en los métodos de solución de problemas físicos.

d)Para estudiantes de College (ciclo básico universitario) pueden ser útiles los problemas tanto

de la parte I como de la II.

El estudiante debe intentar resolver los problemas por sí mismo, pero si siente que no puede, o

que le faltan elementos, podrá pasar a los problemas resueltos. No obstante, las habilidades de la

operatoria de solución las adquirirá sólo (a) si realiza los pasos algebraicos que a veces en el

libro solamente se indican, pero no se desarrollan paso a paso, y (b) si hace el trabajo de cálculo

Page 6: problemas selectos

6

numérico (a veces extenso) para comprobar que la respuesta es la que ofrece el texto y ver las

aproximaciones y redondeos que se hicieron.

Lamentablemente, el libro no incorpora ejercicios experimentales: el trabajo experimental sola-

mente puede realizarse en el laboratorio con los instrumentos apropiados. Es la base del trabajo

de la Física, pues una teoría por sí sola, sin el correspondiente aval de la experiencia, es material

muerto y ocioso. El estudiante que aspire a las Olimpiadas de Física y al trabajo profesional con

esta ciencia deberá procurarse el entrenamiento experimental necesario: los montajes y medi-

ciones que deben realizarse para contestar problemas físicos son a veces verdaderos problemas

de ingenio. En el libro, el estudiante podrá hacerse una vaga idea de este ingenio experimental en

la parte II, en la sección de análisis numéricos con gráficos, donde la sola obtención del gráfico

que se da como dato es toda una tarea experimental.

Las fuentes de algunos de estos problemas están distribuidas en una veintena de libros y manua-

les empleados por el autor durante veinte años, más problemas que son del propio autor. Las

discusiones desarrolladas son todas del autor, no meras traducciones.

Las fuentes utilizadas son:

En inglés:

-International Physics Olympiads: Edited by Waldemar Gorzkowski; Polish Academy of

Science, 1990.

-IPhO; http://www.jyu.fi/ipho/

-Feynman Lectures on Physics. Exercises; Addison-Wesley, 1964

-Optics; B. Rossi; Addison Wesly, 1957

-Waves and Oscillations; F.S.Crawford; Mc Graw Hill, N.Y., 1966

-Fundamental Physics; J. Orear; J. Wiley, 1964.

En ruso:

-Problemas de las Olimpiadas de Moscú; A.I. Buzdin, V.A. Ilin; Redacc: S.S. Krotov; Moscú,

Ed. Naúka 1988.

-Revista Kvant, Moscú.

-Olimpiadas de Física de la URRS; I.Sh. Slobodetski, V.A. Orlov; Moscú, Ed Prosvishenie, 1982

-Un problema, dos problemas, …; A.I. Busdin, A.R. Zilverman, S.S. Krotov; Moscú, Ed. Naúka,

1990.

Page 7: problemas selectos

7

En español:

-Problemas seleccionados de la Física Elemental; B.B. Bujovtsev, V.D. Krichenkov; Ed. Mir,

Moscú, 1979.

-Problemas de Física General; L.E. Irodov; Ed. Mir, Moscú, 1985.

-Problemas de Física; dirigido por S. Kosel; Ed. Mir. Moscú, 1986.

-Preguntas y Problemas de Física; L. Tarásov y A. Tarasova; Ed. Mir, Moscú, 1988.

-Problemas de Física; dirigido por O.Ya. Savchenko; Ed. Mir, Moscú, 1989.

-Física en ejemplos y problemas; E. Butilov, A. Bicov, A. Kondratiev; Ed. Mir, Moscú, 1991.

-OIbF; http://oc.uan.edu.co/oibf/oibf.htm

-Física PSSC; Ed. Reverté; México, 1968.

-Mecánica; R. Portuondo, M. Pérez; Ed. Pueblo y Educación, Cuba, 1983.

-Leyes de Conservación (Curso Facultativo); R. Portuondo; Poligráfico “Haydé Santamaría”,

Cuba, 1989.

-Introducción a la Física General; R. Portuondo; ENPES; Univ. de La Habana, 1990.

-Introducción a la Teoría de la Relatividad; R. Portuondo; RUM, PR, 2008.

Page 8: problemas selectos

8

Page 9: problemas selectos

9

PRIMERA PARTE:

FISICA FRAGMENTADA

En esta parte, los problemas de Física están agrupados por capítulos según el tema físico con el

que se relacionan:1-Medidas y estimaciones (5 problemas); 2-Cinemática (9); 3-Fuerzas (9); 4-

Conservación y variación de cantidades mecánicas (12); 5-Sólido Rígido (11); 6-Hidrostática

(5); 7-Termodinámica I (11); 8-Física Molecular (13); 9-Termodinámica II (9); 10-Oscilaciones

(6); 11-Campo eléctrico (6); 12-Circuitos (13); 13-Magnetismo (6); 14-Óptica (10); 15-Física

Moderna (9).

En el Apéndice al final del libro se presentan las definiciones matemáticas de las cantidades físi-

cas y sus significados. El estudiante encontrará también las ecuaciones físicas necesarias para

resolver los problemas, subdivididas en ecuaciones de definiciones y ecuaciones de leyes. Las

primeras son aplicables siempre, dentro de su marco de definición clásico (a velocidades próxi-

mas a las de la luz, algunas definiciones se transforman), pero las leyes son aplicables solamente

dentro de determinadas condiciones, aún dentro de la física clásica: el estudiante deberá conocer

las condiciones de aplicación de cada ley para utilizarlas solamente en las situaciones admisibles.

Para resolver los problemas, el estudiante necesitará además tener habilidades operatorias de

Algebra (incluyendo ecuaciones cuadráticas y sistemas de ecuaciones), Geometría y Trigonome-

tría. Y, en general, deberá razonar finamente con los elementos físicos del problema.

Los problemas de esta parte son, en general, problemas avanzados para el nivel de los grados 11

y 12. Constituyen un entrenamiento inicial para estudiantes de ese nivel que aspiren a participar

en competencias internacionales de Física. Son adecuados también para estudiantes del nivel de

College en Ciencias e Ingenierías.

Page 10: problemas selectos

10

Page 11: problemas selectos

11

1A-Medidas y estimaciones. Problemas.

1.1-Cinco relojes son puestos a prueba en un laboratorio. Exactamente al mediodía, de acuerdo

con la señal mundial estándar de tiempo WWV, se leen los relojes en días sucesivos de una

semana, y se obtienen las lecturas mostradas en la tabla. Ordene los cinco relojes de acuerdo con

el valor relativo de sus lecturas como buenos medidores del tiempo, de mejor a peor. Justifique

su respuesta.

Reloj Domingo Lunes Martes Miércoles Jueves Viernes Sábado

A 12:03:59 12:02:49 12:01:54 12:01:52 12:01:32 12:01:22 12:01:12

B 11:59:59 12:00:02 11:59:57 12:00:07 12:00:02 11:59:56 12:00:03

C 12:26:40 12:26:56 12:27:12 12:27:27 12:27:44 12:27:59 12:28:14

D 12:50:45 12:51:43 12:52:41 12:53:39 12:54:37 12:55:35 12:56:33

E 12:03:59 12:02:52 12:01:45 12:00:38 11:59:31 11:58:24 11:57:17

1.2-Un litro de aceite es derramado sobre un lago tranquilo. Si la mancha de aceite se expande

uniformemente hasta producir una capa con el espesor justamente de una molécula, con molé-

culas adyacentes en contacto, estime el diámetro de la mancha de aceite. Asuma que las molé-

culas de aceite tienen un diámetro de 2 ×10-10

m.

1.3-Usted está volando de USA a Europa, a 45o de latitud. ¿Qué distancia debe viajar directa-

mente al Este para adelantar su reloj seis horas? (Radio de la Tierra: 6400 km).

1.4-Teniendo en cuenta los movimientos de traslación y rotación de la Tierra, calcule el tiempo

que la Tierra demora en dar una vuelta completa sobre sí misma.

1.5-Una persona de 1.75 m de altura, parada a la orilla del mar, nota que una lancha que se aleja

perpendicularmente a la costa a 20 km/h demora unos 15 minutos en perderse tras el horizonte.

La misma persona, acostada a nivel del mar, ve ocultarse el Sol tras el horizonte, pero si se para

vuelve a ver el borde del Sol durante 10 s más hasta que se oculta nuevalmente. La persona se

encuentra en la zona ecuatorial. Estime con estos datos el radio de la Tierra.

Page 12: problemas selectos

12

1B-Medidas y estimaciones. Soluciones.

1.1-El mejor reloj será el que muestre más consistentemente la constancia de su periodicidad. Si

cada día se atrasa (o adelanta) exactamente lo mismo que el día anterior, bastará hacer la co-

rrección de esa cantidad constante cada día para tener la hora exacta con toda confianza.

Los mejores son D y E: el D se adelanta siempre 58 s de un día al siguiente y el E se atrasa siem-

pre 67 s. Luego el C, que tiende siempre a adelantarse cada día, entre 15 s y 17 s, lo que no es

muy caótico. Luego el B, que se adelanta o atrasa cada día entre 3 s y 10 s, lo cual ya es imprede-

cible si irá hacia atrás o hacia adelante, pero las desviaciones no son muy grandes. Finalmente el

A sufre variaciones diarias que pueden oscilar desde 70 s hasta 10 s, lo cual es muy grande

comparado con los restantes relojes.

1.2-El volumen del litro de aceite será igual al volumen del disco de aceite expandido sobre el

agua del lago: V = πR2h. De aquí: R = , donde V = 1L = 1×10

-3 m

3 y h = 2×10

-10 m.

Haciendo el cálculo:

R = 1.3×103 m = 1.3 km

1.3- Un paralelo de la Tierra barre 360o de Este a Oeste. Las 24 horas del día distribuidas en 360

o

significa que cada 15o (un “uso horario”) cambian una hora física los relojes sobre los distintos

territorios (salvo ligeras modificaciones geopolíticas locales).

A la latitud de 45o la circunferencia del paralelo terrestre tiene un radio r = RT cos45

o donde RT

es el radio terrestre.

Entonces: r = 4525 km. Para adelantar el reloj 6 horas, habrá que barrer un ángulo θ de 6×15 =

90o = π/2 rad. Si se hace por el paralelo de 45

o de latitud N, el arco recorrido tendrá una longitud:

s = rθ = 4525×π/2 = 7109 km

1.4-Cuando decimos que el día demora 24 horas estamos indicando que cada 24 horas (como

promedio anual) el Sol pasa por el cenit (el punto más alto de la “trayectoria” del Sol respecto a

nosotros). Pero éste no es el tiempo de una vuelta de la Tierra sobre sí misma.

La Fig. 1.4S muestra la Tierra rotando alrededor del Sol y alrededor de sí misma (no está escala).

Si de A a B transcurre un día, puede notarse que el observador P que en A miraba al sol en su ce-

nit ha rotado algo más de 360o (una vuelta) para ver en su cenit al Sol 24 horas más tarde.

Page 13: problemas selectos

13

Δθ =0.986o ≈ 1

o

Esto es, la Tierra rota casi 361o cada día. Y como en esos 361

o invierte 24 h, en la vuelta

completa de 360o invertirá solamente:

Δt = 24×(360/361) = 23.9 h = 23 h y 54 min

Éste es el verdadero período de rotación de la Tierra (que mide la duración del día respecto a las

estrellas fijas, o día sideral), no el día solar de 24 horas. Podrá darse cuenta que en el año de 365

días, la Tierra da 366 vueltas sobre sí misma.

1.5-Si la lancha desaparece en unos 15 minutos (o ¼ hora), el hombre parado en la orilla del mar

ve el horizonte a: s = vt = 5 km (distancia de A a B en la Fig. 1.5S).

Pero s = θR, donde R es el radio terrestre, por lo que R = s/θ = 5 km / (7.2×10-4

) = 6.9×103 km,

que es próximo al valor aceptado de 6.4×103 km.

R θ

A B

s

Fig. 1.5S

La visión del borde del Sol estando acostado se logra con

una visual tangente a la esfera terrestre desde su punto de

observación (A en la Fig. 1.5S), y la vision del borde del

Sol ya parado se tapa en el punto B distante unos 5 km de

A. Esta distancia es la rotada por la superficie terrestre en

los 10 s que transcurrieron entre los ocultamiento del Sol

estando acostado y parado. Teniendo en cuenta que la Tie-

rra da una vuelta en casi 24 h, u 86 400 s, en 10 s la Tierra

rotará:

θ = (10/86 400)×2π = 7.2×10-4

rad.

Tierra

Sol

Sol

ecuador

A

B P

P

Δθ

Δθ

Tierra

Sol

Asumiendo que la trayectoria de la órbita de la

Tierra alrededor del Sol es casi circular, el ángulo

Δθ extra rotado cada día (muy exagerado en la

figura), multiplicado por 365 y ¼ de días, valdrá

360o pues la Tierra y el observador P estarán en

la posición inicial de su órbita anual. Entonces:

365.25 Δθ = 360

Y de aquí: Fig. 1.4S

Page 14: problemas selectos

14

2A-Cinemática. Problemas.

2.1-Un helicóptero despega de una pista verticalmente con aceleración a =3.0 m/s2 y velocidad

inicial vo= 0. Después de cierto tiempo t1 el piloto apaga el motor. El sonido del motor en el lu-

gar del despegue deja de ser escuchado al cabo de un tiempo t2= 30 s después que despegó. De-

termine la velocidad del helicóptero en el momento de apagar el motor. Considere la velocidad

del sonido u = 320 m/s.

2.2- Una carretera recta y una línea de ferrocarril son paralelas una a otra. Un carro de carreras

está en reposo en un punto de la carretera y un tren de 100 m de largo está pasando junto a él a

81.0 km/h. Justo cuando pasa al lado del carro el último vagón, el carro arranca en la misma

dirección del tren, con una aceleración constante de 4.0 m/s2.

a)¿Al cabo de cuánto tiempo pasa el carro a la locomotora, asumiendo que todo el rato mantiene

la misma aceleración?

b)¿Qué distancia ha recorrido el carro en ese tiempo?

c)¿Qué velocidad tiene el carro en el instante en que pasa a la locomotora?

d)¿Cuál fue la mayor distancia que hubo entre el carro y el frente de la locomotora antes de que

el carro pasara a la locomotora?

2.3-Tres partículas se encuentran en los vértices de un triángulo equilátero de lado l y se mueven

con rapidez constante, v, de tal modo que siempre la primera se dirige en dirección a la segunda,

la segunda hacia la tercera y la tercera hacia la primera. ¿Qué tiempo demoran en encontrarse en

el centro del triángulo?

2.4-Dos proyectiles se lanzan simultáneamente desde el mismo punto, ambos en dirección hori-

zontal, pero en sentidos opuestos, con rapideces iniciales v1 y v2. ¿Qué distancia los separa en el

instante en que sus velocidades son mutuamente perpendiculares?

2.5-Cierta estructura articulada consta de tres rombos cuyos lados están en la relación 3:2:1 (Fig.

2.5: próxima página). La bisagra A3 se desplaza horizontalmente con velocidad vo. Determine las

velocidades de las bisagras A1, A2 y B1 en el instante cuando todos los ángulos de la estructura

son rectos.

Page 15: problemas selectos

15

2.6-Dos pequeños anillos O y O’ se encuentran en varillas verticales fijas AB y A’B’. Un hilo

inextensible amarrado al punto A’ y al anillo O pasa a través del anillo O’ (ver Fig. 2.6). Con-

siderando que el anillo O’ se mueve hacia abajo a velocidad v1, determine la velocidad v2 del

anillo O cuando <AOO’= θ, para θ < π/2.

be existir entre v, h, r y θ para que la esfera logre escapar del pozo después de chocar con sus

paredes y su fondo. (Los choques son perfectamente elásticos).

2.8-Una lancha de motor navega a favor de la corriente de un río y adelanta a un tronco que flota

libremente. Transcurrido un tiempo τ desde este encuentro, la lancha se detiene y emprende de

inmediato el retorno con la misma rapidez respecto a la corriente encontrándose de nuevo con el

tronco una distancia l más abajo que la primera vez. Calcule la velocidad de la corriente.

θ

r v

A

2.7-La Fig.2.7 muestra una vista superior de una

esfera que se mueve en un plano horizontal con

velocidad constante v y cae, en el punto A, dentro

de un pozo cilíndrico vertical de profundidad h y

radio r, cuyo fondo es un piso plano y horizontal.

La velocidad v de la esfera forma un ángulo θ con

el diámetro del pozo que pasa por el punto A.

Asuma que todas las superficies del pozo son

perfectamente lisa y duras, por lo que la esfera

rebota en cada choque como un rayo de luz con un

espejo. Determine la relación matemática que de-

Fig.2.7

A0 A1 A2 A3

B1

B2

B3

Fig. 2.5

vo

Fig.2.6

A

B

A’

B’

O’

O

θ

Page 16: problemas selectos

16

2B-Cinemática. Soluciones.

2.1-El helicóptero estuvo ascendiendo con aceleración constante un tiempo t1 durante el cual

alcanzó una velocidad v = at1. Y llegó a una altura h = ½ at12. El sonido del motor, al apagarlo,

viaja un tiempo Δt para bajar la altura h hasta el suelo, de tal modo que h = uΔt. Entonces:

½ at12

= uΔt, de donde resulta Δt = ½at12/u. Pero t2 = t1 + Δt = t1 + ½at1

2/u. De aquí resulta una

ecuación cuadrática en t1:

½at12/u + t1 – t2 = 0

O: 0.00469 t12 + t1 – 30 = 0

Resolviendo: t1 = 26.7 s

Sustituyendo en la ecuación de la velocidad: v = at1 = 80 m/s.

2.2-Tomemos un eje x en la dirección en que se mueve el tren. Sea x = 0 la posición en donde se

encuentra el carro y sea t = 0 el instante en que el último vagón pasa junto al carro y éste arranca.

La velocidad del tren en m/s es: v= 22.5 m/s. Las ecuaciones de movimiento del carro y de la

locomotora serán:

H

R

B

A

ω

2.9-El eje de una rueda de radio R está ubicada a una altura H

sobre el piso y rota con una velocidad angular ω. Desde el

borde de la rueda se desprende una gota que cae en el piso en

un punto B justamente debajo del eje de la rueda (Fig. 2.9).

Encuentre el tiempo de caída de la gota y el punto A de la

rueda en el que la gota se desprende de la rueda. Hint: Analice

el recorrido de la gota desde que sale de A hasta llegar a la

vertical bajo el eje asumiendo que no hubiese gravedad. Con-

sidere luego el efecto de la gravedad.

Fig.2.9

Page 17: problemas selectos

17

Carro: xc = ½ act2 = 2.0t

2.

Locomotora: xl = 100 + 22.5t.

a)Cuando el carro alcanza y pasa a la locomotora: xc = xl, esto es: 2.0t2 = 100 + 22.5t , que es

una ecuación cuadrática, que resuelta da: t = 14.7 s.

b)La distancia recorrida por el carro en ese tiempo es: xc = 2.0t2 = 430 m.

c)La velocidad del carro en ese instante es: vc = at = 59 m/s ≈ 212 km/h. (Unas 130 mill/h).

d)En el instante inicial el tren se aleja, luego el carro acelera y alcanza la velocidad del tren, y en

lo adelante supera su velocidad y llega a pasar la locomotora. En el instante en que ambos tienen

la misma velocidad, el tren deja de alejarse y a partir de ahí el carro empieza reducir la distancia

a la locomotora. Ése es, por tanto, el instante en que la distancia entre ambos fue máxima. En el

momento en que eso ocurre: vl = vc.

O sea: 22.5 = 4.0t, por lo que t = 5.6 s. En ese instante la separación s entre ambos es:

s = xl – xc = 100 + 22.5t – 2.0 t2 = 163 m.

2.4-Las componentes x de la velocidad de cada proyectil son constantes: v1x = v1 y v2x= −v2. Las

componentes y de las velocidades de los dos proyectiles crecerán de la misma manera para

ambos, en dirección hacia abajo: v1y = v2y = -gt, donde consideramos negativa la dirección hacia

abajo. El ángulo θ que forma cada vector velocidad con la horizontal será tal que: tanθ = vy/vx.

Luego: tanθ1 = -gt/v1 y tanθ2 = gt/v2.

C

P1

P2

P3

P1

P2

P3

2.3-Las tres partículas se mueven de manera que siem-

pre están en los vértices de un triángulo equilátero que

se va achicando, y terminarán reuniéndose en el punto

central C (Fig. 2.3S).

La velocidad relativa de aproximación entre dos cuales-

quiera de las partículas es:

V = v + v cos60 = (3/2) v

La distancia que tiene que recorrer cada una para encon-

trar a su vecina es l. Entonces:

t = 2l / (3v)

v

v

V cos60

Fig. 2.3S

Page 18: problemas selectos

18

Y cuando dos direcciones son mutuamente perpendiculares se cumple: tanθ1tanθ2 = −1, según se

sabe de geometría (por ejemplo: tan(240)×tan(330) = (-0.57735)×1.73205= -1, donde la dife-

rencia entre las dos direcciones vectoriales del ejemplo es: 330 – 240 = 90 ).

Entonces: (-gt/v1)(gt/v2) = -1, por lo que t = . En este tiempo las partículas se han

desplazado: x1 = v1t = y x2 = -v2t = −

. La separación es entonces:

Δx = x1 – x2 = (v1 + v2) g .

2.5-La distancia de A1 a la pared es el segmento s1 = AoA1. La distancia de A2 a la pared es:

s2 = AoA1 + A1A2.

Y la distancia de A3 a la pared es: s3= AoA1 + A1A2 +A2A3.

La proporcionalidad que existe entre los lados de los rombos es la misma entre sus diagonales,

por lo que AoA1 : A1A2 : A2A3 = 3:2:1. Por tanto: A1A2 = 2A2A3 y AoA1 = (3/2)A1A2 = 3A2A3.

En consecuencia:

s1 = 3A2A3, s2 = 3A2A3 + 2A2A3 = 5A2A3 y s3 = 3A2A3 + 2A2A3 + A2A3 = 6A2A3.

O sea: s1 : s2 : s3 = 3:5:6. Y cuando estas distancias cambien, mantendrán siempre esa propor-

ción. Su rapidez de cambio en el tiempo satisfará la misma relación: v1 : v2 : v3 = 3:5:6.

Pero v3 = vo por dato. Entonces: v1/vo = 3/6, de donde resulta v1 = vo/2.

Y: v2/vo = 5/6. Por tanto: v2 = (5/6) vo

Para calcular la velocidad de la bisagra B1 en el instante cuando todos los ángulos de la estruc-

tura son rectos basta notar que B1 rota alrededor de Ao, y que cuando los lados AoB1 y B1A1

forman un ángulo recto, la velocidad de B1 está dirigida hacia A1 a lo largo de la varilla B1A1. De

este modo, la velocidad del punto A1 de la varilla B1A1 coincide con la velocidad de B1 a lo largo

de la varilla en ese instante (son varillas rígidas), y la componente horizontal de esta velocidad

coincide con la velocidad horizontal de la bisagra A1, que es vo/2. Entonces:

vB1 cos 45 = vo/2

De aquí: vB1 = vo /2

2.6-Sea un eje y positivo hacia abajo con el origen en el nivel de la recta AA’. Sean v1 la

velocidad de O’ respecto a AA’, v2 la de O respecto a AA’, y vOO’ la de O respecto a O’.

Entonces:

Page 19: problemas selectos

19

distancia que baja O’ es la misma longitud de hilo que se recoge en el segmento OO’. Pero v1 es

positiva hacia abajo y vr es negativa pues está dirigida hacia el origen O’ (reduciendo la distancia

r entre O y O’), por lo que vr = –v1. Sustituyendo vr en (3): vOO’ = –v1 / cosθ. Y sustituyendo

vOO’ en (2):

v2 = v1 (1 – 1/cosθ).

Este resultado es válido mientras θ<π/2, pues para ese ángulo la solución tiende a infinito. En la

práctica el anillo debe frenar antes de llegar a ese ángulo, para el cual el segmento OO’ ya estaría

horizontal, y si el anillo O’ se sigue moviendo romperá el hilo por ser éste inextensible. Además,

como 1/cosθ > 1, la solución indica que v2 < 0 y el anillo O se mueve hacia arriba, como

corresponde al hecho de que el hilo entre O y O’ se recoge.

2.7-La esfera en su caída describe un arco de parábola. En cada choque con la pared cilíndrica la

esfera cambia su dirección como una reflexión pura (ángulo de incidencia igual a ángulo de

reflexión) sin modificar el valor de la velocidad en cada rebote. La Fig. 2.7Sa (próxima página)

muestra una vista superior de cómo avanza la esfera dentro del cilindro, rebotando con las

paredes y el fondo del mismo.

Las distancias horizontales entre choque y choque son iguales: AB = BC = CD = DE = …, pero

se producen a distintas alturas de la trayectoria parabólica como muestra la Fig. 2.7Sb. La

condición para que la esfera pueda escapar es que al hacer un número entero de rebotes, n,

alcance su altura máxima, h, justamente en el borde del pozo cilíndrico y en vez de rebotar se

vaya rodando como llegó.

A A’

B B’

O’

O

θ

Fig.2.6S

vOO’

vr

vb

v2 = vOO’ + v1 (1)

Ahora bien, la velocidad vOO’ está dirigida hacia

arriba, pues no hay aproximación horizontal entre O

y O’, por lo que la ecuación (1) puede reescribirse

escalarmente, ya que todas las velocidades invo-

lucradas son paralelas al eje y:

v2 = vOO’ + v1 (2)

De la Fig. 2.6S se ve que: vOO’ = vr / cosθ (3)

Y, por otro lado, la componente vr de aproximación

relativa entre O y O’ es la velocidad con que se

recoge el segmento de hilo OO’, que es la misma

velocidad con que desciende el anillo O’, ya que la

Page 20: problemas selectos

20

Si llamamos l a la distancia horizontal que avanza la esfera entre pico y pico, y llamamos d a la

distancia horizontal que avanza la esfera entre rebote y rebote, entonces, la condición para que la

esfera escape será: kl = nd, donde tanto n como k son números enteros: 1, 2, 3, …

Ahora, el tiempo de caída desde una altura h es tc = , por lo que el tiempo total de bajada

y subida a la misma altura es tbs = 2 . Y la distancia horizontal que avanza la bola de pico

a pico es: l = vtsb = .

Por otra parte, la distancia horizontal d que avanza la esfera entre choque y choque puede calcu-

larse a partir de relaciones geométricas en el triángulo isósceles que se forma dentro del círculo

de la Fig. 2.7Sa:

AB = d = 2r cosθ

Entonces, la condición kl = nd necesaria para que se escape la esfera de este pozo cilíndrico es:

k = n 2r cosθ

O: (r cosθ) / ( ) = k/n donde n y k son dos enteros cualesquiera.

2.8-La velocidad de la lancha respecto al agua es vl. La velocidad del tronco respecto a la orilla

es V, que es la velocidad de la corriente. Como el tronco está fijo al agua, la rapidez de la lancha

respecto al tronco es la misma en la ida que en la vuelta, pues el motor desarrolla la misma

velocidad respecto al agua en ambos casos. Lo cual significa que la lancha demora el mismo

tiempo alejándose del tronco que retornando al tronco: 2τ. Y en ese tiempo el tronco, con la

corriente, avanzó una distancia l respecto a la orilla. Significa que la velocidad de la corriente es

V = l/(2τ).

A

B

C

D

E

A B

C D

A B C D n =1 n =2 n =3 d d d

h h h h

l l l k =1 k =2 k=3 k=4 l

θ

θ

θ

r

r

Fig.2.7Sb Fig.2.7Sa

Page 21: problemas selectos

21

llevaría la gota hasta el punto B en el suelo, separado por una distancia H del centro de la rueda,

dada por:

H = + ½ gt2

que es una ecuación cuya única incógnita es el tiempo de caída, t, pues v = ωR. Pasamos el tér-

mino ½gt2 al lado de la H, elevamos al cuadrado ambos miembros y arreglamos, de modo que se

obtiene:

g2t4/4 – (Hg + v

2 ) t

2 + (H

2 – R

2 ) = 0

que es una ecuación cuadrática en t2. Resolviéndola:

t2=

Se nota que (v2 + Hg) es mayor que el radical, por lo que ambas soluciones de t

2 son positivas,

pero solamente la menor solución será nuestra respuesta, o sea:

t2=

(La solución que da un tiempo mayor, correspondería a otra gota que se zafaría por el otro lado

de la rueda, cuando va en ascenso: la gota subiría, haciendo una parábola ascendente, y final-

mente caería en el punto B, si no chocara con la rueda en su descenso).

Sustituimos v = ωR, arreglamos y sacamos raíz al resultado anterior, y se obtiene finalmente

t =

El punto A del cual se zafa la gota se localiza por el ángulo υ que forma la vertical con el radio

hasta el punto A (ver Fig. 29S): tan υ = vt/R, donde t está dada ya por la respuesta anterior.

2.9-Si no hubiese gravedad, la distancia

recorrida por la gota desde A sería vt en

línea recta, tangente a la rueda, y estaría

entonces bajo el centro de la rueda en un

punto P a una distancia d del centro, dada

por Pitágoras (ver Fig.2.9S):

d =

Por efecto de la gravedad la gota descen-

dería verticalmente una distancia

s = ½ gt2

La suma de los dos desplazamientos, vt y s,

R

H

B

vt d

½ gt2

θ

(v =ωR)

P

Fig.2.9S

A

ω

Page 22: problemas selectos

22

3A-Fuerzas. Problemas.

a)Calcule la fuerza F con que el cuerpo 2 empuja al 1.

b)Calcule bajo qué ángulo de inclinación θA del plano los cuerpos resbalan con rapidez cons-

tante.

3.2-En el sistema de la Fig. 3.2, la viga vertical A puede deslizar hacia abajo verticalmente sin

fricción con la cuña B, ni con las paredes del orificio vertical por el que desciende. La cuña tiene

un ángulo de inclinación θ. La relación de masas de la viga y la cuña es mA / mB = β. Calcule la

aceleración vertical aA con que desciende la viga y la aceleración horizontal aB con que se mueve

la cuña.

3.3-En el extremo de una tabla de longitud L y masa M se encuentra un pequeño bloque de masa

m (Fig. 3.3, próxima página). La tabla puede deslizarse sin fricción sobre una superficie hori-

zontal. El coeficiente de fricción entre el bloque y la tabla es μ. ¿Qué velocidad mínima vo hay

que comunicar a la tabla con un empujón instantáneo para que ella se deslice bajo el bloque y

logre quitárselo de arriba?

B θ

A

Fig. 3.2

θ

m1

m2

Fig. 3.1

3.1-Dos cuerpos 1 y 2 están en contacto, am-

bos sobre un plano inclinado con ángulo de

inclinación θ, como muestra la Fig. 3.1. Sus

masas son m1 y m2. Sus coeficientes de fric-

ción cinética con el plano son μ1 y μ2, con

μ1 > μ2. El ángulo de inclinación del plano es

tal que los cuerpos resbalan juntos por el pla-

no hacia abajo.

Page 23: problemas selectos

23

3.4-En el extremo de un resorte vertical de masa despreciable se cuelga una pesa de masa m.

Seguidamente se cuelga otra pesa de igual masa en el medio del resorte ya estirado. Determinar

la longitud del resorte estirado con las dos pesas. Considere conocidas la constante elástica del

resorte, k, y su longitud natural, lo.

3.5-Un acróbata que se halla sobre la superficie lateral de un cilindro, mueve las piernas y

desplaza al cilindro con velocidad constante (Fig. 3.5). El cilindro está sobre una superficie

horizontal rugosa. Considerando que el coeficiente de fricción entre las zapatillas y el cilindro es

μ, determine el ángulo límite θo que puede formar con la vertical el radio del cilindro que pasa

por el punto en que se encuentra el acróbata. ¿Cuál será la fuerza de fricción entre el cilindro y el

acróbata en este caso? La masa del acróbata es m.

3.6-Las masas de dos estrellas son m1 y m2 y la distancia ente ellas es l. Calcule el período de

rotación de estas estrellas por órbitas circulares alrededor de su centro de masas.

3.7-Desde los polos Sur y Norte de la Tierra se lanzan simultáneamente dos cohetes con las

mismas velocidades iniciales, horizontalmente dirigidas. Después de un tiempo τ = 3 h 20 min

los cohetes están a la máxima separación uno del otro. Determine la distancia máxima d entre los

θo

Fig. 3.5

v constante

corre hacia acá

rueda hacia acá

m

M

L

vo

Fig. 3.3

Page 24: problemas selectos

24

cohetes. La aceleración de caída libre sobre la superficie terrestre se asume conocida. El radio de

la Tierra es Ro = 6 400 km.

l

r

eje

m

A

Fig. 3.9

3.9-Un péndulo está formado por una vari-

lla ligera con una esfera pequeña y muy

densa en su extremo inferior. A su extremo

superior se acopla un casquillo cilíndrico

ligero de radio interior r que envuelve a un

eje horizontal en rotación, accionado por

un motor (Fig.3.9). La longitud desde el

centro de la esferita hasta el centro del

casquillo es l. El coeficiente de fricción

cinética entre el eje y el casquillo es μ. De-

termine el ángulo de inclinación de la va-

rilla respecto a la vertical cuando quede en

equilibrio. (Considere toda la masa del sis-

tema concentrada en el centro de la esfe-

rita).

m1 m2

l1 l2

3.8-Dos esferitas de masas m1 = 56 g y m2

= 28 g se cuelgan mediante dos hilos de

longitudes l1 = 7 cm y l2 = 11 cm al extre-

mo inferior de una barra que cuelga libre-

mente (Fig. 3.8). Determine la velocidad

angular ω con que debe rotar la barra alre-

dedor de la vertical que pasa a lo largo de

sí misma para no separarse de dicha posi-

ción vertical.

θ1 θ2

Fig. 3.8

Page 25: problemas selectos

25

3B-Fuerzas. Soluciones.

3.1-La Fig. 3.1S muestra las fuerzas sobre cada cuerpo y las direcciones tomadas como positivas.

Ahora: F12 = F21 por acción y reacción, y les llamaremos simplemente F, que es el valor de la

fuerza del bloque 2 sobre el 1. Como los cuerpos resbalan juntos: a1 = a2 ≡ a. Las ecuaciones

pueden reducirse a:

m1gsenθ + F – μ1m1gcosθ = m1a (1) y m2gsenθ – F – μ2m2gcosθ = m2a (2)

Sumando miembro a miembro (1) y (2), y despejando a:

a = g[senθ (m1 + m2) – cosθ (μ1m1 + μ2m2)] / (m1 + m2) (3)

Sustituyendo en (1) y despejando: F = m1 m2 g [(μ1 − μ2) / (m1 + m2)] cosθ

b)Si la rapidez es constante, la aceleración en (3) la hacemos cero y obtenemos:

tanθ = (μ1 m1 + μ2 m2) / (m1 + m2). De aquí: θ = tan-1

[(μ1 m1 + μ2 m2) / (m1 + m2)]

Fsenθ = mBaB (2)

B θ

A

θ Δy

Δx

mAg

F

F

3.2-En la Fig. 3.2S se muestra las fuerzas que influyen en los

movimientos analizados, y se indica la relación geométrica de

los desplazamientos en x y y de los cuerpos.

Tomando positiva la dirección vertical hacia abajo:

mAg – Fcosθ = mAaA (1)

Tomando positivo hacia la derecha:

θ

m1

m2

Fig. 3.1S

Planteando la segunda ley de Newton por

componentes a cada cuerpo, y recordando que

f =μN, tendremos:

m1gsenθ + F21 – μ1N1 = m1a1

N1 – m1gcosθ = 0

m2gsenθ – F12 – μ2N2 = m2a2

N2 – m2gcosθ = 0

m1g

m2g

f1

f2

N2

N1

F21

F12

(+)

(+)

Fig. 3.2S

Page 26: problemas selectos

26

Además: Δy = Δx tanθ. Se desprende de aquí que:

vA = vB tanθ

Y sus variaciones en el tiempo darán las aceleraciones:

aA = aB tanθ (3)

Despejando aB de (1), (2) y 3), y arreglando, se obtiene:

aB = g β /(cotθ + β tanθ)

Y sustituyendo en (3): aA = = g β / (cot2θ + β)

3.3-La velocidad instantánea que se comunique a la tabla respecto a la superficie horizontal es la

misma que adquiere el pequeño bloque respecto a la tabla. Lo que se pide es que esa velocidad vo

alcance al bloquecito para recorrer la distancia L y quedar casi en reposo cuando esté a punto de

salir de la tabla:

0 = vo2 – 2arel L (1)

Aquí arel es la aceleración del bloquecito respecto a la tabla, que a su vez está acelerada respecto

al piso.

La fricción entre la tabla y el bloquecito es f = μmg, y produce en la tabla una aceleración aM de

frenado hacia la izquierda respecto al piso (Fig. 3.3S), y una aceleración de impulsión hacia la

derecha, am, sobre el bloquecito (ambas aceleraciones respecto al piso, que es el sistema iner-

cial). Aplicando la segunda ley de Newton a cada cuerpo, en valores absolutos:

MaM = μmg y mam = μmg

De estas ecuaciones: aM = μmg/M y am = μg (valores absolutos)

Asumiendo positiva la dirección hacia la derecha: aM = −μmg/M y am = μg

La aceleración relativa del bloquecito respecto a la tabla es arel = am – aM = μg (1 + m/M)

Sustituyendo en (1) y despejando: vo =

m M

L

v

Fig. 3.3S

f, aM

f, am

Page 27: problemas selectos

27

3.4-Con el peso mg colgado al extremo inferior del resorte, éste se estira mg/k, lo que significa

que cada mitad se estira mg/2k. Si del extremo inferior no se cuelga más peso, la mitad inferior

no se estirará más, pero si de la mitad superior se cuelga otra carga mg, esto equivaldrá a duplicar

la tensión en la mitad superior del resorte, por lo que esa mitad se estirará el doble:

2(mg/2k) = mg/k.

El alargamiento total será entonces el de la mitad superior más el de la mitad inferior:

mg/k + mg/2k = (3/2) mg/k.

La longitud total de resorte será entonces: l = lo + (3/2) mg/k.

3.5-Como el cilindro se mueve con velocidad constante es entonces un sistema inercial y pode-

mos analizar el equilibrio del hombre respecto al cilindro con las leyes de Newton. Considerando

al hombre como partícula, el diagrama de fuerzas sobre él se muestra en la Fig. 3.5Sa.

Además: fs,m = μN

De estas tres ecuaciones resulta: tanθo = μ, por lo que: θo = tan-1

μ.

Entonces: f = μN = μmg cosθo

El valor obtenido para la tangente permite plantear el triángulo

cosθo = 1 /

Entonces: f = μmg/

3.6-Las estrellas se atraen gravitacionalmente con fuerzas iguales y opuestas. La fuerza de gravi-

tación que experimenta cada estrella actúa como fuerza centrípeta sobre ella. Las estrellas deben

describir circunferencias con centro común en el centro de masas del sistema: las estrellas

estarán siempre en lados opuestos respecto al centro de masas, por lo que las frecuencias de

rotación de ambas deben ser iguales.

θo

θo

Como el hombre se mantiene en esa posición respecto al cilindro,

la suma de fuerzas sobre él es nula, estando sostenido por la fric-

ción estática máxima, fs,m, si está ya en el ángulo para el cual el

resbalón es inminente:

fs,m – mg senθo = 0 y N – mg cosθo = 0.

mg

N fs,m

θo

1

μ

Fig. 3.5Sb

Fig. 3.5Sa

auxiliar mostrado al lado, del cual se deduce el valor del coseno:

Page 28: problemas selectos

28

Sea un sistema de referencia inercial con su origen en el centro de masas del sistema. Sean l1 y l2

las distancias desde cada estrella al origen del sistema, y l, la distancia entre las estrellas (se

cumple: l = l1 + l2). Entonces:

Gm1m2/ l2 = m1ω

2l1 y Gm1m2/ l

2 = m2ω

2l2

Simplificando términos: Gm2/ l2 = ω

2l1 y Gm1/ l

2 = ω

2l2

Sumando miembro a miembro: G(m1 + m2) / l2 = ω

2( l1 + l2) = ω

2l

De aquí: ω2 = G (m1 + m2 ) / l

3 .

Pero ω = 2π/T. Entonces: T =

3.7-Sea un sistema de referencia con su origen en el centro de la Tierra y sus ejes orientados ha-

cia las estrellas fijas (el sistema no rota con la Tierra). Los tipos de órbitas que siguen los cohetes

lanzados desde los polos son elipses como se muestran en la Fig. 3.7S.

ramos estas órbitas con otra conocida.

El período de estas órbitas es conocido, a saber, el doble del tiempo η en que recorrieron la mitad

de esas órbitas. Según la tercera ley de Kepler hay una proporcionalidad entre el cuadrado del

período y el cubo del semieje mayor, que para estos cohetes sería L/2. La órbita de parámetros

conocidos a que podemos recurrir para completar la proporcionalidad es la de un satélite que da

la vuelta a la Tierra pegado a su superficie, cuyo semieje mayor sería el propio radio terrestre, Ro,

y su período es fácilmente calculable por la relación:

g = ω2Ro = 4π

2Ro/T

2 , de donde T =

Entonces, por la tercera ley de Kepler: [2η / ( )]2 = [(L/2) / Ro]

3.

Despejando L y evaluando parámetros: L ≈ 5.6 Ro. Sustituyendo en (1): d = 9.2 Ro ≈ 5.9×104

km.

L

L

Tierra

Son elipses iguales cuyos ejes mayo-

res valen L; la separación entre los

vértices opuestos es:

d = L + (L – 2Ro) = 2L – 2Ro (1)

y esta es la máxima separación posi-

ble entre los cohetes.

Para determinarla podemos hacer uso

de la tercera ley de Kepler si compa-

Fig. 3.7S

Page 29: problemas selectos

29

3.8-Sea un sistema de referencia fijo al techo del cual cuelga la barra. Apliquemos la segunda ley

de Newton por componentes vertical y horizontal, con la condición de que la barra permanezca

vertical. Esto implicará que las dos esferitas describan circunferencias con un centro común, en

la vertical de la barra: la fuerza resultante sobre cada una de estas esferitas será una fuerza cen-

trípeta. Si llamamos θ1 y θ2 al ángulo de cada hilo con la vertical, tendremos:

Para la de masa m1: T1 cosθ1 – m1g = 0 T1 senθ1 = m1ω2(l1 senθ1)

Para la de masa m2: T2 cosθ2 – m2g = 0 T2 senθ2 = m2ω2(l2 senθ2)

Además, la resultante de fuerzas horizontales sobre la barra debe ser cero: T1 senθ1 = T2 senθ2

Se obtiene así un sistema de cinco ecuaciones con cinco incógnitas: T1, T2, θ1, θ2 y ω.

Despejando:

≈ 14 rad/s2

l

r

eje

mg

θ

θ

N

μN

3.9-La incógnita a determinar es el ángulo υ de la Fig. 3.9S. En el

equilibrio, la línea de acción del peso debe pasar por el punto de

suspensión del sistema, y la fuerza total hacia arriba debe com-

pensar al peso del sistema. Entre la normal N y la vertical hay un

ángulo θ en el punto de suspensión A en el equilibrio. En tal situa-

ción:

mg cos θ = N y mg sen θ = μN

Dividiendo miembro a miembro la segunda ecuación por la

primera: tan θ = μ (1)

Además, de los triángulos de la figura: l sen υ = r sen θ

por lo que: sen θ = (l/r) sen υ (2)

y: tan θ = sen θ / cos θ = sen θ / (1 – sen2 θ)

1/2

O: tan θ = 1/(sen-2

θ – 1)1/2

(3)

Sustituyendo (1) y (2) en (3):

μ = 1 / [(l/r)-2

sen-2

υ – 1]1/2

Despejamos sen υ: sen υ = (r/l) μ/ (μ2 + 1)

1/2

Y de aquí: υ = arc sen [(r/l) μ/ (μ2 + 1)

1/2]

A θ

mg

Fig. 3.9S

Page 30: problemas selectos

30

4A-Conservación y variación de cantidades mecánicas. Problemas.

Determine la velocidad de cada cuerpo después del choque.

4.4-Tres pequeños cuerpos cuyas masas se relacionan como 3:4:5 se mantienen en tres puntos

diferentes de la superficie interior lisa de una taza semiesférica de radio r. El punto más bajo de

la taza descansa en un plano horizontal. En cierto momento los cuerpos se sueltan y se aban-

Fig. 4.3

4.3-Sobre un grupo de tres cubos lisos idénticos que

descansan sobre una superficie horizontal lisa (Fig.

4.3) incide un cilindro liso con velocidad v. La masa

de cada cubo es igual a la masa del cilindro. El diá-

metro del cilindro y su altura son iguales al lado de

cada cubo.

v m

m

m

m

Vista superior

M M

m

h

Fig. 4.2

4.2-Dos cuñas móviles idénticas de

masa M poseen transiciones suaves

con el plano horizontal (Fig. 4.2).

Desde la cuña izquierda resbala un

cubito de masa m desde una altura h.

¿Qué máxima altura hmax se eleva el

cubito sobre la cuña derecha? Des-

precie la fricción.

m1

m2 r

4.1-Sobre una superficie horizontal li-

sa, junto a una pared, descansa un blo-

que simétrico de masa m1 con una

oquedad de forma semiesférica, de

radio r (Fig. 4.1). Desde la posición

inicial indicada resbala un pequeño

bloque de masa m2, sin fricción. En-

cuentre la velocidad máxima del blo-

que grande en su movimiento. Fig. 4.1

Page 31: problemas selectos

31

donan a sí mismos. Sabiendo que la masa del más pequeño es m, calcule la máxima cantidad de

calor Q que puede desprenderse de tal sistema. ¿Para qué posiciones iniciales de los cuerpos esto

ocurre? Considere que todos los choques son absolutamente plásticos.

4.5-Sobre un apoyo de altura h = 5 m se halla una esfera de masa M = 200 g. Una bala de masa m

= 10 g que avanza en dirección horizontal con velocidad v = 500 m/s perfora la esfera exac-

tamente en la dirección de su diámetro horizontal. ¿A que distancia l cae al suelo la bala si la

esfera cae a una distancia L = 20.0 m a partir de la base de la mesa? ¿Qué parte de la energía

cinética de la bala se transforma en energía interna cuando la bala perfora a la esfera? Despreciar

la resistencia del aire.

4.6-Una esfera rígida de masa 2m se dispara verticalmente hacia arriba con velocidad vo. A la

esfera está ligado un hilo ligero de longitud l (l < vo2/ 2g) perfectamente inextensible. En el otro

extremo del hilo hay ligada otra esfera rígida de masa m. El lanzamiento de la primera esfera

ocurre prácticamente desde el mismo punto donde se encuentra la segunda. ¿Después de cuánto

tiempo y a qué distancia del punto de lanzamiento chocan las bolas? Despreciar los diámetros de

las esferas.

4.7-Un péndulo matemático se inclinó un ángulo de 90º respecto a la vertical y se soltó. En el

momento en que el péndulo pasaba por su posición de equilibrio, su punto de suspensión

comenzó a moverse hacia arriba con aceleración a. ¿Cuál será el máximo ángulo que se separará

el péndulo de la vertical?

El gas es expulsado con una velocidad de 10000 m/s respecto a la nave ¿Qué cantidad de gas

expulsó la nave en la maniobra? (Radio de la Luna: 1700 km; aceleración de la gravedad en la

superficie lunar: 1.7 m/s2).

Luna

v

4.8-Una nave de 1000 t (toneladas métricas) está en ór-

bita circular alrededor de la Luna a 100 km sobre su su-

perficie. Para acercarse casi rasante a la Luna, expulsa

instantáneamente un chorro de gas hacia delante en su

órbita, de modo que frena un poco y pasa a una nueva

órbita que le permite pasar rasante a la Luna por un pun-

to diametralmente opuesto al punto en que lanzó la masa

de gas (Fig. 4.8).

u w R

h

Fig. 4.8

Page 32: problemas selectos

32

4.10-Una cinta transportadora de longitud l se mueve con velocidad v respecto al suelo. Por su

extremo derecho se introduce un bloque de masa m que se moverá en contra de la cinta (ver Fig.

4.10). Determine la velocidad vo (respecto al suelo) con la cual es necesario impulsar el bloque

contra el movimiento de la cinta para que la cantidad de calor liberada por el frenado del bloque

por la cinta sea la máxima posible. Considere conocido el coeficiente de fricción μ entre la cinta

y el bloque. Determine la cantidad de calor liberada en el proceso de frenado sobre la cinta. (En

la situación planteada se cumple que v < (2μlg)1/2

).

4.12-Un cuerpo de masa m = 0.5 kg está unido con dos paredes verticales mediante dos resortes

ligeros y realiza oscilaciones moviéndose rectilíneamente por una superficie horizontal (Fig.

4.12). Las amplitudes de dos elongaciones sucesivas respecto al punto de equilibrio a la derecha

A

M

m

l

4.11-Una varilla sin peso, de longitud l, con una bolita de

masa m en su extremo superior, está articulada por el

punto inferior A como indica la Fig.4.11, y se encuentra

en posición vertical, rozando una caja de masa M sobre

un piso sin fricción. Producto de un ligero golpecito el

sistema se pone en movimiento. ¿Bajo qué relación de

masas M/m la varilla formará un ángulo θo= π/6 con la

horizontal en el momento en que se despegue de la caja?

¿Cuál es la velocidad de la caja en ese instante? Se

desprecia cualquier fricción.

Fig. 4.11

v

vo l

μ

m

R vo

h

Fig. 4.9

4.9-Una partícula es impulsada horizontalmente con velo-

cidad vo dentro de un casco semiesférico liso de radio R, en

contacto con la superficie interior, desde un punto loca-

lizado en el borde del casco (Fig. 4.9). Calcule la altura h

que descenderá la partícula antes de volver a subir por el

casco.

Fig. 4.10

Page 33: problemas selectos

33

y a la izquierda son A1= 10 cm y A2= 7 cm. Determine el coeficiente de fricción cinética μ entre

el cuerpo y la superficie sabiendo que la constante elástica de ambos resortes es k = 15 N/m.

4B-Conservación y variación de cantidades mecánicas. Soluciones.

4.1-Puesto que todas las superficies son lisas, no habrá pérdidas de energía mecánica por fric-

ción. Mientras el bloquecito cae empuja al bloque grande contra la pared, pero ésta lo aguanta y

no le permite movimiento. Al ascender por la otra parte de la semiesfera, empuja al bloque

grande hacia adelante y le comunica cantidad de movimiento horizontal, así como energía ciné-

tica. Al llegar a su punto de máxima altura, retorna y vuelve a caer, y en esa caída continúa

empujando al bloque grande hacia la derecha. Al llegar a la parte más baja en ese retorno co-

mienza a subir por la oquedad y ya empieza a frenar al bloque grande. Por tanto, la máxima

velocidad del bloque grande se alcanza cuando el pequeño pasa por el punto más bajo, de retorno

hacia la pared.

La energía inicial del sistema es E = Ug = m2gr. Al llegar abajo en el primer descenso, la energía

se convierte en cinética y la velocidad que alcanza hacia la derecha será v02 = . Su mo-

mento lineal será en ese momento p02 = m2v02, y éste será el momento total horizontal del

sistema en ese instante; y se conservará en lo sucesivo, pues ya se elimina la acción horizontal

externa de la pared.

Cuando el bloquecito llega al punto más bajo en su retorno, su rapidez será v2 hacia la pared y la

del bloque grande será v1 hacia la derecha. Entonces:

m2v02 = m1v1 – m2v2 y ½ m2v022 = ½ m1v1

2 + ½ m2v2

2, donde v02 ya se conoce. Resolviendo

para v1:

v1 =2m2v02/(m1+m2) = 2m2 / (m1+m2)

m

k k

Fig. 4.12

Page 34: problemas selectos

34

04.2-En la situación planteada se conservan el momento lineal horizontal del sistema y la energía

mecánica total, dado la ausencia de fricción y de cualquier otra fuerza horizontal externa al

sistema cuñas-cubito.

Durante el descenso por la primera cuña, ésta adquiere movimiento hacia la izquierda, con

velocidad V, y el cubito adquiere velocidad v hacia la derecha. Se cumplirá:

mgh = ½ mv2 + ½ MV

2 y 0 = mv – MV

De estas dos ecuaciones se obtiene: v = (1)

Cuando el cubito asciende por la segunda cuña, la empuja hacia adelante y cuando llega a su

máxima altura, h’, comparte la velocidad horizontal de la cuña, V’, y se cumplirá:

mv = (m + M)V’ y ½ mv2 = ½ (m + M) V’

2 + mgh’

Eliminando V’ de estas dos ecuaciones: 2gh’= v2/ (1 + m/M)

Y teniendo en cuenta (1):

h’= h / (1 + m/M)2

4.3-El cilindro entra en contacto simultáneamente con los cubos laterales, con los cuales coli-

siona centralmente. Las fuerzas de contacto en el choque se producen perpendicularmente a las

caras de los cubos (Fig. 4.3S), por lo que cada cubo sale con velocidad u (iguales por la simetría

de la situación), sin empujar al cubo central. El cilindro a su vez quedará con una velocidad v’

(hacia adelante o hacia atrás). Estos choques serán elásticos y se conservarán la energía y el

momento del sistema:

Resolviendo estas dos ecuaciones se obtiene v’= 0 y u = ( /2) v. Significa que el cubo central

no recibe impacto porque el cilindro queda en reposo, por lo que no se mueve, en tanto los cubos

laterales salen con las velocidades calculadas.

Fig. 4.3S

v’

u

u

½ mv2 = ½ mu

2 + ½ mu

2 + ½ mv’

2 y

mv = mu cos45 + mu cos45 + mv’

O:

v2 = 2u

2 + v’

2 y v = 2u cos45 + v’

Pero cos45 = /2, lo cual modifica la segunda

ecuación:

v2 = 2u

2 + v’

2 y v = u + v’

Page 35: problemas selectos

35

4.4-En los choques plásticos los cuerpos se pegan, pero pueden quedar pegados y con velocidad

hacia alguna parte, o podrían quedar pegados y en reposo. La energía de cada cuerpo es

inicialmente potencial, y respecto al fondo de la semiesfera es mgr, para el primero, (4/3)mgr

para el segundo y (5/3)mgr para el tercero. Cada una se transforma en cinética en esa misma

cantidad cuando llega a la parte de abajo del interior de la semiesfera. Si las tres esferas se

sueltan a la vez desde el borde superior, llegan a la vez al fondo con la máxima energía cinética

posible, KA + KB + KC = 4mgr, y si se pegaran y quedaran en reposo, toda esa energía se

convertiría en calor Q, la máxima cantidad posible si quedan en reposo. Ahora, para que las tres

queden pegadas en reposo después del choque no solamente hay que soltarlas simultáneamente

desde la misma altura, sino desde posiciones que al llegar abajo tengan una cantidad de movi-

miento total nula antes del choque, pues sólo así quedarán en reposo, conservando el momento

lineal del sistema. Las posiciones iniciales están sugeridas por las relaciones entre las masas,

3:4:5, pues son los números pitagóricos típicos: al llegar abajo todos con la misma rapidez,

4.5-La Fig. 4.5S (próxima página) muestra la situación. La bala empuja a la esfera horizontal-

mente cuando la atraviesa y, despreciando las dimensiones de la esfera en relación con la altura

de la mesa, ambas llegan a la vez al piso. La componente horizontal del momento lineal total de

la bala y la esfera se conserva pues no hay fuerzas horizontales externas (se desprecia el aire), y

la fuerza externa de gravedad altera solamente a la componente vertical del momento lineal, no a

la componente horizontal.

Ahora bien: el momento lineal total del sistema es igual a la masa total del sistema por la velo-

cidad del centro de masa del sistema. El centro de masas del sistema se mueve horizontalmente

con la misma velocidad V antes y después del impacto: (M + m)V = mv + Mvoe donde voe = 0 es

la velocidad de la esfera antes del choque. Por tanto: V = mv / (m + M) = 23.8 m/s

Vista

superior

v = , sus momentos lineales estarán en

la relación de sus masas: pA : pB : pC = 3:4:5.

Así, habrá que colocarlos en el borde superior

de la semiesfera, en tres puntos A, B, C tales

que cuando los cuerpos desciendan al centro

de la semiesfera, sus momentos lineales pA,

pB, pC queden formando un triángulo pita-

górico, de ángulos 53o y 37

o. La Fig. 4.4S

muestra la situación. En las posiciones ini-

ciales, la de masa m debe estar en A, a 90o de

la de masa (4/3)m, en B, y a 127o de la de

masa (5/3)m en C. Fig. 4.4S

A

B

C

53

37 90

pB

pC

m

(4/3)m

(5/3)m

pC

Page 36: problemas selectos

36

Cuando la bala y la esfera llegan al piso, ha transcurrido en la caída un tiempo:

t = = 1.01 s

En ese tiempo el centro de masas avanza horizontalmente: xcm = Vt = 24.0 m

Pero el centro de masas de dos cuerpos satisface la relación:

xcm = (M xe + mxb) / (M + m)

Y sabemos que xe = 20.0 m y xcm = 24.0 m. Entonces:

xb = [(M + m) xcm – Mxe] / m = 104 m

Para calcular la fracción de energía perdida necesitamos las velocidades de la bala y la esfera al

terminar el choque. Tales velocidades horizontales podemos calcularlas por las distancias hori-

zontales que avanza cada una durante el tiempo de caída:

v’e = x2/t = 19.8 m/s y v’b = xb/t = 103 m/s. Entonces:

ΔK / K = (½mvb2 - ½mv’b

2 - ½Mv’e

2) / (½mvb

2) = 0.93.

4.6-En el instante en que se pone vertical y tirante el hilo, la velocidad de la esfera de masa 2m

es:

v = y ha transcurrido un tiempo t0 tal que v = vo – gt0, por lo que:

t0 = (vo – v) /g =

Instantáneamente el hilo tirante frena a la esfera de masa 2m y le imprime una nueva velocidad

u1, a la vez que levanta a la segunda esfera con una velocidad inicial u2. Este tirón de la cuerda se

comporta como un choque elástico entre las bolas (son indeformables las esferas e inextensible el

hilo), por lo que:

2 mv = 2 mu1 + mu2 (conservación del momento lineal)

x

xb xcm xe O

Fig. 4.5S

Page 37: problemas selectos

37

Y: ½ (2m) v2 = ½ (2m) u1

2 + ½ m u2

2 (conservación de la energía cinética)

Resolviendo el sistema: u1 = v/3 y u2 = 4v/3

La esfera de abajo arranca con más velocidad que la que tiene la primera después del tirón, por lo

que la alcanzará y chocara con ella. Después del tirón, ambas esferas continúan hacia arriba

influidas solamente por la fuerza de gravedad (el hilo flojo no hace fuerza) y satisfacen:

y1 = l + u1 t1 – ½ gt12 y y2 = u2 t1 – ½ gt1

2

Cuando choquen: y1 = y2

Entonces: l + u1 t1 – ½ gt12 = u2 t1 – ½ gt1

2

O sea: t1 = l / (u2 – u1) = l /(4v/3 – v/3) = l / v = l /

Así, chocan un tiempo t1 después de haberse producido el tirón del hilo. El tiempo t transcurrido

desde que se tiró la primera esfera es:

t = t1 + t0

ambos conocidos ya. El punto en que se produce el choque está por arriba del punto de lanza-

miento una distancia dada por:

y2 = u2 t1 – ½ gt12 = (4v/3)(l/v) – ½ g (l/v)

2 = l {4/3 − 1/ [2(vo

2/gl - 2)]}

4.7-Durante el descenso del péndulo matemático (péndulo simple), la masa desciende una altura

igual a la longitud del péndulo, l, y por conservación de la energía llega al punto inferior con una

velocidad horizontal de rapidez v = . Justo al comenzar su ascenso el punto de suspensión

se acelera verticalmente, por lo que en el sistema no inercial asociado al punto de suspensión

aparece una fuerza de gravedad aparente mayor, dada por m(g+a).

En este sistema no inercial, la energía potencial aparente de la masa del péndulo, cuando éste

suba una altura h será U = m (g+a)h. Conservando la energía durante el ascenso, y teniendo en

cuenta que en el punto inferior de la masa del péndulo la velocidad horizontal es la misma en el

sistema fijo a Tierra y en el sistema no inercial acelerado verticalmente hacia arriba, tendremos:

½ mv2 = m (g+a) h

Sustituyendo el valor encontrado para v y despejando: h = l g / (g+a) (1)

Se ve que el péndulo asciende una altura menor que l.

Page 38: problemas selectos

38

4.8-Llamemos m a la masa de la nave y Δm, a la masa de gas expulsada. La nave está inicial-

mente en una órbita circular que satisface:

mv2/(R + h) = GmM/(R + h)

2

donde M es la masa de la Luna. Simplificando:

v2 = GM/(R + h) = gR

2/(R + h)

donde g = GM/R2 es la aceleración de la gravedad en la superficie lunar. Esta velocidad es la de

la nave en la órbita inicial.

Evaluándola: v = 1652 m/s

Al expulsar gas instantáneamente, en ese mismo punto de la órbita la nave se frena a un nuevo

valor de velocidad w con el que realiza una elipse, cuya velocidad es u al pasar rasante a la

superficie lunar. En los extremos del eje mayor de la elipse se cumplirá:

½ mw2 – GmM/(R+h) = ½ mu

2 – GmM/R (conservación de energía)

mw(R + h) = muR (conservación del momento angular)

De estas dos ecuaciones se despeja w:

= = 1628 m/s

Tenemos entonces las velocidades de la nave antes y después de expulsar la masa Δm de gas.

Por conservación del momento lineal al expulsar el gas:

mv = (m – Δm)w + (Δm)(w + vgn) donde vgn= 10000 m/s

Despejamos Δm: Δm = m (v – w) / vgn = 2.4 t

θ

l

l

h

Fig. 4.7S

El máximo ángulo θ que asciende el péndulo se

encontrará por trigonometría, teniendo en cuenta

que (Fig. 4.7S):

h = l – l cosθ, que da un ángulo menor que 90o.

Igualando con la expresión (1) y despejando:

cosθ = a/(a + g)

Page 39: problemas selectos

39

Por otra parte, la fuerza normal de contacto apunta siempre hacia el punto O en el centro de la

base del hemisferio, en el eje vertical de simetría (Fig. 4.9S), por lo que no produce torque ver-

tical respecto a ese eje (sólo produce torque horizontal). Y el peso apunta hacia abajo, por lo que

produce un torque horizontal respecto a O en cada momento, y este torque tampoco tiene com-

ponente vertical jamás; significa que la componente vertical del momento angular de la partícula

se conserva (Lo,vert = Lf,vert ). En el punto más alto de la trayectoria y en el punto mas bajo, la

partícula tiene velocidad horizontal y momento angular solamente vertical, por lo que podremos

plantear:

mvoR = mvr (2)

donde R y r se miden perpendicularmente desde el eje vertical a la partícula en cada punto (Fig.

4.9S). De la misma figura se ve que:

r = (3)

De (2) y (3): v = vo R/r = vo R/ = vo /

Sustituyendo en (1):

vo2 = vo

2 / [1 – (h/R)

2] – 2gh

Da una ecuación cúbica en h:

h3 + (vo

2/2g) h

2 – R

2h = 0

Una solución es h = 0, que corresponde a la altura inicial. La otra solución sale de la cuadrática:

h2 + (vo

2/2g) h – R

2 = 0

Obteniéndose: h = −(vo2/4g) ±

Es válida la solución positiva: h =

Si vo = 0 la partícula desciende h = R hasta el fondo del casco antes de volver a subir. Si vo ≠ 0,

la partícula desciende una altura h<R, como puede demostrarse a partir de la solución obtenida.

R vo

h N

r mg

Fig. 4.9S

4.9-No hay fricción y la fuerza normal de contacto no trabaja

sobre la partícula, por lo que la única fuerza que trabaja es la

de gravedad, que es conservativa; significa que la energía

mecánica se conserva en este movimiento. Llamando v a la

velocidad en el punto más bajo, cuando la partícula descien-

de una altura h podemos plantear:

½ mvo2 = ½ mv

2 – mgh (1)

O

R

Page 40: problemas selectos

40

4.10-La máxima cantidad de calor liberada corresponde al caso en que el bloque recorra la mayor

distancia posible respecto a la cinta, para que la fricción cinética trabaje a lo largo de la mayor

distancia. Por tanto, el bloque no puede lanzarse tan rápido que enseguida llegue al extremo

opuesto del transportador y se salga de la cinta; ni tan despacio que el bloque frene enseguida

sobre la cinta y ésta la traiga de regreso a su punto de partida sin más deslizamiento.

La velocidad que hay que comunicarle al inicio debe llevar al bloque justo hasta el extremo

opuesto del transportador, y que ahí se detenga respecto al suelo. En tal punto, con el bloque en

reposo respecto al suelo, pero con la cinta moviéndose bajo el bloque, seguirá habiendo una

fricción cinética que comenzará a impulsar al bloque en su retorno. Esta fricción cinética

impulsará al bloque hasta que su velocidad respecto al suelo sea la misma de la cinta, en cuyo

caso ya no habrá más deslizamiento. Y hasta que esto ocurra se estará liberando calor.

El bloque pesa mg, la normal que lo sostiene en la cinta es N = mg, y la fricción cinética que se

desarrolla es f = μmg. Como ésta es la única fuerza horizontal sobre el bloque, ella es la resul-

tante que determine la aceleración del mismo. O sea: ma = μmg determina que a = μg.

En la ida hasta el extremo opuesto, ésta será la aceleración de frenado respecto al suelo, pero en

el regreso, ésta será la aceleración de impulsión del bloque respecto al suelo hasta que alcance la

velocidad v de la cinta. En la ida al extremo opuesto:

v2 = vo

2 – 2ad = vo

2 – 2μgd

Y queremos que cuando se haya desplazado la longitud completa del transportador, d = l, su

velocidad se haga cero, v = 0:

0 = vo2 – 2μgl

Y de aquí, la velocidad inicial necesaria respecto al suelo será: vo =

Se ve que es mayor que la velocidad de la cinta, pues por dato v < (2μlg)1/2

.

Ahora, el trabajo de la fuerza de fricción, que se convertirá en calor, se calcula por W = fd, donde

d representa la distancia total recorrida por el bloque sobre la cinta, que es mayor que l en la ida,

y aún hay que sumarle algo más en el retorno. Para averiguar la distancia deslizada en contacto

con la cinta necesitamos saber qué tiempo demora el bloque deslizándose sobre la cinta

(experimentando fricción cinética). Multiplicando este tiempo por la velocidad de la cinta

sabremos la longitud que avanzó la cinta mientras el bloque se deslizaba en la ida. El tiempo de

ida, ti, podemos obtenerlo de v = vo – at, sabiendo que al cabo del tiempo de ida el bloque se

detiene respecto al suelo:

0 = (2μgl)1/2

– μg ti

De aquí: ti =

Page 41: problemas selectos

41

La cinta avanzó durante la ida:

Y la longitud de deslizamiento en la ida será el largo del transportador, l, más lo que se mueve la

cinta durante el tiempo de ida:

sida = [ ] (1)

Hay que agregar ahora el deslizamiento mientras retorna el bloque, lo cual ocurre durante un

tiempo de retorno tr tal que el bloque pasa de velocidad cero a la velocidad v de la cinta:

v = 0 + atr = μg tr, por lo que: tr = v / μg

En este tiempo la cinta recorre: scinta = v tr = ( v2 / μg)

En el propio tiempo, el bloque avanza respecto al suelo una distancia

sbloque = 0 + ½ atr2 = ½ (μg)(v/μg)

2 = (v

2 / 2μg)

que es menor que la longitud completa del transportador, pues por dato v < (2μlg)1/2

.

La distancia con deslizamiento efectivo sobre la cinta en el retorno es entonces:

sretorno = scinta − sbloque = ( v2 / μg) − ( v

2 / 2μg) = (v

2 / 2μg) (2)

La distancia total con deslizamiento en la ida y en el retorno es la suma de (1) y (2):

d = [l + v(2l / μg)1/2

] + (v2 /2μg) = [v + ]2

/ (2μg)

El trabajo, convertido en calor, será entonces:

W = Q = fd cos180= −( m/2) [v + ]2

Puede verse que este trabajo equivale a la pérdida de energía cinética del bloque respecto a la

cinta, ya que al entrar en ella el bloque tiene velocidad v + vo = v + y energía cinética

K = ½m [v + ]2. Esta energía la pierde el bloque totalmente al quedar en reposo sobre al

cinta.

(La variación de energía cinética respecto al sistema fijo al suelo es otra:

ΔK = ½mv2 – ½mvo

2 = ½m (v

2 −2μgl)

Y está de acuerdo con el trabajo de la fricción –fuerza resultante en ese sistema−, dado por:

W = f lidacos180 + f sbloque-retorno cos0 = −μmgl + μmg (v2/2μg) = ½m (v

2 −2μgl)

Respecto al suelo la fricción hace trabajo positivo sobre el bloque en el retorno, impulsando al

bloque desde una velocidad cero hasta la velocidad de la cinta, v. Este trabajo positivo no revela

Page 42: problemas selectos

42

el calor liberado en ese proceso de impulsión. Ni el trabajo de frenado en la primera parte

revela el calor liberado. El cálculo del calor liberado debe considerar la distancia física reco-

rrida por el cuerpo sobre la superficie con fricción).

aceleración horizontal de la caja, ax, y el ángulo θ de la varilla con la horizontal que es el mismo

que el que existe entre la aceleración tangencial y el vector peso, mg.

Mientras estén en contacto la bola y la caja, la componente horizontal de la aceleración de la bola

es la misma que la aceleración horizontal de la caja. Para la bola, su vector aceleración a puede

expresarse en función de sus componentes tangencial y normal: a = at + (v2/l)n

donde n es el vector unitario normal a la trayectoria, que apunta hacia la articulación A. De la

figura se ve, en la bola, que:

ax = at senθ – (v2/l) cosθ

que es también la aceleración de la caja, que satisface: Max = N

O sea: N = M at senθ – M (v2/l) cosθ

Por tanto, en ese instante: at = g cosθo

Sustituyendo en (1): g cosθo senθo = (v2/l) cosθo

De aquí: v = (2)

que es la velocidad de la bola al despegarse de la caja. La velocidad de la caja en ese momento es

la componente horizontal de la velocidad de la bolita con la que estaba en contacto:

A

M

l

Y en el momento del despegue: N = 0, lo que implica:

at senθo = (v2/l) cosθo (1)

Pero, al despegarse, la única fuerza sobre la bolita

capaz de producir aceleración tangencial es la fuerza de

gravedad, pues la que le hace la varilla, F, está dirigida

hacia A (no hay fricción en A) (Fig. 4.11Sb).

θ

mg mat

V

θ

V

v θ

F

A

M N

l

4.11-La Fig.4.11Sa muestra la varilla y la caja en un

momento cualquiera de sus movimientos. Se repre-

sentan el peso mg de la bolita superior y la fuerza N

con que se le resiste el bloque; no se representa la

fuerza de la varilla sobre el peso superior.

Se indican también la aceleración total a de la bola, su

aceleración normal, an, su aceleración tangencial, at, la

θ

mg a

at

an θ

ax

ax

θ

Fig. 4.11Sa

Fig. 4.11Sb

Page 43: problemas selectos

43

V = v senθo = (3)

Por conservación de la energía tendremos:

mgl =mgl senθo + ½mv2 + ½MV

2

Haciendo uso de (2) y (3), y teniendo en cuenta que senθo = sen(π/6) = ½, tendremos:

mgl = ½mgl + ½mgl(½) + ½Mgl(1/8)

Arreglando: M/m = 4

Además, como θo = π/6, entonces, de (3): V =

4.12-La energía potencial acumulada en cada extremo de la oscilación está dada por la suma de

las energías potenciales de los dos resortes: U = ½ kA2 + ½ kA

2 = kA

2.

Como en los extremos no hay energía cinética, la energía perdida ΔE al moverse el bloque de un

extremo al otro es la variación de energía potencial:

ΔE = ΔU = kAf2 –kAi

2 = k (Af

2 – Ai

2)

Y esta pérdida de energía será igual al trabajo total de la fricción en el recorrido d = (Ai + Af):

μmgdcos180o = k (Af

2 – Ai

2)

Despejando μ y evaluando: μ = k (Af2 – Ai

2) / [−mg (Ai + Ad)] = k(Ai – Af) / (mg) = 0.090

Page 44: problemas selectos

44

5A-Sólido Rígido. Problemas.

5.1-Una varilla sin peso se mantiene en equilibrio horizontal. Su longitud es 3l y tiene acoplada

dos pesas de masas m1 y m2 a iguales distancias una de otra y de los extremos; cuelga de hilos

verticales (Fig. 5.1). Determine la fuerza de tensión T en el hilo izquierdo en el instante en que se

acaba de cortar el hilo derecho.

5.2-A un cubo homogéneo de lado l se le saca un octante, quedando el cuerpo de la Fig. 5.2. Cal-

cule la posición de su centro de masas.

5.3-Un lápiz de sección hexagonal se coloca sobre un plano inclinado un ángulo θ con la hori-

zontal. Se coloca primero de modo que la longitud del lápiz queda perpendicular a la recta

generatriz del plano (recta de intersección del plano con su base horizontal) y se observa que el

lápiz queda en reposo. Se coloca luego con la longitud del lápiz paralela a la recta generatriz del

plano inclinado, y se observa que el lápiz se va rodando cuesta abajo. Determine el mínimo

ángulo υ entre la longitud del lápiz y la recta generatriz bajo el cual el lápiz queda aún en reposo

y no rueda cuesta abajo (Fig. 5.3).

θ

No resbala

Rueda

υ

Ángulo mínimo

para que no ruede

Fig. 5.3

m1 m2

l

l l

Fig. 5.1 Fig. 5.2

x y

z

Page 45: problemas selectos

45

5.4-Dos planos lisos están inclinados respecto al horizonte y entre sí 60º. ¿Cómo es necesario

colocar un cubo entre estos planos para que él se encuentre en equilibrio? ¿A qué altura de la

base de los planos quedará el centro de masas del cubo? La fricción entre el cubo y los planos es

despreciable.

5.8-Un disco de masa M y radio R gira alrededor de un eje vertical con velocidad angular Ω.

Sobre él sueltan otro disco, de radio r y masa m, cuyo eje de simetría se mantiene rigurosamente

vertical, pero no coincide con el eje del primer disco. La distancia ente los ejes es d (se cumple

que r < d < R). Entre las superficies de los discos hay fricción. La base del disco chiquito cae

íntegramente dentro del disco grande. Determine la velocidad angular final del disco chiquito.

No hay fricción en el eje del disco grande.

m

m

l

l

v

Fig. 5.7

5.7-Una varilla delgada homogénea de masa m y largo l resbala

con velocidad v sobre una superficie horizontal sin fricción y

choca plásticamente con otra igual en reposo como indica la Fig.

5.7. Calcule la velocidad de traslación y rotación del conjunto

después del choque, así como la energía perdida en el choque.

Vista superior

l

M

M

v

5.6-Una fina varilla de masa M y longitud l reposa sobre una su-

perficie horizontal totalmente lisa. Un pequeño bloque de la mis-

ma masa M, con velocidad v dirigida perpendicularmente a la

varilla, choca sobre un extremo de ella, al cual se pega instantá-

neamente. ¿Cuáles son las velocidades de traslación y de rota-

ción del sistema después del choque? (Fig. 5.6).

Fig. 5.6

Vista superior

m

M

F

Fig. 5.5

O

5.5-Un bloque de masa M se encuentra

sobre una mesa horizontal lisa por la cual

puede moverse sin fricción. Sobre el

bloque está un cubo de masa m que se

apoya en un pequeño saliente O (Fig.

5.5). ¿Cuál es la máxima fuerza F aplica-

da al bloque horizontalmente que no pro-

duce el volteo del cubo?

Page 46: problemas selectos

46

5.9-Considere una bola de billar como una esfera homogénea perfecta. La bola descansa en

reposo sobre una mesa horizontal con fricción y recibe un golpe central con el taco, o varilla, que

le imprime instantáneamente una velocidad de traslación pura sobre la mesa, y con ello, cierta

cantidad de energía cinética (Fig. 5.9a). La bola empieza a rotar al deslizarse sobre la mesa (Fig.

5.9b) hasta que en un momento dado comienza a moverse en rodadura pura, sin deslizar más

(Fig. 5.9c). Calcule la fracción de energía que pierde la bola en el proceso de deslizamiento hasta

alcanzar la rodadura pura. (El momento de inercia de la esfera respecto a su eje central está dado

por I = (2/5)mR2

)

5.10-Un cuerpo está formado como indica la Fig.5.10, por un cilindro central y dos cilindros

iguales en sus extremos, cada uno de mayor radio que el cilindro central (como un eje con dos

ruedas). Los tres cilindros son de igual longitud, pero el radio de los extremos es el doble del

radio del cilindro central. El cuerpo se coloca sobre un plano inclinado estrecho, con fricción, de

manera que solamente el cilindro central hace contacto con el plano, en tanto los cilindros

laterales quedan en el aire. El cuerpo comienza a rodar en rodadura pura por el plano inclinado, y

pasa a un piso horizontal con fricción (sin saltar) donde los cilindros laterales son los que hacen

contacto con el piso. Si el cuerpo desciende una altura h por el plano inclinado, calcule la

velocidad final con que avanzará por el piso en rodadura pura. Los cilindros son compactos,

homogéneos y del mismo material.

h vf, ωf

vo v

ω

fk fk (a) (b) (c)

Fig. 5.10

Fig. 5.9

Page 47: problemas selectos

47

5-Sólido Rígido. Soluciones.

5.1-La Fig. 5.1S muestra las fuerzas en la varilla y en cada pesa un instante después de cortado el

hilo derecho.

Las incógnitas son T, N1 y N2. Las fuerzas normales de contacto son de direcciones des-

conocidas: podrían representarse lo mismo para arriba que para abajo. Si se es consecuente en el

planteamiento de ecuaciones (de fuerzas, torques y ligaduras entre aceleraciones) con los signos

correspondientes a las direcciones planteadas para estas fuerzas en el diagrama, el resultado será

correcto. Si la dirección real de alguna de estas normales N es opuesto al representado en la figu-

ra, al calcular su valor resultará con signo negativo, pero el valor absoluto será el correcto.

m1g

m2

m2g

m1

N2

N2

N1

N1

T

l l

l

5.11-En los extremos y en el medio de una varilla ligera de

longitud l fueron colocadas esferitas idénticas. La varilla se

coloca verticalmente y se suelta (Fig. 5.11). Considere que la

fricción entre la superficie y la esfera inferior está ausente y

encuentre la velocidad de la esfera superior cuando llega al

piso. ¿Cómo cambia la respuesta si la esfera inferior está fija a

un gozne sin fricción?

Fig. 5.11

Fig. 5.1S

Page 48: problemas selectos

48

Sea el sistema de referencia fijo al soporte del sistema, con un eje positivo hacia abajo. Apli-

cando la segunda ley de Newton a cada cuerpo:

N1 − N2 – T = mvarilla avarilla = 0 (1) (pues mvarilla = 0)

m1g − N1 = m1a1 (2)

N2 + m2g = m2a2 (3)

La suma de torques sobre la varilla respecto al extremo donde está T, es:

N1l – N2(2l) = Ivarilla αvarilla = 0 (4) (pues Ivarilla = 0)

Aparecieron dos incógnitas más, a1 y a2, por lo que se requiere una ecuación más (cinco incóg-

nitas requieren cinco ecuaciones). La quinta ecuación la da la relación entre las dos aceleraciones

(ecuación de ligadura), que se deduce del hecho de que, al girar la varilla, el cuerpo de masa m1

tiene la mitad del radio de giro del cuerpo de masa m2:

a2 = 2a1 (5)

Resolviendo el sistema de cinco ecuaciones: T = m1m2g/ (m1 + 4m2)

5.2-Tomemos tres ejes x, y, z como indica la Fig. 5.2, y llamemos l al lado del cubo. Si al cubo

no le faltase ningún pedazo, su centro de masas estaría en las coordenadas (l/2, l/2, l/2). A su vez,

el centro de masas del octante que falta estaría en las coordenadas (3l/4, 3l/4, 3l/4). Si llamamos

m a la masa del cubo completo, la del octante sería m/8 y la del resto del cubo sería 7m/8, cuyo

centro de masa estaría en las coordenadas (xc, yc, zc). Entonces, las coordenadas del cubo

completo podrían calcularse a partir de sus dos masas componentes. Por ejemplo, para la coor-

denada x:

l/2 = (1/m)[(m/8) 3l/4 + (7m/8) xc].

Y despejando: xc = (13/28) l

E igual para las otras dos coordenadas. El centro de masas del cubo sin el octante estaría corrido

del centro del cubo completo en l/28 por cada componente, hacia el origen del sistema x, y, z.

5.3-El hecho de que el lápiz no resbale al ponerlo a lo largo del plano significa que hay suficiente

fricción estática para no dejarlo resbalar en ningún caso. Si el lápiz rueda al ponerlo paralela-

mente a la recta generatriz del plano, es porque el centro de masas queda fuera de la base de

apoyo del lápiz en esa situación (Fig. 5.3Sa) y el peso hace un torque respecto al borde de la base

de apoyo que lo hace voltearse.

Page 49: problemas selectos

49

El problema consiste en hallar qué ángulo υ tiene que inclinarse el lápiz respecto a la recta gene-

ratriz para que la fuerza peso pase justamente por un punto R del borde del rectángulo de apoyo

(Fig. 5.3Sb) cuyo extremo es la base inferior del hexágono en contacto con el plano, de manera

que aún el torque del peso respecto a ese borde no pueda voltear el lápiz.

Los lados del hexágono son todos iguales, de longitud l, así como los lados del triángulo equi-

látero con vértice en el centro del hexágono (Fig. 5.3Sc).

La semibase del hexágono será PQ = l/2 (el mismo segmento PQ en las Fig. 5.3Sb y 5.3Sc).

Cuando el lápiz gira un ángulo υ respecto a la recta generatriz, también la base del hexágono rota

el mismo ángulo y la semibase PQ forma entonces el mismo ángulo υ con la recta ascendente

que corre a lo largo del plano inclinado, perpendicularmente a su recta generatriz (Fig. 5.3Sb).

La recta ascendente del plano inclinado fija la dirección sobre la cual se corre la vertical del peso

sobre la base de apoyo. La distancia QR (Fig. 5.3Sb) está dada por:

QR = PQ / cos υ = (l/2) / cos υ

de acuerdo con la trigonometría aplicada al triángulo PQR.

Por otra parte, la distancia del centro C del hexágono al punto Q de su base será (Fig. 5.3Sc):

d = l cos 30

l

l l

l l

l l

l Fig. 5.3Sc

d

θ Fig. 5.3Sd

d

s θ

P Q

Q

H

C

Q C

θ

Fig. 5.3Sa

mg P

Q

Dirección

de la recta

generatriz

Dirección

de la recta

ascendente

ascendente υ

υ

Fig. 5.3Sb

R

Borde

inferior

del plano

inclnado

Base de

apoyo del

lápiz

Base del

hexágno

Page 50: problemas selectos

50

Ésta será siempre la distancia del centro de masas del lápiz al centro del rectángulo que le sirve

de base de apoyo en cualquier caso.

Y la distancia QH que hay desde el punto H de la recta ascendente (donde la vertical que pasa

por el centro de masas corta al plano inclinado) hasta el punto Q (centro de la base rectangular

que sirve de apoyo al lápiz), para una inclinación θ del plano inclinado (Fig. 5.3Sd) será:

QH = d tan θ = l cos 30 tan θ

Mientras esta distancia QH sea menor o igual que la distancia QR, la vertical que pasa por el

centro de masas no se saldrá de la base de apoyo rectangular del lápiz, y no habrá un torque del

peso que lo haga voltear.

Entonces, la condición para no rodar es: QH ≤ QR

O: l cos 30 tan θ ≤ (l/2) / cos υ

Sustituyendo cos 30 = / 2 y arreglando: cos υ ≥ 1 / ( tan θ)

El ángulo mínimo según el cual puede orientarse el lápiz sin rodar será entonces:

υ = cos-1

[1 / ( tan θ)]

(siempre que el ángulo de inclinación del plano, θ, garantice que el lápiz no resbale cuando se

coloque perpendicularmente a la recta generatriz, como se dijo por dato).

5.4-La Fig. 5.4 muestra las maneras de colocar el cubo de manera que permanezca en equilibrio:

en ambos casos el centro de masas del cubo queda sobre la recta de simetría del sistema, por lo

que las fuerzas de contacto con los dos planos serán iguales y provocarán torques iguales y

opuestos al centro de masas sin provocar giros.

h2 = l / 2 + (l / 2) cot30o = (l / 2) ( + 1)

Resulta h2 > h1, por lo que el caso (a) es el de equilibrio estable, y el (b), el de equilibrio ines-

table.

30 30

60 60 60 60

45

La altura del centro de masas del cubo en

cada caso se podrá expresar en función de

la longitud de la arista del cubo, que lla-

maremos l. En el primer caso, Fig. 5.4a:

h1 = (l cos45o) cot30

o = l / 2

En el segundo caso, Fig. 5.3b:

(a) (b)

Fig. 5.4

Page 51: problemas selectos

51

5.5-Cuando un cuerpo descansa en equilibrio sobre una superficie horizontal, la fuerza normal de

contacto que experimenta pasa justamente por el centro de masas del cuerpo. Pero si la superficie

se acelera hacia la derecha, la normal se corre hacia la izquierda, como habremos notado cuando

estamos en el pasillo de un ómnibus, de frente a las ventanillas, y el ómnibus arranca: auto-

máticamente el pie que queda más atrás en el pasillo es sobre el que se recarga el cuerpo, y la

fuerza normal sobre ese pie es mayor que sobre el de alante.

ya sobre el saliente O. La fuerza de inercia Fi = ma (1), y la fuerza Fs del saliente, hacia ade-

lante, crean un torque que tiende a rotar el cubo en contra de las manecillas del reloj. La fuerza

de inercia queda aplicada al centro de masas del bloque.

La normal y el peso mg crean un torque recuperador a favor de las manecillas del reloj que se

opone a que el cubo se voltee. En el sistema no inercial ambos torques se contrarrestan mientras

el cubo no se voltee. Llamando l al lado del cubo tendremos, cuando el cubo está ya a punto de

voltearse: Fil/2 – mgl/2 = 0, por lo que Fi = mg. Comparando con (1): a = g. Ésta es la ace-

leración que debe adquirir como mínimo el bloque para que el cubo esté a punto de voltearse.

Ahora, la fuerza F que hay que aplicar al bloque de masa M debe arrastrar al bloque y al cubo,

por lo que debe valer: F = (m + M) g.

5.6-La velocidad del sistema varilla-bloque después del choque es la velocidad de su centro de

masas vc. Como en el sistema se compensan las fuerzas externas (pesos y normales de contacto),

el momento lineal del sistema se conserva y la velocidad del centro de masas no varía con el cho-

que. Por conservación del momento lineal:

Mv = 2Mvc.

De aquí: vc = v/2, que es la velocidad final del sistema.

Por otra parte, los torques externos también se anulan, por lo que se conserva el momento angu-

lar del sistema respecto a cualquier punto, o eje, de un sistema inercial: Lantes = Ldespués, y de esta

condición podremos calcular la velocidad de rotación, ω, después del choque. Tanto el piso sobre

el que se desliza el sistema varilla-bloque, como un sistema ligado al centro de masas del con-

junto, sirven como sistemas inerciales de referencia. Aquí tomaremos un eje de momentos fijo al

piso (en el próximo problema se escogerá un sistema ligado al centro de masas), que entre en el

mg

Fi

Fs

N

l

l

En un cubo, la normal se corre en dirección opuesta a la acele-

ración a que se le somete, y se corre más mientras mayor sea la

aceleración. Si la normal llega al borde posterior del cubo y aún

la aceleración aumenta más, ya la normal no podrá correrse más

atrás y el cubo se voltea. La Fig. 5.5S muestra la situación

extrema, observada desde el sistema no inercial que se mueve

con aceleración a (junto con el bloque), cuando la normal N está Fig. 5.5S

O

Page 52: problemas selectos

52

piso por el punto en que el centro de masas del conjunto se encuentre en el momento del impacto

del bloque y la varilla, punto que estará a una distancia l/4 del extremo de la varilla y del centro

de la misma (Fig. 5.6S).

parte:

I = Ivarilla + Ibloque = [Ml2/12 + M (l/4)

2] + M (l/4)

2 = (5/24)Ml

2

donde se tuvo en cuenta el teorema de los ejes paralelos para la varilla. Entonces, por conser-

vación del momento angular:

Mvl/4 = (5/24) Ml2ω

Y despejando: ω = (6/5) v/l

5.7-Las fuerzas sobre las dos varillas se compensan (el peso de cada una con la normal de con-

tacto), y no introducen torques externos. Por tanto, se conservan el momento lineal y el momen-

to angular del sistema de varillas.

La conservación del momento lineal nos lleva a plantear: mv = 2mvc, donde vc es la velocidad del

centro de masas del conjunto, que vale lo mismo antes que después del choque. Entonces: vc =

v/2, y representa la velocidad del conjunto después del choque.

Para calcular la velocidad de rotación debere-

mos tener en cuenta la conservación del mo-

mento angular del conjunto. La Fig. 5.7S mues-

tra la rotación de las varillas después de pegarse

en el choque: rotan alrededor del centro de

masa del conjunto, a favor de las manecillas del

reloj. Para los cálculos, tomemos el centro de

masas del conjunto como sistema inercial de re-

ferencia (todo el tiempo se mueve con veloci-

dad rectilínea v/2).

Fig. 5.7S

CM

Vista superior

v m

l

l/4

m

l

l/4

l/4

M

M

v

Fig. 5.6S

Vista superior

l/4

v/2 Centro masas

Centro varilla

Respecto a este punto fijo, solamente el bloquecito

tiene momento angular antes del choque, por lo que

Lantes = Mvl/4; y después del choque, todo el sistema

rota unido como un solo cuerpo, con momento angular

de espín, Lespín = Iω y sin momento angular orbital del

centro de masas, puesto que dicho centro se mueve

sobre la recta que pasa por el eje escogido (Lorb = 0).

El momento de inercia del sistema respecto al eje es-

cogido es la suma de los momentos de inercia de cada

parte:

ω

Page 53: problemas selectos

53

El momento de un sólido respecto a un punto es la suma vectorial de su momento angular orbital,

Lorb=mVr senθ y su momento angular de espín, Lspin= Iω; en nuestro caso se reduce a la suma

algebraica pues ambos apuntan perpendicularmente al plano del papel.

Antes del choque, respecto al centro de masa móvil del conjunto, ambas varillas tienen momento

angular orbital, y ninguna lo tiene de espín, pues ninguna rota; la rapidez de ambas respecto al

centro de masa es v/2, en direcciones opuestas, pero ambas con momento angular a favor de las

manecillas; el brazo de momento de cada una es rsenθ = l/4, por lo que las dos juntas tienen

momento angular orbital antes del choque:

Lorb = m(v/2) l/4 + m(v/2) l/4 = mvl/4.

Después del choque, las dos varillas juntas forman un solo cuerpo que rota alrededor de su centro

de masas común, por lo que su momento angular se expresará como Lespin = Iω, donde ω es la

velocidad angular del conjunto y la I representa el momento de inercia de las dos varillas

respecto al centro de masas común. Para cada varilla, este momento puede calcularse por el

teorema de los ejes paralelos, puesto que el eje de rotación está desplazado l/4 del eje central de

cada varilla: I1 = ml2/12 + m(l/4)

2 = (7/48)ml

2. Las dos varillas juntas aportan un momento de

inercia total de I = 2I1 = (7/24)ml2. Entonces, después del choque: Lespín = (7/24)mωl

2.

Y por conservación del momento angular: mvl/4 = (7/24)mωl2.

Despejando: ω = (6/7)v/l

La energía perdida es la diferencia de las energías cinéticas antes y después del choque:

ΔE = ½ mv2 – ½ Mvc’

2 – ½ Iω

2

donde M = 2m, I =(7/24)ml2 ) y vc = v/2. Sustituyendo:

ΔE = (1/7)mv2

5.8-La velocidad angular ω que adquiere el disco chiquito es la de rotación alrededor del eje ver-

tical fijo del disco grande cuando finalmente quede en reposo sobre el grande. Ésta será también

la velocidad angular final ω del disco grande.

La fuerza de fricción entre los discos es interna del sistema compuesto por dichos discos, por lo

que no varía el momento angular del sistema. La fuerza de gravedad sobre el disco pequeño

introduce un torque externo respecto al eje vertical central del disco grande, pero tal torque se

compensa con el que produce la fuerza de reacción que ejerce el eje sobre el disco, impidiendo

que se voltee el sistema. En resumen, la resultante de torques externos sobre el sistema de los dos

discos es nula, por lo que el momento angular vertical del sistema se conserva respecto al eje

central. Entonces, teniendo en cuenta el teorema de los ejes paralelos:

Page 54: problemas selectos

54

½ MR2Ω = ½ MR

2ω + (½ mr

2 + md

2) ω

Despejando: ω = Ω / [1 + (m/M) (r2/R

2 + 2d

2/R

2)]

5.9-En (a) la esfera tiene energía cinética de traslación pura: Ko = ½ mvo2. En (c) la esfera tiene

energías cinéticas de traslación y de rotación, y puesto que se mueve en rodadura pura se

cumplirá además que la velocidad con que avanza, v, y la de rotación, ω, se relacionan por la

ecuación v = ωR, donde R es el radio de la esfera. Así, la energía final es:

K = ½ mv2 + ½ Iω

2 = ½ mv

2 + ½ (2/5)mR

2(v/R)

2 = (7/10)mv

2 (1)

Y debemos calcular la fracción: (Ko – K)/Ko.

La fricción cinética que surge es fk = μkN = μkmg. De aquí: ma = μkmg, por lo que la aceleración

de frenado es a = μkg.

La propia fricción produce un torque alrededor del eje central de la esfera que produce la

aceleración angular que la hace rotar: η = fkR = Iα.

O sea: μkmgR = (2/5)mR2α, por lo que μkg = (2/5)αR y se tendrá: α = (5/2) μkg/R.

La aceleración a refleja un frenado en la traslación de la esfera (disminuye v) y la aceleración α

refleja un incremento de la velocidad angular de rotación (aumenta ω). En este proceso de

aumento de ω (a partir de cero) y disminución de v (a partir de vo) llegará un momento en que se

cumplirá v = ωR, y de ahí en adelante se producirá una rodadura pura (como en la Fig. 5.9-c). En

el tiempo t que dura este proceso la esfera alcanza una velocidad de traslación:

v = vo – at = vo – μkg t (2)

Y en ese mismo tiempo la esfera alcanza una velocidad de rotación:

ω = 0 + αt = (5/2) μkg t/R

De aquí: ωR = (5/2) μkg t

Pero al final de este tiempo t se establece la rodadura pura, por lo que v = ωR y quedará:

v = (5/2) μkg t. De aquí se obtiene que: μkg t = (2/5)v. Sustituyendo en (2):

v = vo – (2/5)v. Despejando: v = (5/7)vo .

Sustituyendo en (1):

K = (7/10) m (25/49) vo2 = (5/14)mvo

2

Finalmente: ΔK / Ko = [½mvo2 – (5/14)mvo

2] / ( ½ mvo

2) = 2/7

Page 55: problemas selectos

55

5.10-Sean m y R la masa y el radio del cilindro central. Los laterales, siendo de la misma

densidad, tendrán masas: ml = ρV = ρlπ4R2 = 4m. El momento de inercia del cilindro central será

½mR2, y el de cada cilindro lateral será ½ml (2R)

2 = 8mR

2. El momento de inercia total del cuer-

po respecto al eje central será Io = (33/2)mR2 y la masa total del cuerpo será M = 9m.

En el descenso por el plano inclinado se conserva la energía mecánica (es una rodadura pura),

por lo que:

Mgh = ½MV2 + ½Ioω

2, donde V = ωR.

Esta V es la velocidad del centro de masas del cuerpo en el borde final del plano inclinado, justo

cuando va a pasar al plano horizontal. Desarrollando:

(9m)gh = ½(9m)V2

+ ½(33/2)mR2(V/R)

2 = (51/4)mV

2

De aquí: V =

La velocidad angular en ese instante es: ω = V/R. Y el borde de los cilindros laterales que hace

contacto con el piso horizontal en ese instante gira con una velocidad de traslación V’= 2ωR =

2V, por lo que va hacia atrás el doble de rápido que lo que el centro de masas avanza en la

horizontal. Esto provoca que haya deslizamiento al pasar al piso horizontal, con una fricción

cinética dirigida hacia adelante (la rueda desliza hacia atrás). Esta fricción hacia adelante hará

aumentar la velocidad de traslación del cuerpo, a la vez que produce un torque opuesto a la

rotación de la rueda que hará disminuir su velocidad angular. De este modo, llegará un momento

en que el valor ωf (2R) = vf, y de ahí en adelante el cuerpo seguirá en rodadura pura.

Así la aceleración de impulsión en la traslación será a = f /M = f / (9m) y la impulsión durará un

tiempo tal que: vf = V + at = V + f t /(9m) , por lo que t = (vf − V)9m/f.

Durante ese tiempo actúa un torque de frenado que cambia la velocidad angular de ω = V/R a

ωf = vf /2R, con una aceleración de frenado: α = η/Io = fR/ [(33/2)mR2].

Se cumple: ωf = ω – αt

Sustituyendo todos los valores: vf /(2R) =V /R – {fR/ [(33/2)mR2]}(vf − V)9m/f

Simplificando: vf = 2V − (36/33)vf + (36/33)V

De aquí: vf = (34/23)V = (34/23) ≈ 1.2

5.11-Si no hay fricción no existirán fuerzas horizontales sobre el sistema, por lo que el centro de

masas del sistema caerá verticalmente. La suma de las tres energías cinéticas de las tres esferitas

será igual a la energía potencial inicial del sistema:

Page 56: problemas selectos

56

½ mv12 + ½ mv2

2 + ½ mv3

2 =mgl/2 + mgl (1)

donde consideramos “1” la esfera inferior y “2”, la del centro. En el instante en que la esfera

superior llega al piso, su velocidad v3 es vertical hacia abajo, al igual que v2 de la esfera central,

pero la velocidad de la esfera “1” es cero: v1 = 0, pues si tuviera aún velocidad horizontal hacia

la izquierda, arrastraría al sistema rígido hacia la izquierda, lo cual es imposible. Entonces,

respecto a la esfera “1”, las velocidades v2 y v3 son tales que v3 = 2v2, pues en el instante final

las otras dos esferas se ven describiendo circunferencias hacia abajo en torno a “1”. Y esta

relación de velocidades es la misma respecto al piso, pues la bola “1” está en reposo en el

instante final. Entonces, la ecuación (1) queda: 0 + ½ m(v3/2)2 + ½ mv3

2 =mgl/2 + mgl

Y despejando: v3 = 2

Si la esfera inferior estuviera fija al gozne, el sistema caería rotando todo el tiempo alrededor de

esa esfera inferior, y el resultado sería el mismo de antes.

Page 57: problemas selectos

57

6-Hidrostática. Problemas.

6.2-Se vierte líquido de densidad ρ en un recipiente cilíndrico vertical, el área de cuyo fondo es

S. ¿En cuánto varía el nivel del líquido en el recipiente si en él se introduce un cuerpo de masa m

de forma arbitraria, con heterogeneidades y huecos en su interior, que no se hunde?

6.4-Una pecera tiene agua (densidad: 1.00 g/cm3) hasta una altura de 50.0 cm. ¿Qué fuerza ejerce

el agua sobre una pared de la pecera de 80.0cm de ancho? ¿Qué fuerza neta sufre la pared hacia

afuera de la pecera? (Presión atmosférica: 1.01×105 Pa)

6.5-En una bolsita ligera de hilo hay 100 monedas de dimensiones idénticas, pero unas de oro y

otras de plata. El peso de la bolsita con las monedas es de 127.39 g en el aire, pero sumergida en

agua tiene un peso aparente de 119.54 g. Cuántas monedas hay de cada una, sabiendo que la

densidad del oro es 19.3 g/cm3 y la de la plata, 10.5 g/cm

3.

h S1

S2

Fig.6.3

6.3-En el fondo de un recipiente cilíndrico chato de área

S1 y altura h abrieron un agujero de área S2 y colocaron

en él un tubo de plástico. La masa del recipiente con el

tubo es m. El recipiente se para sobre una placa de goma

dura, con su fondo hacia arriba, como un embudo inver-

tido (Fig. 6.3). Por el tubo vierten agua. ¿Hasta qué altu-

ra puede subir el agua por el tubo sin que se salga? No

tenga en cuenta la tensión superficial.

S

h

Fig.6.1

6.1-Un cilindro de finas paredes lleno de gas,

de masa m, altura h y área de la base S, flota en

agua como muestra la Fig. 6.1. Como resultado

de la pérdida de hermeticidad en la parte infe-

rior del cilindro, la profundidad de inmersión

aumenta un Δh. Determine la presión inicial p1

del gas en el cilindro. La presión atmosférica es

igual a po y la temperatura no cambia.

Page 58: problemas selectos

58

6-Hidrostática. Soluciones.

6.1-Sea p2 la presión del gas en el interior del cilindro cuando éste se encuentra en el estado final

de equilibrio. El cilindro va a estar sometido a fuerzas de presiones por su interior y por su

exterior, más la fuerza de gravedad. Entre las fuerzas por su exterior están las que lateralmente

ejercen la atmósfera y el agua, pero éstas se compensan por aparecer en puntos opuestos del

cilindro, al mismo nivel. Por debajo, el agua ejerce fuerza de presión hacia arriba, pero se

compensa con la que hace hacia abajo el agua en el interior del cilindro, a la misma profundidad

h respecto a la superficie libre del agua. Las fuerzas de presión por el interior, sobre las paredes,

también se compensan por actuar sobre puntos opuestos. Quedan solamente sin compensarse la

fuerza de presión que hace la atmósfera hacia abajo por la cara superior y la fuerza de presión

que hace el gas interior sobre la misma cara, hacia arriba. Pero la diferencia entre estas dos

fuerzas debe compensarse con la fuerza de gravedad sobre el cilindro. Luego:

p2S = poS + mg.

De aquí: p2 = po + mg/S (1)

Y por la ley de Boyle: p2 (h – Δh) S = p1S h (2)

De (1) y (2): p1 = (po + mg/S) (1 – Δh / h)

6.2-Si el cuerpo introducido flota, la fuerza de empuje será igual al peso del cuerpo introducido.

Por Arquímedes, esta fuerza es igual al peso del volumen de fluido desplazado por el cuerpo. Si

Δh es la altura del líquido desplazado:

mg = g (ρSΔh)

Luego: Δh = m / (ρS)

6.3-En la medida que se echa agua y sube el nivel por el tubo estrecho aumenta la presión hacia

arriba que ejerce el agua sobre la superficie horizontal del recipiente, de área S1 – S2. La fuerza

de presión hacia arriba va igualmente aumentando, hasta que levantará el recipiente cuando

sobrepase la fuerza de gravedad sobre el cilindro. La situación límite ocurre cuando:

ρg (H – h)(S1 – S2) = mg

donde H es la altura total que asciende el liquido sobre la superficie de apoyo del recipiente. De

aquí:

H = h + m/[ρ(S1 – S2)]

Page 59: problemas selectos

59

6.4-a)La fuerza que ejerce el agua está dada por la presión del agua sobre la pared. Esta presión

crece linealmente con la profundidad, por lo que la presión media sobre el área de la pared será

el valor de la presión a la mitad de la altura del agua, h = 25.0 cm = 0.250 m:

p = ρgh + po = 1.00×103 ×9.80×0.250 + 1.01×10

5 = 1.03×10

5 Pa

La fuerza es entonces: F = pA = 1.03×105 ×0.500×0.800 = 4.14×10

4 N

b)Del lado de afuera, la presión atmosférica empuja hacia adentro al vidrio, por lo que la fuerza

neta del agua hacia afuera es:

Fneta

= ρghA = 1.00×103 ×9.80×0.250×0.500×0.800 = 980 N

Una fuerza igual y opuesta estarán haciendo los soportes de la pared para mantenerla en equi-

librio.

6.5- Sean: WA = 127.39 gf el peso total de las monedas en el aire, WW = 119.54 gf su peso total

en el agua, wo el peso de cada moneda de oro, wp el peso de cada moneda de plata, ρo, ρp, sus

respectivas densidades y ρ, la densidad del agua. Las cantidades de cada tipo de moneda serán No

y Np. Entonces:

WA = Nowo + Npwp (1)

y el peso aparente será el peso real menos la fuerza boyante :

WW = (Nowo + Npwp) – (Nowoρ/ρo + Npwp ρ/ρp) (2)

De (1): Nowo = WA − Npwp. Sustituyendo en (2) y arreglando :

WW + WA (ρ/ρo – 1) = Npwp (ρ/ρo − ρ/ρp)

Despejando y evaluando: Npwp = 28.77 gf. Y sustituyendo en (1): Nowo = 98.62 gf. Dividiendo

miembo a miembro estas dos igualdades:

Npwp / Nowo = 0.2917 (3)

Pero como las monedas son de volúmenes iguales: Npwp / Nowo = Npρp / Noρo = 0.5440 Np / No.

Sustituyendo en (3) y arreglando: Np / Na = 0.5362. Y por regla de las proporciones:

(Np + No)/ No = (0.5362 + 1) / 1, donde Np + No = 100. O sea:

100/No = 1.5362. De aquí No = 65, y en consecuencia: Np = 100 – 65 = 35.

Page 60: problemas selectos

60

7A-Termodinámica I. Problemas.

7.1-Dos cuerpos con masas m1 y m2 se mueven al encuentro uno del otro, en direcciones

mutuamente perpendiculares, con velocidades v1 y v2. Como resultado del choque los cuerpos se

pegan. Determine la cantidad de calor Q liberada como resultado del choque.

7.2-Dos esferas iguales están a la misma temperatura. Una de las esferas se encuentra sobre una

superficie horizontal aislante y la otra pende de un hilo aíslante. A ambas se les comunican

iguales cantidades de calor, tan rápidamente que no hay pérdidas de calor al ambiente. ¿Serán

exactamente iguales, o serán diferentes las temperaturas de las esferas después de calentarlas?

7.3-En un recipiente térmicamente aislado se encuentran dos líquidos con temperaturas iniciales

T1 y T2, y calores específicos c1 y c2, separados por un tabique no conductor. El tabique es

retirado y después del establecimiento del equilibrio térmico la diferencia entre la temperatura

inicial del líquido 1 y la temperatura que se establece en el recipiente es la mitad de la diferencia

entre las temperaturas iniciales de los líquidos. Encuentre la razón de las masas de los líquidos,

m1 / m2 .

7.4-En un calorímetro aislante se vierten m1 = 0.50 kg de agua a temperatura T1 =+15ºC. En el

agua se coloca un pedazo de hielo de masa m2 = 0.50 kg a temperatura T2 =−10ºC. Encuentre la

temperatura de la mezcla cuando se alcanza el equilibrio térmico. Calores específicos del agua y

del hielo: c1 = 4.2×103 J/kg.K, c2 = 2.1 ×10

3 J/kg.K. Calor de fusión del hielo: L = 3.3×10

5 J/kg.

7.5-Un calorímetro de cobre de masa m1, con una masa de agua m2, está a la temperatura T1. En

el calorímetro se echa hielo de masa m3 y temperatura T2. (a) Determine la temperatura y las

masas del agua y del hielo después de establecerse el equilibrio térmico, para valores cuales-

quiera de m1, m2, m3, T1 y T2. Escriba la ecuación del balance térmico del sistema. (b) Determine

la temperatura y las masas de agua y hielo en estado de equilibrio térmico si m1 = 1.0 kg, m2 =

1.0 kg, m3 = 2.0 kg, T1 = 10oC y T2 = −20

oC. Despreciar las pérdidas de calor al ambiente.

Considere la presión atmosférica como la normal. Calores específicos del cobre, del agua y del

hielo: c1 = 0.39 kJ/kg o

C, c2 = 4.2 kJ/kg o

C, c3 = 2.1 kJ/kg o

C. Calor de fusión del hielo: L =

330 kJ/kg.

Page 61: problemas selectos

61

7.6-En un recipiente hay un volumen V1 = 3.0×10-4

m3 de tolueno a una temperatura t1 = 0ºC y

en otro, un volumen V2 = 1.1×10-4

m3 de tolueno también, a una temperatura t2 = 100ºC. ¿Qué

volumen V ocupa el tolueno después de mezclarlos, si su coeficiente de dilatación volumétrica es

α = 0.0001 K-1

? Despreciar las pérdidas de calor.

7.7-Dos cilindros verticales tienen secciones transversales diferentes y están conectados por sus

bases mediante un fino tubo. Las partes superiores de cada cilindro están cerradas por sendos

pistones, de masas m1 = 1.0 kg y m2 = 2.0 kg. Por la parte superior de los pistones hay vacío

total, y por la parte inferior hay un gas ideal a temperatura constante. Los pistones están situados

a alturas iguales, ho = 0.20 m. ¿Cuál será la diferencia de alturas de los pistones si se aumenta la

masa del primer pistón hasta la masa del segundo?

7.9- Un intercambiador térmico de longitud l consiste en un tubo de sección transversal de área

2S por dentro del cuál pasa otro tubo de sección transversal de área S (Fig.7.9). El tubo interior

es de paredes finas y conductoras. Toda la construcción está aislada térmicamente del medio

exterior. Por ambos tubos circula líquido de densidad ρ y calor específico c, y en ambos tubos la

T1i

T2i T2f

T1f

v

v

v

S 2S

Tubo 1

l ( l >> )

Fig.7.9

Tubo 2

Fig.7.8

7.8-Un fino tubo en forma de U, cerrado por uno de sus

extremos, consiste en tres tramos de longitud l = 250 mm

cada uno, doblados en ángulos rectos. Las partes verticales

del tubo están llenas de mercurio hasta la mitad (Fig.7.8).

Lentamente se calienta el gas de la parte cerrada del tubo, de

tal modo que expulsa del tubo todo el mercurio. Determine

qué trabajo W realiza el gas en el tubo para expulsar total-

mente el mercurio. La presión atmosférica es po=1.0×105 Pa,

la densidad del mercurio es ρ = 13.6×103 kg/m

3 y la sección

transversal del tubo es S = 1.0 cm2.

Page 62: problemas selectos

62

rapidez con que circula el líquido es v, pero moviéndose en direcciones opuestas. Las tempe-

raturas del líquido en las entradas de cada tubo al intercambiador son T1i (ambiente) y T2i

(caliente). Determine las temperaturas T1f y T2f del líquido en las salidas del intercambiador.

Considere que la cantidad de calor transferido en cada punto, por unidad de tiempo a través de la

unidad de área, es proporcional a la diferencia de temperaturas, con un coeficiente de propor-

cionalidad conocido, k. La conducción térmica del propio líquido a lo largo del flujo es des-

preciable.

7.10-La Fig.7.10 muestra un calorímetro de flujo continuo. Consiste en un tubo de vidrio sobre

dos soportes aislantes, Z, en cuyo interior hay una resistencia eléctrica de calentamiento, R, que

se extiende a lo largo del tubo, acoplada a un circuito externo a través de dos tapones aislantes en

los extremos del tubo. Se introducen dos termómetros, T1 y T2, cerca de la entrada y de la salida

del flujo de líquido en el tubo. Por A entra el líquido cuyo calor específico se desea medir, y sale

por B hacia un tanque M Se usa un mecanismo (“frasco de Mariotte”) que asegura un flujo

constante de líquido todo el tiempo que dure la medición.

En un experimento de medición se trabaja con un líquido cuya densidad es 0.88 g/cm3. Se

establece el flujo de líquido por el tubo y se conecta la resistencia eléctrica. Durante un tiempo

cambian las temperaturas de los termómetros, hasta que a los pocos minutos se estabilizan,

registrando el termómetro T1 una temperatura estable de 40.0oC y el T2, una temperatura de

32.5oC. En ese momento se introduce un recipiente H a la salida del tubo para recoger líquido

por 60.0 s, durante los cuales salen 475 cm3 de líquido. La resistencia eléctrica estuvo conectada

durante ese tiempo a una fuente eléctrica alterna de 120 V (voltaje eficaz) que proveía una

corriente de 2.20 A (corriente eficaz).

a)Calcule con estos datos el calor específico del líquido.

Z Z

R

T1 T1

A

B

M

H

Fig.7.10

Page 63: problemas selectos

63

b)Explique cómo afectarían al resultado la absorciones de calor por parte de todos los com-

ponentes del calorímetro.

c)Explique bajo qué rango de valores debe establecer las mediciones para minimizar las pérdidas

de calor al ambiente

7B-Termodinámica I. Soluciones.

7.1-Al pegarse los cuerpos (choque plástico) la energía cinética perdida se convierte en calor.

Además, el momento lineal del sistema compuesto por los dos cuerpos se conserva en el choque.

Sea un sistema de referencia fijo al plano por el que se mueven los dos cuerpos. Tomemos el eje

x en la dirección en que se mueve el cuerpo de masa m1, y el eje y en la dirección en que se

mueve el cuerpo de masa m2. Llamando ux y uy a las componentes de las velocidades de los

cuerpos unidos después del choque:

m1v1 = (m1 +m2)ux y m2v2 = (m1 + m2)uy

De estas ecuaciones pueden calcularse ux y uy: ux = m1v1/(m1+m2) y uy = m2v2/(m1+m2)

T1

T2

Tfusión

l2

l1

7.11-Un largo tubo vertical de ladrillos está lleno de hierro

colado (Fig. 7.11). El extremo inferior del tubo se mantiene a

una temperatura T1>Tfusión, y el extremo superior a T2<Tfusión

(donde Tfusión es la temperatura de fusión del hierro colado). La

conductividad del hierro colado líquido es k veces mayor que la

del sólido. La propagación del calor por convección en el hierro

colado líquido es despreciable frente al calor propagado por

conducción. Determine que fracción del tubo está ocupada por

el metal fundido.

Fig.7.11

Page 64: problemas selectos

64

El calor liberado es:

Q = (energía mecánica antes del choque) – (energía mecánica después del choque)

O sea: Q = ½ m1v12 + ½ m2v2

2 – ½ (m1+m2) (ux

2 + uy

2)

Sustituyendo ux y uy y desarrollando:

Q = ½(m1m2) (v12 + v2

2) / (m1 + m2)

7.2-Al ganar calor las esferas se dilatan y aumentan su diámetro. El centro de masas de cada

esfera sigue estando en su centro, pero éste cambia de posición de manera diferente en cada

esfera, pues una está sujeta por debajo (en contacto con la superficie horizontal) y la otra está

sujeta por arriba (la que cuelga del hilo). Para la que descansa en la superficie, sube su centro de

masas y gana energía potencial gravitatoria; significa que parte del calor ganado no incrementa la

energía cinética de sus moléculas, sino se acumula como energía potencial gravitatoria. Y para la

que cuelga, desciende su centro de masas, por lo que pierde energía potencial gravitatoria que

reaparece como energía cinética de sus moléculas. En definitiva, la que cuelga gana más energía

cinética molecular que la que está sobre la superficie, por lo que su temperatura asciende más.

7.3-Sean T la temperatura final de la mezcla, T1 la temperatura inicial del líquido 1, y T2 la

temperatura inicial del líquido 2. Supondremos T1>T2. Entonces, el calor perdido por el líquido

más caliente es igual al ganado por el líquido más frío:

m1c1 (T1 – T) = m2c2 (T – T2)

De aquí: m1/m2 = [c2 (T – T2)] / [c1 (T1 – T)]

Pero por dato: (T1 – T) = ½ (T1 – T2).

Y de aquí: (T1 – T) = (T – T2)

Entonces: m1/m2 = c2/c1

7.4-Las masas de agua, m1, y hielo, m2, son iguales; el agua tiene doble calor específico que el

hielo, y la diferencia de temperaturas con el 0oC es mayor para el agua. Significa que para llegar

a cero grados centígrados el agua desprenderá más calor que el que necesita absorber el hielo

para llegar al cero. En definitiva, el exceso de calor que puede entregar el agua alcanza para

fundir hielo. Lo que tendremos que averiguar es si se funde sólo una parte (quedando el sistema a

cero grados), o si se funde todo (quedando el sistema justamente a cero grados), o si se funde

Page 65: problemas selectos

65

todo y aún pudiera subir más la temperatura de la mezcla (quedando el sistema por arriba de cero

grados).

El agua, al enfriarse hasta 0oC, desprendería: Q1 = m1c1(0 – T1) = -3.15×10

4 J

El hielo, al calentarse hasta 0oC, absorbería: Q2= m2c2(0 – T2) = 1.05×10

4 J

La diferencia es: ΔQ=│Q1│− Q2 = 2.10×104J, que alcanza para fundir una masa de hielo:

Δm = ΔQ/L = 0.064 kg,

que es menor que la masa existente de hielo. En resumen, el sistema se equilibra a los 0oC, con

un aumento de la fase líquida de 0.064 kg.

7.5-a)Como hay que resolver el problema de manera general, habrá que considerar tres posi-

bilidades: i) cuando toda el agua quede líquida, sobre 0oC, ii) cuando toda el agua quede sólida,

por debajo de 0oC, iii) cuando quede parte sólida y parte líquida, a 0

oC.

En el primer caso, la temperatura final T será T>0, y la ecuación de balance térmico será:

m3c3(0 –T2) + m3L + m3c2(T – 0) = (m1c1 + m2c2)(T1 – T), y de aquí se despeja T.

En el segundo caso, la temperatura final T será T<0, y la ecuación de balance térmico será:

m3c3(T –T2) = m1c1(T1 – T) + m2c2(T1 – 0) + m2L + m2c3(0 – T), y de aquí se despeja T.

En el tercer caso, si llamamos Δm a la masa de agua o hielo que cambia de estado y asumimos

que es un poco de hielo el que se transforma en agua líquida:

m3c3(0 – T2) + LΔm = m1c1(T1 – 0) + m2c2(T1 – 0).

En este último caso la temperatura final es 0oC y la incógnita a despejar es la masa Δm. Si Δm da

positiva y menor que m3, es hielo el que se transforma en líquido, como asumimos al plantear la

ecuación. Si Δm da negativa y |Δm| < m2, es agua líquida la que se transforma en hielo. Si Δm es

positiva y mayor que m3, todo termina en fase líquida y debe calcularse T por el primer caso (i).

Si Δm da negativa y |Δm| >m2, todo termina en fase sólida y la temperatura T debe calcularse por

el segundo caso (ii).

b)Los datos numéricos de este problema conducen al caso (iii) y se obtiene Δm = -0.12 kg, lo

que significa que 0.12 kg de agua líquida se transforman en hielo y el sistema queda en equilibrio

térmico a 0oC.

Page 66: problemas selectos

66

7.6-Sean V01 y V02 los volúmenes de tolueno a 0oC. Entonces, sus volúmenes a las temperaturas

T1 y T2 son V1= V01(1+αT1) y V2= V02(1+αT2). Las masas de uno y otro volumen serán m1=ρoV01

y m2=ρoV02, donde ρo es la densidad del tolueno a cero grados centígrados. Al unirse las dos

muestras líquidas se establece un intercambio de calor hasta alcanzar una temperatura de

equilibrio T tal que: m1c1 (T – T1) = m2c2 (T2 – T), donde c1 = c2 = ctolueno. Despejando T, y

escribiendo las masas en función de los volúmenes a cero grados, se obtiene:

T = (V01T1 – V02T2) / (V01 + V02)

El volumen de la mezcla a la temperatura final será V = (V01 + V02) (1 + αT).

Sustituyendo T y reordenando se obtiene:

V = V01(1+αT1) + V02(1+αT2) = V1 +V2 = 4.1×10-4

m3

Significa que lo que se contrae una masa compensa justamente lo que se expande la otra y la

final el volumen sigue siendo el mismo del inicio.

7.7-Al inicio, dado el equilibrio existente, las presiones en ambos cilindros son iguales, y sufí-

cientes para sostener el peso de cada pistón. Cuando se coloca más masa sobre el pistón menor

presionará más sobre el gas de ese cilindro y se romperá el equilibrio: fluirá gas del cilindro de

mayor presión al de menor presión. En este último, la presión no puede aumentar porque el peso

del pistón sigue siendo el mismo. En consecuencia, fluirá gas hasta que el pistón menor choque

con el borde inferior del cilindro donde surgirá la fuerza normal de contacto necesaria para

detener el pistón. Todo el gas del primer cilindro pasará al segundo sin variar la presión que

había inicialmente, ni la temperatura de acuerdo con los datos. Por tanto el volumen total sigue

siendo el mismo al final:

S1ho + S2ho = S2h

donde h es la altura final del segundo pistón. Luego: h = ho( 1 + S1/S2)

El cociente S1/S2 puede calcularse a partir de la condición inicial de equilibrio de presiones:

m1g/S1 = m2g/S2, por lo que: S1/S2 = m1/m2.

Entonces: h = ho( 1 + m1/m2) = 0.30 m

7.8-El trabajo total que realiza el gas puede descomponerse en la suma del trabajo que realiza

contra la presión atmosférica, más el trabajo contra la fuerza de gravedad al elevar el centro de

masas del sistema líquido desde una altura inicial hasta la altura l en que se encuentra el extremo

abierto por donde se derrama el líquido.

Page 67: problemas selectos

67

El trabajo de expansión contra la atmósfera es: W1 = poS(2l + l/2) = (5/2)poSl

El centro de masa del líquido está inicialmente a una altura h sobre la parte inferior dada por:

h = (1/M) [(M/4)(l/4) + (M/4)(l/4) + (M/2)(0)] = l / 8

donde M representa la masa total del mercurio: M = ρ(2Sl).

La altura final a la que la presión del gas levanta todo el líquido es l, por lo que el incremento de

altura será: Δh = (7/8) l.

El trabajo realizado contra la gravedad es igual al aumento de la energía potencial gravitatoria de

la masa liquida: W2 = ΔU = MgΔh = (7/4) ρgSl2.

El trabajo total es entonces: W = W1 + W2 = (5/2) poSl + (7/4) ρgSl2 = 7.7 J.

7.9- Los flujos de líquido avanzan por secciones de áreas iguales, S para el tubo 2 y (2S – S) = S

para el tubo 1.

Esto significa que en cada punto a lo largo de los tubos hay la misma diferencia de temperaturas

y se producen iguales intercambios de calor. La distribución de temperaturas será como muestra

la Fig. 7.9S. En cada punto del intercambiador, la diferencia de temperaturas entre el líquido frío

y el caliente será:

ΔT= T2i −T1f = T2f −T1i (1)

El calor cedido lo largo del tubo 2 en un tiempo t será:

Q2 = kAintercambio t (T2i −T1f) donde Aintercambio =2πrl, con r = (S/π)1/2

, por lo que:

Aintercambio = 2πl(S/π)1/2

= 2l(S π)1/2

, y: Q2 = k2l (S π)1/2

t (T2i −T1f) (2)

T2i

T2f

T1f

T1i

x

T

ΔT= T2i −T1f

Como la velocidad de los líquidos es la

misma, y sus áreas de avance son las

mismas, sus gastos de masa son iguales.

Por cada sección del intercambiador

avanzan flujos líquidos iguales y opues-

tos, por lo que al intercambiar calor, la

misma variación de disminución de tem-

peratura que sufre el líquido en el tubo 2

la experimenta como un aumento el

líquido en el tubo 1 (en cada punto, o

sección del sistema de tubos).

Fig.7.9S

Page 68: problemas selectos

68

Por otra parte, el calor ganado por el líquido del tubo 1 se puede expresar:

Q1 = mcΔT = ρVc (T1f −T1i), donde V = Svt es el volumen de líquido que pasa por el tubo en el

tiempo t. Así:

Q1 = ρSvtc (T1f −T1i) (3)

Pero (2) y (3) representan la misma cantidad de calor:

k2l (S π)1/2

t (T2i −T1f) = ρSvtc (T1f −T1i)

Despejando y hacienda arreglos: T1f = T2i − (T2i −T1i)[ 1 + 2(π/S)1/2

lk/(ρvc)]-1

Sustituyendo en (1) y despejando: T2f = T1i + (T2i −T1i)[ 1 + 2(π/S)1/2

lk/(ρvc)]-1

7.10-(a) La masa que entra por segundo, Δm/Δt, recorre el tubo alrededor de la resistencia y se va

calentando. El calor generado por la resistencia en cada segundo, ΔQ/Δt, equivale a su potencia

eléctrica, P = IeVe (donde Ie y Ve son los valores eficaces de corriente y voltaje para la corriente

alterna).

Este calor al inicio calienta a la propia resistencia, al líquido que fluye y al líquido termométrico.

Para los tapones y el vidrio el calentamiento es pequeño pues son muy buenos aislantes y demo-

ran mucho en calentarse. Cuando los termómetros no varían más y se estabiliza la diferencia de

temperaturas entre los extremos de la resistencia, en cada punto del tubo de flujo ya no hay

diferencia de temperaturas entre el líquido y la pared interior del tubo, por lo que el flujo de calor

radial al exterior es mínimo y todo el calor generado en la resistencia se consume en calentar el

líquido de un extremo al otro. Si pasa una masa Δm desde un extremo al otro de la resistencia,

en un tiempo Δt, sufrirá un aumento de temperatura ΔT que se registra con los termómetros ya

estabilizados, y el calor absorbido será:

ΔQ = (Δm)cΔT.

Por unidad de tiempo será: ΔQ/Δt = (Δm/Δt) cΔT.

Sustituyendo por la potencia eléctrica: IeVe = (Δm/Δt) cΔT.

Pero Δm = ρΔV, por lo que: IeVe = (ρΔV/Δt) cΔT.

Despejando: c = IeVe / [(ρΔV/Δt) ΔT]

donde: Ie = 2.20 A, Ve = 120 V, ρ = 0.88×103 kg/m

3, ΔV = 475×10

-6 m

3, Δt = 60 s y

ΔT =47.0 – 32.5 = 15.5oC. Sustituyendo valores: c = 2.4 kJ/kg.

oC

Page 69: problemas selectos

69

b)Cuando se estabiliza la diferencia de temperaturas entre los termómetros, los componentes del

calorímetro prácticamente no absorben del calor generado en un breve intervalo de tiempo, como

es un minuto (60.0 s). El calor que inicialmente absorben, durante lo cual la diferencia de tem-

peraturas entre los termómetros crece, no entra en los cálculos.

c)Para que la aproximación anterior en (b) sea los más aceptable posible será necesario no

establecer diferencias de temperaturas muy altas entre el tubo y el medio ambiente. Con unos

pocos grados de diferencia de temperaturas casi no se transporta calor del tubo al ambiente (ni

por conducción, ni por convección, ni por radiación).

Pueden mejorarse los resultados haciendo las mediciones para varios voltajes durante el mismo

tiempo y con el mismo flujo, haciendo un gráfico de P contra ΔT, y determinando la pendiente de

la mejor recta que surge, que sería igual a (ρΔV/Δt)c, de donde se podría calcular c.

7.11-La corriente de calor hacia arriba por el hierro es la misma en todas partes del tubo en el

estado estacionario. Esta corriente de calor satisface la ley de conductividad térmica:

ΔQ/(SΔT) = KΔT/Δl

Entonces: Kliq (T1 – Tfus) /l1 = Ksol (Tfus – T2) /l2

Por datos: Kliq = kKsol, por lo que k (T1 – Tfus) /l1 = (Tfus – T2) /l2

Despejando: l1 / l2 = k (T1 – Tfus) / (Tfus – T2)

Usando las propiedades de las proporciones:

11/ (l1 + l2) = k (T1 – Tfus) / [k (T1 – Tfus) + (Tfus – T2)]

que da la fracción solicitada.

Page 70: problemas selectos

70

8A-Física Molecular. Problemas.

8.1-Las paredes de un recipiente, en el cual hay gas a temperatura absoluta T, poseen una tempe-

ratura Tp. ¿En qué caso la presión del gas sobre las paredes del recipiente es mayor: cuando las

paredes del recipiente están más frías que el gas ( Tp< T), o cuando están más calientes ( Tp> T)?

8.2-Un recipiente de capacidad V = 30 litros está dividido en tres partes iguales por delgados

tabiques semipermeables (Fig. 8.2). En la parte izquierda se introdujo mH = 30 g de hidrógeno,

en el medio, mO = 160 g de oxígeno y en la derecha, mN = 70 g de nitrógeno. A través del

tabique izquierdo solamente puede difundirse el hidrógeno, y a través del derecho, el hidrógeno y

el nitrógeno. ¿Qué presión habrá en cada una de las tres partes después que se establezca el

equilibrio, si el sistema se mantiene a una temperatura constante de 300 K?

8.3-En un cilindro vertical con un área S de sección transversal y bajo un pistón de masa M se

encuentra un gas ideal monoatómico. En cierto instante, bajo el pistón se enciende un calentador

eléctrico que da al gas una cantidad de calor q por unidad de tiempo. Determine la velocidad v

del movimiento del pistón bajo la condición de que la presión del gas bajo el pistón se mantiene

constante y que el gas se expande contra la presión atmosférica, po. El gas bajo el pistón está

térmicamente aislado.

8.4-Supongamos que a un planeta de masa M y radio ro lo rodea una atmósfera de densidad

constante, compuesta por un gas de masa molar μ. Determine la temperatura T de la atmósfera

sobre la superficie del planeta si el espesor de la atmósfera es h (h<< ro).

8.5-En tiempo frío, desde la boca sale “vapor” con el aire respirado. Si se entreabre la puerta de

una cabaña caliente en un día helado, entonces entra “vapor” en la habitación. Explique estos

fenómenos.

Fig.8.2

H2 O2 N2

Page 71: problemas selectos

71

8.6-Es conocido que si al agua común se la sala, su temperatura de ebullición aumenta. Deter-

mine cómo cambia en esta situación la densidad de los vapores saturados en el agua salada en la

temperatura de ebullición.

8.7-En cierto planeta la presión de vapor saturado del agua es igual a po = 760 mm Hg. Deter-

mine la densidad ρ del vapor de agua.

8.8-Para muchas sustancias existe un valor de temperatura TT y de presión pT tales que bajo esas

condiciones coexisten en equilibrio las tres fases (sólido, líquido, gas): el llamado punto triple de

la sustancia. Por ejemplo, para el agua: TT = +0.0075oC y pT = 4.58 mm Hg. El calor de

vaporización del agua en el punto triple es Lv = 2.48×103kJ/kg y el calor de fusión del hielo es Lf

= 0.34×103 kJ/kg. Encuentre el calor de sublimación del agua, Ls, en el punto triple.

8.9-Demostrar que en una atmósfera isoterma la fuerza ascensional de un aerostato de envoltura

muy elástica no cambia con la altura a que se eleva el aerostato bajo cualquier ley de variación

de la presión con la altura.

8.10-Se hace un alto vacío en un recipiente térmicamente aislado. La atmósfera que rodea al reci-

piente está a presión po y a temperatura To, y puede ser considerada como un gas ideal mono-

atómico. En cierto momento se abre una llave del recipiente y se produce su llenado con gas at-

mosférico. Haga una valoración aproximada de la variación de temperatura del gas en el reci-

piente después de llenado.

h2

h1

c2

c1

8.11-Es sabido que la presión de vapor saturado sobre

una solución acuosa de azúcar es menor que la del

agua pura, para la cual el valor es psat. La diferencia

entre ambos valores es Δp = 0.05psatc, donde c es

[(moles de azúcar) / (moles de solución)] (la concen-

tración molar de la solución). Un estrecho recipiente

cilíndrico lleno de solución acuosa de azúcar hasta una

altura h1=10 cm lo colocan bajo una ancha campana;

su concentración es c1 = 2×10-3

. Sobre la superficie

horizontal bajo la campana echan el mismo tipo de

solución, pero con concentración c2 = 10-3

, cuyo nivel

Fig.8.11

Page 72: problemas selectos

72

h2 satisface: h2 << h1 (Fig. 8.11). Determine el nivel h de la solución del cilindro después que se

establezca el equilibrio. La temperatura se mantiene constante e igual a 20oC. El vapor sobre la

superficie de la solución contiene sólo moléculas de agua; la masa molar del vapor de agua es

μ = 18×10-3

kg/mol.

8.12-En un recipiente cúbico de volumen V = 1 litro se encuentra una masa M = 0.01 g de helio

a una temperatura T = 300 K. Aproximadamente, ¿cuántas veces, η, choca una misma molécula

contra la tapa inferior del cubo en un tiempo de t = 1 min?

8.13-En un vaso echaron dos líquidos no miscibles: tetracloruro de carbono (CCl4) y agua. A

presión atmosférica normal el tetracloruro hierve a 76.6oC y el agua a 100

oC. Si se calienta

uniformemente el vaso en un baño de agua, la ebullición en la frontera divisoria de los líquidos

comienza a los 65.5oC. Si de cada líquido se echaron 50 g, determine cuál de los líquidos se

consume más rápido por la ebullición de frontera, y cuántas veces más rápido. La presión de va-

por saturado del agua a 65.5oC es 192 mm Hg.

8B-Física Molecular. Soluciones.

8.1-Recordemos primero el modelo microscópico que explica la presión del gas sobre la pared.

Cuando el gas y la pared están a la misma temperatura, las moléculas y la pared no intercambian

energía, en valores medios. En tal caso, como promedio, los choques de las moléculas con la

pared son elásticos y las moléculas rebotan con la misma energía cinética y el mismo valor

modular del momento lineal. Si llamamos px a la componente del momento lineal perpendicular

a la pared, la variación del momento lineal de una molécula en un choque es 2│px│, y propor-

cional a esta variación es la fuerza que la molécula ejerce contra la pared, y la presión del gas

sobre la pared por causa de todos los choques promediados en el tiempo.

Si la pared está más caliente que el gas, éste ganará energía de la pared, por lo que en los cho-

ques sus moléculas rebotarán con mayor energía cinética y mayor momento lineal, esto es, si

llega con px rebotará con │px│+ │Δpx│. La variación de momento lineal en el choque será

ahora: 2│px│+ │Δpx│. La fuerza sobre la pared por el choque de cada molécula será mayor, y la

presión aumentará correspondientemente.

Page 73: problemas selectos

73

Si la pared está más fría que el gas, las moléculas del gas rebotarán más lentamente, con menor

cambio de momento lineal, menor fuerza y menor presión.

En conclusión, la presión sobre las paredes es mayor cuando las paredes están más calientes que

el gas.

8.2-El análisis de este problema requiere recordar que la difusión de un tipo de gas a través de

una membrana semipermeable está controlada por la diferencia entre las presiones parciales de

ese tipo específico de gas a ambos lados de la membrana, con independencia de las presiones

parciales que ejerzan otros tipos de gases. La difusión a través de la membrana es un proceso de

tipo microscópico en el que las moléculas de un gas pasan del lado A al B, y del B al A; macros-

cópicamente la difusión deja de notarse cuando pasan de A a B tantas moléculas como de B a A

en un intervalo de tiempo dado, y para que esto ocurra tiene que haber igual presión parcial del

gas a ambos lados.

En conclusión, para este problema cesará la difusión de cada gas cuando el hidrógeno se reparta

por todo el volumen del recipiente (30 L) y cuando el nitrógeno se reparta en dos de las seccio-

nes del recipiente (20 L). El oxígeno no puede pasar por ninguna de las membranas, así que se

limitará a permanecer en su volumen inicial (10 L) sin interferir en la difusión de los otros gases.

La presión de cada gas en su estado final de equilibrio es calculable por las ecuaciones:

pHV = (mH/μH)RT, pOV/3 = (mO/μO)RT, pN(2/3)V = (mN/μN)RT,

donde μH= 2.0 g/mol, μO= 32.0 g/mol, y μN= 28.0 g/mol. Se calculan estas tres presiones (pH =

12.46 atm, pN = 3.11 atm y pO = 12.46 atm) y después, teniendo en cuenta la ley de Dalton de las

presiones parciales, la presión en cada sección será la suma de las presiones de los gases ahí con-

tenidos:

p1 = pH = 12.5 atm, p2 = pH + pO + pN = 28.0 atm y p3 = pH + pN = 15.6 atm.

8.3-La velocidad del pistón hacia arriba se determina por la razón v =Δh/Δt, o sea, la variación de

altura por unidad de tiempo. Esta variación de altura Δh está relacionada con la variación de

volumen del gas al expandirse por calentamiento: ΔV = SΔh, donde S es el área de la sección

transversal del pistón. A su vez, la variación de volumen puede relacionarse con la variación de

temperatura del gas mediante la ley de los gases ideales pV = nRT. En este caso, como el gas se

expande a presión constante (la de la atmósfera, po, más la debida al peso del pistón, Mg/S), se

tendrá:

pΔV = nRΔT

O lo que es igual: (po + Mg/S) SΔh = nRΔT (1)

Page 74: problemas selectos

74

Ahora, durante la expansión del gas se cumple la primera ley de la termodinámica: Q = ΔU + W

Aquí: ΔU = nCVΔT = (3/2)nRΔT puesto que CV = (3/2)R para gases monoatómicos; W = pΔV

por ser una expansión isobárica; y Q = PΔt = qΔt, donde P es la potencia del calentador y Δt el

tiempo que el calentador está conectado. Sustituyendo las tres cantidades en la ecuación de la

primera ley:

qΔt = (3/2)nRΔT + pΔV

Teniendo en cuenta (1):

qΔt = (3/2)(po + Mg/S)SΔh + (po + Mg/S)SΔh

Despejando:

Δh/Δt = (2/5) q / (poS + Mg)

8.4-Si h<< ro, entonces puede aceptarse que la aceleración de la gravedad, g’, es constante en

toda la atmósfera; como además la densidad de la atmósfera es constante según los datos,

podremos plantear que la presión atmosférica sobre la superficie del planeta obedece la ley p =

ρg’h. Pero si llamamos M a la masa del planeta y G a la constante de gravitación universal,

entonces g’= GM/ ro2 (sale de igualar mg’ = GMm/ ro

2 para cualquier cuerpo en la superficie del

planeta), por lo que:

p = ρh (GM/ ro2) (1)

En un pequeño volumen V de la atmósfera, de masa m, próximo a la superficie del planeta, se

cumplirá:

pV = nRT = (m/μ)RT

De aquí: p = (m/V)RT/μ = ρRT/μ (2)

Igualando (1) y (2): ρh(GM/ro2) = ρRT/μ

Despejando: T = μGM h/(Rro2)

8.5-El vapor de agua contenido en el aire expelido por los pulmones (a unos 37oC) tiene una

presión de vapor menor que la presión de vapor saturado del aire a temperaturas calientes, pero

mayor que la presión de vapor saturado del aire frío. Por ello no se condensa cuando sale de los

pulmones al aire caliente, pero sí se condensa en el aire frío: el aire admite vapor de agua a cada

temperatura siempre que la presión de ese vapor no sobrepase la “presión de vapor saturado” a la

Page 75: problemas selectos

75

temperatura que esté el aire, y si lo sobrepasa, se condensa parte del vapor para disminuir su

presión e igualarse a la del vapor saturado que corresponde a esa temperatura.

Es lo mismo que ocurre cuando entra aire frío desde el exterior a la habitación: en ésta la tem-

peratura es mayor y tiene una presión de vapor saturado mayor. Al entrar aire frío y enfriar el

volumen donde entra, la presión de vapor saturado baja localmente en esa zona y el vapor en

exceso que había en esa zona se condensa para disminuir su presión de vapor saturado al valor

que corresponde a la nueva temperatura más fría.

8.6-En las burbujas de agua que se forman durante la ebullición se cumple que:

p = nRT/V = ρRT/μ

En las burbujas solamente hay vapor de agua (no hay moléculas de soluto, salvo las que puedan

meterse en las burbujas por minúsculas fluctuaciones estadísticas). Puesto que p no varía (las

burbujas se siguen formando al igualar su presión interna con la atmosférica), ni varía μ,

entonces el aumento de la temperatura de ebullición implica una disminución de la densidad ρ de

los vapores saturados, para que se satisfaga la igualdad anterior.

8.7-Sabemos que la ebullición del agua ocurre cuando la presión del vapor saturado en las

burbujas que intentan formarse es igual a la presión que se ejerce sobre la superficie del líquido

(despreciando la pequeña sobrepresión que se añade a distintas profundidades de la burbuja en el

recipiente, ρgh, y la pequeña sobrepresión debido a la tensión superficial que surge en la

membrana de la burbuja). Conocemos además que la presión de vapor saturado del agua a 373 K

es justamente 760 mm Hg (que es por lo que el agua hierve a esta temperatura en la Tierra).

Entonces, en el interior de las burbujas que se forma en la ebullición se conocen su presión y

temperatura. Por la ley de los gases ideales tendremos:

p = nRT/V = ρRT/μ

con μ = 18×103 kg/mol para el vapor de agua. De aquí:

ρ = pμ/(RT) = 0.58 kg/m3.

8.8-La Fig. 8.8S (próxima página) muestra un diagrama p-T con el punto triple de una sustancia

cualquiera, donde se señalan tres temperaturas próximas a la temperatura del punto triple, TT, y

las temperaturas de fusión, de vaporización y de sublimación en la trayectoria cerrada mostrada.

Si la sustancia cambiase de sólido a líquido desde el estado (p1, T1) hasta el estado (p2, T2), la

primera ley de la termodinámica, Q12 = ΔU12 + W12, nos permitiría escribir:

Page 76: problemas selectos

76

mLv + mcliq(Tvap – T2) + mcgas(T3 – Tvap) = ΔU23 + W23 (2)

Y al cambiar de gas a sólido del estado (p3, T3) al (p1, T1):

−mLs + mcgas(Tsub – T3) + mcsol(T1 – Tsub) = ΔU31 + W31 (3)

En esta última ecuación, el calor intercambiado en la sublimación es negativo pues la masa debe

entregar calor para pasar de gas a sólido. Las diferencias de temperaturas son también negativas,

como puede apreciarse a partir de los valores relativos en el gráfico, y el trabajo también es

negativo pues el gas se contrae al hacerse sólido.

Si sumamos las tres ecuaciones miembro a miembro y reducimos ahora el radio de la “trayec-

toria” de calentamiento-enfriamiento hasta prácticamente un diferencial alrededor del punto

triple, las variaciones de temperatura ΔT tenderán a cero y quedará:

mLf + mLv −mLs = ΔU12 + W12 + ΔU23 + W23 + ΔU31 + W31

Como el estado inicial y el final es el mismo se anulará la suma de las tres variaciones de energía

interna:

mLf + mLv −mLs = W12 + W23 + W31

Como todos estos procesos están ocurriendo prácticamente a presión constante (pt), los trabajos

pueden tratarse como isobáricos:

W12 + W23 + W31 = pt ΔV12 + pt ΔV23 + pt ΔV31 = pt (ΔV12 + ΔV23 + ΔV31)

Pero el volumen final y el inicial son los mismos:

líquido

sólido

gas

p

T

T1 T2 T3 TT

mLf + mcsol(Tfus – T1) + mcliq(T2 – Tfus) =

= ΔU12 + W12 (1)

donde: mLf : calor de cambio de estado;

mcsol(Tfus – T1): calor de calentamiento del sóli-

do desde T1 hasta Tfus;

mcliq(T2–Tfus): calor de calentamiento del líqui-

do desde Tfus hasta T2; y W12: trabajo de expan-

sión.

De manera similar, al cambiar de líquido a gas,

del estado (p2, T2) al (p3, T3), podemos escribir:

Tvap Tfus Tsub

p1

p2

p3

Fig.8.8S

Page 77: problemas selectos

77

(ΔV12 + ΔV23 + ΔV31) = 0

Entonces: mLf + mLv −mLs = 0

Por tanto: Ls = Lf + Lv = 2.82×103kJ/kg

8.9-La fuerza ascensional sobre el globo es la fuerza de empuje de Arquímedes, o fuerza boyan-

te:

Fb = gmaire-desalojado = gρaireVdesal.

El volumen desalojado de aire es igual al volumen del gas dentro del globo (se desprecia el vo-

lumen de la envoltura) y satisface:

Vdesal = Vgas = ngasRT/pgas

Pero como la envoltura es muy elástica, la presión del gas en su interior es casi igual a la at-

mosférica a cada altura (la sobrepresión que añade el material elástico tratando de contraerse es

despreciable frente a la presión atmosférica, y el globo se dilata con facilidad), y:

patmosf = RTρaire/μaire

donde T es tanto la temperatura del aire como la del gas en el globo, pues todo el ascenso es

isotérmico. Sustituyendo patmosf = pgas en la expresión de Vdesal, y éste en la expresión de la fuerza

boyante:

Fb = ngasμaireg

Como el número ngas de moles de gas es constante, μaire también y g es prácticamente constante

en unos pocos kilómetros de altura, queda demostrado que la fuerza boyante sobre el globo muy

elástico es constante con independencia de que la densidad de la atmósfera vaya disminuyendo

con la altura. Por supuesto, en la práctica el material del globo podrá expandirse según lo

previsto mientras no sobrepase el límite elástico. Después ya no se cumplirá lo analizado: o no se

expande más el globo, o estalla.

8.10-Por ser un llenado rápido y un recipiente térmicamente aislado, el aire que entra no recibe ni

pierde calor. El efecto neto aproximado será como si la atmósfera, de presión constante, haya

trabajado sobre una masa m de aire que cabe en un volumen Vo a presión po y temperatura To, sin

fuerza que se oponga, pues el recipiente en que se meterá esa masa de aire está vacío inicial-

mente. Este “empujón” de aire hacia el vacío le dará una energía cinética a la masa de gas en su

conjunto, pero como en la práctica el gas queda constreñido a un volumen inmóvil, esa energía

comunicada se transformará en energía interna del gas. Para hacer un calculo aproximado de este

Page 78: problemas selectos

78

trabajo, podemos modelarlo por un proceso cuasiestático en el que la atmósfera se expande en

equilibrio hasta ocupar el volumen de todo el recipiente (el flujo real de aire al interior del

cilindro no es en equilibrio). Así, el trabajo aproximado es W = poVo y el aumento de energía

interna del gas ideal monoatómico se calcula por su variación de temperatura: ΔU = (3/2)nRΔT.

Entonces: poVo = (3/2)nRΔT

Pero n = poVo /RTo

Sustituyendo en la ecuación anterior:

poVo = (3/2)(poVo /RTo) RΔT

De aquí: ΔT = (2/3) To

La variación aproximada será un aumento de temperatura de 2/3 de la temperatura inicial que

tenía el aire en la atmósfera. En la realidad, la variación será menor.

8.11-La presión del vapor sobre el líquido es, en cada zona: pvap = psat – Δp = psat − 0.05psat c

Tanto en el recipiente cilíndrico como en el fondo de la campana hay intercambio de moléculas

de agua con el vapor (evaporación-condensación). Esto trae variaciones de concentración funda-

mentalmente en el estrecho recipiente cilíndrico, no así en el ancho fondo de la campana, donde

c2 ≈ constante. Este intercambio se balanceará cuando la diferencia de concentraciones en el

cilindro y en la campana provoque una diferencia de presiones del vapor saturado sobre el

cilindro y sobre el fondo de la campana igual a la diferencia de presiones hidrostáticas del vapor

entre el fondo de la campana y la superficie del líquido en el cilindro.

Sea h la altura del líquido en el cilindro en el equilibrio. La diferencia de alturas entre los dos

niveles del líquido en el equilibrio será: Δh = h – h2.

La concentración c de equilibrio será tal que: c/c1 = h1/h (teniendo en cuenta que la concentra-

ción es inversamente proporcional al volumen de solución, y este volumen es proporcional a la

altura del líquido en el cilindro). Por tanto:

c = (h1/h) c1 (1)

Entonces, por el criterio de equilibrio expresado más arriba:

pvap,2 – pvap,1 = 0.05psat c – 0.05psat c2 = ρvapg (h – h2)

De aquí: h – h2 = 0.05 psat (c – c2) / (ρvapg) (2)

La densidad del vapor se determina por: pV = nRT, ecuación que despejada da:

Page 79: problemas selectos

79

p = (M/μ)RT/V = ρRT/μ. De aquí: ρvap= psat μ/(RT). Sustituimos en (2):

h – h2 = 0.05 psat (c – c2) / [g psat μ/(RT)] = 0.05 [(h1/h) c1 – c2] / [g μ/(RT)]

donde se tuvo en cuenta (1). En este problema: c1 = 2c2. Entonces:

h – h2 = 0.05c2RT [2(h1/h) – 1] / (g μ)

Sustituyendo por los datos del problema, con R = 8.3 J/molK, T = 273 +20 = 293 K, h1= 0.10 m

y g = 9.8 m/s2, queda: h – h2 ≈ 0.689(0.2/h − 1) . Y despejando:

h2 + h(0.689 – h2) – 0.1378 = 0

En esta cuadrática no conocemos h2. Pero, por dato, h2<<h1 = 0.10, por lo que si h2<<0.10,

entonces h2<<0.689 y se puede despreciar en la ecuación, que se reduce así a:

h2 + 0.689h – 0.1378 = 0

Resolviendo la cuadrática y tomando la solución positiva: h = 0.16 m = 16 cm

O sea, se diluye más la solución que estaba más concentrada.

8.12-El tiempo t de 1 minuto es suficientemente largo para que una molécula a 300 K recorra

muchas veces el recipiente de tapa a tapa. Así que todas las moléculas habrán golpeado la pared

inferior del recipiente la misma cantidad de veces en promedio, y el número de choques de una

molécula será el cociente del total de los choques sobre la tapa, Nchoq, y el número total de

moléculas en el recipiente, N: η = Nchoq/N.

El número total de choques sobre una tapa se puede calcular a partir de la presión sobre la tapa:

p = F/A = [2m|vz,avg| Nchoq/t] / A

donde: |vz,avg| es la velocidad promedio de las moléculas en el eje z (perpendicular a la tapa); m es

la masa de una molécula; 2m|vz,avg| es la variación de momento lineal a lo largo del eje z de una

molécula que choca contra una tapa en el plano xy (como sería la tapa inferior); y A es el área de

la tapa.

Pero esta presión satisface también la ley p = nRT/V, por lo que:

nRT/V = [2m|vz,avg| Nchoq/t] / A

Despejando: Nchoq = (nRTAt) / (2m|vz,avg|V )

Entonces: η = Nchoq/N = (nRTAt) / (2m|vz,avg|VN )

Pero: Nm = nμ (masa total del gas), donde μ es la masa molar del helio. Al sustituir queda:

Page 80: problemas selectos

80

η = (RTAt) / (2|vz,avg|μV ) . Ahora: vz,avg ≈ [(vz2)]

1/2 = [(v

2)/3]

1/2 = [RT/μ]

1/2

Además: V/A = l, representa la arista del cubo: l = 1 dm3/1dm

2 = 1 dm = 0.1 m.

Entonces: η = (t /2l) [RT/μ]1/2

Sustituyendo datos: η ≈ 2×105 veces.

8.13-En líquidos poco profundos, como en vasos, la ebullición comienza cuando la presión de

vapor que produce el líquido se iguala con la presión atmosférica (despreciando la pequeña

presión extra que añaden la poca profundidad y la tensión superficial en la superficie de las

burbujas: Δp = ρgh + 2γ/r << patm).

Cuando se tiene dos líquidos no miscibles, las burbujas que se forman en la interfase contienen

vapores de ambos líquidos, por lo que la presión total en estas burbujas es la suma de las presio-

nes parciales de ambos líquidos, lo que da una presión mayor que la de cada líquido por separa-

do, por lo cual la ebullición comienza en la frontera a menor temperatura que la de los dos líqui-

dos por separado: 65.5 < 76.6 < 100.

Llamemos pa a la presión de vapor del agua y pt, a la del tetracloruro. Entonces, en la ebullición:

patm = pa + pt, con patm = 760 mm Hg y pa = 192 mm Hg, por dato. Entonces:

pt = 760 -192 = 568 mm Hg

Pero las presiones parciales dentro de una burbuja son una medida indirecta de los moles que hay

de cada gas o vapor, y, por tanto, una medida indirecta de la rapidez con que se convierte en

vapor cada líquido.

La presión parcial de cada gas se puede expresar como p = nRT/V. En la burbuja, el volumen V

es el mismo para cada vapor, así como la temperatura. Entonces:

nt /na = pt /pa = 568/195 = 2.9

Significa que se vaporizan casi tres moléculas de tetracloruro por cada molécula de agua en la

ebullición de cada líquido. En cuanto a las masas que esto representa, recordemos que m = nμ,

donde m es la masa total de sustancia y μ, su masa molecular (μa = 18 g/mol, μt = 154 g/mol).

Entonces:

mt /ma = (nt /na)( μt /μa) = 2.9×(154/18) ≈ 25

La masa 50 g de tetraclururo se consumirá 25 veces más rápidamente que igual masa de agua en

esa ebullición de frontera.

Page 81: problemas selectos

81

9A-Termodinámica II. Problemas.

9.1-Con n moles de gas ideal se realiza un proceso cíclico cerrado 1-2-3-4-1 que consiste en dos

isobaras, 2-3 y 4-1, de la isocora 1-2 y de un proceso 3-4 representado en el diagrama pV como

una línea recta (Fig. 9.1). Las temperaturas del gas en los estados 1, 2 y 3 son iguales a T1, T2 y

T3 respectivamente, y los puntos 2 y 4 están en una misma isoterma. Determine el trabajo W del

gas en el ciclo.

9.2-Con 3.0 moles de gas ideal monoatómico se realiza el ciclo mostrado en la Fig. 9.2. Las

temperaturas del gas en los diferentes estados son: T1 = 400 K, T2 = 800 K y T3 = 2400 K y T4 =

1200 K. Encuentre el trabajo en el ciclo.

9.3-Determine el trabajo W que realiza un gas ideal en el ciclo cerrado 1-4-3-2-1 representado en

la Fig. 9.3, si p1 = 1.0×105 Pa, po = 4.5×10

5 Pa, p3 = 7.0×10

5 Pa, V2 – V1 =10 litros y las partes

del ciclo 4-3 y 2-1 son paralelas al eje de las V.

p

V

2

3 4

1

Fig. 9.3

0

p3

po

p1

V1 V2

p

V

2

3

1

Fig. 9.4

V

2 3

4 1

Fig. 9.1

p p

T

2 3

4

1

Fig. 9.2

Page 82: problemas selectos

82

9.4-Sobre un gas se realizan dos procesos térmicos, calentándolo desde un mismo estado inicial

hasta una misma temperatura final. En un diagrama pV los procesos se muestran mediante dos

líneas rectas como se ve en la Fig. 9.4 (página anterior). Determine en cuál de los procesos se co-

munica al gas una mayor cantidad de calor.

9.5-Demuestre que la eficiencia térmica de una máquina térmica que realiza un ciclo compuesto

de dos isotermas y dos isocoras es menor que la eficiencia de una máquina de Carnot que trabaja

entre esas dos mismas temperaturas extremas.

9.6-En un primer caso, en una probeta se echa agua a 20ºC. Su fondo se sumerge entonces en una

gran cantidad de agua a 80ºC y el agua de la probeta se calienta en un tiempo t1 hasta los 80ºC.

En un segundo caso se echa en la probeta agua a 80ºC y su fondo se sumerge en una gran

cantidad de agua a 20ºC. El agua en la probeta se enfría hasta 20ºC en un tiempo t2. ¿Qué tiempo

es mayor: t1 o t2? Explique.

9.7-Es conocido que si la temperatura en la calle es de −20ºC, entonces dentro de la habitación

con cierto calefactor la temperatura es de +20ºC. Pero si en la calle la temperatura es de −40ºC,

entonces en la habitación, con el mismo calefactor, se estabiliza la temperatura en +10ºC. En-

cuentre la temperatura T del calefactor que calienta la habitación.

9.8-Un objeto cósmico posee forma de esfera de radio R. Por todo su volumen se distribuyen

uniformemente fuentes que aseguran una liberación de calor con rapidez constante. La potencia

calorífica liberada por unidad de área de la superficie del objeto es proporcional a la cuarta po-

tencia de su temperatura termodinámica (la absoluta). ¿En cuántas veces cambiará la temperatura

estacionaria del objeto si su radio disminuye en 2 veces?

9.9-La envoltura de una estación cósmica representa una esfera negra, la temperatura de la cual

se mantiene igual a T = 500 K como resultado del trabajo de los aparatos dentro de la nave. La

cantidad de calor liberada por unidad de área de la superficie es proporcional a la cuarta potencia

de su temperatura termodinámica. Determine la temperatura Tx de la envoltura si a la nave se la

rodea con una fina pantalla esférica de un radio casi igual que el de la propia envoltura.

Page 83: problemas selectos

83

9B-Termodinámica II. Soluciones.

W = (p2 – p1)( V3/2 + V4/2 – V2)

Ahora, los estados 2 y 3 están enlazados por un proceso isobárico, así como los estados 1 y 4

entre sí. En cada uno de estos procesos se puede aplicar la ley V/T = const., por lo que:

V2/ T2 = V3/ T3 y V4/ T4 = V1/ T1.

De la primera ecuación: V3= V2T3/T2.

De la segunda ecuación: V4= V1T4/T1

Y como T4 = T2 (por dato), entonces V4 = V1T2/T1.

A su vez, los procesos 1 y 2 están relacionados por un proceso isocórico y satisface la ley

p/T = const., por lo que: p2/T2 = p1/T1, de donde se obtiene p2 = p1 T2/T1.

Sustituyendo V3, V4 y V2 en función de V1, así como p2 en función de p1, obtenemos:

W = ½p1V1(T2/T1 – 1)(T3/T2 + T2/T1 – 2 ).

Y como p1V1 = nRT1, finalmente queda todo expresado en función de las temperaturas:

W = ½nR(T2 – T1)(T3/T2 + T2/T1 – 2 ).

9.2- Los estados 1 y 2, así como los estados 3 y 4, están ligados por procesos representados por

rectas que pasan por el origen del plano p-T, por lo que corresponden a proporcionalidades del

tipo:

9.1-El trabajo mecánico que realiza un gas en

un proceso cíclico se corresponde con el área

contenida en la curva cerrada que describe al

ciclo en un plano pV. En el presente problema,

la curva cerrada es un trapecio rectangular: su

área puede calcularse de acuerdo con la siguien-

te expresión:

W = (p2 – p1)(V3 – V2) + ½(p2 – p1)(V4 – V3)

Desarrollando:

V

2 3

4 1

Fig. 9.1

p

Page 84: problemas selectos

84

Ahora: p2V3 = p3V3 = nRT3 , p2V2 = nRT2 , p1V1 = nRT1 y p1V4 = p4V4 = nRT4

Sustituyendo en la expresión del trabajo:

W = nR(T1 + T3 – T2 – T4)

Evaluando: W = 7.4×103 J

9.3-Cuando se realiza un proceso cerrado en el plano pV, el área dentro del ciclo representa el

trabajo realizado en el mismo: si el ciclo se recorre a favor de las manecillas del reloj, el trabajo

es positivo, y en caso contrario es negativo. En el presente problema, el orden del recorrido, 1-4-

3-2-1, es tal que el triángulo superior es recorrido en contra del reloj, y el inferior, a favor. Por lo

que el trabajo en el triángulo superior es negativo y el que corresponde al triángulo inferior es

positivo.

Estos dos triángulos son semejantes, con una constante de proporcionalidad k entre sus lados que

puede determinarse a partir de la relación entre sus alturas: k = (p3 – p0) / (p0 – p1)

Sustituyendo valores: W ≈ 860 J

p

V

2

3 4

1

Fig. 9.3

0

p3

po

p1

V1 V2

La relación entre sus áreas será k2, que será jus-

tamente la relación entre sus trabajos en valores

absolutos: Wsup = k2Winf

Ahora, el trabajo en el triángulo inferior está

dado por su área:

Winf =½ (p0 – p1) (V2 – V1) = 1750 J

El trabajo total será:

W = Winf + Wsup = Winf – k2 Winf =

= Winf [1 – (p3 – p0)2/ (p0 – p1)

2]

p

T

2 3

4

1

Fig. 9.2

p1/T1 = p2/T2 y p3/T3 = p4/T4

Pero ésta es la ley de Gay-Lussac propia de los

procesos del gas ideal a volumen constante, en

los cuales el gas no realiza trabajo. El trabajo lo

realiza el gas en los procesos 2-3 y 4-1 que son a

presión constante, y el trabajo en cada uno se

calcula como W = pΔV.

Luego: W = p2(V3 – V2) + p1(V1 – V4)

Page 85: problemas selectos

85

9.4-Los procesos 1-2 y 2-3 muestran dos expansiones del gas a partir del mismo estado inicial,

mediante los cuales el gas llega a la misma temperatura final, o sea, a un mismo estado de ener-

gía interna final. Si en ambos procesos el cambio de energía interna es el mismo, entonces consu-

bajo, W23>0 (y la energía interna no varía). En el proceso 3-4 no hay trabajoW34 = 0 y se cede

calor, pues hay enfriamiento. En el proceso 4-1 el trabajo se hace sobre el sistema (es una con-

tracción), por lo que W41<0 y el calor se sigue cediendo, pues en el proceso isotérmico del gas

ideal no hay variación de energía interna y W = Q.

En resumen: Wneto

= W23 + W41 (cada trabajo con su signo) y Qabs

= Q12 + Q23 (positivos

ambos).

Ahora bien, el trabajo en un proceso isotérmico está dado por WAB= nRT ln (VB/VA). Por tanto:

W23 = nRT2 ln(V3/V2) y

W41 = nRT1 ln(V1/V4) = -nRT1 ln(V4/V1) = -nRT1 ln(V3/V2) = -W23 T1/T2 .

Entonces: Wneto

= W23 + W41 = W23(1 – T1/T2)

Por otra parte, Q23 = W23, por lo que Qabs

= Q12 + W23

Sustituyendo Wneto

y Qabs

en la definición de la eficiencia térmica:

2

3

4

1

p

V Fig. 9.5

9.5-La eficiencia térmica, e, se define por el co-

ciente: e =Wneto

/Qabsorb.

(trabajo neto dividido por calor absorbido).

Comencemos por ver en qué partes del ciclo hay

trabajo y en cuáles se absorbe calor. En el pro-

ceso 1-2, W12= 0 y se absorbe calor pues se pasa

a una temperatura más alta: Q12>0. En la isoter-

ma 2-3 se absorbe calor, Q23>0, y se realiza tra-

T2

T1

p

V

2

3

1

Fig. 9.4S

me más calor el que realiza mayor trabajo, ya

que Q = ΔU + W. Este trabajo queda represen-

tado por el área bajo la curva de cada proceso.

Del gráfico de la Fig. 9.4S puede verse que el

proceso 1-3 tiene mayor área bajo la recta, por

lo que este proceso comunica mayor cantidad de

calor al gas.

Page 86: problemas selectos

86

e = (1 – T1/T2)/ (1 + Q12/W23). Y como Q12 y W23 son positivos, el denominador es mayor que la

unidad, por lo que e <(1 – T1/T2).

Pero el paréntesis representa la eficiencia de la máquina de Carnot que opera entre las dos

mismas temperaturas extremas. Entonces, queda demostrado que en este ciclo: e < eCarnot.

9.6-En el primer caso la fuente caliente está en la parte inferior, por lo que para ceder calor al

agua fría de la probeta colaboran los fenómenos de conducción, convección y radiación del calor

(las corrientes convectivas llevan calor de abajo hacia arriba.

En el segundo caso el calor debe pasar de la probeta (nivel superior) al agua fría del nivel

inferior, por lo que solamente colaboran la conducción y la radiación, pero no la convección.

Significa que el primer proceso se produce más rápidamente que el segundo: t1<t2. (Por demás,

la conducción térmica del agua es muy baja, de tal modo que la convección es el modo de

propagación de calor más rápido en el agua, y en los fluidos no metálicos en general).

9.7-La cantidad de calor que cede un cuerpo caliente por unidad de tiempo al medio ambiente es

proporcional a la diferencia de temperaturas entre el ambiente y el cuerpo caliente. En este

problema se tiene un calefactor caliente a temperatura T cediendo calor a la habitación, a

temperatura Th, y la propia habitación actúa como cuerpo caliente que cede calor a la calle (o sea,

al exterior), a temperatura Tc.

En la primera situación planteada se tiene Th1= 20oC y Tc1 = −20

oC; en la segunda, Th2 = 10

oC y

Tc2 = −40oC. Y en ambos casos la temperatura del calefactor es la misma.

La potencia calorífica P1 entregada por el calefactor a la habitación en el primer caso satisface la

relación P1 = k1(T – Th1), donde k1 es una constante de proporcionalidad que depende del área de

la superficie del calefactor y su conductividad térmica; semejantemente, la potencia calorífica

entregada por la habitación a la calle, P1’, satisface la relación P1’= k2(Th1 – Tc1), donde k2 es una

constante de proporcionalidad que depende del área de la pared de la habitación en contacto con

la calle y de la conductividad térmica de la pared. Cuando se alcanza el régimen estacionario en

el que se estabiliza la temperatura de la habitación, la potencia calorífica entregada por el cale-

factor a la habitación es igual a la potencia calorífica que entrega la habitación a la calle:

P1= P1’

O sea: k1(T – Th1) = k2(Th1 – Tc1) (1)

Análogamente, en el segundo caso, al alcanzarse el estado estacionario de temperatura:

k1(T – Th2) = k2(Th2 – Tc2) (2)

Page 87: problemas selectos

87

En estas ecuaciones, k1 y k2 valen lo mismo en ambas ecuaciones. Dividiendo miembro a miem-

bro la ecuaciones (1) y (2), y despejando T:

T = (Th2Tc1 – Th1Tc2) / (Th2 + Tc1 – Tc2 – Th1) = 60oC

9.8-La potencia calorífica P liberada por el objeto cósmico es proporcional a su volumen, dada la

uniformidad con que se reparten en su interior las fuentes caloríficas. Esto permite plantear una

relación del tipo P = k1R3, donde R es el radio del objeto esférico y k1, una constante de

proporcionalidad que dependerá de la concentración de fuentes caloríficas en el objeto y de la

potencia de radiación de cada una. Por otra parte, la potencia radiada por unidad de superficie es

proporcional a la cuarta potencia de la temperatura termodinámica (ley de Stephan-Boltzman),

por lo que la potencia total radiada por el objeto será proporcional a T4 y a R

2: P’= k2T

4R

2. En el

estado térmico estacionario, toda la potencia producida en el interior se irradia al exterior: P =

P’, por lo que: k1R3 = k2T

4R

2. De aquí: T = [(k1/k2)R]

1/4.

Entonces, si T resulta proporcional a R1/4

, podemos concluir que si el radio disminuye a la mitad,

la temperatura estacionaria de la superficie del objeto disminuye 21/4

= 1.19 veces.

9.9-Como en el problema anterior, la temperatura absoluta de la superficie de la estación orbital

se establece cuando toda la potencia calorífica generada en el interior de la estación se irradie al

exterior; y cuando la estación no se recubre con la fina pantalla, la temperatura que se alcanza

en su envoltura es de 500 K, y se cumple P = kT4. Pero al colocar la pantalla esférica envol-

viendo a la estación y a su envoltura original, ahora parte del calor irradiado por la envoltura es

reflejado por la pantalla negra. Esta pantalla al recibir el calor irradiado por la envoltura, se

calienta y se convierte en un cuerpo negro que irradia a su vez desde sus superficies: una interna

y otra externa. La irradiación externa se pierde en el vacío, pero la irradiación interna retorna a la

superficie de la envoltura que comienza entonces a sobrecalentarse, pues ahora recibe calor

desde su interior (por los aparatos internos) y desde su exterior (reflejado por la pantalla). La

superficie de la envoltura se calentará hasta que irradie una potencia calorífica que sea la suma

de la que se genera en su interior, más la que le llega de la pantalla. La pantalla, por su parte,

llegará a una temperatura en la cual el calor que irradie hacia afuera iguale a la que le llega del

interior de la estación. Pero su superficie interior (de área casi igual a la exterior y casi igual a la

de la estación) irradia hacia a dentro la misma cantidad que hacia afuera. Por tanto, con la

pantalla externa, la envoltura de la estación irradiará la potencia que genera en su interior más

una cantidad igual que le llega de la pantalla, por lo que irradiará el doble de lo que irradiaba sin

la pantalla externa. Y si la envoltura propia de la estación debe aumentar al doble su potencia

calorífica irradiada, se cumplirá en el nuevo estado estacionario: 2P = kTx4.

O sea: 2kT4 = kTx

4. Por lo que: Tx = 2

1/4T ≈ 600 K.

Page 88: problemas selectos

88

10A-Oscilaciones. Problemas.

(En la sección de “Resortes y Algo Más” se presentan más problemas de oscilaciones, en siste-

mas termodinámicos y electromagnéticos)

10.1-Sobre una superficie horizontal reposan dos bloques de masas m y 2m, unidos por un

resorte ligero de constante k. Encuentre el período T de oscilaciones longitudinales del sistema.

Despreciar la fricción.

casos (a, b, c) las oscilaciones son de igual período. ¿Dónde se halla el centro de masas y cuál es

el período de las oscilaciones?

(a)

(b) (c)

42 cm

10 cm

10.3-Del péndulo AB, con esfera de masa M, es colgado el

péndulo BC con esfera de masa m (Fig. 10.3). El punto A

realiza oscilaciones pequeñas en la dirección horizontal

con período T. Encontrar la longitud l del hilo BC si se

sabe que el hilo AB permanece vertical todo el tiempo.

10.4-Se tiene un gancho de alambre que oscila con am-

plitud pequeña en el plano del dibujo con respecto a las

posiciones dadas del punto de suspensión (Fig. 10.4). En

las posiciones (a) y (b) el lado largo está horizontal. Los

otros dos lados del gancho son iguales entre sí. En los tres

A

B M

m C

Fig. 10.3

m

M

Fig.10.2

10 cm

42 cm

(c)

(b)

Fig. 9.2

10.2-Sobre dos rieles horizontales sin fricción se encuentra un bloque

de masa M. Acoplado a él cuelga una esfera de masa m, como un

péndulo simple. El bloque puede moverse solamente a lo largo de los

rieles. La esfera es sacada de su posición de equilibrio para dejarla

oscilar. Encuentre la relación de los períodos T2/T1 de las oscilaciones

pequeñas en planos paralelo y perpendicular a los rieles. (Fig. 10.2).

Fig. 10.4

Page 89: problemas selectos

89

10.5-El punto de suspensión de un péndulo matemático de longitud l realiza oscilaciones hori-

zontales; su coordenada x varía con el tiempo t de acuerdo con la ley x =A cosωt. Considerando

oscilaciones pequeñas, encuentre la amplitud y fase de las oscilaciones forzadas del péndulo.

10B-Oscilaciones. Soluciones.

10.1-Sea un sistema de referencia fijo a la superficie sobre la que descansan el resorte y los blo-

ques. Puesto que la masa de uno de los bloques es el doble de la otra, el centro de masas de los

bloques estará en un punto O que divide al resorte en dos partes tales que una es el doble de largo

que la otra (Fig. 10.1S). Para el resorte sin deformar se cumplirá:

lom/lo2m = 2m/m = 2

Como no hay fuerzas externas horizontales, las oscilaciones de los bloques ocurrirán sin que se

mueva el centro de masas del conjunto (o que lo haga con velocidad constante), por lo que cuan-

do el resorte esté deformado durante las oscilaciones, las distancias lm y l2m desde cada bloque

hasta el centro de masas seguirá cumpliendo con la relación anterior: lm/l2m = 2m/m = 2.

En consecuencia, las elongaciones de cada bloque respecto al centro de masas satisfacen la

relación: Δlm = 2Δl2m

lm l2m

m 2m

CM

km k2m Fig. 10.1S

R

b

l 10.6-Sobre un cilindro horizontal fijo de diámetro R se

encuentra una tabla horizontal homogénea y uniforme; la

tabla está en equilibrio horizontalmente y tiene longitud

l y grosor b (Fig. 10.6). La fricción estática es suficiente

entre el cilindro y la tabla como para que no resbale bajo

pequeñas desviaciones de la tabla de su posición de

equilibrio, mostrada en la figura. Encuentre: (a) La con-

dición para que, bajo pequeñas desviaciones de la hori-

zontal, la tabla retorne al punto de equilibrio y produzca

oscilaciones. (b) El período de oscilaciones pequeñas ba-

jo tales condiciones.

O

C

Fig. 10.6

Page 90: problemas selectos

90

Y la elongación completa del resorte en cada momento sería:

Δl = Δlm + Δl2m = Δlm + Δlm/2 = (3/2)Δlm

Ahora bien, si las oscilaciones se producen sin que se mueva el centro de masas del sistema, todo

ocurre como si el cuerpo de masa m oscilara unido a un resorte fijo al centro de masas (CM) con

una longitud equivalente a 2/3 de la longitud del resorte completo (y el de masa 2m oscilaría

unido a un resorte con 1/3 de la longitud del resorte completo). A cualquiera de ellos se le podría

aplicar la ley del período de un sistema cuerpo-resorte, esto es, T = 2π(m/k)1/2

. El asunto es

encontrar la constante equivalente del resorte de 2/3 de longitud en función de la constante k del

resorte completo. Para ello basta tener en cuenta que la fuerza de tensión elástica es la misma a

lo largo de todo el resorte; pero para el resorte completo F = kΔl y para el segmento de longitud

2/3 del total, acoplado a la masa m, es F = kmΔlm = km(2/3)Δl, por lo que kΔl = km(2/3)Δl, y de

aquí: km = (3/2)k. Entonces:

T = 2π(2m/3k)1/2

De haber usados la otra masa (2m) el resultado sería el mismo, pues k2m = 3k. Los dos bloques

oscilan con el mismo período, en direcciones opuestas.

10.2-Cuando la oscilación sea en el plano perpendicular a los rieles, el bloque no puede salirse de

estos y el péndulo oscilará desde los rieles, con toda la longitud del hilo meciéndose desde el

punto superior en que se acopla al bloque. El período sería: T1= 2π . Pero al oscilar parale-

lamente a los rieles, el bloquecito oscilará también hacia atrás y hacia adelante, permaneciendo

inmóvil el centro de masas del conjunto (por debajo de los rieles). Entonces, la esfera oscilará

como un péndulo fijo al centro de masas. El centro de masas divide la distancia que separa a las

masas en longitudes inversamente proporcionales a las masas. Entonces, si x es la distancia del

centro de masas a la esferita: m / M = (l-x) / x.

De aquí: x = lM / (M+m). Y el período será T2 = 2π

Entonces: T2 / T1 = .

10.3-El hecho de que el hilo superior permanezca vertical implica que no tiene componente hori-

zontal de tensión sobre él. Esto significa que el hilo superior no hará ninguna fuerza horizontal

sobre la esfera B de masa m: ésta experimentará solamente la fuerza horizontal que le hace el

hilo inferior (componente horizontal de la tensión en el hilo inferior). Pero esta tensión es una

fuerza interna al sistema de las dos esferas, B y C, por lo que no alterará el movimiento, o

reposo, del centro de masas de las dos esferas. La Fig. 10.3S muestra cómo ocurre la oscilación

pequeña de la esfera C. Todo ocurre como si la esfera C de masa m oscilara colgada del centro de

Page 91: problemas selectos

91

masas del conjunto. Si determinamos la distancia x de la esfera inferior al centro de masas, ésa

será la longitud del péndulo simple equivalente, y su período se determinará por la ley T =

2π(x/g)1/2

. El centro de masas de dos partículas (nuestras esferas) es tal que:

10.4-El período de un péndulo físico está dado por T = 2π [I/(mgd)]1/2

donde I es el momento de

inercia del péndulo respecto al punto de suspensión, d es la distancia entre el punto de suspen-

sión y el centro de masas del cuerpo, m es la masa del cuerpo y g, la aceleración de la gravedad.

Aplicando el teorema de los ejes paralelos, I = Io + md2, donde Io es el momento de inercia del

péndulo respecto a un eje que pasa por su centro de masas paralelo al de suspensión real. El

período queda entonces: T = 2π [(Io + md2) / (mgd)]

1/2

Elevando al cuadrado y arreglando: d2 – dg (T/2π)

2 + Io/m = 0 (1)

estamos forzados a admitir que dos de las tres distancias son iguales. Por las dimensiones del

gancho de los datos, solo cabe admitir que da = db = 10/2 = 5 cm. La distancia dc sale de aplicar

el teorema de Pitágoras al triángulo rectángulo que se forma con dc como hipotenusa:

dc = [(42/2)2 + 5

2]

1/2 ≈ 21.6 cm

Con los valores de da, db y dc queda bien localizado el centro de masas. Usando ahora en (1) los

valores ya conocidos de dc y da se obtienen dos ecuaciones en las que no se conocen ni T, ni Io, ni

m (pero que se sabe que en ambas ecuaciones tienen que valer lo mismo):

da2 – dag(T/2π)

2 + Io/m = 0 y dc

2 – dcg(T/2π)

2 + Io/m = 0

Restando miembro a miembro ambas ecuaciones: da2 – dc

2 – (da − dc) g (T/2π)

2 = 0

Fig. 10.4S

42 cm

10 cm da

db

dc CM

Esta ecuación la satisface el gancho para cualquiera de

los tres casos indicados en este problema. La Fig.10.4S

señala las distancias da, db y dc que separa al centro de

masas del gancho del punto de suspensión en cada uno

de los tres casos (a), (b) y (c). Como en los tres casos el

período T es el mismo (por dato), y la ecuación de

segundo grado en d solamente admite dos soluciones,

M

m C

mx = M(l – x)

De aquí: x = lM / (M + m)

Entonces: T = 2π [(l/g)M / (M + m)]1/2

Y despejamos de aquí la longitud del hilo BC:

l = (gT2/4π

2) (M + m) /M

x

l - x CM

Fig. 10.3S

Page 92: problemas selectos

92

Expresamos la diferencia de cuadrados como suma por diferencia y despejamos T:

T = 2π [(da + dc) /g]1/2

= 1.0 s

10.5-La Fig.10.5S muestra el punto de suspensión oscilando la distancia horizontal a = 2A.

En el caso (a), el punto de suspensión oscila con una frecuencia menor que la frecuencia propia

del péndulo simple de longitud l: ω < (g/ l )1/2

. La esferita ajusta su frecuencia a la del punto de

suspensión, para lo cual debe oscilar como si el hilo fuese de longitud mayor, L > l; y se mueve

en fase con el punto de suspensión. En el caso (b), la frecuencia del punto de suspensión es

mayor que la frecuencia propia del péndulo de longitud l: ω > (g/ l)1/2

. La esferita ajusta de nue-

vo su frecuencia a la del punto de suspensión, para lo cual debe oscilar ahora como si el hilo

fuese de longitud menor, L < l; y se mueve en contra fase con el punto de suspensión.

La amplitud de oscilación es distinta en cada caso, pero en cada uno se puede calcular por seme-

janza de triángulos en relación con la amplitud del punto de suspensión. La amplitud del punto

de suspensión, A, es tal que a = 2A y la de la esfera del péndulo, B, es tal que b = 2B. Entonces:

(a): b/a = L / (L – l) y (b): b/a = L / (l – L)

O: (a): B/A = L / (L – l) y (b): B/A = L / (l – L)

Despejando B en cada caso:

(a): B = A L / (L – l) y (b): B = A L / (l – L)

Pero en cada caso, la longitud L del péndulo equivalente con el que oscila la esfera se relaciona

con la frecuencia aplicada por la expresión: L =g/ω2. Sustituyendo para cada caso:

(a): B = A g / (g – ω2l) y (b): B = A g / (ω

2l – g)

Puede notarse que en ambos casos, si la frecuencia impuesta al punto de suspensión es igual a la

frecuencia propia del péndulo ω = (g / l)1/2

, la amplitud del péndulo tiende a infinito: es cuando

el péndulo entra en resonancia, sin fuerza amortiguadora.

l l

L

L

a

a

b b

Fig. 10.5S (a) (b)

Cuando el punto de suspensión del péndulo es

forzado a realizar oscilaciones horizontales, el

péndulo puede adoptar dos modos de oscilación,

en dependencia de la frecuencia impuesta al

punto de suspensión. Estas dos formas se mues-

tran en la Fig. 10.5S, (a) y (b). En el caso (a), la

longitud efectiva L del péndulo es mayor que la

longitud del hilo, l: L > l. En el segundo caso

ocurre a la inversa: L < l

Page 93: problemas selectos

93

coincide con A), y al inclinarse, se levanta hasta una altura:

y = R cosθ + (BA’) senθ + (b/2) cosθ

Para ángulos pequeños: (BA’) ≈ θR, por lo que: y = (R + b/2) cosθ + Rθ senθ

Además, para ángulos pequeños: senθ = θ y cosθ = (1 – sen2θ)

1/2 = (1 – θ

2)1/2

≈ 1 – ½ θ2

Sustituyendo en la expresión de y: y = (R + b/2) (1 – ½ θ2) + Rθ

2

La tabla retornará a su posición de equilibrio si el centro C se eleva en el desplazamiento, esto es,

si y > yo, pues de lo contrario se alejará más del punto de equilibrio o quedará retenida en la

nueva posición por la fricción estática. O sea, la condición de retorno es:

(R + b/2)(1 – ½ θ2) + Rθ

2 > R + b/2

Arreglando: (R – b/2) θ2/2 > 0

(a)Y como θ2 es siempre positiva, la condición se reduce a R>b/2. O sea, el diámetro del cilindro

(2R) debe ser mayor que el grueso de la tabla (b), de lo contrario no se lograrán oscilaciones de

la tabla sobre el cilindro.

(b)Las variaciones de energía potencial gravitatoria de la tabla respecto a su valor en la posición

de equilibrio están dadas por:

ΔU = mg(y – yo) = ½(mg/2)(2R – b)θ2

Ésta es equivalente a una energía potencial de torsión elástica del tipo: ΔU = ½ Kθ2, con constan-

te elástica de torsión: K = (mg/2)(2R – b). En el sistema mecánico de torsión (péndulo de tor-

sión), la frecuencia propia está dada por ω = (K/I)1/2

, donde I es el momento de inercia del sis-

tema respecto al eje de rotación. El período será T = 2π

R

b/2

10.6-La Fig. 10.6S muestra la tabla inclinada un ángulo

θ respecto a la horizontal. Al hacerlo, la recta de contac-

to A’ que recorre la base de la madera por su centro (un

punto en la figura) se separa de la recta A que recorre el

lomo del cilindro y se desplaza hasta B. El ángulo θ de

inclinación de la tabla es también el ángulo que forman

con la vertical las líneas indicadas en la figura. (El punto

O representa el centro del cilindro y C es el centro de

masas de la tabla).

Inicialmente, en el equilibrio, el punto C queda a una al-

tura yo = R + b/2 sobre el centro del cilindro (cuando A’

O

θ

θ

C

A A’

θ B

Fig. 10.6S

Page 94: problemas selectos

94

Para pequeñas oscilaciones puede considerarse que éstas ocurren alrededor del eje que pasa por

A’, por el centro de la cara inferior de la tabla. El momento de inercia de la tabla respecto a tal

eje es: I = Io + mb2/4 de acuerdo con el teorema de Steinner, donde Io = (m/12) (l

2 + b

2), de mo-

do que I = (m/12) (l2 +4b

2).

Sustituyendo I y K en la expresión del período se obtiene:

T = 2π

Page 95: problemas selectos

95

11A-Campo eléctrico. Problemas.

11.1-El cable (conductor con corriente) de un tranvía se rompe y cae al suelo. Una persona con

zapatos conductores puede acercarse al cable sólo con pasos muy cortos: dar pasos grandes

resulta peligroso. Explique por qué.

11.3-Tres esferitas metálicas idénticas no cargadas se encuentran en los vértices de un triángulo

equilátero. Las esferitas se unen mediante un alambre conductor por turnos, una cada vez, con

una esfera metálica muy grande y cargada, cuyo centro se halla en la recta que pasa perpendi-

cularmente al triángulo equilátero, por su centro. Como resultado la primera esferita adquiere

carga q1 y la segunda, q2 (q2<q1). ¿Qué carga q3 adquiere la tercera?

11.4-Sobre una mesa horizontal lisa se encuentra una varilla metálica de longitud l y masa m que

puede moverse por la mesa sin fricción. A uno de los extremos de la varilla se fija un hilo no

conductor que pasa por una polea ideal, fijada al final de la mesa. En el otro extremo del hilo

cuelga una varilla exactamente igual a la primera. A continuación se abandona el sistema a sí

mismo y comienza a moverse. Encuentre la diferencia de potencial que aparece entre los extre-

mos de cada varilla. La fricción en el eje de la polea y la masa del hilo son despreciables.

11.5-Un capacitor plano está lleno de un dieléctrico cuya constante dieléctrica depende del

voltaje aplicado al capacitor según la ley ε = αV, donde α = 1.0 V-1

. En paralelo con este capa-

citor “no lineal”, que está descargado, conectan otro capacitor igual, pero sin dieléctrico, el cual

está cargado a un voltaje Vo = 156 V. Determine la tensión final que se establece en los capa-

citores.

r1

r2

11.2-A una esfera metálica de radio r1, cargada hasta un po-

tencial V1, la rodean mediante un capa esférica conductora

muy fina de radio r2 (Fig. 11.2). Determine el potencial final

V2 de la esfera después que ella sea conectada por cierto

tiempo con la capa conductora.

Fig. 11.2

Page 96: problemas selectos

96

11B-Campo eléctrico. Soluciones.

11.1-Cuando la punta del cable toca el suelo, la corriente sale casi radialmente del cable y se

esparce por la tierra. La corriente viaja de mayor a menor potencial, y a las líneas radiales de

corriente corresponden superficies equipotenciales esféricas (o semiesféricas hacia adentro de la

tierra). La relación entre campo y potencial a lo largo de una línea de corriente es E = –ΔV/Δr, lo

cual nos dice que donde el campo sea más intenso hay mayores variaciones ΔV en iguales des-

plazamientos radiales Δr. Y como el campo es más intenso cerca de la punta del cable, ahí serán

mayores las diferencias de potencial eléctrico para la distancia radial que corresponde a un paso.

Haciendo los pasos más cortos según nos acercamos al cable caído disminuye el intervalo Δr

entre los puntos del suelo que tocan los dos pies, y disminuye la diferencia de potencial entre los

dos pies. Esto último reduce las posibilidades de descargas eléctricas entre ambos pies, descarga

que pasaría por las piernas y el cuerpo.

11.2-Inicialmente la esferita de radio r1 adquiere una carga q1 = V1 r1/k.

Al conectarla a la capa esférica metálica de radio r2 que la envuelve, toda esta carga pasa a la

superficie de la capa, que adquiere un potencial en su superficie V2 = kq1/r2 = V1r1/r2. Y éste será

también el potencial de la esferita conectada a la grande, pues el potencial eléctrico en el interior

de una esfera metálica es constante e igual que el potencial de su superficie.

11.6- Entre qué dos puntos del circuito mostrado en la Fig.

11.6 hay que conectar una fuente de corriente para que se

carguen todos los capacitores, cuyas capacitancias son todas

iguales.

A

B

C D

Fig.11.6

Page 97: problemas selectos

97

Éste será también el potencial en la superficie de cada esferita, donde su carga no se distribuirá

uniformemente por la superficie debido a la influencia de la carga de la esfera grande. Por otra

parte, la carga adquirida por cada esferita es tan pequeña que no perturbará la distribución

esférica de la carga en la esfera grande, y su potencial a cualquier distancia s de su centro seguirá

dada por la ley V = kQ/s.

Por otro lado, en cada esferita el potencial en su superficie es igual al potencial en todos los

puntos de su volumen, y en particular, igual a su potencial en el centro (recordar que las cargas

se distribuyen en la superficie del metal de forma tal que la intensidad del campo eléctrico sea

nulo en su interior, por lo que todo el volumen debe estar al mismo potencial). Y el potencial en

el centro de una esferita puede calcularse sumando los potenciales de todas las cargas sobre su

superficie (a una distancia r del centro) con los potenciales de todas las cargas externas a la

esferita:

Vcentro = kΣqi superf

/r + ΣViexternos

donde r es el radio de la esferita (todas las cargas superficiales están a la misma distancia r del

centro). Puede reescribirse:

Vcentro = kq /r + ΣViexternos

(1)

donde q es la carga total en la esferita. Aplicaremos esta ecuación (1) a cada esferita, que siente

el efecto de la esfera grande y el de las otras esferitas. Llamemos l a la distancia entre el centro

de cada esferita y el centro de la esfera grande (lados inclinados de la pirámide en la figura). Sea

r el radio de cada esferita y sea d la distancia de centro a centro de las esferitas (lados de la base

triangular en la figura). Puesto que la carga Q de la grande prácticamente no varía en el proceso,

su potencial tampoco cambia sustancialmente y las esferitas, puestas en contacto con la grande,

tendrán en sus supericies el mismo potencial Vo producido por la esfera grande en su superficie.

r

r

r d

d

d l

l l

Q

q1

q2

q3

R

11.3-La Fig.11.3S presenta la situación descrita en el

problema. Cuando dos esferas metálicas se conectan

intercambian carga hasta que se igualan los poten-

ciales de sus superficies. O sea, si una, de radio r,

estaba inicialmente descargada y la otra, de radio R,

tenía carga Q, el intercambio de carga δq será tal que:

k(Q-δq) / R = kδq / r, o δq / (Q-δq) = r/R. Se ve que si

R>>r entonces (Q-δq) >> δq, lo que significa que

(Q-δq) ≈ Q. O sea, la carga de la esfera grande prácti-

camente no varía cuando le cede algo de carga a cual-

quier esferita muy pequeña. Así, el potencial de la

esfera en su superficie será todo el tiempo Vo = kQ/R,

aunque intercambie carga con las esferitas.

Fig. 11.3S

Page 98: problemas selectos

98

Al conectar la primera esferita a la grande, el potencial Vo que adquiere será expresable de

acuerdo con (Ec. 1): Vo = kq1/r + kQ/l (2)

Al conectar la segunda esferita, su potencial será: Vo = kq2/r + kQ/l + kq1/d (3)

Al conectar la tercera: Vo = kq3/r + kQ/l + kq1/d + kq2/d (4)

De (2) y (3): q2/q1 = (1 – r/d) (5)

De (2) y (4): q1(1 – r/d) = q2 r/d + q3 (6)

Sustituyendo (1 – r/d) de (5) en (6) y reordenando: q3 = q2(1 – r/d)

Empleando (5) nuevamente: q3= q22/q1

11.4-La diferencia de potencial que surge entre los extremos de cada varilla está ligada al campo

eléctrico interior que surge de extremo a extremo en cada una, campo responsable de acelerar

con cada varilla a los electrones de conducción de cada una. En el modelo de la conducción eléc-

trica en un metal, se asume la existencia de una nube de electrones libres dentro del metal (como

un gas de electrones libres en un recipiente cuyas paredes son las propias fronteras de la pieza

metálica). Cuando este “recipiente de electrones libres” se acelera, los electrones tienden a reza-

garse, acumulándose un poco en la pared “trasera” de la varilla y produciéndose una carga

positiva neta en la pared “delantera” por la ausencia de electrones; estas cargas netas en los

extremos de la varilla producen el campo eléctrico necesario para que todos los electrones del

interior se aceleren con la varilla y no se queden afuera. Aclaramos que los electrones no llegan a

escapar del metal porque la salida de unos pocos de ellos generaría una carga positiva neta en la

varilla, impidiendo el escape de otros electrones.

La aceleración de cada varilla se calcula de forma inmediata aplicando la segunda ley de Newton

a cada una. La tensión T del hilo acelera a la varilla horizontal hacia el borde de la mesa,

cumpliéndose: T = ma (no hay fricción), y en la varilla vertical se cumplirá: mg – T = ma. En

ambas ecuaciones se ha planteado la misma masa m para cada varilla y la misma aceleración (en

magnitud). De estas ecuaciones resulta: a = g/2. Cada electrón en cada varilla se moverá con esta

aceleración debido al campo eléctrico E1 y E2 surgido en el interior de cada varilla.

En la varilla horizontal se tendrá, para cada electrón:

eE1 = mea , de donde resulta: E1 = me a/e = meg/2e.

En la varilla vertical se tendrá, para cada electrón: meg – eE2 = mea = meg/2.

De aquí: Ee = meg/2e = E1.

Page 99: problemas selectos

99

Ambos campos son de igual intensidad: el horizontal acelera a los electrones y el vertical los

frena sin dejarlos caer libremente por gravedad.

Estos dos campos son uniformes dentro de cada varilla y a ellos aplica la ecuación E = −ΔV/Δr,

de donde podemos calcular el ΔV entre los extremos de cada varilla. Despejando: ΔV= −EΔr =

−megl/2e. Como el cociente me/e es del orden de 10-12

kg/C, esta diferencia de potencial entre

los extremos de una varilla normal (digamos de 1 m de largo) es indetectable.

11.5-La capacitancia del capacitor podemos expresarla como C = εCo, donde Co es su capaci-

tancia sin dieléctrico. Con dieléctrico no lineal, la capacitancia será: C = αVCo. Al ponerse en

paralelo, ambos capacitores igualan los potenciales de sus placas en contacto, e igualan sus dife-

rencias de potencial entre placa y placa, a un valor V. La carga en cada uno será: qvacío =VCo y

qno lineal = αV2Co.

Ahora: la carga total que hay para ambos capacitores es la que acumulaba inicialmente el capaci-

tor vacío, qo = VoCo. Y por la ley de conservación de la carga: qno lineal + qvacío = qo.

Sustituyendo cada carga por su expresión en función de los voltajes: αV2Co + VCo = VoCo.

Cancelando Co se obtiene: αV2 + V − Vo = 0, y resolviendo esta ecuación cuadrática para V, se

obtienen dos raíces, una positiva y otra negativa. La positiva es: V = 12 V.

11.6-No hay forma de conectar la batería y que se carguen todos los capacitores. Siempre hay

uno que quedará entre dos potenciales iguales y no se carga. La manera más fácil de verlo es

poniendo el circuito en forma tridimensional de tetraedro regular, o pirámide (Fig. 11.6S).

A

B

C

D

+ Batería

Fig.11.6S

Siempre habrá un capacitor en cada lado de la pirá-

mide, y entre cualesquiera dos vértices que se co-

necte la batería, siempre entre los dos otros vértices

quedará un capacitor cuyos dos extremos estarán a

iguales potenciales, por lo que no se cargará. En la

Fig.11.6S, el capacitor de la rama AB quedará sin

cargar.

Page 100: problemas selectos

100

12A-Circuitos. Problemas.

(El símbolo representa un resistor)

12.1-El circuito de la Fig. 12.1 se compone de resistores con resistencias R1=1.0 Ω, R2= 4.0 Ω,

R3=2.0 Ω y R4=3.0 Ω. ¿Qué corriente circula a través del amperímetro A1 si la corriente a través

del amperímetro A2 es de 5.0 A? ¿Cuál es la lectura del voltímetro V? Considere ideales los

instrumentos de medición.

12.2-Cuatro amperímetros idénticos y un resistor están conectados tal como lo indica la Fig.

12.2. El amperímetro A1 marca I1 = 2.0 A, el A2, I2 = 3.0 A. ¿Qué corrientes pasan por los

amperímetros A3 y A4 y por el resistor? Encuentre la relación r/R entre la resistencia interna r del

amperímetro y la resistencia R del resistor.

12.3-Se monta un circuito de acuerdo con el esquema de la Fig. 12.3. El voltímetro se conecta en

paralelo con el resistor de resistencia R1 =0.40 Ω y registra un voltaje V1 = 34.8 V. La tensión en

los bornes de la fuente de corriente se mantiene constante e igual a 100 V. Encuentre la relación

entre la intensidad de la corriente que pasa por el voltímetro y la de la que pasa por el resistor

R2, de resistencia R2 = 0.60 Ω.

12.4-Encuentre cómo cambia el voltaje en el resistor R del circuito que se muestra en la Fig.12.4

en dependencia del valor de la resistencia R de dicho resistor. La fem de cada una de las fuentes

es igual a E y sus resistencias internas son r. Considerar el diodo como ideal (resistencia cero en

polarización directa e infinita en polarización inversa).

R2

1

R1

V1 Fig. 12.3

100 V

Fig. 12.4

R

E

E E

A

B

C

R1

R2

1

R3

R4

A1 A2

V Fig. 12.1

11.2

A1 A2

A3

A4

Fig. 12.2

Page 101: problemas selectos

101

12.5-El elemento S posee la característica voltampérica representada en la Fig. 12.5 (es la carac-

terística idealizada de un estabilitrón). Se conectan en serie un resistor de resistencia R, el

elemento S y una batería de fem E. ¿Cuál es la magnitud de la corriente en el circuito?

12.6-El elemento B posee la característica voltampérica mostrada en la Fig. 12.6. Se conectan en

serie un elemento B, una batería de fem E y un resistor de resistencia R. ¿Qué tensión se produce

en B?

12.7-Encuentre las corrientes a través de los elementos S y las tensiones en los elementos B de

los circuítos mostrados en las Fig. 12.7(a) y (b), para el caso en que E > IoR + Vo. Las carac-

terísticas voltampéricas de los elementos S y B se dan en los dos ejercicios anteriores.

12.9-En el circuito de la Fig. 12.9 (próxima página) la fem (fuerza electromotriz) de la batería es

de E = 100 V, su resistencia interna es r = 100 Ω, la capacitancia es de C = 200 μF y la

resistencia es R = 10.0 Ω; esta resistencia representa un calentador.

R R

R R

A

B

Fig. 12.8

12.8-Calcule la resistencia entre los puntos

A y B del circuito eléctrico infinito que se

muestra en la Fig. 12.8, si todas las resisten-

cias del circuito son idénticas.

R R

S B

S B

(b) (a)

E

E

V Vo

I

0

Fig. 12.5

Elemento S

V

I

Io

-Io

Fig. 12.6

Elemento B

Fig. 12.7

Page 102: problemas selectos

102

[η = (potencia de calentamiento en la resistencia) / (potencia entregada por la batería)]

K L

C1

C2

Vo

Fig.12.12

12.12-En el circuito de la Fig.12.12, cierran el interruptor

K. Encontrar la máxima corriente a través de la bobina de

inductancia L en función de los datos de la figura. Encon-

trar la tensión máxima en el capacitor C1. Considere idea-

les los elementos del circuito (incluyendo batería recar-

gable).

K

R

R C

C

Vo

12.11-En el circuito mostrado en la Fig. 12.11, R = 100 Ω,

C = 10 μF, Vo =10 V, las resistencias internas de la batería y

del amperímetro son despreciables. El interruptor K se cierra

periódicamente por un tiempo t1 = 1×10-3

s y se abre por un

tiempo t2 = 20×10-3

s. Bajo tal régimen, la aguja del ampe-

rímetro marca un valor prácticamente constante. ¿Qué valor

marca el amperímetro?

A

Fig.12.11

200 Ω 120 Ω

220 Ω 180 Ω 80 Ω

15.0 V 10 μF

A

B C

D Fig.12.10

12.10-Determine la carga eléctrica que se

acumula en el capacitor de la Fig.12.10:

(a)Para el circuito tal y como se muestra.

(b)Para el circuito, con el capacitor y el re-

sistor de 200 Ω intercambiados. (c)¿Qué

cantidad de electrones pasa por la batería

en 10 min. en el caso (a)?

K

2 1

R

C

E ,r

El interruptor K hace contacto entre los puntos 1 y 2

a razón de 10 veces por segundo. Cuando el interrup-

tor se encuentra en la posición 1 el capacitor se carga,

y al pasarlo a la posición 2, se descarga.

Calcule el rendimiento térmico de este circuito, η, y

establezca cuántas veces mayor es este rendimiento

(con el conmutador periódico) que si se conectara di-

rectamente la batería a la resistencia de calentamien-

to, sin capacitor. Fig. 12.9

Page 103: problemas selectos

103

12B-Circuitos. Soluciones.

su resistencia interna es cero (amperímetro ideal), significa que la diferencia de potencial entre

sus extremos P y Q es cero (una resistencia cero entre dos puntos a diferentes potenciales oca-

sionaría una corriente enorme, de cortocircuito, pues I = V/r). En este caso, las caídas de poten-

cial en las resistencias R1 y R2 son iguales: I1R1 = I2R2, de donde resulta I1 = 4I2. Pero I1 + I2 =

5.0 A (que es lo registrado en el amperímetro A1). Entonces: I1 = 4.0 A e I2 = 1.0 A. Además, las

caídas de potencial en R3 y R4 tienen que ser iguales también, pues representan las variaciones de

potencial desde P y Q hasta un punto común T por el que se retiran las corrientes I3 e I4, que

deben sumar 5.0 A nuevamente. Procediendo como se hizo con I1 e I2 se obtiene: I4 = 2.0 A e

R1

R2

1

R3

R4

A1 A2

V

Fig. 12.1S

I1

I2

I5 I3

I4

P

Q

5.0 A T

12.1-En la Fig.12.1S se representan las corrientes

I1, I2, I3, I4 e I5, y las direcciones que les supone-

mos. Aplicando la ley de los nodos al nodo P:

I5 = I1 – I3.

Como el amperímetro 1 está registrando una co-

rriente finita (no inmensa, no en cortocircuito), y

R C

diodo

Fig.12.13

12.13-En un circuito común rectificador de media onda (ver

Fig.12.13): C = 1000 μF, R = 500 Ω. La frecuencia del

circuito es f = 50 Hz. Considerando ideal el diodo encuentre:

(a) El coeficiente de pulsaciones de voltaje, ΔV/Vmax, en el

resistor. (b) Diseñe un circuito rectificador de onda completa

con cuatro diodos ideales, y las mismas resistencia R y capa-

citancia C.

Page 104: problemas selectos

104

I3 = 3.0 A. De los cálculos realizados se ve que por el amperímetro A1 debe pasar una corriente

I5 = I1 – I3 = 1.0 A. Y el voltaje del voltímetro es: V = I1R1 + I3R3 = I2R2 + I4R4 = 10 V.

12.2-En la Fig.12.2S, se indican las direcciones que se le proponen a las corrientes, con I1=2.0 A

e I2 = 3.0 A. La caída de potencial entre los nodos P y Q tiene que ser la misma, calculada por la

suma de voltajes en los amperímetros A1 y A3, o calculada por la caída de voltaje del amperíme-

con lo cual quedan determinadas todas las corrientes.

Para determinar la relación r/R, calculemos la diferencia de potencial entre S y T por la rama que

contiene a R y por la que contiene a A3 y A4: VST = IR = I3 r + I4 r.

De aquí: r/R = I/(I3 + I4) = 0.20

Sustituyendo I2 por la expresión (1):

IV = (E - V1)/R2 – V1/R1 (2)

De (1) y (2): IV/I2 = 1 – V1R2 / [(E - V1) R1] ≈ 0.20

12.4-Para encontrar la dependencia del voltaje V que cae en R, respecto al valor de la propia re-

sistencia R, analizaremos el comportamiento del circuito para valores extremos de R (R grande y

R2

1

R1

V1 Fig. 12.3

100 V

12.3-En R2 el voltaje que cae es V2 = E - V1, por lo que la

corriente que pasa por esa resistencia es:

I2 = (E - V1)/R2 (1)

A su vez, I2 = I1 + IV = V1/R1 + IV, donde IV es la co-

rriente en el voltímetro.

Despejando: IV = I2 – V1/R1.

A1 A2

A3

A4

P

Q S

T

I1 I2

I3

I4 I R

tro A2: VPQ = I2r = I1 r + I3 r.

De aquí: I3 = I2 – I1 = 1.0 A.

Aplicando la ley de nodos al nodo Q:

I4 = I2 + I3 = 4.0 A.

Y en el nodo S: I = I1 – I3 = 1.0 A, Fig. 12.2S

Page 105: problemas selectos

105

R pequeño) para tratar de detectar en qué rango de valores el diodo queda en polarización directa

y en qué rango, en inversa. La Fig. 12.4Sa muestra el circuito con la posible corriente I’ por el

diodo (cuando éste conduzca) y la corriente I por la rama de las dos baterías en serie. Las figuras

12.4Sb y 12.4Sc muestran cómo quedaría el circuito equivalente completo cuando el diodo no

conduzca (caso b) y cuando conduzca (caso c).

Consideremos primero altos valores de R (R>>r). En este caso la corriente total del circuito será

baja, por lo que lo serán I e I’. Los potenciales entre los puntos A y B, señalados en la figura (a),

se relacionarán: VB = VA + 2E – 2Ir ≈ VA + 2E.

Y entre A y C: VC = VA + E – I’r – Vdiodo ≈ VA + E – Vdiodo.

Pero: VC = VB, por lo que: VA + 2E = VA + E – Vdiodo.

De aquí: Vdiodo = – E.

Esto significa que el diodo está polarizado en inversa para altos valores de la resistencia R y no

pasará corriente por la rama del diodo, I’ = 0, por lo que el circuito completo se reduce al

equivalente de la Fig. 12.4Sb, en el que la corriente por la resistencia R es la misma que la de la

rama con las dos baterías, I. Aplicando la ley de Ohm a este circuito: I(R + 2r) = 2E. Entonces el

voltaje en la resistencia es:

VR = 2E R/(R + 2r) ≈ 2E (1) para R grande

Al analizar el otro extremo, R →0, aumenta la caída de potencial en las resistencias internas de

las baterías en la rama AB, por lo que el potencial VB en B empieza a disminuir, y con él, VC,

pues: VB = VA + 2E – 2Ir = VC

Cuando 2Ir alcance el valor E se tendrá VB = VA + E y se llega al límite de la polarización in-

versa del diodo (Vdiodo= 0, con I’= 0). Aún en este límite, la corriente I’ es nula por lo que la

corriente en la resistencia R es la misma que la de la rama AB, I, y sigue siendo válido el circuito

R

Fig. 12.4

R

Fig. 12.4Sb

R

Fig. 12.4Sc

R

Fig. 12.4Sa E

E E

E E

I’ E

E E

E

E E

r

r

r

r r r r

IR

IR

IR

I1

I

I2

A

B

C

Page 106: problemas selectos

106

equivalente de la figura (b). Se cumplirá entonces: 2E – 2Ir – IR = 0, con 2Ir = E, por lo que:

4Ir – 2Ir = IR, de donde: R = 2r.

O sea, con este valor de R se llega al límite de la polarización inversa del diodo. Para valores de

R menores que 2r, el diodo comenzará a conducir y el circuito equivalente del sistema es el de la

figura (c).

Si R<2r, planteamos las ecuaciones de mallas y nodos al circuito de la Fig. 12.4Sc y obtenemos:

2E – 2I2r – IRR = 0, E – I1r – IRR = 0, I1 + I2 = IR

Resolviendo este sistema de ecuaciones: IR = 4E / (2r + 3R)

El voltaje en la resistencia R será entonces:

VR = 4E R/ (2r + 3R) (2) para R pequeño

Las soluciones (1) y (2) representan las soluciones extremas de este problema. Note además que

para R = 2r, las expresiones (1) y (2) coinciden en valor, o sea: VR = E.

12.5- Al conectarse en serie el elemento S con la resistencia, por la ley de Ohm: E = IR + VS.

Si E > Vo, pasará corriente por el circuito, el voltaje en el elemento S será Vo e I = (E – Vo)/R.

Pero si E < Vo, la resistencia del elemento se hace infinita y no pasa corriente: I = 0. Todo el

voltaje cae entonces en el elemento S.

12.6- Si se conectan en serie el elemento B y la resistencia, y se cumple que IoR < E , circulará

entonces la corriente Io que fija el elemento B, y: E = IoR +VB, de donde resulta: VB = E − IoR.

Por otra parte, si IoR > E, el elemento B se hace conductor ideal, en el que no cae voltaje, VB = 0,

y pasaría una corriente menor que Io, tal que I R = E.

12.7-En la Fig. 12.7(a) los elementos S y B están en serie con la batería. Se cumplirá:

E = IoR + Vo + VB. Así, queda fijada la corriente Io por el elemento B, y en S cae el voltaje Vo.

El voltaje en B será: VB = E − IoR – Vo, si E > IoR + Vo.

Cuando los elementos se conectan en paralelo, como indica la Fig. 12.7(b):

VR = IR = E – Vo, ya que el elemento S fija el voltaje del paralelo.

De aquí: I = (E − Vo) / R.

Page 107: problemas selectos

107

Pero I = Io + IS. Entonces: IS = (E − Vo) / R − Io, si E > IoR + Vo.

muestra la Fig. 12.8S. Pero la resistencia total de este segundo circuito es fácilmente calculable

(es R en serie con el paralelo de R y RAB):

RAB = R + RRAB/(R + RAB). Despejando: RAB = (1 + )/2 ≈ 1.62 R

Bajo el proceso de carga, con el interruptor K en 1, el capacitor alcanza un voltaje V =E y una

carga q = CE.

El trabajo realizado por la batería en mover toda esa carga será W = qE = CE 2. La energía acu-

mulada en el capacitor será U = ½ CE 2, que es la mitad de lo trabajado por la batería.

Durante esta carga, el calor liberado en la resistencia de calentamiento es:

Q1 = Σ Ii2RΔti = R Σ Ii

2Δti.

Y el calor liberado en la resistencia interna de la batería es:

Q2 = Σ Ii2rΔti = r Σ Ii

2Δti.

Entonces: Q1/Q2 = R/r (1)

Además, este calor total liberado representa la otra mitad del trabajo hecho por la batería:

Q1 + Q2 = ½ CE 2

(2)

K

2 1

R

C

E ,r

12.9-Comencemos por señalar que el tiempo carac-

terístico de carga del capacitor es ηc = (R + r) C =

0.022 s, y el tiempo de descarga será ηd = RC =

0.002 s. Por otro lado, cada ciclo dura 1/10 = 0.1 s,

que equivale a unos 5 tiempos ηc de carga y unos 50

tiempos ηc de descarga, por lo que pueden conside-

rarse cargas y descargas completas en cada ciclo con

error menor del 1%.

Fig. 12.9

RAB R

R

A

B

Fig. 12.8S

12.8-Como el circuito del problema es una ca-

dena de infinitos eslabones idénticos, la resis-

tencia RAB que exhibe entre los puntos A y B

en la entrada del primer eslabón será la misma

que la que exhibe entre los puntos análogos de

la entrada del segundo eslabón, RAB, como

Page 108: problemas selectos

108

De (1) y (2): Q1 = ½ CE 2 R/(R + r) (3)

Este es el calor liberado en la conexión de K con 1. Al pasar el interruptor K a 2, se descarga el

capacitor y libera en la resistencia R toda la energía que había acumulado: U = ½ CE 2, por lo que

en el ciclo completo de carga y descarga, el calor liberado en la resistencia de calentamiento R

es:

Q = ½ C E 2 [1 + R/(R + r)]

El rendimiento será: η = Pcalent /Pbatería = Q(1 ciclo)/W(1 ciclo) = ½[ 1 + R/(R + r)] = 0.54 (A)

Si se conecta la resistencia directamente a la batería, sin capacitor ni conmutador, la corriente

sería: I = E / (R + r) y la potencia de la batería sería: Pbat = IE = E 2/(R + r).

La potencia en la resistencia sería: Pcalent = I 2R = E 2

R / (R + r)2. Entonces:

η’ = Pcalent /Pbatería = [E 2R / (R + r)

2] / [E 2

/(R + r)] = R/(R + r) = 0.09 (B)

O sea, el circuito con carga intermitente es 6 veces más eficiente: 0.54/0.09 = 6.

12-10-(a)Cuando el capacitor se cargue no pasará corriente de B a C, por lo que el puente será

equivalente a dos ramas en paralelo, de resistencias:

Rabd = 340 Ω y Racd = 380 Ω

La corriente que sale de la batería es: Ib = E /RT = 15.0/259.4 = 57.82×10

-3 A = 57.82 mA

El voltaje en el paralelo es: Vad = IbRp = 0.05782×179.4 = 10.37 V

La corriente en cada rama es: Iabd = Vad/Rabd = 10.37/ 340 = 30.51×10-3

A = 30.51 mA

y Iacd = Vad/Racd = 10.37/ 380 = 27.30×10-3

A = 27.30 mA

200 Ω 120 Ω

220 Ω 180 Ω 80 Ω

15.0 V 10 μF

A

B C

D

Fig.12.10

La diferencia de potencial entre B y C será el

voltaje de carga del capacitor. Para determinar

este voltaje necesitamos las corrientes en cada

rama del paralelo. La resistencia total del pa-

ralelo, R, satisface:

1/Rp = 1/340 + 1/380 = 0.005573, por lo que

Rp = 179.4 Ω.

La resistencia total del circuito será:

RT = 80 + 179.4 = 259.4 Ω

Page 109: problemas selectos

109

La caída de voltaje desde A hasta B es: Vab = 220 Iabd = 6.71 V

Y la caída de voltaje desde A hasta D es: Vac = 180 Iacd = 4.91 V

Entonces el voltaje entre B y C es: Vbc = 6.71 – 4.91 = 1.8 V

Y la carga en el capacitor será: q = CVbc = 10×1.8 = 18 μC

La corriente de la batería es ahora: Ib = 15.0/339.3 = 44.2 mA.

El voltaje en el paralelo será: Vp = 0.04421×139.3 = 6.158 V.

La corriente en la rama ACB será: Iacb = 6.158/380 = 1.62 mA;

y la corriente en la rama BD es la misma que en la batería, Ibd = 44.2 mA.

Entonces: Vcd = Vcb + Vbd =0.00162×200 + 0.0442×120 = 5.6 V

La carga en el capacitor será: q = CVcd = 56 μC

c)La corriente de la batería en el caso (a) es Ib = 57.8 mA. La corriente se define como I = Δq/Δt,

pero se cumple que Δq = Ne (la carga es siempre un múltiplo entero de la carga del electrón), por

lo que:

I = Ne/Δt

Y: N = IΔt/e = 0.0578×600/(1.60×10-19

= 2.171020

electrones

12.11-La solución de este problema es rápida si notamos que el ciclo de carga se logra sin pasar

prácticamente por los resistores (ciclo de carga casi instantáneo, pues RC→0), como muestra la

Fig.12.11Sa (próxima página). En tanto el ciclo de descarga se produce a través de los re-

sistores, como muestra la Fig.12.11Sb.

En el ciclo de carga, el voltaje que adquiere cada capacitor es Vo/2, de manera instantánea,

200 Ω

120 Ω

220 Ω 180 Ω

80 Ω

15.0 V

10 μF

A

B C

D

b)Intercambiando el capacitor con la resistencia de 200 Ω queda

un circuito equivalente al que muestra la Fig. 12-10S. En él, las

resistencias de 180 Ω y 200 Ω quedan en serie, para una resis-

tencia equivalente en serie igual a 380 Ω. Estas dos quedan en

paralelo con la de 220 Ω, para una resistencia equivalente en

paralelo de 139.3 Ω. Todo este paralelo estaría en serie con las

resistencias de 80 Ω y 120 Ω, para una resistencia total en el

circuito de 339.3 Ω. El voltaje de carga del capacitor sería ahora

la diferencia de potenciales entre los puntos C y D.

Fig.12.10S

Page 110: problemas selectos

110

tores. Como el interruptor está cerrado un tiempo t1, la carga que pasa por el amperímetro, por

causa de los resistores es: qr = IRt1 = Vot1/(2R). La carga total que pasa por el amperímetro

durante el tiempo en que el interruptor K está cerrado es:

q1 = qr – qc = Vot1/(2R) − CVo/2

donde consideramos positiva la corriente hacia la derecha en el amperímetro. (El análisis realiza-

do se basa en el principio de superposición: calcular qué carga pasa por el amperímetro debido a

una u otra causa, y luego se superponen las cargas por una suma algebraica).

En el ciclo en que se abre el interruptor K, los capacitores se descargan, cada uno a través de una

resistencia, formando dos lazos de descarga que envían corriente en la misma dirección sobre el

amperímetro, dirección que coincide con la que consideramos positiva anteriormente. El tiempo

en que cada capacitor se descarga en un 63% es η = RC = 100×10×10-6

= 1×10-3

s, que es 20 ve-

ces menor que el tiempo t2 con el interruptor abierto, lo cual quiere decir que los capacitores se

descargan totalmente en cada ciclo a los efectos prácticos. Como los dos capacitores envían sus

cargas en la misma dirección por el amperímetro, la carga total que pasa por éste durante el tiem-

po t2 es:

q2 = 2qc = CVo

El valor constante que marcará el amperímetro será el de la corriente media en el ciclo completo

de duración t1 + t2. O sea:

Im = (q1 + q2) / (t1 + t2) = [Vot1/(2R) − CVo/2 + CVo] / (t1 + t2) = [Vot1/(2R) + CVo/2] / (t1 + t2)

Sustituyendo valores: Im = 4.8×10-3

A = 4.8 mA

El ciclo entero dura t1 + t2 = 21×10-3

s = 21 ms, y en ese tiempo la inercia mecánica de la aguja

del amperímetro no le permite ir y volver a cero; apenas vibrará imperceptiblemente cerca del

valor de corriente media.

K

C

C

Vo A

R

R C

C

A

+

+

+

+

− −

y la carga que acumula cada capacitor es qc=

CVo/2, que es la carga que circula por el am-

perímetro por causa de los capacitores. Aho-

ra, las resistencias en paralelo con cada capa-

citor adquieren también instantáneamente el

voltaje Vo/2, y con ello se produce una co-

rriente directa sobre las resistencias de valor

IR = Vo/ (2R), que en el amperímetro es

opuesta a la corriente de carga de los capaci- Fig.12.11Sa Fig.12.11Sb

Page 111: problemas selectos

111

La energía acumulada entre los dos capacitores es: Uo = ½ Vo2 C1C2/ (C1 + C2)

Al cerrar el interruptor, los dos capacitores y la batería mandarán carga a través de la bobina. La

máxima corriente que pase por la bobina provocará que en ese momento no haya fem en la mis-

ma, pues E = - L dI/dt (en el máximo de la función I la derivada es cero). En ese momento el ca-

pacitor C2 debe haberse descargado totalmente (ya no tiene mas carga que enviar y la corriente

llegó a su máximo). En ese instante, el voltaje en C1 es Vo (ya que C2 está descargado y en la bo-

bina la fem es cero), por lo que la energía total en el circuito en ese instante es:

U = ½LI2

max + ½C1Vo2

La diferencia entre esta energía y la inicial es:

ΔU = ½LI2

max + ½C1Vo2 − ½ Vo

2 C1C2/ (C1 + C2)

Este aumento de energía en el circuito se debe al trabajo hecho por la batería mandando carga al

circuito, ΔU = VoΔq, donde el Δq se mide por la diferencia entre la carga inicial en los capacito-

res y la carga existente en C1 en ese instante:

Δq = C1Vo - Vo C1C2/ (C1 + C2)

Entonces:

½LI2

max + ½C1Vo2 − ½ Vo

2 C1C2/ (C1 + C2) = Vo [C1Vo - Vo C1C2/ (C1 + C2)]

Despejando: Imax = C1Vo/[L(C1 + C2)]1/2

lo cual contesta la primera pregunta.

Para contestar la segunda, notemos que C1 y C2 están en una misma malla, por lo que en todo

momento: Vo = V1 + V2. Esto implica que cuando V1 sea máxima, V2 será mínima, pues Vo es

constante. Además, en el contorno L-C2 se tendrá que cuando V2 esté en un valor extremo

(máximo o mínimo), en L estará ocurriendo una inversión de corriente, lo que significa que I = 0

en esos instantes; y por conservación de la carga en el nodo, implica que tampoco en C1 hay

corriente en ese momento. Entonces, por conservación de la energía para el instante en que V1 es

máximo:

½C1V12 + ½C2(Vo – V1)

2 = ½ Vo

2 C1C2/ (C1 + C2) + Wbat

K L

C1

C2

Vo

12.12- Antes de cerrar K, los dos capacitores están en serie,

cargados, y la carga en los dos capacitores es la misma, pues se

cargan estando en serie. La capacitancia total de ambos en serie

es:

Ceq= C1C2/ (C1 + C2)

Y la carga en cada uno es: qo = Vo C1C2/ (C1 + C2)

Fig.12.12

Page 112: problemas selectos

112

donde ahora: Wbat = VoΔq’ = Vo [C1V1 - Vo C1C2/ (C1 + C2)]

Sustituyendo en la ecuación anterior y arreglando:

V12 (C1 + C2)/2 – V1 (C1+ C2) Vo + [½Vo

2 (C2

2 + 2C1 C2 ) / (C1 + C2 ) ] = 0

Resolviendo la cuadrática:

V1 = [(C1 + C2) Vo ± C1Vo] / (C1 + C2)

Con soluciones: V1,max = Vo[1 + C1/(C1 + C2)] y V1,min = Vo[1 − C1/(C1 + C2)]

tiempo que está descargándose el capacitor en cada ciclo es casi igual al período del ciclo (el

tiempo de carga es muy breve en cada ciclo). Si ΔV<<Vmax, el voltaje que experimenta el resis-

tor es casi constante y la corriente será directa en el resistor.

(a)La variación de voltaje en el resistor será igual a la variación de voltaje en el capacitor (están

en paralelo): ΔV = Δq/C = IΔt/C ≈ (Vmax/R)Δt/C .

De aquí: ΔV/Vmax ≈ Δt/RC

donde Δt ≈ T, por lo que: ΔV/Vmax = T/(RC).

Sustituyendo valores: ΔV/Vmax ≈ 4%

Vmax

ΔV

t

V

R C

diodo

12.13-Para los valores dados en el circuito, el tiempo carac-

terístico de descarga del capacitor será: η = RC = 0.5 s

Y el período de la señal alterna será: T = 1/f = 0.02 s

O sea, se cumple: η >> T (25 veces mayor).

El tipo de voltaje que caerá en el resistor tendrá la forma

aproximada mostrada en la Fig. 12.13Sa, donde se ve que el Fig.12.13

Fig.12.13Sa

Page 113: problemas selectos

113

(b)El rectificador de onda completa tiene la forma de la Fig. 12.13Sb.

Vmax

t

V

Fig.12.13Sc

ΔV

Ahora los dos lóbulos pasan rectificados por la resisten-

cia en la misma dirección (ver Fig. 12.13Sc). Como la

caída de voltaje en la descarga es tan lenta se aproxima

a una línea recta, en cuyo caso los ΔV se reducen a la

mitad, y ΔV/Vmax = 2%. El voltaje resultante sobre la

resistencia es casi constante, como lo indica la línea

horizontal de puntos en el gráfico.

Fig.12.13Sb

Page 114: problemas selectos

114

13A-Magnetismo. Problemas.

13.3-Por una superficie horizontal no conductora rueda sin deslizar un anillo de masa m con

velocidad constante, que posee una distribución de cargas uniforme. Tras conectar un campo

magnético uniforme horizontal de inducción B perpendicularmente al plano del anillo, la fuerza

de presión del anillo sobre la superficie disminuye a la mitad. ¿Con qué velocidad rodaba el

anillo si su carga total es q?

(a)¿Cuál debe ser la inducción magnética B de un campo magnético uniforme, perpendicular al

plano de la figura, para que los electrones que salen del cañón alcancen el blanco? (b) ¿Cuál debe

T T’

M

θ

13.4-En un cañón electrónico, un haz de electrones es

acelerado en cierto recorrido por un campo eléctrico

entre dos puntos bajo una diferencia de potencial V =

1.0 kV. Después de salir del cañón por el punto T (Fig.

13.4) los electrones se mueven por la recta TT’. En el

punto M, a la distancia d = 5.0 cm del punto T, se

encuentra un blanco, de tal modo que la recta TM forma

un ángulo θ = 60o con la recta TT’.

Fig. 13.4

Fig.13.2

vo

e

O

y

x

B 2B

13.2-En un electroimán de construcción especial exis-

ten dos zonas de campo magnético, separadas por un

eje y, como muestra la figura, donde el vector B de in-

ducción magnética en una zona es el doble que en la

otra, saliendo del papel en ambos casos. Perpendicu-

larmente al vector inducción B y al eje y sale una

partícula beta (electrón) de una muestra radiactiva en

el origen O, con velocidad vo (Fig.13.2). Calcule la

velocidad efectiva (o media) que adquiere el electrón

a lo largo del eje Oy. (Su respuesta debe quedar

solamente en función de vo, sin que aparezcan la ma-

sa del electrón, o su carga, ni B).

r

I

Fig. 13.1

13.1-Un alambre conductor tiene la forma de la Fig. 13.1: son arcos

de circunferencia de radio r, de 90o cada uno. Por el conductor pasa

una corriente I gracias a una pequeña batería. Calcule el vector

inducción magnética en el centro C de los arcos.

C

Page 115: problemas selectos

115

ser la inducción magnética Bp de un campo magnético uniforme y paralelo a la recta TM para

que los electrones alcancen el blanco? Considerar que la magnitud de los vectores B y Bp no su-

peran los 0.03 T.

13.5-Una varilla conductora está colgada horizontalmente de dos conductores ligeros en un cam-

po magnético cuya inducción está dirigida verticalmente hacia arriba y cuyo módulo es B = 1 T

(Fig.13.5). La longitud de la varilla es l = 0.2 m, su masa es m = 10 g y la longitud de los con-

ductores es l1 = 0.1 m. A los puntos de suspensión de los conductores se conecta un capacitor de

capacidad C = 100 μF cargado hasta un voltaje V = 100 V. .

13.6-Sobre un núcleo toroidal de ferrita, con permeabilidad magnética μ = 2000, enrollaron dos

bobinas, la primera de n1 = 2000 vueltas y la segunda, de n2 = 4000 vueltas. Cuando en la

bobina primaria se aplica una tensión alterna V1 = 100 V, en la secundaria, a circuito abierto, hay

V2 = 199 V. Encuentre qué tensión se producirá en la bobina secundaria, a circuito abierto tam-

bién, si se utiliza otro núcleo de iguales dimensiones, pero de una ferrita con μ’= 20. Desprecie

la dispersión de flujo magnético y las pérdidas en el núcleo.

13B-Magnetismo. Soluciones.

r

I

Fig. 13.1S

13.1-Se sabe que un anillo de corriente crea un campo B en su

centro de magnitud B = μoI/(2r), cuya dirección está dada por la

regla de la mano derecha, perpendicular al plano del anillo. En

nuestro caso hay tres medios anillos (cada uno compuestos por dos

cuartos de anillo), en cuyo caso la magnitud del campo magnético

que produce cada uno en el centro del anillo es de valor B = μoI/(4r).

En la Fig. 13.1S se muestra el campo Ba que crean juntos los dos

arcos “a”, así como los campos Bb y Bc creados por los arcos “b” y

“c” respectivamente.

a

a

b

b

c

c

Ba Bb

Bc

C

Q

B l

l1 l1

C

Determinar: (a) El máximo ángulo θ de inclinación del

sistema desde su posición de equilibrio después de la des-

carga del capacitor, considerando que la descarga ocurre

en un tiempo muy corto. (b) La capacidad C’ de un capa-

citor que al descargarse incline al sistema un ángulo α=3o

sabiendo que con un capacitor de capacidad C* = 10 μF,

cargado hasta el mismo voltaje (desconocido), el sistema

se inclina un ángulo β = 2o bajo la descarga.

Fig. 13.5

Page 116: problemas selectos

116

El campo total será de valor B =

= μoI/(4r) y estará dirigido a lo largo de la línea de

puntos CQ que forma un mismo ángulo con los vectores Ba, Bc y Bc.

El tiempo t que consume el recorrido de la media circunferencia mayor es tal que:

va t = πr = πm vo /eB, por lo que t = πm/eB.

Por igual razonamiento se llega a que el tiempo t’ que consume el recorrido de la media circunfe-

rencia menor es t’ = πm/2eB.

Se ve que el electrón avanza por el eje Oy de tal modo que se desplaza

Δy = 2r − 2r’ = m vo /eB

en un tiempo total Δt = t + t’ =(3/2) πm/eB

La velocidad media con que sube el electrón por el eje Oy es entonces:

vy = Δy/ Δt = 2vo /(3π)

13.3-Mientras el anillo rueda horizontalmente en ausencia de campo magnético, la fuerza de

gravedad mg que experimenta hacia abajo se compensa con la fuerza de contacto normal N de la

superficie de apoyo, por lo que N = mg. Si la fuerza normal se reduce a la mitad, N = mg/2,

significa que la otra mitad del peso es compensada por la fuerza magnética hacia arriba, que vale

entonces: FB = mg/2.

vo

e

O

y

x

B 2B

Fig.13.2S

13.2-La Fig.13.2S muestra la trayectoria que

seguirá el electrón, acorde con la regla de la mano

derecha para determinar la dirección de la fuerza

magnética. Recorre medias circunferencias en cada

campo y en promedio va subiendo por el eje Oy. La

rapidez en cada circunferencia no se altera y sigue

siendo vo. La fuerza magnética actúa como fuerza

centrípeta en cada media circunferencia. En el

campo de la derecha: evo B = m vo 2/r, de donde se

obtiene el radio: r = m vo /eB

En el campo de la izquierda se obtiene:

r’ = m vo /2eB

Page 117: problemas selectos

117

Ahora bien, la fuerza magnética hacia arriba es la resultante de la fuerza magnética que actúa so-

bre cada porción del anillo; y para cada porción de carga δqi la fuerza magnética está dada por

δFB = δqi vi × B, donde vi sería la velocidad de cada segmento, que es la suma de la velocidad de

traslación de la rueda más la de rotación alrededor del eje: vi = vtras + vi,rot .

O sea: FB = Σ δqi (vtrasl + vi,rot )×B

Pero a cada porción del anillo de carga δqi corresponde otra porción diametralmente opuesta con

una velocidad de rotación igual y opuesta, de modo que la suma Σ δqi vi,rot × B = 0 y queda

FB = Σ δqivtrasl×B = q vtrasl×B,

que de acuerdo con la regla de la mano derecha apunta verticalmente. Su valor absoluto es

FB = qvtraslB sen 90 = qvtraslB

Y esta fuerza debe ser igual a la mitad del peso: qvtraslB = mg/2

De aquí: vtrasl = mg / (2qB), que representa la velocidad con que rueda el anillo.

13.4-La diferencia de potencial acelerador V da al electrón una energía eV que se manifiesta

como energía cinética: ½ mv2= eV, de donde resulta v = .

(a)Al entrar el electrón en una zona donde las líneas del campo B uniforme son perpendiculares a

su dirección de movimiento, sufrirá una fuerza magnética FB = evB que actuará como fuerza

centrípeta y describirá un movimiento circular (Fig. 13.4Sa). En tal caso:

mv2/R = evB, y de aquí: B = mv/(eR) = (1/R) , donde R es el radio del arco de circun-

ferencia que describe el electrón. De la Fig. 13.5Sa puede inferirse que: senθ = (d/2)/R, por lo

que R = d/(2senθ). Entonces, sustituyendo en la expresión de B: B = (2senθ/d) . Eva-

luando: B = 3.7 mT.

T T’

M

θ

Fig. 13.4Sa

θ

R

R

d

Fig. 13.4Sb

T’

M

T

vn θ

vt

v

Eje del cilindro

vn θ

v

vt

Page 118: problemas selectos

118

(b)En el segundo caso el electrón realiza una hélice desde T hacia M que estará contenida en la

superficie de un cilindro cuyo eje es paralelo a la recta TM (Fig.13.4Sb); el cilindro es tangente a

la recta TM a todo lo largo de la misma, y se desarrolla entrando y saliendo en el plano de la

figura. La velocidad de salida del electrón desde el cañón electrónico sigue siendo la misma,

v = , pero tiene ahora una componente tangencial a las líneas del campo Bp, vt = vcosθ,

y otra componente normal a dicho campo, vn = v senθ. Esta última es la responsable de que surja

una fuerza magnética sobre el electrón que le obliga a realizar una trayectoria circular en el plano

perpendicular a las líneas del campo, en tanto la componente vt hace que el electrón avance a lo

largo de las líneas del campo y se desarrolle la hélice. El radio R’ del movimiento circular queda

determinado por la relación: evnBp = mvn2/R’. De aquí: R’= (senθ/Bp) .

Para que el electrón que sale de T dé en el blanco en M debe cumplirse que en el tiempo en que

el electrón avance la distancia d entre estos dos puntos se produzca un número entero de circun-

ferencias de longitud s = 2πR’. El tiempo en que el electrón describe una circunferencia está

dado por T = 2πR’/vn.

En ese tiempo el electrón avanza una distancia s = vtT = 2πR’ (vt /vn) = 2πR’cotθ (vea en la

Fig.13.3Sb el triángulo de las componentes vt, vn y θ). Entonces, el electrón dará en el blanco si

d = ks, donde k = 1, 2, 3, …

O sea: d = k 2πR’cotθ = (2πk cosθ/ Bp) .

Despejando Bp: Bp = (2πk cosθ/d) .

Debido a la limitación de que el campo no puede tomar valores superiores a 0.03 T, los valores

posibles son para k = 1, 2, 3, y 4, con valores 0.0067 T, 0.013 T, 0.020 T y 0.027 T. Para k = 5,

ya se sobrepasa el valor límite para B1.

13.5-(a)Asumir que la descarga es muy rápida equivale a asumir que la varilla recibe un impulso

instantáneo de la fuerza magnética al pasar la rápida corriente de descarga.

Tal fuerza es horizontal (hacia adentro del papel o hacia afuera, dependiendo de la dirección de

la corriente de descarga). El impulso comunicado será:

p =ΣFiΔti = ΣBlIiΔti = Bl ΣIiΔti = BlQ

donde Q es la carga total que pasa por la varilla, igual a la que acumulaba el capacitor: Q = CV.

Así: mv = BlQ = BlCV.

De aquí, la velocidad horizontal que adquiere instantáneamente la varilla es: v = BlCV/m.

Page 119: problemas selectos

119

Con esta velocidad horizontal instantáneamente adquirida, la varilla inicia un movimiento pendu-

lar hacia arriba hasta subir una máxima altura dada por h = l1 (1 – cos θ) (Fig.13.5S, próxima

página). Y por conservación de la energía mecánica en ese ascenso: ½mv2 = mgh.

Dividiendo miembro a miembro:

cosα / cosβ =[2gl1 – (BlC’V/m)2] / [2gl1 – (BlC*V/m)

2]

Despejando: C’= [m/(BlV)]{2gl1 – (cosα/cosβ)[2gl1 – (BlC*V/m)2]}

1/2

Pero esta solución queda en función de V (desconocida), por lo que debemos buscar otra expre-

sión. En la ecuación (1) notamos que el factor (1 – cosθ) es equivalente a 2sen2(θ/2). Entonces,

la solución (1) se convierte en:

sen(θ/2) = BlCV / [2m(gl1)1/2

]

Aplicado al caso (b): sen(α/2)/sen(β/2) = C’/C* = α/β (ángulos pequeños)

Despejando y evaluando: C’ ≈ C* α/β = 15 μF

13.6-Se trata de un transformador con el secundario a circuito abierto y con corriente en el

primario.

La ley de Ohm para corriente alterna en el circuito primario (con L y R) permite plantear:

V1

V2

R

XL

En la Fig.13.6S se muestra el circuito equivalente, con la

resistencia R equivalente de los alambres y la reactancia

inductiva de la bobina primaria.

En el transformador se cumple: E1/ E2 = n1 / n2 = k

En nuestro problema la fem E1 en el primario es algo menor

que el voltaje alterno aplicado, V1, debido a la caída de vol-

taje que producen los alambres de la bobina y el circuito.

Fig. 13.6S

h

l1

θ l1

Fig. 13.5S

O sea: ½(BlCV/m)2 = gl1(1 – cosθ) (1)

De aquí: cosθ = [2gl1 – (BlCV/m)2]/ (2gl1) (2)

Sustituyendo valores: cosθ = 11.59 ≈ 12o

(b)Aplicando la solución (2) a los dos casos planteados en la nueva

situación:

cosα= [2gl1 – (BlC’V/m)2]/ (2gl1) y cosβ = [2gl1 – (BlC*V/m)

2]/ (2gl1)

Page 120: problemas selectos

120

I1 = V1/ [R2 + XL

2]

1/2, donde XL = ωL.

Entonces: E 1 = I1XL = V1XL/ [R2 + XL

2]1/2

= V1/ [1 + (R/XL)2]

1/2 (Se ve que E 1 < V1)

En el secundario se producirá un voltaje: E 2 = k E 1= k V1/ [1 + (R/XL)2]

1/2 = V2 (1)

(Note que, a circuito abierto, V2 = E 2). Ahora, la inductancia de una bobina es proporcional a la

permeabilidad magnética del material que la llena: L(con ferrita) = μL(vacío), por lo que también

XL = μXLo.

De este modo: XL’/XL = μ’/μ. Y: XL’= XL(μ’/μ).

Para la segunda ferrita que se considera, se cumplirá:

E 2’ = k V1/ [1 + (R/XL’ )2]

1/2 = k V1/ [1 + (R/XL)

2(μ/μ’ )

2]

1/2 (2)

Pero de (1): (R/XL)2 = (k V1/V2)

2 – 1

Sustituyendo en (2):

E 2’ = V2’= k V1/ {1 + [(k V1/V2)2 – 1] (μ/μ’ )

2]

1/2

Sustituyendo valores: V2’ ≈ 20 V

Con la segunda ferrita el transformador baja el voltaje en vez de subirlo.

Page 121: problemas selectos

121

14A-Óptica. Problemas.

del segundo prisma. Determine el índice de refracción del material de los prismas.

14.2-Un par de prismas iguales invertidos, de secciones equiláteras de 10.0 cm de lado, están

frente a una pantalla como muestra la Fig. 14.2a. Cada prisma tiene un índice de refracción de

1.422 para la luz roja de ciertos láseres que se usan en la situación que se analiza. Los dos rayos

laser viajan en un mismo plano horizontal, llegan a las caras inclinadas de cada prisma a la mitad

de sus alturas, los atraviesan perpendicularmente al eje de cada prisma, y cada uno desvía el rayo

hacia su base. Vea en la Fig. 14.2b los datos de distancias.

a)Calcule la separación vertical h que se produce entre los dos puntos luminosos en la pantalla.

b)Calcule la apertura angular entre los dos rayos que llegan a la pantalla.

20.0 cm

10.0 cm

h

Vista lateral

Fig. 14.2a Fig. 14.2b

A

B

C

D

E

F

E’

D’

C’

B’

A’

F’

14.1-Dos prismas hexaédricos regulares ABCDEF y

A’B’C’D’E’F’ de un material transparente están pues-

tos muy pegados por las caras ennegrecidas DE y D’E’

(Fig. 14.1). Las caras restantes poseen un recubrimiento

que elimina la luz reflejada. La cara AB se ilumina con

un ancho haz de luz paralela y a lo largo del plano de la

sección hexaédrica de los prismas. Para cierto valor del

ángulo de incidencia se logra que todo el flujo luminoso

que incide por la cara AB salga a través de la cara A’B’ Fig. 14.1

Page 122: problemas selectos

122

El rayo entra en la gota, se refleja parcialmente en su interior y sale de nuevo al aire. Calcule la

desviación angular entre los rayos rojo y violeta que salen de la gota después de la reflexión

interna.

14.5-Una persona estándar tiene una acomodación visual que le permite ver nítidamente desde

25 cm (punto de visión cercana) hasta el infinito (punto de visión lejana). Para ello, el cristalino

del ojo cambia su radio de curvatura gracias a unos músculos ciliares, lo cual le permite obtener

una imagen bien enfocada en la retina de cualquier objeto que esté entre sus puntos de visión

cercana y lejana. Considere el ojo como una esfera, y el sistema óptico “córnea-humor vítreo-

cristalino-humor acuoso” considérelo como un solo “lente equivalente” a una distancia de 2.0 cm

por delante de la retina.

a)Calcule entre qué potencias dióptricas fluctúa la acomodación del ojo estándar.

b)Si un ojo miope tiene 15 cm de visión cercana y 80 cm de visión lejana, ¿cuántas dioptrías

requiere su lente correctivo?¿Qué visión cercana tendrá con los espejuelos puestos? (Asuma

despreciable la distancia ojo-lente correctivo).

(a)

(b)

14.4-Se obtiene una imagen del Sol sobre una pan-

talla puesta a 90º con los rayos provenientes del Sol,

usando una lente plano-convexa (Fig. 14.4a). Se corta

la lente por un diámetro y se unen sus dos mitades por

sus lados planos para obtener una lente compuesta

(Fig. 14.4b). Con esta segunda lente se obtiene una

segunda imagen del Sol sobre la pantalla. Determine

en cuántas veces cambia la iluminación de la segunda

imagen respecto a la primera.

(Iluminación: cantidad de energía de luz visible que

llega por unidad de área y por unidad de tiempo a la

pantalla).

Fig. 14.4

60o

15o

14.3-El Sol está 15.0o

sobre el horizonte y sus rayos de

luz entran en una gota esférica de agua que cae en una

llovizna. Al momento de entrar en la gota, un rayo de

luz lo hace por un punto de la gota que está localizado

bajo un ángulo de 60.0o con el diámetro horizontal, me-

dido desde el centro de la gota (Fig.14.3). Asuma que

los índices de refracción del agua en los extremos del es-

pectro visible van desde 1.330 hasta 1.338.

Fig. 14.3

Page 123: problemas selectos

123

c)Si la pupila tiene 5 mm de diámetro, ¿a qué máxima distancia podrá el ojo estándar distinguir

por separado dos fuentes de luz de los faros de un carro que viene desde el frente por una larga

carretera recta? Considere los faros como dos fuentes puntuales separadas 1.5 m y la longitud de

onda visible de mayor sensibilidad visual, de 550 nm.

14.6-¿Cómo varía la iluminación de la imagen de un planeta observado en un telescopio cuyo

objetivo posee un diámetro D = 80 mm y distancia focal f = 800 mm, si se cambia el ocular de

distancia focal f1 =50 mm por otro de distancia focal f2 = 100 mm, o por otro de f3 =25 mm? El

diámetro de la pupila del ojo tómelo como dp = 5 mm.

14.7-Sobre la superficie plana de un semicilindro de vidrio caen rayos de luz con un ángulo 45o.

Los rayos de luz pasan perpendicularmente al eje del semicilindro. ¿De qué zona de la super-

ficie lateral del semicilindro saldrán los rayos de luz? Índice de refracción del vidrio: n = .

14.8-Una red de difracción tiene 3150 rayas/cm y se ilumina con luz blanca que incide normal-

mente a la red. Se forma un espectro en una pantalla a 30 cm de la red. Si se marca un cuadrado

de 1.0 cm en la pantalla, con su borde interior a 5.0 cm del máximo central (Fig. 14.8), ¿qué

colores aparecen en el cuadrado?

14.9-Una placa plana de plástico transparente se prepara como red de difracción bidimensional,

con líneas periódicamente distribuidas paralelamente a los ejes x y y (horizontal y vertical res-

pectivamente). Un fino haz de luz laser (λ = 620 nm) atraviesa la red perpendicularmente a su

superficie y cae sobre una pared 4.0 m por delante de la red, dando como patrón de interferencia

una cuadrícula de pequeñas manchas circulares brillantes separadas 20 cm por la horizontal y 30

cm por la vertical, en la zona central del patrón (Fig. 14.9). Calcule la densidad de líneas en la

red a lo largo de las direcciones x y y.

θ

30 cm

5cm

Fig. 14.8

5.0 m

30 cm

Fig. 14.9

20 cm

Page 124: problemas selectos

124

14.10-Una lente delgada planoconvexa de diámetro transversal 2r, radio de curvatura R e índice

de refracción no, se coloca en posición tal que a la izquierda hay aire (n1 = 1) y a la derecha hay

otro medio transparente con un índice de refracción n2>1 (la cara convexa se halla del lado del

aire). En el aire, a una distancia do de la lente, sobre el eje óptico, se coloca una fuente puntual de

luz monocromática.

(a) Demuestre que: f1/do + f2/di = 1,

donde di es la distancia entre la imagen y la lente, f1 es la distancia focal de la lente en aire y f2 es

la distancia focal de la lente en el medio de índice de refracción n2.

(b)La lente se corta perpendicularmente a la cara plana en dos partes iguales (Fig. 14.10), las

cuales se desplazan una distancia δ<<r. En el eje de simetría de este sistema, a una distancia

d (d>f1) a la izquierda de la lente, se coloca una fuente puntual de luz S. A la derecha del

sistema, sobre una pantalla P colocada paralelamente a la lente, a una distancia l, se forman N

franjas de interferencia. A ambos lados de la lente ahora hay aire. Determine el número N de

franjas de interferencia en dependencia de la longitud de onda λ. (Los índices de refracción

indicados son los absolutos).

Fig. 14.10

δ

l do

S

Page 125: problemas selectos

125

14B-Óptica. Soluciones.

sar por dentro de exaedro paralelamente a la cara BC, por lo cual es fácil hallar los ángulos de

incidencia y refracción a partir de los ángulos del exaedro. Puesto que cada ángulo interior del

exaedro es de 120o, entonces 120 + 2θ = 180, por lo que θ = 30

o. Además, la recta AC es normal

a la cara CD (puesto que el ángulo <BCD vale 120o), por lo que θ = 90 – 30 = 60

o. De este

modo, los ángulos θ y θ son los de incidencia y refracción en la cara CD para todos los rayos que

llegan ahí. Por la ley de Snell:

n senθ = senθ

De aquí: n = senθ/ senθ = sen 30 / sen 60 = 1.73

14.2-a)La Fig. 14.2S (próxima página) muestra la trayectoria del rayo laser a través de un prisma

hasta la pantalla. La desviación del rayo respecto a su dirección original es la distancia h/2. El

otro prisma produce una desviación similar hacia arriba, por lo que la separación vertical entre

las dos manchas luminosas en la pantalla es h. En la Fig. 14.2S se ve que la desviación h/2 es la

suma de la desviación a que ocurre entre la entrada y la salida del rayo en el prisma y la

desviación b posterior al salir del prisma: h/2 = a + b.

Para calcular a y b deberemos hacer uso de la ley de la refracción y de la geometría plana. En la

figura se han indicado los valores de los ángulos que se van deduciendo:

i)Refracción en la entrada del prisma: 1.00 sen 30.0 = 1.422 senθ , de donde resulta θ = 20.6o.

ii)El complementario de este ángulo, que se forma entre el rayo y la cara del prisma es 69.4o.

iii)La suma de los ángulos interiores de un triángulo es 180o, por lo que el ángulo entre el rayo en

el interior del prisma y la cara de salida del rayo es 50.6o.

iv)El ángulo de incidencia con la normal en la cara de salida es el complementario del anterior, o

sea es de 39.4o.

A

B

C

D

E

F

E’

D’

C’

B’

A’

F’

14.1-Para que todo el haz que

incide sobre la cara AB salga

por la cara A’B’ debe darse la

situación que describe la figu-

ra adjunta: el rayo que entra

por B debe recorrer la cara BC

y salir horizontalmente de C a

C’. Y todos los rayos que en-

tran por la cara AB deben pa-

θ

θ θ θ

θ

Fig. 14.1S

Page 126: problemas selectos

126

v)El ángulo de refracción a la salida del prisma se obtiene de: 1.422 sen 39.1 = 1.00 senθ,

resultando θ = 64.5o

El corrimiento a es: a = l cos 50.6, donde l se obtiene de: l / sen 60.0 = 5 / sen 50.6, o sea, l =

5.60 cm. Y: a = 3.6 cm

El corrimiento b es: b = s tan 34.5, donde s = 22.5 – l sen 50.6 = 18.1 cm. Y: b = 12.4 cm. (El

ángulo de 34.5o se deduce de la diferencia de 64.5 – 30.0; el de 30.0 se deduce del esquema).

Por tanto: h/2 = 3.6 + 12.4 = 16.0 cm.

Entonces: h = 32.0 cm.

b)La desviación angular que sufre hacia abajo el rayo que atraviesa el prisma de la figura es de

64.5 – 30.0 = 34.5o. El otro rayo sufre una desviación angular similar hacia arriba. La apertura

angular entre ambos rayos será 2 × 34.5 = 69.0o.

14.3-En la Fig.14.3Sa (próxima página) se muestra el recorrido que hace el rayo de luz rojo

dentro de la gota. El ángulo de incidencia con la normal es 90o – 15

o – 30

o = 45

o y el de

refracción θ1 al entrar se obtiene de:

sen45.0 = 1.330 senθ1; θ1 = 32.1o

Los triángulos AOB y BOC son isósceles, con iguales ángulos en sus bases los dos (la reflexión

en B es bajo ángulos iguales). A la salida al aire, el rayo formará también el ángulo de 45o con la

normal en el punto C.

20.0 cm

h/2

a

b

a

h/2 = a + b

30.0

30.0 30.0

60.0 60.0

20.6

69.4

50.6 39.4 64.5

Fig. 14.2S

10 cm

5 cm

5 cm

l s 30.0

34.5

2.5 cm

60.0

Page 127: problemas selectos

127

yos dará la desviación angular entre ambos al salir de la gota. Las figuras 14.3Sb muestran grá-

ficamente cómo calcular la desviación de un rayo de luz en una refracción (Δθa = |θinc – θrefr |) y

en una reflexión (Δθb = 180 - 2θinc).

En el caso de la gota, las dos refracciones y la reflexión producen desviaciones siempre a favor

de las agujas del reloj. En cada refracción la desviación del rojo y del violeta son:

Δθa,rojo = 45.0 – 32.1 = 12.9o y Δθa,viol = 45.0 – 31.9 = 13.1

o

Teniendo en cuenta las dos refracciones de cada color: Δθa,rojo = 25.8o y Δθa,viol = 26.2

o

En la reflexión de cada color, la desviación es:

Δθb,rojo = 180 − 2×32.1 = 115.8o y Δθb,viol = 180 − 2×31.9 = 116.2

o

Estas desviaciones se añaden a las de las refracciones, para dar unas desviaciones totales:

Δθrojo = 25.8 + 115.8 = 141.6o y Δθviol = 26.2 + 116.2 = 142.4

o

θinc

θrefr

c

Δθa

θinc

Δθb

θinc

θinc

Fig. 14.3Sb

60o

15o

Algo similar ocurrirá con el rayo violeta, pero bajo án-

gulos diferentes en los triángulos isósceles internos,

pues el de refracción en A será:

sen45.0 = 1.338 senθ1; θ1 = 31.9o

Este rayo se pega más a la normal en la entrada a la

gota, lo que correrá el punto B algo más abajo (a un B’),

y correspondientemente se correrá el punto C más a la

izquierda (a un C’). El rayo violeta saldrá formando

también un ángulo de 45o con la normal que pasa por el

punto C’, corrido respecto a C.

Lo que nos interesa es la diferencia angular entre los dos

rayos a la salida de la gota. Para ello bastará calcular la

desviación total que ha sufrido cada rayo al pasar la

gota, y la diferencia entre las desviaciones de los dos ra-

C

θ1

θ1

θ1

θ1

A

O

C’

Rojo

Violeta

30o

45o

Δθrojo-viol

Fig. 14.3Sa

B

B’

Page 128: problemas selectos

128

Entonces, la separación angular entre ambos rayos emergentes es:

Δθrojo-viol = 142.4 – 141.6 = 0.8o ≈ 1

o

Para un observador que reciba en tierra estos rayos reflejados, aparecería un arcoíris con un

espesor angular de 0.8o (equivaldría aproximadamente a mirar una cinta de 1 cm de ancho al fi-

nal de los brazos extendidos). Desde gotas superiores en el cielo llegarán al ojo los rayos rojos

(más inclinados hacia abajo) y desde gotas inferiores llegarán los rayos violetas: en el arco iris, el

rojo se ve arriba y el violeta abajo. (A veces se observan dos arcoiris bajo ángulos apropiados del

Sol sobre el horizonte. El segundo arcoíris es el resultado de una doble reflexión interna en las

gotas, lo cual invierte el orden de colores en este segundo arcoíris).

4.4-La lente inicial tiene una distancia focal f que satisface:

1/fa = (n -1)(1/r1 + 1/r2 ) = (n – 1)/R

pues r1 →∞ y r2 = R

La lente biconvexa tendrá una distancia focal:

1/fb = (n -1)(1/r1 + 1/r2 ) = 2(n – 1)/R = 2/fa

O sea: fb = fa / 2

Así, la segunda lente tiene la mitad de la superficie de luz solar incidente, pero enfoca esa luz en

la mitad de distancia. Hay que calcular el diámetro del círculo solar en la pantalla para calcular la

iluminación, J, de cada una.

La iluminación J es, de acuerdo a su definición: J = (1/A) ΔE/Δt. Tenemos: ΔEa = 2ΔEb pues la

lente completa recoge el doble de luz que la lente compuesta.

Por otra parte, los diámetros dI y dO de una imagen y un objeto circulares en una lente están en

relación directa con sus distancias i y o a la lente: dI /dO = i/o, por lo que dI = (dO/o)i. La distan-

cia al objeto (el Sol) y su diámetro son las mismas en ambas lentes, pero las distancias imagen

coinciden prácticamente con las distancias focales de cada una, por lo que: dIa = 2 dIb.

Entonces el área de la imagen del Sol de la lente plano convexa tiene un área 4 veces mayor (A =

πd2/4) que la de la lente biconvexa: Aa = 4 Ab. En consecuencia:

Jb = (1/Ab) ΔEb /Δt = (4/Aa) (1/2 ΔEa)/Δt = 2Ja

Así, la imagen de la lente biconvexa tiene dos veces más iluminación a pesar de recibir menos

energía luminosa total (pues la recibe más concentrada).

Page 129: problemas selectos

129

14.5-La potencia dióptrica de una lente es el recíproco de su distancia focal: P = 1/f.

a)Si el ojo enfoca en la retina la imagen de una estrella que se encuentra infinitamente alejada de

la lente, significa que la imagen se formó en el plano focal y que la distancia focal es justamente

la distancia entre la “lente equivalente” del ojo y la retina: f = 2.0 cm = 0.020 m. Entonces:

Pl = 1/f = 1/0.020 = 50 dioptrías (potencia dióptrica del ojo para la visión lejana)

Para la visión cercana, la distancia imagen sigue siendo di = 2.0 cm, pero ahora la distancia

objeto es do = 25 cm. Entonces, la distancia focal se obtiene de:

1/f = 1/di + 1/do = 1/0.020 + 1/0.25 = 54 D = Pc

valor que representa la potencia dióptrica del ojo en el punto cercano. Entonces, la acomodación

visual del ojo normal está entre las 50 D y las 54 D.

b)El lente correctivo que necesita el miope es para que le ayude a ver enfocado desde su punto de

visión lejana hasta el infinito (en las cortas distancias le sobra potencia dióptrica natural). Para

lentes adyacentes una a continuación de otra, la potencia dióptrica del conjunto es igual a la su-

ma de las potencias dióptricas de cada lente, por lo que deberemos encontrar la potencia dióptri-

ca de este ojo miope para su punto de visión lejana, y las dioptrías que le sobren o le falten para

llegar a 50 D (punto estándar de visión lejana) serán las del lente correctivo a añadir ante el ojo.

La potencia dióptrica para la visión lejana de este ojo se obtiene de:

P = 1/f = 1/di + 1/do = 1/0.020 + 1/0.80 = 51.25 D

Para llegar a las 50 D de visión lejana habrá que añadirle una lente de:

Plente = 50 – 51.25 = −1.25 D

O sea, este ojo miope necesita un lente divergente de distancia focal f = 1/D = 80 cm, que es

mucho mayor que la distancia entre el ojo y la lente (alrededor de 1 cm), lo cual hace válido la

aproximación a lentes adyacentes.

Con la ayuda de esta lente, el cristalino amoldará su curvatura convenientemente para ver desde

los 80 cm al infinito. Pero para ver a cortas distancias, esta lente le resta sus posibilidades natu-

rales, y su nuevo punto de visión cercana debe estar más lejos de los 15 cm. La visión cercana

natural de este ojo miope corresponde a una potencia dióptrica de:

P = 1/f = 1/di + 1/do = 1/0.02 + 1/0.15 = 56.67 D

Pero con el lente correctivo disminuye la potencia dióptrica a: P’ = 56.67 – 1.25 = 55.42 D, a la

que corresponde una distancia focal f’= 1/D’= 1.8 cm

La distancia objeto que produce una imagen enfocada en la retina con esta distancia focal es:

Page 130: problemas selectos

130

do = dif’ / (di – f’ ) = 2.0×1.8/(2.0 – 1.8) = 18 cm

O sea, el punto de visión cercana se le corrió al ojo miope de 15 cm a 18 cm. No es un gran

precio a pagar por ganar visión desde los 80 cm hasta el infinito. (A muchos miopes les es más

cómodo quitarse el espejuelo para leer con su potencia dióptrica natural)

c)Esta pregunta se resuelve aplicando el criterio de Rayleigh de resolución de imágenes puntua-

les. La luz de una fuente puntual se difracta en un agujero circular de manera que forma una

mancha circular de difracción. El ángulo bajo el cual aparece el borde de la mancha (posición del

primer mínimo de difracción) respecto al centro de dicha mancha está dado por:

Dsenθ1 = 1.22λ

donde D es el diámetro del hueco circular. El criterio de Rayleigh establece que para ver separa-

das dos manchas de difracción, el borde de una de ellas debe pasar como tope por el centro de la

mancha vecina: si el solapamiento de las dos manchas es más fuerte (que cada mancha circular

se extienda más allá de la mitad de la otra), las dos manchas se verán como una sola.

En este problema, las fuentes puntales son dos faros separados 1.5 m, y cada fuente produce una

mancha circular de difracción en la retina al pasar por el hueco de la pupila. Si estas manchas en

la retina se solapan más que lo indicado por el criterio de Rayleigh, las dos luces se verán como

una. El ángulo bajo el cual se localiza el borde de la mancha se calcula por:

senθ1 = 1.22 λ/D = 1.22×550×10-9

/(5.0×10-3

) = 1.34×10-4

Como es un ángulo pequeño: θ1 = 1.34×10-4

rad

Bajo este ángulo, la mancha en la retina tendrá un radio: Rimag = (2.0 cm)×θ1 = 2.7×10-4

cm. Y

ésta es la menor distancia que debe separar el centro de una mancha, C1, del centro de la otra,

C2, para que se vean por separado. De la Fig. 14.5S se ve que a esa distancia de separación de

los centros de las manchas corresponde una distancia x al carro tal que:

x/2.0 = 150/(2.7×10-4

) = 1.12×106 cm ≈ 11 km

Hasta unos 11 km se pueden ver por separado los faros por un ojo bien enfocado a esa distancia.

Más allá, las luces pueden parecer como una sola luz. (En el problema se ha supuesto claridad

total en la atmósfera, sin humo o partículas difusoras y sin reverberaciones por cambios de

temperatura en la distancia x; con estos efectos, la distancia calculada se reduce bastante).

θ1 1.5 m=150 cm

x

C1

C2

Faro 2

Faro 1

θ1

2.0 cm Fig. 14.5S

Page 131: problemas selectos

131

14.6-La iluminación Jl (en lux: 1 lx = 1 lm/m2) de una imagen se define como el flujo luminoso

Φl (en lumen: lm) que incide por unidad de área A (en m2) sobre la superficie donde se proyecta

la imagen: Jl = Φl/A. El flujo luminoso Φl (en lm) es la potencia luminosa (energía por unidad de

tiempo) que percibe el ojo (pero matizada por el factor fisiológico de la sensibilidad del ojo para

distintos colores).

Para contestar al problema debemos comparar la potencia luminosa que entra por la pupila y

forma su imagen en la retina en cada caso, al fondo del ojo. El sistema óptico se representa en la

Fig.14.6Sa: el lente objetivo, el lente ocular, el lente cristalino (del ojo) y la retina. En un telesco-

pio astronómico de refracción el foco del objetivo y el foco del ocular coinciden.

El diámetro d del haz de luz que sale por el ocular es d = D foc / fob, por lo que en cada caso será:

d1 = 80×50/500 = 5 mm, d2 = 10 mm, d3 = 2.5 mm

En los casos 1 y 3, toda la luz que sale del ocular llega a la retina, Φoc = Φ1 = Φ3, pero en el ca-

so 2 se pierde parte del haz: Φ2 = (dp/d2)2 Φoc = ¼ Φoc

Debemos tener también en cuenta el aumento angular Mθ que produce el telescopio de la imagen

del planeta. El aumento angular está dado por Mθ = θimag / θobjeto y para el telescopio de refrac-

ción se evalúa como Mθ = fob / foc, distinto para cada ocular en este problema. (El aumento angu-

lar es la relación entre el ángulo bajo el cual se observa directamente el diámetro del planeta y el

ángulo bajo el cual se observa el diámetro de la imagen del planeta con el telescopio: el telesco-

pio no muestra una imagen mayor que el planeta en sí, sino una imagen mayor que la imagen que

percibimos mirando directamente el planeta).

La imagen que observa el ojo en cada caso está contenida dentro de un ángulo:

θimag = Mθ θobjeto = ( fob / foc) θobjeto = ( fob / foc) θplaneta

y bajo este ángulo es que entra al ojo la luz que pasa por el telescopio para formar la imagen final

en la retina (Fig.14.6Sb, próxima página), por lo que el diámetro de esa imagen en la retina será

di = Dojo θimag, donde Dojo es la distancia del cristalino a la retina, la misma en los tres casos.

Objetivo

Ocular

Cristalino

Retina

Telescopio

fob foc

Ojo

Fig.14.6Sa

D

Page 132: problemas selectos

132

Entonces: di = Dojo( fob / foc) θplaneta

Y el área de la imagen en la retina será: A = π di2/4 = (π/4)(Dojo θplaneta fob / foc)

2

La iluminación de cada imagen será: Jl = Φ/A. Aplicando a cada una de los tres oculares:

J1 = Φ1 /A1 = Φoc/[(π/4)(Dojo θplaneta 800 / 50)2] = Φoc/ [256(π/4)(Dojo θplaneta)

2]

J2 = Φ2 /A2 = (1/4)Φoc/[(π/4)(Dojo θplaneta 800 / 100)2] = Φoc/ [256(π/4)(Dojo θplaneta)

2]

J3 = Φ3 /A3 = Φoc/[(π/4)(Dojo θplaneta 800 / 25)2] = Φoc/ [1024(π/4)(Dojo θplaneta)

2]

En consecuencia:

J2/J1 = 1 y J3/J1 = 0.25

O sea, los dos primeros oculares producen iguales iluminaciones en la retina, pero el tercero la

disminuye. El primer ocular debe ser el mejor de los tres, pues tiene alta iluminación y mayor

aumento angular.

14.7-La Fig. 14.7S muestra algunos rayos cayendo sobre la cara plana del semicilindro con un

ángulo de 45o.

El ángulo límite θ de reflexión total interna para los rayos que van del vidrio al aire satisface:

45o 45o

1

1

2

2

β θ1

θ2 β

φ

φ

Fig. 14.7S

Todos los rayos se refractarán en el material

con el mismo ángulo β. Los rayos que inciden

muy próximos a los bordes de la cara plana

sufren posteriormente una reflexión total inter-

na en la superficie curva del semicilindro. En

la Fig. 14.7S aparecen los rayos límites, 1 y 2,

para los cuales se produce reflexión total inter-

na por los dos extremos: nótese que los ángu-

los de salida al aire forman 90o con la normal

(el radio del semicilindrocilindro) en el punto

de salida.

Ojo

θimag θimag

Dojo

Imagen producida

por el telescopio di

Fig. 14.6Sb

Page 133: problemas selectos

133

n senθ = 1 sen90o, con n = por lo que θ = 45

o

Por otra parte, el rayo que entra en el vidrio bajo un ángulo de 45o se refracta bajo un ángulo β tal

que: sen45o = n senβ, por lo que senβ = ½ y β = 30

o. Observando los dos triángulos de la figura,

cuyos ángulos interiores deben sumar 180o, se nota que:

(90o – β) + θ + θ1 = 180

o, por lo que θ1 = 75

o

Y: (90o + β) + θ + θ2 = 180

o, por lo que θ2 = 15

o

Así, los rayos saldrán de la superficie lateral del semicilindro por el sector central comprendido

entre los rayos límites localizados por los ángulos θ1 y θ2.

14.8-El período de la red es d = 1/3150 = 3.1746×10-4

cm. El ángulo θi bajo el cual se localiza el

borde interior de la abertura satisface: tan θi = 5/30 = 1/6, θi = 9.46º. Y el ángulo θe del borde

exterior satisface: tan θe = 6/30 = 1/5, θe = 11.31º.

La luz blanca tiene longitudes de onda comprendidas entre unos 700 nm (rojo) y unos 400 nm

(violeta). El asunto es averiguar cuáles de estas longitudes de onda producen máximos de

interferencia en esa “ventana” angular entre los 9.46º y los 11.31º.

La condición de máximos para redes de rayas es:

n λ = d sen θn , con n = 0, 1, 2, …

Las longitudes de onda que se refuerzan en los extremos de la ventana angular las calculamos

por despeje de la ecuación anterior: λ = d sen θn /n

Para n = 0 todas las longitudes de onda forman un máximo central, que no cae en la ventana.

Para n = 1: λi = d sen θi = 522 nm y λe = d sen θe = 623 nm

que son longitudes de onda entre el verde y el rojo.

Para n = 2: λi = (d/2) sen θi = 261 nm y λe = (d/2) sen θe = 311 nm

que son longitudes de onda en el ultravioleta.

En resumen, en el cuadrado aparece una fracción del arcoíris de primer orden, entre las longitu-

des de onda de 522 nm y 623 nm (del verde al rojo).

14.9-a)La separación de las manchas que se expanden paralelas al eje x se deben a la inter-

ferencia de la luz difractada por las líneas verticales de la red (líneas orientadas paralelamente al

Page 134: problemas selectos

134

eje y). Para ellas vale la condición de máximos de interferencia de una red de líneas verticales:

dxsenθm = mλ

Teniendo en cuenta que la distancia horizontal entre manchas es Δx = 20 cm y que la distancia a

la pared es L = 400 cm, podemos calcular el ángulo del primer máximo por tanθ1 = Δx/L =

0.050. Pero este valor corresponde a ángulos pequeños, por lo que podemos asegurar directa-

mente que también senθ1 = 0.050. Entonces:

dx (0.050) = (1)(620). Por lo que: dx = 1.24×104 nm = 1.24×10

-2 mm

La densidad de líneas verticales será: 1/dx = 80.6 líneas /mm

Semejantemente: dysenθm ≈ dytanθm = mλ, donde tanθ1 = 30/400 = 0.075, y de aquí:

dy(0.075) = (1)(620). Por tanto: dy = 0.827×10-2

mm y la densidad de líneas horizontales será:

1/dy = 121 líneas/mm

14.10-(a) La Fig. 14.10Sa muestra la lente, la fuente puntual S, su imagen S1 si del lado plano de

la lente hubiese aire, y su imagen S2 cuando del lado plano de la lente está el medio de índice de

refracción n2.

Se representa también un rayo que sale de la fuente S formando un ángulo pequeño con el eje

óptico (ángulo agrandado en la figura), que se refracta en la cara convexa de la lente e incide

bajo un ángulo θo sobre la cara plana. La refracción posterior se produce bajo un ángulo θ1 si hay

aire tras la cara plana, y bajo un ángulo θ2 si está el medio de índice n2. El rayo sale de la cara

plana a una distancia x del eje óptico. La distancia de S a la lente es do, de S1 a la lente es di,aire y de S2 a

la lente es di.

Cuando hay aire en los dos lados de la lente se cumplirá: 1/do + 1/di,aire = 1/f1, por lo que:

f1/do + f1/di,aire = 1

La relación que debemos demostrar es: f1/do + f2/di = 1

Fig. 14.10Sa

x

di,aire

do

S

S1 (en aire) S2 (en n2)

di

θo

θ1 θ2

θ1 θ2

Page 135: problemas selectos

135

Comparando estas dos ecuaciones se ve que lo que debemos probar es: f1/di,aire = f2/di (1)

Para ello, analicemos las refracciones en la cara plana de la lente:

no senθo = senθ1 , cuando hay aire tras la lente, y

no senθo = n2 senθ2 , cuando está el medio de índice n2.

Se desprende de aquí que: senθ1 = n2 senθ2

Y dado que los ángulos son pequeños se puede aproximar a: θ1 = n2 θ2 (2)

Los ángulos θ1 y θ2 son iguales respectivamente a los de las bases de los triángulos rectángulos

que forman los rayos emergentes con el eje óptico. En estos triángulos el cateto de longitud x es

prácticamente igual al arco de circunferencia que puede trazarse desde S1 con radio di,aire, o

desde S2 con radio di. En tal caso:

θ1 = x / di,aire y θ2 = x / di

Sustituyendo en (2) y arreglando: di = n2di.aire (3)

Si la fuente S estuviera muy alejada de la lente, las imágenes S1 y S2 se formarían en el foco de la

lente, en el aire y en el medio de índice n2, y las distancias di,aire y di serían:

di,aire = f1 y di = f2

Y la relación (3) quedaría: f2 = n2f1 (4)

Dividiendo miembro a miembro la ecuación (4) por la (3):

f2/di = f1/di,aire

que es la ecuación (1) con la que se demuestra que: f1/do + f2/di = 1

(b)La Fig. 14.10Sb muestra la situación descrita en esta parte del problema.

La fuente S produce ahora dos imágenes reales puntuales S’ a una distancia di de la bilente; las

dos imágenes están separadas una distancia d y se comportan como dos fuentes puntuales cohe-

Fig. 14.10Sb

δ

l

do

S

S'

S'

d

di L

D

Page 136: problemas selectos

136

rentes que emiten luz hacia la pantalla que está a la distancia l de la lente. La luz que avanza

desde estas imágenes puntuales, proyectarán dos haces sobre la pantalla que se solaparán en la

zona de espesor D señalada en la figura. En esta zona se producirán las franjas de interferencia.

Sea L la distancia desde las imágenes puntuales S’ hasta la pantalla, do, como antes, la distancia

de la fuente S a la bilente y δ, la distancia de separación entre las partes de la bilente. Lo primero

será averiguar la posición en que surgen las imágenes S’ para conocer su distancia L hasta la

pantalla (L = l – di) y poder analizar el patrón de interferencia. Se cumplirá:

1/do + 1/di = 1/f1, por lo que: di = dof1/ (do – f1)

Por otra parte, el triángulo que se forma entre la fuente S y las dos imágenes S’ es semejante al

triángulo con base en el segmento δ de la bilente y vértice en la fuente S. De aquí:

d/ (do + di) = δ/do, por lo que d = δ (do + di)/ do

Sustituyendo aquí la expresión anterior de di y arreglando: d = δdo/ (do – f1)

Conocida la distancia entre las fuentes podemos calcular la distancia entre las franjas en la panta-

lla. En el segmento D, las franjas de máximos de interferencia satisfacen la condición:

kλ = d senθk , con k = 0, ±1, ±2, …

donde θk es el ángulo que localiza a cada máximo respecto al centro del patrón, medido desde el

punto medio de las dos fuentes S’. Para ángulos pequeños: kλ ≈ θkd. La distancia desde cada

máximo al centro del patón será: xk = Lθk = L kλ/d. La distancia entre dos máximos sucesivos, k

y k + 1, será: Δx = Lλ/d, donde:

L = l – di = [l (do – f1) – dof1] / (do – f1)

Luego: Δx = λ [l (do – f1) – dof1] / (δdo)

Averiguando la longitud del segmento D en la pantalla podremos calcular la cantidad de franjas

N que se forman sin más que dividir N = D/Δx Por la semejanza de triángulos entre el que tiene

de base D y el que tiene de base δ (ambos con vértice en S), podemos plantear: D/ (l + do) = δ/do

Entonces: N = D/Δx = (δ2/λ) (1 + do) / [l (do – f1) – dof1] = 46.6

Contando solamente franjas completas: N = 46.

Esta respuesta ha supuesto que l>di y D>Δx, esto es, que la pantalla se coloque más atrás de

donde aparecen las imágenes puntuales S’, y que el espesor de la zona de solapamiento de los

dos haces sea mayor que la distancia entre dos franjas sucesivas.

Page 137: problemas selectos

137

15A-Física Moderna. Problemas.

15.1-Se emplea una red de 600 líneas/mm para analizar la luz de una galaxia, y el primer máxi-

mo lateral de la luz roja del hidrógeno emitida por la galaxia se obtiene bajo un ángulo de 25.3o

respecto al máximo central. Diga si la galaxia se aleja del observador o si se aleja de éste, y

calcule con qué velocidad lo hace.

15.2-Un electrón es acelerado a partir del reposo a través de una diferencia de potencial de

2.00×105V y después viaja a velocidad constante. (a) ¿Qué tiempo toma al electrón viajar 8.00 m

después de alcanzar su velocidad final? (b) ¿Qué distancia recorrió medida en el sistema del

electrón en reposo?

15.3- Un positrón con una energía cinética de 0.739 MeV choca con un electrón en reposo y se

aniquilan, dando lugar a dos fotones. Uno de los fotones emerge a 90º con la dirección del

positrón incidente. ¿Cuáles son las energías de los dos fotones emergentes y en qué dirección

emerge el segundo fotón? (El electrón y el positrón son de iguales masas en reposo, de 0.511

MeV/c2, y cargas iguales y opuestas: son una pareja de partículas análogas de materia y antima-

teria).

15.4- Una partícula de masa en reposo m y energía cinética relativista 4mc2 choca y se pega a

una partícula estacionaria de masa en reposo 2m. Encuentre la masa en reposo M de la partícula

compuesta resultante.

15.5- (Efecto Compton relativista). Un fotón de rayos X de longitud de onda λ incide sobre un

electrón en reposo y se desvía un ángulo θ y tiene entonces una longitud de onda mayor λ’. El

electrón que recibe el impacto se mueve después del choque con una velocidad relativista.

Considere conocidas la masa en reposo del electrón, m, la velocidad de la luz, c y la constante de

Planck, h. Calcule la variación de longitud de onda del fotón en función del ángulo θ que se

desvía. (Para una partícula relativista se cumple el “invariante relativista”: E2 − p

2c

2 = m

2c

4).

15.6-a) Un fotón de energía E es absorbido por una partícula estacionaria de masa en reposo m.

¿Cuál es la velocidad y la masa en reposo de la partícula compuesta resultante?

Page 138: problemas selectos

138

b) Repita la parte (a) remplazando el fotón por una partícula de masa en reposo m y una rapidez

v=0.80c.

15.7- Un pion neutro se desintegra en dos fotones. La masa en reposo del pion es de 135 MeV/c2.

Suponga que el pion está en un haz secundario con una energía cinética relativista de 1.00 GeV.

a) ¿Cuáles son las energías de los fotones si ellos son emitidos en direcciones opuestas a lo largo

de la dirección original de movimiento del pion?

b) ¿Qué ángulo se forma entre los dos fotones si ellos son emitidos bajo ángulos iguales respecto

a la dirección de movimiento original del pion?

15.8-(a) Un neutrón lento, 1n, es absorbido por un núcleo de

235U que se transforma momen-

táneamente en un núcleo de 236

U que se fisiona rápidamente y da lugar a un núcleo de 140

Ce, otro

de 94

Zr y dos neutrones lentos, de acuerdo con la reacción:

235

U + 1n →

140Ce +

94Zr + 2

1n

Las masas de los núcleo se indican más abajo en unidades de masa atómica (1 u= 931.5 MeV/c2).

a) Calcule la cantidad de energía que se libera en una reacción nuclear como ésta.

b) Si una bomba es de 57 kg de uranio enriquecido (235

U) y sólo el 10% de este material llega a

fisionarse durante la explosión, ¿qué cantidad total de energía se libera? (235 g de 235

U contienen

6.022×1023

núcleos: número de Avogadro).

c) Si hace falta un kilotón (mil toneladas de TNT) para liberar 2.6×1025

MeV, ¿de cuántos kilo-

tones es la bomba analizada en este problema?

Masas: 1n: 1.08866 u,

235U: 235.0439 u,

140Ce: 139.9054 u,

94Zr: 93.9063 u

15.9- El flujo de energía radiante del Sol que llega a la Tierra es Φ =1370 J/(s-m2), sobre cual-

quier superficie en la Tierra perpendicular a los rayos del Sol.

a) ¿Cuál es la fuerza total que ejercería la luz sobre toda la Tierra si toda la energía luminosa

fuese totalmente absorbida.

b) ¿Cuánta masa solar es convertida en energía por segundo para suministrar toda la que llega a

la Tierra? (Radio de la Tierra: RT ≈ 6.4 × 106 m, distancia Tierra-Sol: d ≈ 1.5 × 10

11 m).

c) ¿Cuál es la masa total consumida y radiada por el Sol en cada segundo?

Page 139: problemas selectos

139

d) Estime la masa de hidrógeno que debe ser convertida en helio por segundo para suplir esta

energía radiante. (Recuerde que la mayor parte de la energía suministrada por el Sol proviene de

la fusión de hidrógeno en helio. La masa de un núcleo de hidrógeno es de 1.67262 × 10-27

kg y la

masa de un núcleo de helio es de 6.64642 ×10-27

kg).

e) Estime por cuánto tiempo el Sol calentará a la Tierra, considerando solamente el proceso de

fusión de hidrógeno (Masa del Sol: MS = 2.00 × 1030

kg).

15B-Física Moderna. Soluciones.

15.1-La luz roja del hidrógeno corresponde a la transición más pequeña de la serie Balmer, desde

el nivel n = 3 al nivel m = 2. Las energías de los niveles permitidos en el hidrógeno satisfacen:

EH = −13.6 (eV)/n2 con n = 1, 2, 3, …

La energía del fotón de luz emitido en la transición del nivel al m al n satisface: ΔEfot = −ΔEH.

Pero la energía de un fotón de frecuencia f está dada por ΔEfot = hf = hc/λ, donde h es la cons-

tante de Planck, h = 6.626×10-34

Js y c = 2.998×108 m/s, es la velocidad de la luz en el vacío.

Entonces:

hc/λ = −ΔEH = 13.6(1/m2 – 1/n

2) = 13.6(1/2

2 – 1/3

2) = 1.888 eV = 3.022×10

-19 J

Y: λ = hc/3.022×10-19

= 657 nm

Ésta es la longitud de onda roja del hidrógeno emitida por átomos casi en reposo en el laborato-

rio. Pero la longitud de onda emitida por el hidrógeno en la galaxia lejana tienen una longitud de

onda λ’ que satisface la condición del primer máximo de interferencia (m = 1):

dsenθ1 = (1) λ’, con d = 1/600 = 1.67×10-3

mm = 1.67×103 nm por lo que:

λ’ =1.67×103×sen25.3 = 712 nm

Esta es una longitud de onda mayor que la observada en la muestra de hidrógeno en reposo, por

lo que la galaxia se aleja: es un corrimiento al rojo.

Por efecto Doppler relativista, para el corrimiento al rojo:

λ’ = λ [(1 + V/c) / (1 − V/c)]1/2

Despejando:

V = c[(1 − λ2/λ’

2) / (1 + λ

2/λ’

2)] = c[(1 – 657

2/712

2) / (1 + 657

2/712

2) = 0.0802 c

Page 140: problemas selectos

140

15.2-a) La energía cinética ganada por el electrón bajo el voltaje aplicado es:

K = qV = 0.200MeV

que es comparable con su energía de reposo: Eo = 0.511 MeV, por lo que se requiere un análisis

relativista de energías.

Tenemos: K = E – Eo, por lo que E = K + Eo = 0.711 MeV.

Pero E = mc2 / , o E = Eo / . Entonces:

v = c(1 – Eo2/E

2 )

1/2 = c [1 – (0.511/0.711)

2 ]

1/2 = 0.695 c

Con esa velocidad constante recorre d = 8.00 m en: t = d/v =3.84×10-8

s.

b) La distancia recorrida en el sistema del electrón en reposo es la de 8.00 m contraída:

d’= d = 8.00× = 5.75 m

15.3-La Fig. 15.3S muestra la situación descrita, que podemos representar por la ecuación:

e+ + e

− → γ1 + γ2

Aplicando las leyes de conservación:

Energía: E + mc2 = E1 + E2 (1)

Momentum en eje x: p = (E2 /c) cos υ (2)

Momentum en eje y: 0 = (E1 /c) − (E2 /c) sen υ (3)

donde el momentum del positrón incidente satisface: p =(1/c) (E2 – m

2c

4)1/2

y su energía E = mc2 + K1 = 0.511 + 0.739 = 1.250 MeV,

e+ : K =0.739 MeV

e−

90º υ

fotón: E2

fotón: E1

x

y

Fig. 15.3S

Page 141: problemas selectos

141

por lo que p = 1.141 MeV/c. Entonces:

1.761 = E1 + E2 (1’)

1.141/c = (E2 /c) cos υ (2’)

0 = (E1 /c) − (E2 /c) sen υ (3’)

De (2’) y (3’): cos2υ + sen

2υ = (1.301 /E2

2)+ (E1

2 / E2

2) = 1,

de donde resulta: 1.301 = E22 – E1

2 (4’)

Combinando (4’) y (1’): E2 = 1.250 MeV y E1 = 0.511 MeV.

Sustituyendo en (2’): υ = 24.1º

15.4-Aplicando leyes de conservación al choque:

(mc2 + 4 mc

2) + (2 mc

2) = Mc

2 / (1)

Y: p + 0 = Mu/ (2)

donde M es la masa que deseamos conocer y u, su velocidad

Pero el momentum inicial en (2) se puede escribir como:

p = (1/c) (E2 – m

2c

4)1/2

= (1/c) (25m2c

4 – m

2c

4)1/2

= mc

Las ecuaciones (1) y (2) quedan así:

7mc2 = Mc

2 / (1 – u

2/c

2)1/2

(1’)

Y: mc = Mu/ (1 – u2/c

2)1/2

(2’)

Dividiendo (2) por (1): u = ( / 7) c = 0.700 c

Sustituyendo en (1’): M = 5m

15.5-La Fig. 15.5S de la siguiente página describe el choque analizado.

Se aplican las leyes de conservación del momentum lineal y de la energía. Llamaremos p al

momentum del fotón incidente y p’ al del fotón dispersado; y por E y E’ sus correspondientes

energías. El momentum del electrón será pe y su energía cinética será K. Entonces:

Page 142: problemas selectos

142

p2 – 2pp’cosθ + p’

2 = pe

2 (4)

La ecuación (3) puede arreglarse teniendo en cuenta que para el fotón E = hf = hc/λ:

hc/λ = hc/λ’ + K por lo que: c(h/λ – h/λ’ ) = K

O sea: c(p – p’ ) = K (5)

Por el invariante relativista del momentum y la energía se tiene: E2 = p

2c

2 +m

2c

4, por lo que para

el electrón:

(mc2 + K)

2 = pe

2c

2 + m

2c

4

Desarrollando: 2mc2K + K

2 = pe

2c

2 , por lo que pe

2 = 2mK + K

2/c

2

Sustituyendo aquí la expresión de K dada en (5):

pe2 = 2mc(p – p’ )

+ (p – p’ )

2

Y teniendo en cuenta (4): p2 – 2pp’cosθ + p’

2 = 2mc(p – p’ )

+ (p – p’ )

2

Desarrollando se llega a: (1 – cosθ) = mc(1/p’ – 1/p )

Pero para el fotón: p = h/λ

Por tanto: λ’ – λ = (h/mc) (1 – cosθ)

que es el llamado “corrimiento Compton” de la longitud de onda del fotón dispersado.

15.6-a) Aplicando leyes de conservación al choque:

E + mc2 = Mc

2 / (1)

Y: E/c = Mu/ (2)

donde M es la nueva masa en reposo de la partícula tras absorber el fotón y u, su velocidad.

Dividiendo (2) por (1) y despejando u: u = cE / (E + mc2)

Sustituyendo en (1) y despejando M: M = m (1 + 2E/mc2)1/2

m v

λ λ’

θ

θ

p = p’ cosθ + pe cosθ (1)

0 = p’ senθ − pe senθ (2)

E = E’ + K (3)

Arreglando (1) y (2):

p – p’cosθ = pe cosθ y p’senθ = pe senθ

Elevando al cuadrado y sumando miembro a miembro:

Fig. 15.5S

Page 143: problemas selectos

143

b) Con v = 0.80c: γv = 1/ = 5/3

Las leyes de conservación se expresan ahora así:

Energía: γv mc2 + mc

2 = γu Mc

2 (con γu = 1/ ) (3)

Momentum: γv m(0.80c) + 0 = γu Mu (4)

Despejando: u = c/2 y M = (4/ ) m

15.7-a) La desintegración es de la forma: πo → γ1 + γ2

Llamaremos M a la masa en reposo del pión, y E1 y E2 a las energías de los dos fotones.

Las leyes de conservación permiten plantear:

Energía: Mc2/ = E1 + E2 (1)

Momentum: Mv/ = E1/c − E2/c (2)

La energía cinética es K = Mc2

[1/ − 1] = 1.00 GeV.

Y la velocidad del pion era: v = c [1 – (1 + K/Mc2)-2

]1/2

= 0.9929c

Sustituyendo los valores de M y v en (1) y (2) obtenemos:

1134.95 MeV = E1 + E2 (1’)

Y: 1126.89 MeV/c = E1/c − E2/c (2’)

Resolviendo: E1 = 1130 MeV y E2 = 4.0 MeV

b) Si los fotones se emiten simétricamente respecto a la dirección original, llevan iguales mo-

menta y energías y forman el mismo ángulo υ a cada lado de la dirección original. Entonces, las

ecuaciones de conservación quedan en la forma:

Energía: Mc2/ = 2E (3)

Momentum: Mv/ = 2(E/c) cos υ (4)

Dividiendo (4) por (3): cos υ = v/c = 0.9929, por lo que υ = 6.83º y el ángulo entre los dos foto-

nes será el doble: 13.7º.

Page 144: problemas selectos

144

15.8-a)La masa transformada en energía en una sola reacción es:

Δm= m(235

U) – m(140

Ce) – m(94

Zr) – m(1n) = 0.1435 u =

La energía liberada en una reacción es entonces: ΔE = c2Δm = 0.1435×931.5 = 133.7 MeV.

b)En 57 kg de uranio hay 57000/235 = 242.6 moles. En cada mol hay 6.022×1023

núcleos de ura-

nio, por lo que en nuestra muestra hay 242.6×6.022×1023

= 1.46×1026

núcleos. De estos solamen-

te reaccionan 1.46×1025

(el 10%). Y en cada una de estas reacciones se libera la energía calcula-

da antes. El total de energía liberada es: 1.46×1025

×133.7 = 1.95×1027

MeV.

c)La cantidad de kilotones equivalentes es: 1.95×1027

/( 2.6×1025

) = 75 kT.

15.9-a) La fuerza que ejerce la luz sobre la Tierra es igual a la variación de momentum que sufre

la luz por unidad de tiempo al ser absorbida por el disco terrestre (perpendicular a los rayos sola-

res), F = Δp/Δt, donde Δp = ΔE/c = ΦAΔt/c = ΦπRT2Δt /c. Entonces: F = ΦπRT

2 /c ≈ 59 × 10

7 N.

Esta es una fuerza equivalente al peso de unas 59 000 toneladas métricas, menor que el peso de

un portaviones. No es una fuerza muy grande para la Tierra.

b) La energía que absorbe la Tierra en un tiempo Δt es ΔE = ΦA Δt. Por unidad de tiempo es:

ΔE/Δt = ΦA. Como ΔE = c2Δm, entonces: c

2Δm/Δt = ΦA.

De aquí: Δm/Δt = ΦA /c2 ≈ 2.0 kg/s

es la masa convertida por segundo en el Sol dirigida a la Tierra.

c) Conocemos el flujo solar Φ a la distancia de la Tierra. Como el Sol irradia por igual en todas

direcciones (al menos en promedio), la energía total que alcanza ese flujo sobre una esfera

imaginaria de radio d (distancia Tierra-Sol) será: ΔE = Φ4πd2Δt. Entonces, la masa total radiada

por el Sol por unidad de tiempo es:

Δm/Δt = Φ4πd2/c

2= 4.3×10

9 kg/s.

d) Cada cuatro átomos de hidrógeno que se convierten en helio pierden una masa en reposo de:

Δm = (4×1.67262 – 6.64642)×10-27

= 4.406×10-29

kg. Si dividimos la masa total consumida por

segundo en el Sol, 4.3 × 109 kg/s, por la masa consumida en una reacción de fusión nuclear,

4.406×0-29

kg, nos dará el número de reacciones nucleares ocurriendo en el Sol por segundo:

ΔN/Δt =(4.3 × 109 )/ 4.406×10

-29 ) = 3.9×10

38 reacc/s. Cada una de estas reacciones involucra la

desaparición de 4 átomos de hidrógeno, por lo que se consumen 4×3.9×1038

(átomos H)/s. La

masa de cada hidrógeno es de 1.67×10-27

kg, por lo que la masa de hidrógeno consumida por

segundo es:

ΔM = 4×3.9×1038

× 1.67×10-27

= 2.6×1012

kg/s.

Page 145: problemas selectos

145

e) La masa del Sol es 2.0×1030

kg, mayoritariamente de hidrógeno. Y si el hidrógeno se consu-

me a razón de 2.6×1012

kg/s, entonces, atendiendo solamente a este proceso de producción de

energía en el Sol, éste puede mantenerse un tiempo de:

(2.0×1030

) / (2.6×1012

) ≈ 7.7×1017

s ≈ 2×1010

años

El tiempo de vida promedio para estrellas como nuestro Sol se estima en 1010

años.

Page 146: problemas selectos

146

SEGUNDA PARTE:

FISICA INTEGRADA

En esta parte, los problemas de Física están agrupados por el tipo de habilidad de razonamiento

que se requiere para resolverlo. Cada problema puede integrar varias partes de la Física.

Las secciones de esta parte son: 16-Problemas con Resortes y Algo Más; (8 problemas); 17-Pro-

blemas con Análisis y Cálculos Gráficos (13); 18-Problemas con Análisis de Simetría (10); 19-

Problemas con Cálculo Diferencial o Integral. (20)

En la sección de Resortes se ligan estos con situaciones termodinámicas y electromagnéticas, y

algunos pueden requerir de Cálculo Diferencial e Integral (se avisa al inicio de cada problema).

Los de la sección de Simetría requieren analizar el tipo de simetría presente en el problema para

simplificar su planteamiento matemático. Los de la sección de Gráficos requieren el análisis

teórico de gráficos y el uso de las cantidades numéricas que se obtienen a partir de ellos para

poder solucionar problemas (incluyendo la solución de ecuaciones trascendentes, de solución

gráfica). Y los de la sección de Cálculo, como lo indica el título, requieren de las habilidades de

derivación e integración, y sus significados analíticos.

A estas dificultades matemáticas deberá añadir los razonamientos físicos necesarios.

Los problemas de esta parte pueden ser útiles para un nivel más avanzado de preparación para

Olimpiadas en los grados 11 y 12, y para estudiantes de College que aspiren a un dominio de

conceptos y leyes de la Física superior al mínimo requerido en muchos cursos de Física de

College.

Page 147: problemas selectos

147

Page 148: problemas selectos

148

16A-Problemas con Resortes y Algo Más.

16.1-Un recipiente cilíndrico térmicamente aislado está dividido en dos partes por un pistón no

conductor, el cual puede desplazarse por el cilindro, sin fricción con sus paredes. En la parte

izquierda del recipiente está contenido n = 1 mol de gas ideal monoatómico, y en la derecha hay

vacío. El pistón está unido a la pared derecha del recipiente por medio de un resorte cuya

longitud en estado libre es igual a la longitud interior del recipiente (Fig. 16.1). Determine la

capacidad calorífica C del gas, despreciando la capacidad calorífica del recipiente, del pistón y

del resorte. Dato: R = 8.31 J/(mol K).

Cuando el pistón se libera, el volumen ocupado por el gas aumenta al doble. ¿Cómo cambian la

temperatura del gas y su presión? Las capacidades caloríficas del cilindro, el pistón y el resorte

se desprecian.

16.3-Se tiene una máquina térmica compuesta de un cilindro lleno de gas, con un pistón que

puede moverse entre los topes AA y BB, y un resorte acoplado (Fig. 16.3, próxima página). El

gas se calienta lentamente mientras el pistón se desplaza desde AA hasta BB, después de lo cual

la base del resorte se corre desde CC hasta DD. Después el recipiente se enfría lentamente hasta

que el pistón alcanza el tope AA, y en ese momento se corre la base del resorte otra vez hasta

CC. A continuación se calienta de nuevo el cilindro y se repite el ciclo. Encuentre el coeficiente

de eficiencia térmica de este ciclo termodinámico. El cilindro está lleno de helio; el área del

pistón es S = 10 cm2; la constante del resorte es k = 10 N/m y su longitud en estado no

Fig.16.2

16.2-En un cilindro dispuesto horizontalmen-

te (Fig.16.2), a la izquierda del pistón se en-

cuentra 1.0 mol de gas ideal. En la parte de-

recha del pistón hay vacío y un resorte colo-

cado entre el pistón y la pared del cilindro se

encuentra en estado no deformado. El cilin-

dro está térmicamente aislado del medio.

resorte

Fig.16.1

Page 149: problemas selectos

149

deformado es lo = 60 cm. La presión externa tómela como nula (vacío externo). Las dimensiones

de la figura considérelas en centímetros para el eje x.

16.4-Una de las placas de un capacitor plano de área S está suspendida de un resorte y la otra es

mantenida inmóvil (Fig.16.4). La distancia entre las placas en el momento inicial es lo. El capa-

citor es conectado a una batería durante un tiempo muy corto y se carga hasta una tensión V.

¿Cuál debe ser el valor de la constante k del resorte para que no se produzca contacto entre las

placas como resultado de su atracción después de cargadas?

distancia de equilibrio entre las placas si el interruptor se cerrara solamente por un breve

intervalo de tiempo, suficiente para cargar las placas, pero sin que éstas tengan tiempo para

moverse durante la carga?

16.6-(Requiere Cálculo) Una esfera de masa m está colgada de un resorte de constante k. El

punto de suspensión del resorte realiza oscilaciones armónicas simples verticales de amplitud A y

frecuencia ω. ¿Cómo se mueve la esfera en el régimen de oscilaciones estacionarias?

16.7-(Requiere Cálculo) Una varilla conductora de masa m y longitud l está colgada de dos

soportes aislantes en sus extremos, con ayuda de dos resortes idénticos de constantes elásticas k.

A B

d

resorte

pared

pared

K Fig.16.5

16.5-La placa A de un capacitor plano es inmóvil y la B

está unida a una pared por un resorte que le permite mo-

verse permaneciendo paralela a la placa A (Fig. 16.5). Si

el interruptor K se cierra de manera permanente, la placa

B se mueve hasta detenerse en una nueva posición de

equilibrio. En esta situación, la distancia d que hay entre

las placas originalmente (cuando el resorte no estaba es-

tirado) disminuye en un 10%. ¿En cuánto variaría la

x(cm) 0 20 40 60 70

A

A B

B

C

C

D

D

resorte base del

resorte

pistón

Fig.16.3 Fig.16.4

lo

Page 150: problemas selectos

150

16.8-Alrededor de un eje vertical gira una varilla rígida horizontal con velocidad angular

constante ω. Por la varilla pueden deslizarse sin fricción dos esferitas de masas idénticas, m. Las

esferitas están conectadas entre sí por un resorte ligero de constante elástica k, la longitud del

cual es lo en estado no deformado. La esfera que se encuentra más cerca del eje está unida al

mismo por un segundo resorte idéntico al primero (Fig. 16.8). Encuentre la longitud de cada

resorte durante la rotación, sabiendo que cada esferita realiza un movimiento circular. ¿Bajo qué

condiciones es que se logra que las esferitas describan circunferencias? (Considere puntuales las

esferitas)

× × × × × × × ×

× × × × × × × ×

× × × × × × × ×

× × × × × × × ×

× × × × × × × ×

× × × × × × × ×

B

l

C

k k

Halado hacia abajo

Fig. 16.7

Soporte Soporte La varilla se encuentra en un campo magnético

uniforme de inducción B, perpendicular al plano en

que están la varilla y los resortes. Halaron la varilla

en el plano vertical desde su posición de equilibrio y

la soltaron. Determine la frecuencia del movimiento

subsecuente de la varilla en el plano vertical, si a los

extremos superiores de los resortes hay conectado

un capacitor de capacitancia C. Desprecie las resis-

tencias, las inductancias y capacitancias parásitas de

todos los conductores (Fig. 16.7).

Fig. 16.8

Page 151: problemas selectos

151

16B-Problemas con Resortes y Algo Más. Soluciones.

16.1- Ponemos un eje x paralelo al eje del cilindro, con el origen en su extremo izquierdo. La

posición del pistón está dada por x. El calor Q que gana el gas (Fig.16.1S) lo usa para hacer

trabajo W en contra del pistón y para incrementar la energía interna del gas, ΔU:

Q = W + ΔU (1)

El trabajo será igual al incremento de energía potencial elástica del resorte al comprimirse desde

x1 hasta x2:

W = ½ kx22 – ½ kx1

2 (2)

En las posiciones inicial y final se cumple que la fuerza que hace el gas sobre el pistón está en

equilibrio con la fuerza elástica:

PA = kx (3)

donde P es la presión del gas y A el area del pistón. Y como PV = nRT:

P =nRT/V = nRT/ (Ax)

Sustituyendo en (3) y despejando: kx2 = nRT. Y sustituyendo esta expresión en (2):

W = ½ nRΔT (4)

Por otra parte, la variación de energía interna de un gas ideal monoatómica está dada por su

variación de temperatura:

ΔU = (3/2) nR ΔT (5)

Sustituyendo (4) y (5) en (1):

Q = W + ΔU = ½ nRΔT + (3/2) nR ΔT = 2nR ΔT = 2R ΔT, pues n = 1.

La capacidad calorífica de una muestra es: C = Q/ΔT. Entonces: C = 2R =16.62 J/(mol K)

x

O

x

x1 x2

Q

x

W

F = PA Fig.16.1S

Page 152: problemas selectos

152

16.2-Al concluir el proceso de expansión del gas, éste ha realizado un trabajo neto W sobre el

resorte que le proporciona energía potencial elástica en una cantidad ½kx2, donde x es la

distancia recorrida por el pistón. Además, la energía interna del gas habrá cambiado en ΔU =

ncV (T2 – T1). Y como el gas está térmicamente aislado, entonces Q = 0.

Y por la primera ley de la Termodinámica, Q = ΔU +W, tendremos:

0 = ncV (T2 – T1) + ½ kx2 (1)

En el estado final, la presión P2 del gas produce una fuerza P2S sobre el pistón que se equilibra

con la fuerza elástica del resorte contraído una distancia x:

kx = P2S (2)

Por otra parte, el aumento de volumen del gas fue Sx y como por datos el volumen V2 es el doble

del inicial, entonces V2 = 2Sx, y en el estado final se cumplirá:

P2 (2Sx) = nRT2. Despejando P2, sustituyéndola en la ecuación (2) y ordenando: kx2 = ½ nRT2.

Sustituyendo esta expresión en la ecuación (1) y despejando T2/T1 se obtiene:

T2/T1 = 1/(1+ R/4cV).

Para calcular P2/P1 podemos aplicar la ley de los gases ideales a los estados inicial y final:

P1V1 = nRT1 y P2 (2V1) = nRT2.

Dividiendo la segunda ecuación por la primera, miembro a miembro, se obtiene:

P2/P1 = 1/ [2(1 + R/4cV)]

Tomando el origen del eje de las x en el fondo del cilindro, la fuerza elástica será en cada posi-

ción del pistón F = kx = kV/S, donde V es el volumen del gas. Y: P = F/S = kV/S2. Hagamos un

gráfico PvsV del ciclo para orientarnos en el problema. La Fig. 16.3S muestra el ciclo en el plano

P-V.

x(cm) 0 20 40 60 70

A

A B

B

C

C

D

D

resorte base del

resorte pistón

Fig.16.3S

16.3-Con el pistón en AA y la base del

resorte en CC, el resorte está comprimido 20

cm y hace una fuerza sobre el pistón del tipo

F = kx (x1=20 cm) con la que comprime al

gas con una presión P1 = kx1/S. Ésta será la

presión inicial del gas en el ciclo. Como el

movimiento del pistón es lento, la presión del

gas será siempre igual a la que ejerce el pis-

tón sobre él, debido a la fuerza del resorte.

Page 153: problemas selectos

153

F3 = k (70 cm – 40 cm) = (10N/m)(0.30 m) = 3 N

Ahora, al enfriarse el gas su presión disminuye a volumen constante hasta alcanzar el valor de la

presión que hace el resorte con la fuerza F3: P3 = F3/S = 3 kPa, alcanzando el punto 3 del ciclo.

A continuación, al seguir el enfriamiento, la presión del gas baja linealmente en equilibrio con la

presión del resorte, hasta alcanzar V4 = V1 = 2×10-4

m3

(pistón en AA nuevamente). Esta bajada

de presión es ahora conforme a la ley P = k (V – Vo)/S2, según se desprende del gráfico (la recta

3-4 tiene la misma pendiente que la recta 1-2, pero desplazada sobre el eje de las V desde V = 0

hasta Vo= 1×10-4

m3, siendo este último valor el intercepto en el eje de V ). Evaluando para V4:

P4 = 1 kPa. Finalmente, al comprimir de nuevo el resorte hasta CC, el pistón ya no se mueve del

tope AA. La presión del resorte aumenta a su valor inicial (2 kPa), pero la presión del gas sigue

siendo de 1 kPa (los topes AA no permiten al resorte transmitirle su presión al gas). Ahora se

calienta el gas a volumen constante y su presión se eleva lentamente hasta igualarse con la del

resorte, que es el mismo valor inicial de 2 kPa. Y después el ciclo se repite.

La eficiencia de este ciclo será e = Wneto

/Qabs

, donde Wneto

está dado por el área cerrada de la

curva del gráfico PV:

Wneto

= (área de paralelogramo) = (largo)(ancho)senθ (ver Fig. 16.3S)

Pero: (largo)senθ = ΔV, por lo que:

Wneto

= (ΔP)(ΔV) = (1×103) (2×10

-4) = 0.2 J

Y el calor absorbido es: Q = Q1-2+Q4-1 pues son las dos partes del ciclo en que se absorbe calor.

Ahora: Q1-2 =W1-2 + ΔU1-2 = ½(P1+P2) (V2 – V1) + (3/2)nR(T2 – T1) y Q4-1 = (3/2)nR(T1 – T4)

Efectuando la suma: Q = Q1-2+Q4-1 = ½(P1+P2) (V2 – V1) + (3/2)nR(T2 – T4). Pero nRT = PV.

Entonces: Q = Q1-2+Q4-1 = ½(P1+P2)(V2 – V1) + (3/2)(P2V2 – P4 V4) ≈ 2.7 J

La eficiencia es entonces: e = Wneto

/Qabs

= 0.2/2.7 = 0.074. O sea, es del 7.4%.

P(kPa)

4

3

2

1

0 V(×10

-4m

3)

4 3 2 1 0

1

2

3

4

En el punto 1: V1 = Sx1 = 200 cm3 = 2×10

-4 m

3

y P1= kV1/S2 = 2×103

Pa = 2 kPa. .

Al dilatarse el gas, la presión aumenta lineal-

mente de acuerdo con el aumento de la fuerza

elástica, hasta que V = V2 = 4×10-4

m3

y .

P2 = kV2/S2 = 4 kPa.

Ahí cesa el calentamiento. Se estira ahora el re-

sorte hasta DD, pero el tope BB aguanta al pis-

tón y la presión del gas no disminuye, aunque la

fuerza elástica si baja su valor a:

Fig.16.3S

θ largo ancho

Page 154: problemas selectos

154

16.4-Al cargarse las placas adquieren cargas +q y –q que permanecen constantes cuando se

muevan pues la batería se desconecta enseguida. El valor de esta carga será: q = CV = VεoS/lo.

La fuerza de atracción entre las placas es F = qE donde E = q/(2εoS) es el campo debido a una

sola de las placas. Entonces: F = q2/(2εoS) y sustituyendo q: F = V

2εoS/2lo

2. Se ve que para dis-

tancias x pequeñas, esta fuerza de atracción eléctrica no depende de la distancia x que se mueva

la placa móvil (la ecuación de campo usada es válida sólo para distancias pequeñas a las placas).

x1 = mg/k. Al cargarse las placas y correrse la posición de equilibrio desde x1 hasta xo, se pro-

ducirá una oscilación alrededor del nuevo punto de equilibrio, xo, con una amplitud (hacia arriba

y hacia abajo) igual a la distancia:

xo – x1 = V2εoS/(2lo

2k)

Para que no haya contacto entre las dos placas, el recorrido total de la placa móvil, 2(xo – x1), no

debe sobrepasar la distancia original que las separaba, o sea, debe cumplirse: 2(xo – x1) < lo.

Sustituyendo: 2V2εoS / (2lo

2k) < lo.

Y despejando: k >V2εoS / lo

3.

A B

d

resorte

pared

pared

K Fig.16.5

16.5-La diferencia entre las dos situaciones está en que

en el primer caso las placas se cargan bajo una diferen-

cia de potencial E, con la capacitancia que haya cuando

la placa móvil se estabilice (q1 = C1E ), y en el segundo

caso, la carga se adquiere con una diferencia de po-

tencial E, con la capacitancia inicial de las placas a la

distancia d (q2 = C2 E ).

lo

Las fuerzas que sufre la placa móvil son su peso, mg, la del resorte, kx, y la

de atracción eléctrica de la otra placa, V2εoS/2lo

2. En el equilibrio de estas

tres fuerzas:

kxo – mg − V2εoS/2lo

2 = 0,

Por tanto: xo= mg/k + V2εoS/(2lo

2k) y representa la posición de equilibrio de

la placa móvil cargada (ver Fig. 16.4S).

La posición de equilibrio, x1, que tenía esta placa antes de cargarse respon-

día al equilibrio entre la fuerza de gravedad y la elástica: kx1 – mg = 0, con Fig.16.4S

xo

x1

Page 155: problemas selectos

155

Para el primer caso: q1 = C1E = [εoS/(d – x1)]E donde S es el área de cada placa y x1 = 0.10d

es el corrimiento de la placa hasta su nueva posición de equilibrio.

El campo total entre las placas será: E1 = E / (d – x1). La mitad de este campo es la producida

por cada placa separadamente, y para calcular la fuerza que experimenta una placa en su atrac-

ción por la otra necesitamos la carga de una placa, q1, y el campo que experimenta por parte de la

otra. Así, el campo que experimenta la placa móvil E = E1/2 = E / [2(d – x1)]. Entonces, la fuerza

de atracción de una placa sobre la otra es:

F1 = q1E = {[εoS/ (d – x1)] E } {E /[2(d – x1)]} = εoS E 2/ [2(d – x1)

2].

Esta fuerza queda compensada por la del resorte: F = kx1:

kx1 = εoSE 2/ [2(d – x1)

2] (1)

En esta expresión tenemos una relación entre la deformación del resorte, x1, y los parámetros del

sistema para el primer caso. Trataremos ahora de hallar una relación similar en el segundo caso,

con una compresión diferente, x2, y poder establecer la relación entre ambas deformaciones.

En el segundo caso: q2 = C2E donde C2 = εoS/d. Cuando la placa móvil alcance el equilibrio de

fuerzas ya no cambiará esta carga q2 porque la batería ya se desconectó. El campo total en el

equilibrio es ahora E2 = ζ/εo = q2/ (Sεo), y el campo de la placa fija sobre la móvil será E2/2. La

fuerza sobre la placa móvil es ahora: F2= q2E2/2 = q22/(2εoS) = C2

2E 2/(2εoS) .

Sustituyendo C2: F2= εoSE 2/(2d

2).

Esta fuerza de atracción eléctrica estará en equilibrio con la fuerza elástica:

kx2 = εoSE 2/ (2d

2) (2)

Dividiendo miembro a miembro la ecuación (1) por la (2): x1/x2 = d2/ (d – x1)

2

Despejando: x2 = x1(d – x1)2/d

2 con x1 = 0.10d. Así: x2 = 0.08d.

16.6-Si el resorte colgara estáticamente, en equilibrio, se estiraría una longitud Δl = mg/k. En la

Fig.16.6S (próxima página) se representa un momento cualquiera en el que el punto de suspen-

sión se ha desplazado Asenωt y la esfera se ha corrido una distancia x respecto a su posición de

equilibrio estático. En el equilibrio estático, el resorte tendría sus extremos entre los puntos O y

P. El punto O indica el centro de oscilación del punto de suspensión móvil, y lo sería la longitud

del resorte sin deformar (sin peso colgado). Se cumplirá para la esfera:

mg – kΔx = m (d2x/dt

2), donde Δx = mg/k + x + A senωt (representa la deformación total del re-

sorte). Sustituyendo Δx en la ecuación anterior se obtiene:

Page 156: problemas selectos

156

Nos corresponde evaluar la amplitud P y el desfasaje θ. Para ello, derivamos la solución estacio-

naria, xs, dos veces y la sustituimos en la ecuación (1):

dxs/dt = Pω cos(ωt + θ) y d2x/dt

2 = −Pω

2 sen(ωt + θ)

Sustituyendo en (1): −Pω2 sen(ωt + θ) + (k/m) P sen(ωt +θ) = −(kA/m) senωt

Arreglando: P[(k/m) – ω2]sen(ωt +θ) = −(kA/m) senωt.

Como esta ecuación debe cumplirse para todo valor de t esto obliga a que θ = 0 (solamente así

ambos miembros serían cero a la vez, y máximos a la vez). Por otra parte, deberá cumplirse tam-

bién que la amplitud que acompaña al seno sea la misma en ambos miembros:

P[(k/m) – ω2] = −(kA/m), de donde: P = (Ak/m)/( ω

2 – k/m)

Pero k/m = ωo2, como ya dijimos antes. Entonces, la amplitud de la oscilación estacionaria será:

P = Aωo2/(ω

2 – ωo

2) = A/(ω

2/ ωo

2 – 1)

y la oscilación estacionaria quedará descrita por la ecuación: xs = A sen ωt / (ω2/ ωo

2 – 1)

Puede notarse que si la frecuencia de la fuerza externa coincide con la propia del sistema, la

amplitud de oscilación tiende a infinito, que es el fenómeno de la resonancia, sin resistencia

mecánica en el sistema.

16.7-En la Fig. 16.7S (próxima página) se ha dibujado un eje x positivo hacia abajo, con el ori-

gen O en el nivel de la posición de equilibrio de la varilla con los resortes (línea quebrada

horizontal). En esa posición ya los resortes están algo estirados respecto a su longitud natural.

x

Suspensión

móvil

lo+mg/k

Asenωt

(+) m

O

m (d2x/dt

2) = −kA senωt – kx

Arreglando: (d2x/dt

2) + (k/m)x = −(kA/m) senωt (1)

Ésta es la ecuación del oscilador armónico simple “forzado” (está

sometido a una fuerza periódica externa aplicada en su punto de

suspensión). Esta ecuación tiene una solución compuesta de dos

partes: la de la oscilación propia (natural) del oscilador, con fre-

cuencia ωo = (k/m)1/2

, y la de la oscilación estacionaria que impone

la fuerza externa con su frecuencia ω. Las dos oscilaciones se

superponen, pero solamente nos interesa la estacionaria (así nos lo

piden en el problema). Se sabe que la oscilación estacionaria se

realiza con la frecuencia impuesta por la fuerza externa, y con algún

posible desfasaje respecto a la fuerza: xs = Psen (ωt +θ).

P

Fig. 16.6S

Page 157: problemas selectos

157

Y teniendo en cuenta (1): −2kx = ma, de donde resulta d2x/dt

2 + (2k/m) x = 0, que es la ecuación

del oscilador armónico simple con frecuencia ω = (3)

Al poner un campo magnético B en la zona, cuando la varilla suba y baje, hará que el flujo mag-

nético varíe dentro del circuito cerrado (se crea un lazo de área variable en el campo magnético).

La fuerza electromotriz (fem) así generada es E = −Blv, de acuerdo con la ley de Faraday. Esta

fem es el voltaje aplicado al capacitor, por lo que la carga q en el mismo, en cada instante, será:

q = CV = CE = −CvBl

Como la velocidad es variable durante la oscilación, la carga en el capacitor lo será también. La

corriente en el circuito será justamente la variación de carga en el capacitor por unidad de tiem-

po: I = dq/dt = −CBl dv/dt = −CBl a

donde a es la aceleración de la varilla. La fuerza magnética sobre la varilla en cada momento

será:

Fmagn = BIl = −CB2l2a

La fuerza total sobre la varilla bajo el campo magnético cambia ahora de la expresión (2) a:

ma = mg – 2k(Δlo + x) − CB2l2a

Teniendo en cuenta de nuevo la ecuación (1) y reagrupando términos, obtenemos:

d2x/dt

2 + [ 2k / (m + CB

2l2 )] x = 0

lo cual corresponde a una oscilación armónica de frecuencia ω = (4)

× × × × × × × ×

× × × × × × × ×

× × × × × × × ×

× × × × × × × ×

× × × × × × × ×

× × × × × × × ×

B

l

C

k k

Halado hacia abajo

Fig. 16.7S

El peso mg de la varilla se compensa con la fuerza

elástica total de los dos resortes, 2k Δlo, donde Δlo

es el estiramiento en esas condiciones:

mg = 2k Δlo (1)

Por lo que Δlo = mg / (2k)

Sin campo aplicado B el resorte oscilaría de tal

modo que en cada posición de la varilla se cum-

pliría mg – 2k Δl = ma, con Δl = Δlo + x, y x esta-

ría medida desde la posición de equilibrio O. Así:

mg – 2k (Δlo + x) = ma (2)

x

O

Soporte Soporte

Δlo

Page 158: problemas selectos

158

La varilla realizará una oscilación de frecuencia menor que la oscilación libre, según se aprecia

de comparar (3) y (4). La fuerza magnética que aparece aumenta la inercia efectiva del sistema,

de m a (m + CB2l2).

16.8-Sea l1 la longitud del resorte que une con el eje a la esfera más cercana, y l2, la longitud del

resorte que une a las dos esferas. Si las esferas realizan circunferencias, se cumple para cada una

la segunda ley de Newton (para fuerzas centrípetas):

mω2l1 = k(l1 – lo) − k(l2 – lo) y mω

2(l2 + l1) = k(l2 – lo)

De aquí: l1 = lo / [1 – 3mω2/k + (mω

2/k)

2] y l2 = (1 − mω

2/k) lo / [1 – 3mω

2/k + (mω

2/k)

2]

Estas soluciones poseen sentido físico solamente si:

1 – 3mω2/k + (mω

2/k)

2 > 0 (1) y (1 − mω

2/k) ≥ 0 (2)

Llamemos: x = mω2/k (x > 0). De (2): 1 – x ≥ 0, o sea: 1 ≥ x > 0 (3)

De (1): x2 – 3x + 1 > 0, de donde resultan: x > (3 + )/2 y x < (3 − )/2

Y teniendo en cuenta (3) solamente es posible: 0 < x < (3 − )/2.

O sea: 0 < mω2/k < (3 − )/2. Significa que la condición para que las esferitas realicen circun-

ferencias es ω < [(k/m) (3 − )/2]1/2

Page 159: problemas selectos

159

17A-Problemas con Análisis y Cálculos Gráficos.

17.1-La Fig. 17.1 muestra dos gráficos, el primero de posición contra tiempo y el segundo de

velocidad contra tiempo. En cada gráfico aparecen las curvas correspondientes a dos bicicletas,

A y B en el gráfico 1 y C y D en el gráficoa 2.

a)¿Se puede decir en qué instante se encuentran las bicicletas A y B en el primer gráfico? ¿Se

puede decir en qué instante se encuentran las bicicletas C y D en el segundo gráfico? Si es

posible, diga en cada caso en qué instante es el encuentro.

b)¿Cuál tiene mayor rapidez en el instante t = 10 s en el primer gráfico? ¿Y en el segundo?

c)¿Cuál tiene mayor aceleración, en valor absoluto, en t = 10 s en el primer gráfico? ¿Y en el

segundo?

d)¿Cuál recorrió la mayor distancia entre t = 0 y t = 10 s en el primer gráfico? ¿Y en el segundo?

e)En t = 10 s, ¿A y B viajan en direcciones opuestas o en la misma dirección? ¿Y C y D?

Fig. 17.2

M m m

1 2 3

17.2-Una bola de masa M incide desde la izquierda

sobre un sistema que consiste de dos bolas de masa

m cada una, en reposo sobre una superficie horizon-

tal lisa y unidas por un resorte ligero (Fig. 17.2). El

choque es frontal y absolutamente elástico. Encuen-

tre aproximadamente la relación de masas m/M = γ

bajo la cual el choque ocurre al menos una vez más.

x(m) v(m/s)

t(s) t(s) 10 10 5 5 0 0

B

A C

D

[ 1 ] [ 2 ] Fig. 17.1

Page 160: problemas selectos

160

17.3-Un pistón móvil divide un cilindro en dos partes de iguales volúmenes, Vo =1.0 ×10-3

m3. En

una parte hay aire seco y en la otra, vapor de agua con una masa de 4.0 g de agua líquida. Se ca-

lienta lentamente el cilindro y el pistón comienza a moverse. Después de un corrimiento equiva-

lente a la cuarta parte de la longitud del cilindro, el movimiento del pistón cesa. ¿Cuánta masa de

aire, Mo, y de vapor de agua, M, había en cada parte del recipiente antes del calentamiento?

¿Cuál era la temperatura inicial To y a qué temperatura T1 dejó de moverse el pistón? (Re-

presentaremos por t a la temperatura en grados Celsius; represente por T a la temperatura en

kelvin). La masa molar del aire seco es de unos 29 g/mol y la del agua es 18 g. La dependencia

de la presión del vapor saturado Ps(t) del agua con la temperatura está dada en la siguiente tabla:

t (oC) 100 120 133 152 180

Ps(kPa) 100 200 300 500 1000

17.4-En un vaso con agua introdujeron un calentador eléctrico y midieron la dependencia de la

temperatura T del agua con el tiempo t, registrándose la siguiente la tabla:

t(min) 0 1 2 3 4 5 6 7 8 9 10

T(oC) 20.0 26.2 31.8 36.8 41.4 45.6 49.3 52.7 55.8 58.5 61.1

(a)¿En cuántos grados centígrados se enfría el agua en 1 min si el calentador se desconecta a una

temperatura del agua de 50oC? (b)¿Comenzará a hervir el agua si el calentador no se desconecta

durante un tiempo suficientemente largo? Considere que la potencia de calentamiento no varía.

17.5-El extremo inferior de un tubo capilar de radio r = 0.20 mm y longitud l = 8.0 cm se

sumerge en agua a una temperatura constante de Ti = 0oC. La temperatura de su extremo

superior es de Ts = 100oC. ¿A qué altura h se eleva el agua en el capilar? Considere que la

conductividad térmica del capilar supera en mucho a la conductividad térmica del agua. El

intercambio de calor con el aire circundante se desprecia. El agua moja bien el capilar. Indica-

ción: utilice la siguiente dependencia de la temperatura y la tensión superficial del agua:

T (oC) 0 20 50 90

ζ (×10-3

N/m) 76 73 67 60

Page 161: problemas selectos

161

17.8-Los géiseres pueden considerarse como grandes reservóreos subterráneos llenos de agua,

calentada por el calor de la Tierra (Fig.17.8a, próxima página) La salida desde ellos a la super-

ficie terrestre se realiza por un estrecho canal, el cual, en períodos de calma, está completamente

lleno de agua. Considerando que el período activo se presenta cuando en el reservóreo comienza

a hervir el agua, y que en el momento de erupción el canal está lleno solamente de agua que es

expulsada al exterior, valore qué parte del agua pierde el reservóreo del géiser durante el tiempo

de una erupción. La profundidad del canal es h = 90 m; el calor de vaporización del agua es Lv =

2.26×106 J/kg. La dependencia de la presión del vapor saturado con la temperatura está dada por

el gráfico de la Fig.17.8b (próxima página).

t(min)

T(oC)

10 20 30

20

40

60

80

00

100

40 50

Fig. 17.7

17.7-En una cacerola no muy grande, fina y metálica,

echaron 0.50 litros de agua y la colocaron sobre una

hornilla. Midiendo la temperatura del agua en dife-

rentes instantes construyeron el gráfico de dependen-

cia de la temperatura con el tiempo. Después botaron

el agua y en el mismo recipiente echaron 0.70 litros

de alcohol. De nuevo colocaron la cacerola en la

misma hornilla y repitieron las mediciones de tempe-

ratura y tiempo e hicieron el gráfico de temperatura

contra tiempo para el alcohol. Ambos gráficos se

representan en la Fig. 17.7. Determine mediante estos

gráficos el calor específico del alcohol y su calor de

ebullición si en 23 minutos de ebullición la cantidad

de alcohol en la cacerola disminuyó a la mitad. La

capacidad calorífica de la cacerola es de 200 J/K. La

evaporación desde la superficie de los líquidos es

despreciable. (Densidad del alcohol: 0.80 g/cm3)

t(min)

T(oC)

20 40 60

1

2

3

17.6-En un balde se encuentra una mezcla de agua y hielo de

masa m = 10 kg. El balde lo llevaron a una habitación y co-

menzó a cambiar la temperatura de la mezcla. La dependencia

de la temperatura con el tiempo está representada en la Fig.

10.6. El calor específico del agua es igual a ca = 4.2 kJ/(kg.K)

y el calor de fusión del hielo es Lf = 340 kJ/kg. Determine la

masa de hielo en el balde cuando se llevó para la habitación.

La conductividad del balde se desprecia.

Fig.17.6

Page 162: problemas selectos

162

c) ¿Se quemará alguna de las lámparas si se cambia el resistor anterior por una tercera lámpara

igual a las dos primeras?

4.0 3.0 2.0 1.0

0.10

0.20

0.30 I(A)

V(V)

Relación I-V para la lámpara

(Resistencia no lineal)

Fig. 17.10

17.10-El voltaje de trabajo de una lámpa-

ra, cuya característica voltampérica está

dada en la Fig. 17.10, es igual a 3.5 V. La

corriente se corta a un voltaje de 4.0 V,

pues la lámpara se quema.

a) Dos lámparas como ésta se unen en se-

rie y se conectan a una fuente de voltaje

que provee 5.0 V. ¿Qué corriente circula

por las lámparas?

b) ¿Qué resistencia es necesario conectar

en paralelo a una de las lámparas para que

el voltaje en la otra sea de 3.5 V?

V (V) 80 60 40 20

I(A)

0.2

0.4

0.6

0.8 17.9-¿Podrían dos lámparas incandescentes

de 60 W y 100 W, calculadas para trabajar a

110 V, conectarse en serie a 220 V de ten-

sión, sabiendo que la sobrecarga admisible

en cada lámpara no puede ser mayor que el

10% del valor nominal? La característica

voltampérica de la lámpara de 100 W se

muestra en la Fig. 17.9.

Fig.17.9

P(MPa)

140

0.8

1.6

180 T(oC)

0.4

1.2

0 100 220

90 m

Fig.17.8b Fig. 17.8a

Page 163: problemas selectos

163

17.12-En la Fig. 17.12a se muestra la característica voltampérica del bombillo L de una linterna

de bolsillo. El bombillo se conecta al circuito mostrado en la Fig. 17.12b. (a)Encuentre gráfica-

mente la corriente en la lámpara L. (b)¿Pará qué posición del punto móvil del potenciómetro P,

el voltaje entre los puntos A y B será nulo? (O sea, ¿cómo debe quedar dividida la resistencia de

40 Ω por el contacto móvil para que el voltaje entre los puntos A y B sea nulo?). (c)¿Para qué

posición del punto móvil del potenciómetro el voltaje entre los puntos A y B prácticamente no

cambia bajo pequeñas fluctuaciones de la fem de la batería?

17.13-Para obtener un voltaje que dependiera poco de la temperatura se montó el circuito

mostrado en la Fig.17.13a (próxima página). La característica voltampérica del diodo D y su

dependencia con la temperatura del medio circundante se muestra en la Fig.17.13b para T1=

125oC, T2= 25

oC y T3= −60

oC. El voltaje de la fuente es V =6.0 V para una temperatura de 25

oC,

y con el aumento de temperatura crece linealmente. El coeficiente de cambio del voltaje con la

temperatura es igual a 25×10-3

V/ oC.

Encuentre el voltaje entre los terminales A y B para T =25oC y la dependencia de este voltaje

con la temperatura.

I(A)

V(V) 1 2 3 4 5 0 0

0.1

0.2

0.3

0.4

Fig.17.12a Fig.17.12b

L

A B

P

R= 40Ω E= 4 V

r= 10Ω

I(A)

10 Vl (V)

17.11-En el gráfico de la Fig. 17.11 se presenta la

dependencia de la intensidad de corriente que pasa

por una lámpara con la tensión Vl aplicada a ella. La

lámpara se conecta a una fuente de voltaje constante

de 10 V, en serie con un resistor de 4.0 Ω. Deter-

mine la potencia de la lámpara.

Fig. 17.11

5

1.0

2.0

Page 164: problemas selectos

164

17A-Problemas con Análisis y Cálculos Gráficos. Soluciones.

17.1-a)En el gráfico 1, en t = 10 s están en la misma posición. En el gráfico 2 se puede decir

solamente que tienen la misma velocidad en t = 10 s, pero se desconoce dónde están las bici-

cletas en cualquier instante.

b)En 1, la inclinación de la recta A es mayor, por lo que esa bicicleta tiene mayor rapidez en t =

10 s. En el gráfico 2, ambas bicicletas tienen la misma velocidad en t = 10 s, por lo que tienen la

misma rapidez.

c)En el primer gráfico las bicicletas tiene ambas velocidades constantes, o sea, no tienen acele-

ración, y en el segundo, C tiene mayor aceleración, pues su pendiente es mayor.

x(m) v(m/s)

t(s) t(s) 10 10 5 5 0 0

B

A C

D

[ 1 ] [ 2 ]

Fig. 17.1

A

B

R1=5.0 kΩ

R2=0.2 kΩ

Vd(V)

I(mA)

0.4 0.8 1.2

0.5

1.0

1.5

T1 T2 T3

D

V

Fig. 17.13a

Fig. 17.13b

Page 165: problemas selectos

165

d)En el primer grafico, en el intervalo de tiempo considerado, el desplazamiento de B es mayor

que el de A (se “lee” el Δx sobre el eje x para cada bicicleta), y como son movimientos sin retro-

cesos para cada uno, la distancia recorrida por B es mayor que la de A. En el segundo gráfico, la

rapidez de la bicicleta D desde t = 0 hasta t = 10 s es mayor que la rapidez de C, por lo que la

distancia recorrida por D es siempre mayor que la recorrida por C en cualquier subintervalo de

tiempo entre t = 0 y t = 10 s, y así será para el intervalo completo. (Note que aquí se pide

comparar distancias recorridas, no desplazamientos, que estarían dados por las áreas).

e)En el gráfico 1, todo el tiempo van en direcciones opuestas, pues las pendientes (velocidades)

tienen signos opuestos. En el 2, van en la misma dirección en t = 10 s, pues ambas tienen

velocidades positivas (entre t = 0 y t = 5 s iban en direcciones opuestas).

17.2-La solución será solamente aproximada, como se pide en el enunciado.

Para que haya un segundo choque deberá cumplirse que M > m, pues de lo contrario la bola 1

rebotaría en el choque y jamás alcanzaría a la segunda en otro choque. O sea, m/M < 1, pero

debemos precisar mejor hasta cuánto debe disminuir el cociente m/M para asegurar otro choque.

Después de chocar 1 con 2, la 1 sigue con una velocidad constante y el centro de masas (CM) de

2 y 3 se mueve con otra velocidad constante, mientras 2 y 3 realizan oscilaciones armónicas

respecto al CM. Es necesario encontrar la ley de movimiento de las esferas 1 y 2 después del

choque y ver bajo qué condiciones vuelven a coincidir (un nuevo choque). Supondremos ins-

tantáneo el choque (equivale a asumir que el tiempo de choque es mucho menor que el período

de oscilación de las esferas). Bajo esta suposición se puede analizar el choque entre 1 y 2 como

un choque entre dos esferas libres, para las que aplican las leyes de conservación del momentum

y de la energía mecánica:

Mvo = Mv1 + mv2 (1) y ½ Mvo2 = ½ Mv1

2 + ½ mv2

2 (2)

De (1) y (2): v1 = v0 (M – m) / (M + m) y v2 = 2vo / (M + m)

O llamando γ = m/M: v1 = v0 (1 – γ) / (1 + γ) y v2 = 2vo / (1 + γ) (3)

Asumiendo que el choque ocurrió en x1 = x2 = 0, la coordenada x1 de la bola 1 (que continúa

libremente) será después del choque:

x1 = v1t = v0 t (1 – γ) / (1 + γ) (4)

Por su parte, el CM del sistema de las bolas 2 y 3 se moverá con velocidad:

vcm = v2/2 = vo/(1 + γ)

La coordenada del CM después del choque será:

Page 166: problemas selectos

166

xcm = xcm,0 + v2t = xcm,0 + vot /(1 + γ) (5)

Respecto al CM de 2 y 3, tomado como sistema de referencia inercial, la bola 2 se movería según

una ley:

x2’= A sen ωt - xcm,0

donde ω = (k/m)1/2

, con k como la constante de medio resorte (desde la bola 2 hasta el centro del

resorte completo). La amplitud A de la oscilación satisfará:

½ kA2 = ½ mv2’

2 = ½ m(v2 – vcm)

2 = ½ m(v2 – v2/2)

2.

Por tanto: ½kA2 = ½ m(v2/2)

2 = ½ mvo

2/(1 + γ)

2 , donde se tuvo en cuenta (3). Entonces:

A = [(m/k) vo2/ (1 + γ)

2]

1/2 = vo/ [ω (1 + γ)] (6)

La ley de movimiento de la bola 2 es entonces: x2 = xcm + x2’= (xcm,0 + vcmt) + (A sen ωt – xcm,0).

De aquí: x2 = vot / (1 + γ) + vo/ [ω (1 + γ)] senωt (7)

Si las bolas chocan de nuevo se cumplirá: x1 = x2, y teniendo en cuenta (4) y (7):

v0 t (1 – γ) / (1 + γ) = vot / (1 + γ) + vo/ [ω (1 + γ)] senωt

Arreglando algebraicamente: sen ωt = −γωt (8)

Llamando θ = ωt nos queda: senθ = −γθ. Si llamamos y1 = sen θ y y2 = −γ θ, la ecuación (8)

Del gráfico 17.2S se ve que esto ocurre solamente si la recta se inclina como máximo hasta pasar

muy cerca del mínimo de la sinusoide, donde θ = 3π/2. Si la pendiente de la recta se hace más

vertical, no habrá otra intersección entre la sinusoide y la recta, por lo que no sucederá de nuevo

que x1 = x2, y no habrá otro choque. Entonces, como máximo debe cumplirse, aproximadamente:

sen(3π/2) = −γ3π/2

de donde resulta γ ≈ 0.21 como valor tope posible.

Así pues, ocurrirá al menos un choque más si la relación de masas entre las bolas satisface:

m/M < 0.2

y

θ

y2= -γz

π 3π/2

y2*= -γz

corresponde gráficamente a la intersec-

ción entre la sinusoide y1 y la recta y2,

como muestra el gráfico de la Fig. 17.2S.

Lo que queremos es las bolas 1 y 2

choquen al menos una vez más después

del primer encuentro en el origen. Esto

significa que la recta y2 debe cortar a la

sinusoide en al menos un punto. Fig. 17.2S

Page 167: problemas selectos

167

17.3-El pistón deja de moverse cuando se evapora toda el agua: cuando el número de moles en

estado gaseoso sea constante en cada parte, el aumento de presión y temperatura ocurrirá a

volumen constante; en tal situación, el aire redujo su volumen de Vo a Vo/2 y el vapor lo aumentó

de Vo a 3Vo/2. La evaporación del agua ocurrirá todo el tiempo en equilibrio de la fase líquida

con su vapor saturado. Sean To y Po la temperatura y la presión al inicio, y T1, P1, los valores

finales. Para el aire se cumplirá:

PoVo/To = P1Vo/(2T1) (1)

Para el vapor de agua, tratado como gas ideal, se tendrá al inicio:

PoVo = (M/μ)RTo con Po = Ps (a temperatura To) (2)

Y al final: P1(3Vo /2) = [(M+m)/μ]RT1 con P1 = Ps (a temperatura T1) (3)

donde μ es la masa molar del agua y m= 4 g es la masa evaporada.

De (1): PoVo = P1 (Vo/2)(To/T1).

Sustituyendo en (2) y cancelando To: P1Vo/(2T1) = (M/μ)R.

Dividiendo miembro a miembro por (3) se obtiene: 1/3 = M / (M+m), y M = m/2 = 2 g, que es la

masa de agua al inicio.

Para averiguar las temperaturas To y T1, así como la masa Mo de aire, necesitamos hacer el

gráfico de Ps vs T, mostrado en la Fig. 17.3S. En ese gráfico hay dos puntos cruciales para

nuestro análisis: el de la presión de vapor al inicio y el de la presión de vapor al final. Para estos

dos puntos se cumplirá, por la ley de los gases ideales:

Ps = (MR/μVo)T = 923T y Ps = [(M+m)2R/3μVo]T = 1848T

con Ps en Pa, T en K, y μ=18 g/mol para el agua.

Estas ecuaciones representan dos líneas rectas de PsvsT. Si las expresamos en grados celsius, con

T = t + 273, estas ecuaciones se convierten en:

Ps = 252×103 + 923t y Ps =504×10

3 + 1848t

con Ps en Pa y t en grados celsius.

O, si lo expresamos en kPa: Ps = 252 + 0.923t y Ps = 504 + 1.85t.

Estas dos rectas se han representado en el gráfico de la Fig. 17.3S (próxima página). Para trazar-

las, se determinaron sus intersectos con el eje de ordenada (252 kPa y 504 kPa) y se evaluó la

presión para t = 100oC (344 kPa y 689 kPa), con lo que determinamos dos puntos de cada recta.

Page 168: problemas selectos

168

To = 413 K y T1 = 443K. Entonces, para el aire seco conocemos que To = 413 K y Po = 3.8×10

5

Pa, con μ = 29 g/mol. Por tanto la masa de aire seco era: Mo = PoVoμ/(RTo) = 3.2 g.

17.4-En la Fig. 17.4Sa se ha representado el gráfico de T vs t. Para responder a las preguntas

formuladas es más conveniente un gráfico de velocidad de calentamiento a cada temperatura.

Así, para varios puntos del gráfico original podemos trazar la recta tangente y medir su pen-

diente, que nos indicará la velocidad de calentamiento, en oC/min, para cada temperatura del

agua. Se obtiene así el gráfico de la Fig. 17.4Sb (se graficaron sólo cuatro puntos).

(El segundo gráfico también se puede construir a partir de los datos de la tabla, restando cada dos

temperaturas consecutivas y dividiendo por el intervalo de 1 min que los separa. Este cociente

representará la velocidad de calentamiento en la mitad del intervalo de temperaturas (aproxi-

mación lineal). Se obtiene así la siguiente tabla que sirve también para hacer el segundo gráfico).

t(min) 0.5 1.5 2.5 3.5 4.5 5.5 6.5 7.5 8.5 9.5

ΔT/Δt

(oC/min)

6.2 5.6 5.0 4.6 4.2 3.7 3.4 3.1 2.7 2.6

T(oC) 23.1 29.0 34.3 39.1 43.5 47.5 51.0 54.3 57.2 59.8

T(oC)

t(min) 0 5 10 0

20

40

60

ΔT/Δt(oC/min)

T(oC)

6

4

2

0

20 40 60 80 0

6.5

3.5

Fig. 17.4Sa Fig. 17.4Sb

0 50 100 150 200 t(

oC)

Ps(kPa)

200

400

600

800

1000

0

252

504

689

344

140 170

380 A

B

Fig. 17.3S

Los dos puntos cruciales que nos inte-

resan deben pertenecer a la curva de la

presión del vapor de agua y a cada una

de las dos rectas analizadas, lo cual

corresponde a los puntos A y B del

gráfico.

El punto A nos dice que la tempera-

tura inicial era de unos 140oC, con una

presión de unos 380 kPa. El punto B

nos informa que la temperatura final

era de unos 170oC. O en kelvin:

Page 169: problemas selectos

169

Del gráfico se ve que la velocidad de calentamiento se hace cero cerca de los 85oC, de modo que

no se calentará hasta hervir. Ya cerca de los 85oC el agua cede calor al ambiente con la misma

velocidad con que lo recibe de la resistencia. Esto responde a la segunda pregunta.

En cuanto a la primera pregunta puede observarse del gráfico de calentamiento que a la tempe-

ratura ambiente, de 20oC, la velocidad de calentamiento es 6.5

oC/min, y a 50

oC la velocidad de

calentamiento se reduce a 3.5oC/min. La disminución de la velocidad de calentamiento es de

3.0oC/min, y se debe a la diferencia entre el calor que retiene el agua a temperatura ambiente y el

que retiene a 50oC. O sea, tal diferencia es la debida al calor que entrega el agua caliente al am-

biente cuando está a 50oC. Entonces, si se apaga la resistencia cuando el agua está a 50

oC, la

velocidad con que se enfriará será de 3.0oC/min, lo que significa que en 1 minuto se enfriará

unos 3oC.

17.5-El calor absorbido por el agua es rápidamente repuesto a lo largo del tubo gracias a la gran

conductividad de éste en relación a la del agua. En el estado estacionario se cumplirá la pro-

porcionalidad: Th/Ts = h/l (1)

donde h es la altura a que sube el agua en el capilar y Th es la temperatura a esa altura. La fuerza

de tensión superficial que levanta a la columna líquida es: F = 2πrζ cosα, donde α = 0 pues el

líquido moja perfectamente al capilar (es el ángulo de contacto líquido-capilar). Esta fuerza equi-

libra a la de gravedad de la columna líquida, Fg = mg = πr2hρg. Así: 2πrζ = πr

2hρg, de donde

resulta: h = 2ζ/(ρgr) (2)

Despejando h de (1) y sustituyendo en (2): l Th/Ts = 2ζ/(ρgr) , y

ζ = ρgr l Th/(2Ts) (3)

que nos da el valor de σ en función de Th, que es una recta de pendiente ρgrl/(2Ts) = 0.78×10-3

N/moC.

Llevando esta recta a un gráfico junto con la curva de ζ vs T que sale de la tabla (Fig. 17.5S), se

obtiene un punto de intersección que nos da el valor del coeficiente de tensión superficial ζ a la

20 40 60 80 T(

oC)

20

40

60

80 σ(×10

-3 N/m)

100

Fig. 17.5S

Page 170: problemas selectos

170

altura de la columna líquida y la temperatura a esa altura del agua: cerca de 60×10-3

N/m y cerca

de 80oC. La altura podemos evaluarla por (1): h = l Th/Ts = 8×80/100 ≈ 6.4 cm.

(O por (2): h = 2ζ/(ρgr) = 2×60×10-3

/(1×103×9.8×2×10

-4) ≈ 6.1 cm)

La habitación tiene unos 20 o 30oC, de modo que la diferencia de temperaturas no cambia

apreciablemente entre la mezcla y el aire de la habitación durante todo el proceso (cambia 2

grados de 20 o 30) por lo que la velocidad de intercambio de calor entre la mezcla y la habitación

puede asumirse prácticamente constante durante los 60 min iniciales. En tal caso, el calor

intercambiado en 50 min es cinco veces mayor que el intercambiado en 10 min, esto es: Q’= 5Q

= 420 kJ. Pero todo este calor fue empleado para derretir hielo, sin calentar la parte líquida.

Entonces, la masa fundida de hielo es:

Mhielo = Q/Lf = 420/340 ≈ 1.2 kg de hielo

t(min)

T(oC)

10 20 30

20

40

60

80

00

100

40 50

Fig. 17.7S

17.7- Llamemos q al calor desprendido por unidad

de tiempo de la hornilla, y: C=200 J/K (capacidad

calorífica de la cazuela), ca = 4.18 kJ/kg.K (calor

específico del agua), cal (calor específico del alco-

hol), ma= ρaV = 0.50 kg (masa de agua), mal = ρalV

= 0.56 kg (masa de alcohol).

En ambos casos los líquidos comienzan a calentar-

se a la temperatura de 20oC, según se desprende

del grafico. Consideraremos el calor desprendido

en un tiempo Δt pequeño, próximo a los 20oC. En

ambos casos, esta cantidad de calor será Q = qΔt, y

se empleará en calentar al líquido (mcΔT), calentar

la cazuela (CΔT) y en algunas pérdidas al exterior.

t(min)

T(oC)

20 40 60

1

2

3

17.6-Del gráfico se desprende que en los primeros 50 min en la

habitación el hielo se estaba derritiendo con el calor ganado de

la habitación y el sistema no cambió de temperatura (perma-

neció en 0oC). De los 50 min en adelante, el sistema era todo

líquido y comenzó a calentarse con el calor recibido de la

habitación. Durante 10 min (de 50 min a 60 min en la Fig.

16.6S) la mezcla subió 2oC y el calor absorbido fue:

Q = MmezclacΔT = 10×4.2×2 = 84 kJ

Fig.17.6S

Page 171: problemas selectos

171

Estas pérdidas al exterior dependen de la diferencia de temperaturas entre el sistema y el exterior

(T – 20), el tiempo de calentamiento Δt, y factores geométricos y ambientales (corrientes de aire,

dimensiones de la superficie de la cazuela, etc.) que agruparemos en una constante α. Entonces,

para el agua:

qΔt = macaΔTa + CΔTa + α (T – 20)Δt (1)

Y para el alcohol: qΔt = malcalΔTal + CΔTal + α (T – 20)Δt (2)

Hemos tenido en cuenta que el agua y el alcohol se calientan desigualmente (ΔTa, ΔTal) en un

mismo tiempo Δt.

Considerando un mismo tiempo de calentamiento inicial, qΔt es el mismo en ambos casos, por lo

que:

macaΔTa + CΔTa + α (T – 20)Δt = malcalΔTal + CΔTal + α (T – 20)Δt

Considerando un Δt pequeño (como 1 min), la temperatura T alcanzada no diferirá grandemente,

por lo que el último sumando de cada lado valdrá casi lo mismo y se cancelan uno con otro:

macaΔTa + CΔTa = malcalΔTal + CΔTal

Dividiendo ambos miembros por el tiempo Δt transcurrido:

macaΔTa/Δt + CΔTa/Δt = malcalΔTal/Δt + CΔTal/Δt

De aquí despejamos cal: cal = {[(ΔTa/Δt) / (ΔTal/Δt)] [(maca + C)/mal]} – C /mal

Son conocidos ma, mal, ca y C; los cocientes ΔTa/Δt y ΔTal/Δt) son las pendientes de las curvas en

las proximidades del instante inicial, que pueden determinarse a partir del gráfico (6.8oC/min

para el alcohol y 4.4oC/min para el agua). Sustituyendo valores: cal = 2.4×10

3 J/kg.K.

Llamando al calor de ebullición del alcohol, Lal, cuando el alcohol haya hervido un tiempo Δt1,

se cumplirá: qΔt1 = Lal Δm + α (Tal – 20) Δt1, donde Tal = 78oC (del gráfico) y se tuvo en cuenta

que la cazuela no se calienta durante la ebullición. De aquí:

Lal = [q – α (78 – 20)] / (Δm / Δt1) (3)

El denominador es la rapidez de ebullición, y puede calcularse a partir del dato de que la mitad

del alcohol ebulle en 23 min: Δm / Δt1 = (0.56/2)/23 = 1.2×10-2

kg/min. Y el corchete con q y α

juntas pueden evaluarse a partir de la ecuación (1), aplicando los datos de la curva del agua a

78oC (en t = 13 min):

q – α (78 – 20) = (maca + C) ΔT/Δt = (0.50×4180 + 200) (78 – 20) / 13 min = 1.02×104 J/min

Sustituyendo en (3): Lal = [1.02×104 J/min] / (1.2×10

-2 kg/min) = 8.5×10

2 kJ/kg

Page 172: problemas selectos

172

17.8-La erupción comienza apenas la presión del vapor saturado del agua en la base del canal

sobrepase ligeramente a la suma de presión atmosférica con la presión hidrostática de la columna

de agua en el canal de salida, bajo una profundidad de 90 m. O sea, se produce la erupción

cuando

P ≈ Patm + ρgh ≈ 0.98×10 6 Pa ≈ 10

6 atm

Del grafico se ve que para esa presión de vapor la temperatura es de 180oC. El calor de la tierra

ha elevado la temperatura del agua subterránea hasta esa temperatura, con la cual la presión de

vapor empieza a superar a la presión exterior que oprime al agua, formándose las burbujas de

vapor que empiezan a subir por el canal por flotación: el geiser expulsa vapor de agua que

arrastra algo de agua líquida.

En la medida que sale vapor sobrecalentado se va enfriando el agua en el canal hasta que su

temperatura baja hasta los 100oC y deja de hervir incluso el agua que está cerca de la salida de la

canal del geiser. En el proceso, la temperatura del agua del reservóreo baja desde los 180oC hasta

unos 100oC, y toda el agua M del reservóreo pierde una cantidad de calor McΔT, con ΔT = 80

oC.

Ese calor perdido por el agua del reservóreo es el empleado en hacer hervir una masa m de agua

que se expulsa como vapor al exterior, calor que vale Lvm. Entonces:

McΔT = Lvm

De aquí: m/M = cΔT/Lv = 4180×80/2260000 = 0.15

O sea, en cada erupción el reservóreo pierde alrededor del 15% de su agua. (Al bajar la presión

en el reservóreo entra más agua de las fuentes subterráneas de alimentación y tras un tiempo se

calienta lo suficiente para repetir otra erupción, y así lo hace cíclicamente).

V (V) 80 60 40 20

I(A)

0.2

0.4

0.6

0.8

17.9-De acuerdo con la potencia y el voltaje de

trabajo de cada lámpara, sus corrientes eléc-

tricas a 110 V son:

I1 = P1/V1 = 60/110 = 0.545 A y

I2 = P2/V2 = 100/110 = 0.909 A

Al colocarlas en serie, la corriente debe ser la

misma en ambas lámparas, y sus voltajes deben

sumar 220 V. Si la de 60 W no admite una

sobrecarga mayor del 10% que la que admite a

110 V, su máxima corriente sin quemarse será

de: 0.545 + 0.054 = 0.599 A ≈ 0.6 A.

Fig.17.9S

Page 173: problemas selectos

173

Con tal corriente la lámpara de 100 W tendría un voltaje de 50 V (Fig. 17.9S), que sumado a los

110 V de la primera lámpara quedaría muy por debajo de los 220 V aplicados. Significa que la

lámpara de 100 W consumirá mayor voltaje que 50 V y permitirá el paso de más de 0.6 A. con lo

cual se quema la lámpara de 60 W.

17.10- a)Si se conectan en serie va a pasar la misma corriente por ambas lámparas y caerá el

mismo voltaje en ambas. O sea, caerán 2.5 V en cada una, y por la gráfica se ve que a ese valor

de voltaje corresponde una corriente de 0.27 A (Ver líneas (a) de la Fig. 17.10S).

b)Si en una lámpara caen 3.5 V, en la lámpara que esté en el paralelo con la resistencia cae la

diferencia 5.0 – 3.5 = 1.5 V. Por la gráfica, la lámpara con 3.5 V tiene una corriente de 0.29 A

(líneas b1); y la que está en el paralelo, con 1.5 V, tiene una corriente de 0.22 A (líneas b2). La

diferencia entre estas dos corrientes es la que pasa por la otra rama del paralelo donde está la

resistencia:

I = 0.29 - 0.22 = 0.07 V.

Significa que esa resistencia tiene un valor de: R = V/I = 1.5/0.07 = 21Ω

c)Si conectamos dos lámparas en paralelo con una tercera en serie, la que está sola en serie so-

portará una corriente doble de las otras dos, y ésa sería la primera que se quemaría. Esto ocurriría

si el voltaje de la lámpara en serie alcanzara los 4.0 V, con una corriente de 0.30 A (líneas c1).

En cuyo caso, las otras dos lámparas tendrían una caída de voltaje de 5.0 – 4.0 = 1.0 V. Pero de

acuerdo con la gráfica, cada lámpara del paralelo bajo 1.0 V de voltaje debe soportar una

corriente de 0.17 A (líneas c2), y ambas juntas consumirían 0.34 A. Pero si “halan” juntas más

corriente que la de 0.30 A, se funde la primera lámpara. Por otro lado, si asumimos que en la

4.0 3.0 2.0 1.0

0.10

0.20

0.30

I(A)

V(V)

Relación I-V para la lámpara

(Resistencia no lineal)

Fig. 17.10S

(a)

(b2)

(b1)

(c2)

(a)

(b2)

(c2)

(b1)

(c1)

(c1)

Page 174: problemas selectos

174

lámpara en serie caiga un voltaje menor de 4.0 V, entonces en las dos lámparas en paralelo caería

un voltaje mayor de 1.0 V, con más corriente aún que 0.17 A, por lo que la lámpara en serie se

quemaría más rápido. Por tanto, no pueden colocarse dos en paralelo con otra en serie pues se

quemaría la que está sola en serie.

17.11-Llamemos V al voltaje de la fuente, Vl al de la lámpara y VR al de la resistencia. Se

cumplirá:

V = Vl + VR = Vl + IR (1)

La corriente de operación de la lámpara es entonces I ≈ 1.1 A y su voltaje de operación es Vl ≈

5.5 A. La potencia de la lámpara es entonces: P = IVl ≈1.1×5.5 ≈ 6 W

17.12- Entre los puntos A y B no hay ningún elemento. Son sólo dos puntos entre los cuales se

produce cierta diferencia de potencial, medible por un voltímetro ideal (de corriente nula). De tal

forma, la misma corriente que pasa por r es la que pasa por la lámpara L.

(a)Sea V el voltaje en la lámpara L. Se cumplirá: E = V + Ir, de donde: I =E /r – V/r = 0.4 – 0.1V.

En un gráfico de I vs V ésta es una línea recta (“línea de carga”) de pendiente 1/r. Esta línea nos

informa cómo depende el voltaje en la lámpara de la corriente que pasa por la resistencia r en

serie con la lámpara.

En la Fig. 17.12Sa hemos representado la línea de carga de la resistencia r superpuesta a la

característica voltampérica de la lámpara. El punto de intersección de ambas líneas fija el “punto

de operación” de la lámpara (su corriente y su voltaje para la resistencia r dada).

I(A)

10

Fig. 17.11S

5

1.0

2.0

De aquí: I = V/R – Vl/R

que es la ecuación de una recta en el plano de I vs Vl

(la “línea de carga”) (Fig. 17.11S). Su intercepto con

el eje de las corrientes será para Vl = 0, en cuyo caso

I = V/R = 10/4 = 2.5 A. El intercepto con el eje de

los voltajes será cuando I = 0, en cuyo caso: Vl = V

= 10 V. Con estos dos puntos se puede trazar la

línea de carga como muestra el gráfico, la cual fija

el punto de operación de la lámpara (intersección de

la línea de carga con la curva de la lámpara), pues

en tal punto la corriente satisface los valores de VR,

Vl y V que cumplen con la ecuación (1).

Vl (V)

Page 175: problemas selectos

175

(b)Para que entre A y B no haya diferencia de potencial, el reóstato, o potenciómetro, debe

quedar dividido en dos resistencias, R1 (parte de abajo) y R2 (parte de arriba) tales que:

R1/R2 = V/ (E – V).

De aquí: R1= (R1+R2) V/E = RV/E = 16 Ω, y en consecuencia: R2 = R – R1 = 24 Ω.

(c)Bajo pequeñas variaciones de la fem se producen pequeñas variaciones de voltaje en todos los

elementos del circuito. Para que las variaciones de voltajes en AB, ΔVAB, sean mínimas, la varia-

ción de voltaje en L deberá ser casi igual a la variación de voltaje en R1 (parte baja de R). La

resistencia R1 es constante, pero RL depende del valor del voltaje V aplicado a ella. Para pequeñas

variaciones ΔV alrededor del punto de operación de la lámpara se producirán variaciones ΔI pro-

porcionales a ΔV: ΔI ΔV (aproximación lineal para variaciones pequeñas). La constante de

proporcionalidad será la pendiente de la recta tangente a la curva de la característica voltam-

périca de la lámpara en el entorno del punto de operación (Fig. 17.12Sb). En consecuencia, bajo

pequeñas variaciones de voltaje ΔV, cerca del punto de operación, la lámpara se comporta como

una resistencia de valor rL = ΔV/ΔI = cot θ, donde θ es el ángulo de inclinación (analítico) de la

recta tangente a la curva. (Ésta es la resistencia dinámica, o diferencial, de la lámpara, bajo co-

rrientes variables). Esta resistencia no relaciona los valores de V e I en el punto de operaciones

(eso lo hace RL), sino que relaciona las variaciones de V con las de I. En nuestro caso:

rL = (3.4 – 0)/(0.40 – 0.08) ≈ 11 Ω (ver valores en el gráfico, Fig. 17.12Sb)

Ahora, la variación ΔV en la lámpara satisface: ΔV =ΔE − r ΔI, donde ΔI = ΔV/rL.

Queda entonces: ΔV =ΔE − ΔV(r/rL).

Y despejando: ΔV = ΔE rL/(r+rL) (1)

Por el divisor de voltaje teníamos: V1 = E R1/R, por lo que ΔV1 = ΔE (R1/R) (2)

Deseamos que ΔV = ΔV1 para que el ΔVAB = 0. Entonces, igualando (1) con (2):

I(A)

V(V) 1 2 3 4 5 0 0

0.1

0.2

0.3

0.4

Fig.17.12Sa

1.6

0.24

V(V) 1 2 3 4 5 0 0

0.1

0.2

0.3

0.4

Fig.17.12Sb

ΔV

ΔI

0.08

3.4

Este punto de operación es de V =1.6 V e I = 0.24 A según se puede apreciar en la Fig. 17.12Sa.

La resistencia de la lámpara, en corriente directa será RL = V/I = 1.6/0.24 ≈ 6.7 Ω.

Page 176: problemas selectos

176

ΔE rL/(r+rL) = ΔE (R1/R).

De aquí: R1 = RrL/(r + rL) = 21 Ω.

O sea, el punto móvil del reóstato debe estar en una posición en la que R1 = 21 Ω. En esta con-

dición: V1 = E R1/R = 2.1 V y V = 1.6 V (punto de operación), por lo que VAB= 0.5 V. Si hubiera

una fluctuación de 1 V en la batería, ΔE = 1 V, los cambios en L y R1 serían:

ΔV = ΔE rL/(r+rL) = 0.524 V y ΔV1 = ΔE (R1/R) = 0.525 V, por lo que la fluctuación de voltaje

entre A y B sería ΔVAB = 0.525 – 0.524 = 0.001 V. Así con una variación de un 25% de la fem

(1 V en 4 V), la variación del voltaje entre A y B sería del 0.2% (0.001 V en 0.5 V). Este circuito

puede ser usado como un estabilizador de voltaje.

ción de esta línea de carga con la curva voltampérica fija el punto de operación del diodo.

De la ecuación (1) se ve que la pendiente de la línea de carga es independiente de la temperatura

T, pues solamente depende de las resistencias, que no nos dicen que dependan de la temperatura.

El intercepto en el eje de las I sí cambia pues depende de V, que es función de la temperatura.

Así, a distintas temperaturas la línea de carga se desplaza arriba y abajo, paralelamente a su

inclinación original.

Se conoce por dato que ΔV/ΔT = 2.5×10-2

V/oC = α y que V2 = 6.0 V a T2 = 25

oC. Entonces, a

T1= 125oC se tendrá: V1= V2 + α(T1 – T2) = 8.50 V y a T3 = -60

oC se tendrá: V3= V2 + α(T3 – T2)

= 3.87 V. Se ve que el voltaje de la fuente varía fuertemente con la temperatura en el intervalo

de interés.

Busquemos el intercepto de la línea de carga con el eje I, b=V/(R1+R2), para cada temperatura:

b1= 1.63 mA, b2= 1.15 mA y b3= 0.69 mA. La pendiente en todos los casos es la misma: m =

1/(5.0+0.2) = 0.192 kΩ-1

≈ 0.2 mA/V

Las tres líneas de carga se muestran aproximadamente en la Fig. 17.13S. Se trazó primero la

recta inferior de pendiente -0.2 (pasa por el eje I en I = 0.2 mA y por el eje Vd en Vd = 1 V).

Vd(V)

I(mA)

0.4 0.8 1.2

0.5

1.0

1.5

T1 T2 T3

T1

T2

T3

17.13-El voltaje que cae en el diodo, Vd, será:

Vd = V – IR1 – IR2, por lo que:

I = V / (R1+R2) – Vd/ (R1+R2) (1)

Esta ecuación representa una recta, I = f(Vd),

en el gráfico de I vs Vd, de pendiente:

m =−1/(Ri+R2) e intercepto b=V/(R1+R2);

esta recta es la “línea de carga”. La intersec- Fig. 17.13S

1.63

1.45

0.69

0.60

1.15

0.20 1.0

Page 177: problemas selectos

177

Después se construyen las tres rectas paralelas a la inferior, pero con sus respectivos interceptos

en el eje I.

Operando a 25oC, se ve del gráfico que I = 1.0 mA y Vd = 0.80 V. La caída de voltaje en la salida

AB será:

VAB = V – IR1 = 6 – 0.001×5000 = 1 V, que es la respuesta a la primera pregunta.

Cuando la temperatura varíe entre -60oC y 125

oC, las variaciones de voltaje en la salida AB se

calcularán como: ΔVAB = ΔV – (ΔI) R1 (2)

Del gráfico se ve que la variación de I entre -60oC y 125

oC es ΔI = 1.45 – 0.60 = 0.85 mA. La

variación por cada grado celsius será: ΔI/ΔT = 0.85/(125 + 60) = 0.0046 mA/ oC. Si dividimos la

ecuación (2) por ΔT tendremos:

ΔVAB/ΔT = ΔV/ΔT – (ΔI/ΔT) R1 = α – (ΔI/ΔT) R1

Para el intervalo total de temperaturas:

ΔVAB/ΔT = 2.5×10-2

V/ oC − 0.0046 (mA/

oC) × 5(kΩ) = 0.002 V/

oC

Ésta es la variación del voltaje de salida por cada grado centígrado que cambie la temperatura.

Asumiendo una variación lineal del voltaje de salida con la temperatura podremos plantear:

VAB (T)= VAB(25oC) + (ΔVAB/ΔT ) (T – 25).

O sea:

VAB (T)= 1 + 0.002 (T – 25)

con T en grados celsius. Ésta es la respuesta a la segunda pregunta.

Page 178: problemas selectos

178

18-Problemas con Análisis de Simetría.

A

B

18.5-La arista de cada cuadrado de la Fig, 18.5 es un

alambre de resistencia r = 10 Ω. En el cuadrado

central se suelda una lámina de un material muy buen

conductor (área sombreada). Hallar la resistencia en-

tre los puntos A y B.

Fig. 18.5

R1

R2

kR1

kR2

Fig. 18.4

A

B

18.4-El circuito de resistores de la Fig. 18.4 consiste en

un número de eslabones muy grande. La resistencia de

los resistores de cada eslabón es k veces mayor que el

valor de las resistencias en el eslabón anterior. Encuen-

tre la resistencia RAB entre los puntos A y B si las resis-

tencias del primer eslabón son R1 y R2.

V

A B

C

D

R

R

R

R

R R

R

E R

Fig. 18.3

18.3-En el circuito de la Fig. 18.3 todos los resistores

tienen la misma resistencia R. La tensión en los bor-

nes es V. Determine la intensidad de la corriente I en

los cables de alimentación del circuito. La resisten-

cia de los cables de alimentación es despreciable.

A

B

R

R R

R R R R

R R

Fig. 18.2

18.2-Determine la resistencia del circuito RAB entre

los puntos A y B de la estructura compuesta por

los nueve alambres iguales mostrados en la Fig.

18.2, de resistencia R cada uno.

Fig. 18.1

C

C C

C C C

18.1-Encuentre la capacitancia del sistema de ca-

pacitores idénticos de la Fig. 18.1. La capacitan-

cia de cada capacitor es C.

Page 179: problemas selectos

179

18.6-(Requiere Cálculo) Un fino alambre muy largo posee una densidad lineal de carga λ

constante en toda su extensión. Calcule la intensidad del campo eléctrico a una distancia r del

alambre, lejos de sus extremos y próximos al alambre. Calcule también la diferencia de potencial

entre dos puntos a las distancias r1 y r2 del alambre.

18.7-(Requiere Cálculo) Una esfera metálica de carga q1 y radio r1 está en el interior de una fina

capa metálica esférica de radio r2 y carga q2; ambas esferas son concéntricas. Calcule la

intensidad del campo eléctrico a diferentes distancias del centro de ambas esferas.

18.8-A una esfera conductora homogénea de masa m se le transfiere una carga q y se fija al

extremo de un hilo no conductor. El otro extremo se liga al extremo superior de un anillo de

radio R, el cual se halla en un plano vertical. Al anillo, elaborado de un material conductor, se le

ha comunicado una carga Q del mismo signo que la carga q, que se distribuye uniformemente

por el anillo. Determine la longitud l del hilo con la cual la esfera se sitúa justamente sobre el eje

horizontal del anillo.

18.9-(Requiere Cálculo) Calcule el valor de la inducción magnética, B, en el interior de una

bobina cilíndrica muy larga (solenoide) de n vueltas por unidad de longitud y sin núcleo

ferromagnético, cuando circula por ella una corriente eléctrica de intensidad I.

18.10-(Requiere Cálculo)Calcule el valor de la inducción magnética, B, en el interior de un

toroide de sección circular que tiene un enrollado uniforme de n vueltas por unidad de longitud,

por el cual circula una corriente de intensidad I. Se conocen el radio de la sección circular, r, y el

radio del toroide, R, cumpliéndose que R >> r. Haga el cálculo asumiendo que el toroide está

vacío e indique cómo variaría el resultado si el toroide estuviese lleno de un material ferro-

magnético. (Toroide: es como una dona, o salvavidas, en la que se enrolla el cable en vueltas

muy apretadas).

Page 180: problemas selectos

180

18-Problemas con Análisis de Simetría. Soluciones.

18.1-En la Fig. 18.1Sa hemos numerado los capacitores, aunque son todos iguales, para repre-

sentar el circuito de una manera más simétrica sin perder de vista cuál es cada capacitor en el

nuevo circuito equivalente. La Fig.18.1Sb presenta un circuito equivalente al dado. En este

nuevo circuito puede notarse con facilidad que el capacitor C3 está conectado a puntos de igual

potencial y que no se cargará, lo que es equivalente a quitarlo del circuito sin que se afecte la

distribución de cargas en el resto.

Pero suprimir a C3 equivale a tener el circuito de la Fig. 18.1Sc, que es tratable como un conjun-

to de series y paralelos bien definidos. Son tres ramas en paralelo con capacitancias C, C/2 y C/2

que sumadas dan una capacitancia total de 2C.

18.2-En este problema hay nueve ramas con diez corrientes diferentes si se incluyen la de en-

trada y la de salida, I0. En la Fig. 18.2S, la recta DE divide la estructura en dos partes simétricas,

con distribuciones equivalentes de resistencias, voltajes y corrientes, lo cual reduce las incógnitas

a seis: I0, I1, I2, I3, I4, I4 e I5, y se simplifican mucho los cálculos. Se han representado las

corrientes que son iguales en las distintas ramas simétricas.

Aplicando ley de nodos en F: I1 = I3 + I5

Aplicando ley de nodos en D: I2 + I3 = I4 + I5

Aplicando ley de mallas FDEF: (I3+I4) R = I5R

Aplicando ley de mallas AFDA: (I1+I3) R = I2R

A

B

R

R R

R R R R

R R

A

B

C D

E F

I0

I1 I1

I2

I2

I3 I3

I4

I5

I5

I0

Fig. 18.2

C3

C4

C1 C2

C6 C5

C

C C

C C

Fig. 18.1Sb Fig. 18.1Sc Fig. 18.1Sa

C1

C2 C3

C4 C5 C6

Fig. 18.2S

Page 181: problemas selectos

181

Resolviendo estas cuatro ecuaciones en función de I1:

I2 = (6/5) I1, I3 = (1/5)I1, I4 = (3/5)I1, I5 = (4/5)I1.

Añadiremos ahora una ecuación de ramas desde A hasta B para evaluar la caída de potencial VAB,

necesaria para calcular la resistencia RAB, que será RAB = VAB/I0, donde I0 = I1+I2 (ver nodo A).

O sea: VAB= [I1 + (4/5) I1+ (6/5) I1] = 3I1R, de donde resulta VAB /I1 = 3R.

Ahora: RAB = VAB/I0 = VAB/ (I1+I2) =VAB/[I1+ (6/5)I1] = (5/11)VAB/I1 = (15/11)R

Así: RAB = (15/11) R.

18.3-En la Fig. 18.3 se han añadido letras para identificar los nodos y las resistencias. Puede no-

tarse que las mallas ABCA y ABDA son equivalentes, por lo que los puntos C y D son equipo-

tenciales, que es como si estuvieran conectados entre sí (sin resistencias de por medio).

La Fig. 18.3S muestra el circuito equivalente, donde C y D se han unido en un solo punto, y es

como si las resistencias AC y AD quedaran en paralelo, así como BC con BD y EC con ED.

Visto así el circuito es uno sencillo de series y paralelos, con una rama BEC de resistencia (3/2)R

en paralelo con la rama BC de resistencia R/2, todo esto en serie con la rama AC de resistencia

R/2, y todo en paralelo con la rama AB de resistencia R.

Así: 1/RCB = 1/(R/2) + 1/(R + R/2), de donde: RCB = (3/8)R

Entonces: RACB = R/2 + (3/8)R = (7/8) R.

Finalmente: 1/RAB = 1/R + 1/[(7/8)R].

Y resolviendo: RAB = (7/15)R

V

A B

C

D

R

R

R

R

R R

R

V

A B R

R/2 R/2

R/2

E E

C,D

R R

Fig. 18.3

Fig. 18.3S

Page 182: problemas selectos

182

18.4-En este circuito podemos mentalmente dividir por k todas las resistencias excepto las del

primer eslabón, R1 y R2. Si hacemos esto nos damos cuentas de que lo que quedaría detrás del

primer eslabón es un conjunto de resistencias igual que el dado entre A y B, de modo que la

secuencia que prosigue tras el primer eslabón es la misma que la cadena completa multiplicada

por k. Este análisis nos permite sustituir el circuito original por el equivalente de la Fig. 18.4S,

que ya es un circuito sencillo de serie y paralelo, donde se cumplirá:

RAB = {kR1 – R2(1 – k) ± }/(2k)

La solución negativa carece de significado físico.

de ambas en paralelo.

A continuación (Fig. 18.5Sb) se muestra el esquema del circuito equivalente del grupo 1, 2, 3, 4,

5, 6, 7, 8, 9, reducido a otros circuitos equivalentes cada vez más simples. Las resistencias

equivalentes que se escriben en cada paso se calculan por las operaciones de dos resistencias

equivalentes en serie o en paralelo: Rserie = Ra + Rb y Rparal = RaRb/(Ra + Rb)

El circuito se ha dibujado partiendo del punto B y siguiendo todos los caminos que llevan al

cuadrado central, A, que tiene un único potencial, simbolizado por la línea inferior del esquema

de circuito de la Fig. 18.5Sb (como si fuera un potencial de “tierra” común).

A

B

18.5-La lámina conductora pone a un mismo potencial

todos los alambres del cuadrado soldado a ella. La dia-

gonal de la Fig.18.5Sa divide el circuito en dos ramas

simétricas conectadas a los puntos de potencial A y de

potencial B, por lo que equivalen a dos ramas en

paralelo. Por la simetría de las dos ramas, los puntos C

y D están al mismo potencial, por lo que las dos aristas

de la esquina inferior no conducen corriente entre C y

D y pueden suprimirse en el análisis. Los lados 1, 2, 3,

4, 5, 6, 7, 8, 9 son simétricos a los lados 1’, 2’, 3’, 4’,

5’, 6’, 7’, 8’, 9’. Calcularemos la resistencia equiva-

lente de las nueve primeras resistencias, igual a la de

las otras nueve, y luego calcularemos la equivalente

C

D

1

2

3

4

5

6

7

8

9

1’

2’ 3’

4’

5’

6’

7’

8’

9’

Fig. 18.5Sa

R1

R2 kRAB

A

B

Fig. 18.4S

RAB = R1 + [1/R2 + 1/kRAB]-1

Despejando RAB surge una ecuación de segundo grado:

kRAB2 + RAB[R2(1 – k) – kR1] – R1R2 = 0

Resolviéndola:

Page 183: problemas selectos

183

A

B

A

B

A

B

A

B

A

B

A

B

A

B

1 3

5 6

7

8

9

2

4

r

r

r

A

B

1

3 5 6

7 2

4

2r r

(2/3)r

r r

(8/3)r r

(8/11)r

r

(19/11)r

(19/30)r

(49/30)r

r

r

Las resistencias 8 y 9 en

serie equivalen a una de

valor 2r.

Ésta de valor 2r está en

paralelo con la resisten-

cia 7 de valor r, y equi-

valen a una de valor 2r/3.

La de 2r/3 está en serie

con las dos de valor r ca-

da una, por lo que todas

juntas equivalen a una de

valor 8r/3.

La de valor 8r/3 está en paralelo con una de valor r

y equivalen a una de valor 8r/11.

La de valor 8r/11 está en serie con una de valor r y

equivalen a una de valor 19r/11.

Ésta de 19r/11 está en paralelo con una de valor r

y equivalen a otra de valor 19r/30.

Finalmente, la de 19r/30 está en serie con otra de

valor r y dan una resistencia equivalente total de

49r/30, para la rama 1, 2, 3, 4, 5, 6, 7, 8, 9.

Esta rama está en paralelo con la rama 1’, 2’, 3’,

4’, 5’, 6’, 7’, 8’, 9’, de igual valor, y las dos juntas

dan una resistencia total R = 49r/60.

Y como r =10 Ω, entonces R = 8.2 Ω

2

2

1

1

1

3

2

3

1

4

Fig. 18.5Sb

Page 184: problemas selectos

184

18.6-El tipo de campo que se genera desde un alambre cargado uniformemente, lejos de los

extremos del alambre y cerca del mismo, es de líneas radiales como muestra la Fig. 18.6S.

Es un campo de simetría cilíndrica. Tomando una superficie gaussiana cilíndrica que contenga al

punto P, que está a una distancia r del alambre como muestra la figura, podemos plantear la ley

de Gauss:

= q/εo, donde q es la carga neta dentro de la superficie gaussiana (o sea, la carga del

alambre que queda dentro del cilindro gaussiano. Así:

∫tapa 1EdA + ∫tapa 2EdA + ∫rollo EdA = q/εo (1)

Pero en las tapas del cilindro el flujo de campo es cero, y en la superficie enrollada el valor de E

es el mismo en todos sus puntos, por lo que la última integral da EArollo = E(2πrl), donde l es la

longitud del cilindro dibujado, y (2πrl) es el área de la superficie del rollo. Sustituyendo en (1):

E2πrl = q/εo

Ahora: q = λl, por lo que: E = λ/(2π εor), que es la intensidad del campo a la distancia r del

alambre.

En cuanto a la diferencia de potencial entre dos puntos a las distancias r1 y r2, basta recordar:

E = − dV/dr, por lo que dV = −Edr = − [ λ/(2π εor)] dr.

E integrando: ΔV =

Esta integración da: ΔV = [λ/(2π εo)] ln(r1/r2)

18.7-El campo es radial, de simetría esférica, dado que las cargas están distribuidas uniforme-

mente por las esferas metálicas, por lo que las superficies gaussianas apropiadas serán esferas de

distintos radios r, acorde con las distintas zonas en que calcularemos la intensidad del campo

(esferas gaussianas representadas por líneas quebradas en la Fig.18.7S, próxima página); en estas

esferas el campo E es colineal con el dA en cada punto de la superficie, por lo que E.dA = E dA,

y la E seá constante en cada integral. Aplicando la ley de Gauss a cada zona resulta:

a) Ea = 0 en r < r1

P

Fig. 18.6S (a) (b) Vista en sección

Tapa 1 Tapa 2 Rollo

r

Vista lateral

Page 185: problemas selectos

185

puesto que una esfera gausssiana de radio r concéntrica con las dos metálicas, pero con su

superficie exterior a ellas, encuentra una carga neta en su interior igual a la suma de q1 y q2. (Vea

la esfera A3 en la figura).

18.8-La Fig. 18.8S muestra la situación planteada. Se han representado los datos q, m, Q y R, y la

forma en que debe quedar el sistema en equilibrio. Se han indicado, además, el ángulo θ que

debe formar el hilo con el eje horizontal, la fuerza de tensión T en el hilo, el peso mg de la esfera,

y un elemento de carga δq del anillo que crea la fuerza de repulsión δF representada, cuyas

componentes δFx paralela al eje de simetría y δFy perpendicular al eje, también se han represen-

tado. Debido a la simetría del anillo, habrá otro elemento de carga δq diametralmente opuesto al

representado que ejercerá una fuerza δF igual a la anterior, con iguales componentes x y y, pero

dirigida hacia arriba, lo que hará que las componentes verticales de las fuerzas eléctricas se anu-

len y solamente queden las componentes axiales, δFcosθ. Igualmente pasará con las restantes

parejas de cargas δq del anillo que estén diametralmente opuestas. La resultante en el eje x de

todas las fuerzas eléctricas será Fx =Σq δEi cosθ, donde δEi = k δqi / l2, por lo que:

Fx = (kq cosθ / l2) Σ δqi = (k qQ / l

2) cosθ (1)

Q

q, m

R

l

δq

T

mg

δF=qδE

θ

θ

δFx

δFy

Fig. 18.8S

r1

r2

r

r

A1

A2

A3 puesto que no hay carga interior en ninguna esfera

gaussiana que se tome en el interior de la esfera de

radio r1, como la esfera A1 de la Fig. 18.7S.

b)Eb = kq1 /r2 en r1 < r < r2

puesto que una esfera gaussiana de radio r tomada

concéntrica con la esfera metálica interior, pero con

su superficie entre la esfera metálica y la capa es-

férica, sólo encuentra en su interior la carga neta

producida por la esfera interior, q1. (Vea la esfera A2

de la figura)

c)Ec = k ( q1 + q2) /r2 en r > r2

q2

q1

Ec

Eb Ea=0

r

Fig. 18.7S

Page 186: problemas selectos

186

Ahora bien, la resultante de fuerzas sobre la esferita debe ser cero, tanto en la vertical como en la

horizontal: Tcosθ – Fx = 0 y Tsenθ – mg = 0.

De estas dos ecuaciones se elimina T y se obtiene: Fx = mg cosθ/senθ (2)

Igualando (1) con (2): (kqQ/l2) cosθ = mg cosθ/senθ

Los cosenos se cancelan y el seno es: senθ = R/l

Queda entonces: kqQR/mg = l3, y de aquí: l = (kqQR/mg)

1/3.

18.9-La Fig. 18.9S muestra un corte de la bobina y sus líneas de campo magnético: es un campo

uniforme en su interior (simetría plana), que se debilita hacia los extremos de la bobina y es

prácticamente nulo por su borde exterior paralelo a la bobina. Se ha dibujado una línea cerrada

de “circulación” en forma de rectángulo, con el lado ab paralelo a las líneas del campo del

interior de la bobina, los lados bc y da perpendiculares a las líneas del campo en el interior de la

bobina y están en un campo nulo por su parte exterior, y un lado, cd, en zona de campo nulo. La

corriente entra en el plano de la figura por la parte inferior de la bobina, y sale por la superior en

cada vuelta.

Al aplicar la ley de Ampere a la trayectoria cerrada escogida, tendremos: = µoI, donde I

representa la corriente neta que atraviesa la superficie cerrada de integración, que en nuestro caso

es N veces la corriente suministrada a la bobina (pasan N vueltas en la longitud l del rectángulo

de “circulación”). Entonces:

= ∫ab B dl cos 0 + ∫bcB dl cos 90 + ∫cd0 dl + ∫da B dl cos 90 = Bl.

Y de acuerdo con la ley de Ampere: Bl = µoNI. De aquí: B = µo(N/l)I.

El cociente N/l representa la densidad de vueltas de la bobina (lo empaquetado que están las

vueltas) y se representa por n = N/l. Entonces: B = µonI.

a b

c d

Fig. 18.9S

l

N corriente sale

corriente entra

Page 187: problemas selectos

187

por su borde exterior, y que las vueltas del enrollado están mucho más apretadas que lo que

aparecen en la figura. En este caso tomamos la línea de circulación como una circunferencia que

coincida con una línea de campo magnético dentro del toroide. Como r << R se puede asumir

que todas las líneas de campo en el interior del toroide tienen prácticamente el mismo radio R.

Entonces: = µoI se convierte en:

cos0 = µo NI,

donde N es el número total de vueltas enrolladas en el toroide. Como B es constante en la

circunferencia, la integral se convierte en B2πR = µo NI y despejando:

B = µo NI / (2πR) = µo nI , donde n = N/(2πR) es la densidad de vueltas.

A diferencia del solenoide, el campo B depende del radio del toroide, R, a través de la densidad

de vueltas, n.

Si el toroide se llena con un material ferromagnético, aumenta la permeabilidad magnética μ del

medio en κ veces (μ = κμo), donde κ es la permeabilidad magnética del medio relativa al vacío.

R

2r

Fig. 18.10S

corriente sale

corriente entra

1

2 B

18.10-La Fig. 18.10S muestra un corte del to-

roide (simetría circular), sus radios r y R, y la

geometría circular de las líneas del campo den-

tro del toroide.

Por fuera del toroide el campo es nulo, tanto

para puntos muy distantes del centro del toroi-

de como para puntos en la zona central, pues

aplicando la ley de Ampere, = µoI, en

circunferencias como la 1 y la 2, se tiene que la

coriente neta en el interior de la circulación es

cero, por lo que en la integral la inducción B

deberá ser cero también.

Asumimos que la corriente sale del plano del

papel por el borde interior del toroide y entra

Page 188: problemas selectos

188

19-Problemas con Cálculo Diferencial o Integral.

ja a ésta horizontalmente contra la pared P. Calcule la aceleración horizontal aM de la cuña

mientras la pesa resbala sobre ella.

19.3-Determine bajo qué coeficiente mínimo de fricción μmin de una barra fina homogénea sobre

el piso puede un hombre, lentamente y sin resbalamiento, levantarla del piso hasta la posición

vertical aplicando al extremo de la barra una fuerza siempre perpendicular a la barra.

19.4-Mediante un empujón comunicaron una velocidad vo = 10 m/s a un barco de juguete. Du-

rante el movimiento del barco actúa sobre él una fuerza de resistencia cuyo valor es propor-

cional a la velocidad respecto al fluido: F = -bv. Encontrar: a) la distancia recorrida por el barco

θ

M

m

Fig. 19.2

P

19.2-La cuña móvil de la Fig. 19.2

tiene una masa M, y por su lado iz-

quierdo tiene un ángulo de inclina-

ción θ. Una pesa de masa m puede

resbalar sin fricción sobre la cuña. El

hilo que une la pesa con la pared P es

ligero e inextensible. La polea es

ideal (sin masa ni fricción). La cuña

puede deslizar sobre la superficie ho-

rizontal sin fricción. Cuando la pesa

empieza a resbalar por la cuña, empu-

P

A

θ

a

hilo

Fig. 19.1

19.1-Sobre una cuña con un ángulo θ de in-

clinación sobre la horizontal está tendido un

hilo ligero inextensible (Fig. 19.1). Un extre-

mo del hilo está unido a la pared por el punto

A, a la altura del vértice superior de la cuña.

En el extremo B el hilo está unido a una pe-

queña pesa P. La cuña está inicialmente pega-

da a la pared, pero en cierto instante empieza

a alejarse de ella con aceleración constante a.

Determine la aceleración aP de la pesa res-

pecto al piso mientras esté sobre la cuña.

Page 189: problemas selectos

189

en el tiempo a lo largo del cual su velocidad se reduce a la mitad; b) la distancia total recorrida

por el barco hasta detenerse. Considere b = 0.50 kg/s, y la masa del modelo, m = 0.50 kg.

19.6-Dentro de la cámara de una rueda de bicicleta de radio exterior R cayó una pequeña piedra.

¿Bajo qué velocidad mínima de la rueda la piedrecita rotará junto con la rueda, si el coeficiente

de fricción estática de la piedra con la cámara es μ?

19.8-Un meteorito va con velocidad vo cuando pasa por el punto de su trayectoria más próximo

al Sol, a una distancia ro del centro del astro. a)¿Bajo qué relación entre vo y ro estaría el meteo-

rito en una órbita cerrada alrededor del Sol? b)Calcule su máximo alejamiento del Sol si la órbita

es cerrada. c)¿A qué distancia del Sol será máxima su componente de velocidad de aproximación

al astro? ¿A qué distancia será máxima su velocidad total? d)¿Al cabo de cuánto tiempo se

aproximará de nuevo el aerolito al Sol?

Fig. 19.9

19.9-Un largo cable homogéneo, de longitud l, se coloca

sobre una mesa horizontal sin fricción, cuya altura sobre

el piso es h (h < l), de modo que inicialmente un extremo

del cable cuelga rozando el piso (Fig. 19.9). Al abando-

narse, el cable comienza a resbalar sobre la mesa y cae al

piso. Calcule la velocidad con que salta al aire el extremo

superior del cable.

h

x O x

O

P

l

19.7-La varilla de la Fig. 19.7 rota con veloci-

dad angular constante ω. Un pequeño dispositi-

vo P parte de O con aceleración constante β

respecto a la varilla. Calcule la magnitud de la

fuerza sobre el dispositivo cuando éste alcance

la mitad de la varilla, sabiendo que la longitud

de ésta es l y la masa del dispositivo es m.

z

x

θ

Fig. 19.7

mg

F N

f

θ

θ

19.5-Por una pasarela de madera que forma un ángulo θ

con la horizontal, halan una caja uniformemente mediante

una cuerda (Fig. 19.5). El coeficiente de fricción de la

caja con la pasarela es μ. ¿Bajo qué ángulo θ con la

horizontal hay que dirigir la cuerda para arrastrar la caja

con el menor esfuerzo?

Fig. 19.5

Page 190: problemas selectos

190

19.10-Un satélite A de 250 kg está en una órbita aproximadamente circular alrededor de la Tierra

(T) a una altura igual al radio terrestre. (a) Calcule la rapidez y el período del satélite.

El satélite pierde energía mecánica con una rapidez promedio de 1.6×103 J por cada revolución

orbital. Asumiendo la aproximación de que la órbita es circular en cada vuelta, y que su radio

disminuye lentamente al cabo de varias vueltas, determine: (b) El período de revolución del

satélite.

Calcule los parámetros de la nueva órbita y explique si caerán a Tierra juntos o quedarán en una

órbita elíptica sin chocar con el planeta. Considere que el choque efectivo con la Tierra comienza

a 50 km sobre la superficie terrestre, donde ya se produce incandescencia sobre los objetos

celestes que penetran la atmósfera.

Datos: Masa Tierra: 5.98×1024

kg; Radio: Tierra: 6400 km; Constante de gravitación: G =

6.67×10-11

N.m2/kg

2)

Fig. 19.11

P

R

h

19.11-Una partícula P es lanzada con velocidad vo sobre la

superficie interior lisa de un cono vertical invertido, con base

de radio R y altura h. Esta velocidad inicial es tangente a la

superficie del cono y dirigida horizontalmente, y se le comu-

nica a la partícula en el mismo borde de la base del cono

(Fig. 19.11). (a) Exprese la velocidad de descenso vz de la

partícula en función de la altura z descendida a lo largo del

eje del cono. (b) Calcule la distancia máxima,|zmax|, que des-

ciende la partícula. (c) Calcule la velocidad vo con que debe

ser lanzada la partícula en el borde superior del cono para que

quede haciendo circunferencias horizontales, sin subir ni

bajar.

vo

A

T

B

h

(c) La altura h de vuelo al cabo de 20000 revolu-

ciones.

(d) Calcule la fuerza promedio retardadora sobre el

satélite.

(d) En esta nueva órbita el satélite choca con un

meteorito B de 50 kg que viaja hacia el centro de la

Tierra con la misma rapidez del satélite (Fig.

19.10). Asuma que el meteorito se incrusta en el

satélite y forman un cuerpo único sin pérdida apre-

ciable de masa.

Fig. 19.10

Page 191: problemas selectos

191

su través del frente de la onda de choque. La masa de la cuña es m y sus dimensiones a, b, c se

muestran en la Fig. 19.13b. La fricción se desprecia. Considerar que la velocidad adquirida por

la cuña es mucho menor que la velocidad de la onda (u<<v).

19.14-En dos recipientes metálicos idénticos ligeros echaron iguales masas de agua. Una pesada

esfera (cuya masa es igual a la del agua y su densidad es mucho mayor que la del agua) se cuelga

de un hilo fino no conductor en uno de los recipientes, de tal modo queda en el centro del

volumen de agua. Los recipientes son calentados hasta alcanzar la temperatura de ebullición del

agua y se ponen luego a enfriar. El tiempo de enfriamiento del recipiente con esfera hasta la

temperatura del medio circundante resulta k veces mayor que el tiempo de enfriamiento del

recipiente sin esfera. Determinar la relación de calores específicos del material de la esfera y del

agua: ce/ca.

Fig. 9.15

19.15-Un mol de gas ideal monoatómico está en una

sección de un recipiente térmicamente aislado. Se abre

un conducto entre esta sección y otra de igual volumen

al vacío y el gas se expande (Fig. 19.15). Calcule el au-

mento de entropía del gas al expandirse.

x x

p

po

2po

m

b

a

c

Fig. 19.13a Fig. 19.13b

19.13-Una onda de choque representa

una región de subida de presión que se

propaga en la dirección positiva del eje

x con gran velocidad v. En el momento

de la llegada de la onda, la presión re-

pentinamente se eleva. Esta dependen-

cia se muestra en la Fig. 19.13a. Deter-

mine qué velocidad u adquiere una cu-

ña inmediatamente después del paso a

Eje de simetría y’

que precesa for-

mando un ángulo

θ con ω

Eje fijo

ω

y

x’

z’

y’

θ

Fig. 19.12

19.12-Un volante que tiene forma de placa fi-

na circular y uniforme, tiene masa de 10.0 kg

y radio de 1.00 m, y está montado sobre un

eje que pasa a través de su centro de masas y

que forma un ángulo de 1° con la perpendi-

cular al plano del volante (Fig. 19.12). Si el

volante gira con relación a este eje con velo-

cidad angular de 25.0 rad/s. ¿Cuál es el torque

del par de fuerzas que actúa sobre la caja de

bolas del volante?

Page 192: problemas selectos

192

19.16-Una línea de campo E sale de una carga puntual positiva, +q1, bajo un ángulo α con la

recta que une a esta carga con otra negativa, −q2, también puntual. ¿Bajo qué ángulo β entra la

línea de campo en la carga negativa?

19.20-Un duro y fino anillo conductor yace sobre una superficie horizontal aislante y se encuen-

tra en un campo magnético uniforme cuyas líneas de inducción son horizontales. La masa del

anillo es M = 2.0 g, su radio es R = 4.0 cm y la inducción magnética es B = 0.50 T. ¿Qué co-

rriente I tendrá que pasar por el anillo para que comience a levantarse de la superficie horizontal?

B

Q

ΦB

19.19-Se distribuye una carga Q uniformemente por

un fino aro dieléctrico, el cual reposa sobre una

superficie lisa horizontal. Un campo de inducción

magnética, perpendicular al plano del aro (Fig.

19.19), varía de 0 a Bo. ¿Qué velocidad angular ad-

quiere el aro?

9.18-Dos capas planas, de espesor d cada una, están

cargadas volumétricamente con densidades +ρ y –ρ. Una

partícula con carga negativa –e y masa m se acerca

volando a la capa cargada positivamente, con velocidad v

y dirigida bajo un ángulo θ con la superficie de la capa

(Fig. 19.18). Determine: a) Bajo qué velocidad máxima la

partícula no podrá penetrar aún en la capa de carga

negativa. b) ¿Al cabo de qué tiempo y a qué distancia del

punto A abandonará la placa positiva cuando la partícula

llega justamente con la velocidad calculada en A?

A

19.17-En un plasma uniforme con concentración n de car-

gas (número de cargas de cada signo por unidad de volu-

men), todos los electrones, que inicialmente se encuentran

en una capa de espesor d, se desplazan una distancia d por

la dirección normal a la capa, como resultado de una fluc-

tuación espontánea. Encuentre la intensidad del campo

eléctrico en puntos de la superficie S de separación entre

las cargas (Fig. 19.17) y la frecuencia de oscilación de las

capas que así se separan en el plasma.

x

d

+

+

+

+

+

+

S

+

+

+

+

+

+

d

x O

Fig. 19.17

d d

+ρ −ρ θ

-e,

m,

v

Fig. 19.18

Fig. 19.19

Page 193: problemas selectos

193

19-Problemas con Cálculo Diferencial o Integral. Soluciones.

Cuando la cuña se acelere a la derecha, cambiarán las coordenadas de la cuña, xc, y las de la

pesa, xp, yp. Si derivamos dos veces respecto al tiempo cada una de estas ecuaciones, obtén-

dremos las aceleraciones por componentes para la pesa, y luego, por Pitágoras, hallaremos su

aceleración total. Derivando una primera vez, teniendo en cuenta que h y L son constantes,

obtenemos las velocidades:

vpx = vc – vccosθ y vpy = vcsenθ

Derivando de nuevo: apx = ac – accosθ y apy = acsenθ

Pero ac = a, por lo que: apx = a – a cosθ y apy = a senθ

Entonces: ap =

= a = 2a sen(θ/2)

Nmcosθ + Tsenθ – mg = maym (2)

θ

M

m

19.2-La Fig. 19.2S muestra dos ejes

x,y coincidentes con la pared y el pi-

so de la Fig. 19.2, con sus direccio-

nes positivas indicadas por las saetas.

El hilo aparece como estaba en la

Fig. 19.2, pero la cuña se ha dibujado

desplazada para una representación

más cómoda de los vectores.

Ecuaciones de fuerza para la pesa:

Nmsenθ – Tcosθ = maxm (1)

Fig. 19.2S

P

θ

mg

Mg

Nm

Nm NM

T

T

T

x

y

xm xM

h θ

θ

θ

θ

19.1-La Fig. 19.1S muestra dos ejes x, y que

nos permitirán localizar la pesa en cualquier

instante por las coordenadas del extremo mó-

vil del hilo. Éste tiene una longitud L cons-

tante. Las coordenadas del extremo del hilo

son:

xp = xc + (L – xc)cosθ

yp = h – (L – xc)senθ

P

A

θ

y

h

xc xp x

yp

Fig. 19.1S

Page 194: problemas selectos

194

Ecuación de fuerza para la cuña:

Tcosθ − Nmsenθ − T = MaM (3)

De (1) y (2): T = m(aym senθ − axm cosθ + g senθ) (4)

y Nm = m(axm senθ + aym cosθ + g cosθ) (5)

Pero las aceleraciones ax y ay de la pesa pueden calcularse en función de aM estableciendo las

relaciones entre sus coordenadas:

xM + (xm – xM) / cosθ = L , donde L es la longitud del hilo (constante), y

ym = h – (xm – xM) tanθ

Derivando dos veces respecto al tiempo cada ecuación:

aM + (axm – aM) / cosθ = 0 Y: aym = – (axm – aM) tanθ = (aM – axm) tanθ

De estas dos ecuaciones se obtiene: axm = aM (1 – cosθ) y aym = aM senθ

Sustituyendo en (4) y (5) las expresiones de axm y aym se obtiene:

Nm = m(aM senθ + g cosθ) y T = maM (1 – cosθ) + mg senθ

Sustituyendo estas dos expresiones en (3) y despejando aM:

aM = −mg senθ / [M + 2m(1 – cosθ)]

El signo negativo indica que la cuña se acelera hacia la pared, como era de esperar.

ni la fuerza F producen torques, y los torques de N y de f deberán compensarse.

Si llamamos l a la longitud de la varilla, estos torques serán:

O

F

N

f

mg

θ

Fig. 19.3S

19.3S-La fuerza F deberá ser variable para compensar en cada

momento a las restantes fuerzas y torques que actúan sobre la va-

rilla. La condición de que no resbale en el proceso de alzarla im-

plica que nunca la fuerza de fricción estática sobrepase al valor

de la fricción estática máxima, f < fmax = μN, o cuanto más, que la

iguale.

La resultante de torques en todo momento debe ser nula si se alza

la varilla lentamente (en equilibrio). La suma de torques se puede

calcular respecto a cualquier punto o “eje”. Tomaremos el punto

O de la figura como eje de torques, donde se cortan las líneas de

acción del peso y de la fuerza F. Respecto a este punto, ni el peso

θ

Page 195: problemas selectos

195

ηN = N(l/2)cosθ y ηf = f [(l/2)senθ + (l/2)/senθ]

donde los brazos de fuerzas se obtiene a partir de la Fig. 19.3S. Como los torques son iguales y

opuestos:

N(l/2)cosθ = f [(l/2)senθ + (l/2)/senθ]

De aquí se despeja: f = N cosθ / [senθ + 1/senθ] = N senθ cosθ / (1 + sen2θ)

O: f = N senθ cosθ / (2sen2θ + cos

2θ) = N / (2tanθ + cotθ)

Si llamamos x = tanθ (1), entonces f = N / (2x +1/x)

Y deberá cumplirse: N / (2x +1/x) ≤ μN O sea: μ ≥ 1 / (2x +1/x) (2)

donde x es una función de θ según (1). La mínima μ necesaria para que no resbale la varilla bajo

ningún ángulo será la que asegure la igualdad en (2) para el máximo valor de la función F(x) =

1/ (2x + 1/x), correspondiente al ángulo que mayor fricción requeriría para no resbalar la varilla.

Para averiguar este máximo valor de F(x) comenzamos por calcular para qué valor de x se

produce el máximo de F(x), o el mínimo de la función H(x) = (2x +1/x): dH/dx = 2 – 1/x2 = 0,

a lo que corresponde x = 1/ .

Entonces: μmin = 1 / (2/ + ) = / 4

19.4-a)Tenemos: F = ma = -bv, o m dv/dt = -bv. De aquí: dv/v = (-b/m)dt.

Integrando entre los límites correspondientes:

=

O: ln(v/10) = −bt/m, de donde resulta: v =10e-bt/m

(1)

Pero v = dx/dt, por lo que dx = 10e-bt/m

dt. E integrando:

=

,

Que da: x = (10b/m) (1 – e-bt/m

) (2)

(Se supuso x = 0 en t = 0, cuando el bote comienza a moverse tras el empujón).

De la ecuación (1) calculamos en qué tiempo la velocidad se reduce a la mitad:

5 = 10e-bt/m

, o ½ = e-bt/m

, y de aquí: t = -(m/b) ln(½) = -(0.5/0.5) ln(½) = 0.69 s

Sustituyendo en (2): x = 10b/m(1 – ½) = 5 m

b)Según (1), la velocidad se reduce a cero cuando t →∞, y en (2) se obtiene para ese tiempo

x = 10 m.

Page 196: problemas selectos

196

En la práctica el bote se detiene en un tiempo tal que lo que avance de ahí en adelante ya no es

observable por el efecto de las pequeñas olas, el viento, los choques moleculares del agua con

las paredes del bote, etc. El tiempo real en el que se detiene es finito, de algunos segundos, o

minutos.

Otro modo de resolver el problema es poniendo v en función de x directamente: de (1) se tiene

que e-bt/m

= v/10. Sustituyendo en (2): x = (10b/m) (1 – v/10) = b/m(10 – v) = 10 − v.

Cuando v = 5 m/s (mitad de la velocidad inicial) se obtendrá x = 5 m, y cuando v = 0: x = 10 m

Fcos(θ – θ) – μ[mg cosθ – Fsen(θ – θ)] – mg senθ = ma. Y despejando F:

F = [ma + mg(senθ + μcosθ)] / [ cos(θ – θ) + μsen(θ – θ)]

Notamos que para cualquier valor de υ el valor mínimo de F se obtendrá cuando a = 0 (ascenso

en equilibriol), quedando entonces:

F = [mg(senθ + μcosθ)] / [cos(θ – θ) + μsen(θ – θ)]

Minimizar el valor de F todo lo posible dependerá ahora del ángulo θ con que se hale la caja. En

particular, hay que ver para qué valor de θ se hace mínimo el cociente:

[(senθ + μcosθ)] / [cos(θ – θ) + μsen(θ – θ)],

lo cual ocurrirá cuando sea máximo el denominador [cos(θ – θ) + μsen(θ – θ)] (notar que el

numerador no depende de θ). Llamemos f(θ) = [cos(θ – θ) + μsen(θ – θ)] y calculemos para qué

valor de θ se hace máxima esta función, o sea, para qué valor de θ se cumple: df/dθ = 0.

Así: df/dθ = −sen(θ – θ) + μcos(θ – θ) = 0, y de aquí: tan(θ – θ) = μ

Por tanto: θ = θ + tan-1

μ.

Halando bajo este ángulo con la horizontal el esfuerzo será mínimo para subir el cuerpo por la

pasarela, en equilibrio (sin acelerar).

mg

F N

f

θ

θ

19.5-Las ecuaciones dinámicas en la dirección ascendente

del plano y en la dirección normal al plano son:

Fcos(θ – θ) – μN – mg senθ = ma

Fsen(θ – θ) + N – mg cosθ = 0

De la segunda ecuación:

N = mg cosθ – Fsen(θ – θ). Sustituyendo en la primera:

Fig. 19.5

Page 197: problemas selectos

197

N + mg cos(π – θ) = mv2/R donde v es la velocidad de la rueda sobre el piso y la de la piedra

alrededor del eje de rotación, ya que la rueda se mueve en rodadura pura (sin resbalar).

Arreglamos: f = mg senθ (1) y N = mv2/R + mg cosθ (2)

Para que la piedra no resbale del punto de la cámara en que se encuentre, la fricción necesaria no

debe sobrepasar la fricción estática máxima, μN, o como caso límite: f = μN. Esto ocurrirá para el

valor Nmin posible. De (2), N será mínima para el ángulo θ cuando v sea la mínima posible:

Nmin = mvmin2/R + mg cosθ

La fricción estática límite será:

f = μNmin = μ(mvmin2/R + mg cosθ)

Pero esta fricción debe satisfacer también la ecuación (1):

mg senθ = μ(mvmin2/R + mg cosθ)

Despejamos vmin2: vmin

2 = gR (μ

-1senθ – cosθ) (3)

Esta es la velocidad mínima permisible a la rueda para que la piedra no resbale en un punto

localizado bajo un ángulo θ con la vertical OQ y dé vueltas alrededor del eje central. Ahora, para

distintos ángulos θ la velocidad mínima será diferente, y con cierto valor de vmin la piedra podría

no resbalar en un punto, pero sí hacerlo en otro superior. Nos interesa el mayor valor de las vmin

dadas por (3) para asegurar que bajo ningún ángulo resbale la piedra. Esto se averigua maximi-

zando la función: f(θ) = (μ-1

senθ – cosθ), presente en (3):

df/dθ = μ-1

cosθ + senθ = 0

De aquí: tanθmin = −1/μ

Para calcular vmin por (3) necesitamos el seno y el coseno de θ.

Podemos auxiliarnos del triángulo adjunto, que satisface

tanθm = −1/μ.

R

19.6-Supongamos la piedra P en una posición cualquiera que

forma un ángulo θ con la vertical OQ, rotando con la rueda. En

la Fig. 19.6S hemos representado la fricción estática f entre la

piedra y la cámara. El sistema de referencia inercial que toma-

mos para plantear las ecuaciones es uno con centro en O y que

viaje con velocidad constante junto con la rueda. Las ecuaciones

en los ejes normal y tangencial a la rueda son:

f – mg sen(π – θ) = 0 pues no hay aceleración tangencial (velo-

cidad constante)

N

f

mg

θ

π-θ

P

O

Q

Fig. 19.6S

θm

Fig. 19.6S

Page 198: problemas selectos

198

Así: senθm = y cosθm =

Entonces: f(θmin) = μ-1

( )

Pero vmin2 tiene que ser positiva, por lo que solamente valdrá la combinación con signos positi-

vos:

vmin2 = gR[ ) ]

Y de aquí: vmin = [(gR/μ) ]1/2

y θ, donde r es la distancia (siempre positiva) del dispositivo al eje z, y θ es el ángulo que

localiza a la partícula sobre la varilla, medido a partir de un eje x fijo (ver figura). En

coordenadas polares hay dos componentes de velocidad mutuamente perpendiculares: vr y vθ,

donde vr mide la velocidad radial de alejamiento y acercamiento al eje z (positiva en el

alejamiento) y vθ es la velocidad de “barrido” alrededor del eje z (positiva en la dirección en que

crece θ). Se cumplen las siguientes relaciones:

vr = dr/dt , vθ = r dθ/dt

La derivada dθ/dt representa la velocidad angular de la varilla: ω = dθ/dt.

Asociada a cada componente de velocidad hay un vector unitario polar, y . El primero apunta

en la dirección creciente de la variable r, y el segundo apunta en la dirección creciente de la

variable θ, normal a la dirección radial. Con tales vectores unitarios se puede expresar la velo-

cidad del dispositivo como:

v = vr + ωr

La aceleración saldrá de derivar esta velocidad respecto al tiempo. Si conocemos la aceleración,

podremos calcular la fuerza de la varilla sobre la partícula. Al derivar hay que tener presente que

aunque los vectores unitarios son constantes en magnitud, no lo son en dirección:

= cosθ i + senθ j y = −senθ i + cosθ j (Vea la Fig. 19.7S)

O

P

l

19.7-El movimiento rectilíneo del dispositivo

con aceleración constante es respecto a la varilla

que rota. Respecto al laboratorio, el dispositivo

realiza una trayectoria compleja (una espiral) y

tiene componentes de velocidad y de acelera-

ción en dos direcciones.

Plantearemos las ecuaciones usando coordena-

das polares. En este caso las coordenadas son r

z

x

θ

Fig. 19.7

Page 199: problemas selectos

199

Sus derivadas son:

d /dt =−senθ dθ/dt i + cosθ dθ/dt j = ω(−senθ i + cosθ j) = ω (1)

d /dt = −cosθ dθ/dt i − senθ dθ/dt j = −ω(cosθ i + senθ) = −ω (2)

La derivada de v será: dv/dt = dvr /dt + vr d dt + ωdr/dt + ωr d /dt

Y usando (1) y (2): dv/dt = dvr /dt + vr ω + ωdr/dt − ωr ω

O sea: dv/dt = ar + vr ω + ωvr − ω2r = ar + 2vr ω − ω

2r (3)

Esta expresión nos da la aceleración del dispositivo respecto al laboratorio en función de las

coordenadas polares asociadas a la varilla que rota. En (3), ar es la aceleración radial observada

respecto a la varilla: ar = β, y vr es la velocidad radial del dispositivo, que se relaciona con la

aceleración radial constante por vr2 = 2βr. Si el dispositivo salió prácticamente del eje z, su velo-

cidad cuando llegue a la mitad de la varilla (r = l/2) será vr = (β l)1/2

. Entonces, la aceleración

(respecto al laboratorio) del dispositivo ligado a la varilla cuando esté a la mitad de su longitud

será:

dv/dt = (β − ½ω2l) + 2ω(β l)

1/2

Entonces, la fuerza resultante sobre el dispositivo será:

F = m[(β − ½ω2l) + 2ω(β l)

1/2 ]

El primer término representa la fuerza radial que actúa sobre la partícula, producto de su meca-

nismo de interacción con la varilla (ruedas dentadas, o algo similar) y la segunda representa la

acción lateral de la varilla sobre el dispositivo para acelerarlo en la dirección de barrido de la

varilla. El peso del dispositivo y una fuerza hacia arriba producida por la varilla se compensan.

Nota:

En el sistema no inercial de la varilla que rota, la única aceleración que se mide al dispositivo es

ar, pues respecto a la varilla el dispositivo no puede moverse en la dirección normal a ella. Si de

la ecuación (3) despejamos el término de aceleración radial se obtiene:

ar = dv/dt − 2vr ω + ω2r

Multiplicamos por la masa del dispositivo y obtenemos:

mar = mdv/dt – 2mvr ω + mω2r

En el sistema de la varilla (no inercial) la fuerza que miden a lo largo de la varilla es la que resul-

ta de sumar las fuerzas de interacción reales (empuje lateral de la varilla, empuje del engranaje),

presentes en el término mdv/dt , con dos “fuerzas de inercia”: la de “Coriolis”, de valor 2mvr ω,

“opuesta” al barrido de la varilla (− ), y la fuerza “centrífuga”, de valor mω2r, que empuja al

θ

θ

x

y

Fig. 19.7S

Page 200: problemas selectos

200

dispositivo alejándolo del eje (+ ). Respecto a la varilla, las fuerzas que hacen el contacto con la

varilla son para oponerse a estas “fuerzas de inercia”.

donde M es la masa del Sol y m, la del meteorito. Si la energía total es negativa, el meteorito

permanece en órbita cerrada alrededor del Sol; si la energía es positiva, el meteorito puede es-

capar para siempre de la atracción solar. Si es nula la energía total, aún podría escapar el meteo-

rito, aunque quedaría casi en reposo al alejarse mucho del Sol. La condición para la órbita ce-

rrada es entonces: E < 0. O:

½ mv2 – GmM/r < 0

Aplicando esta condición al punto en que pasa más próximo al Sol:

½ mvo2 – GmM/ro < 0

Despejando: vo < que será la condición de órbita cerrada para este meteorito.

b)El punto más lejano está a una distancia rl del Sol y el meteorito pasará por allí con velocidad

vl (ver Fig. 19.8). Se conservan la energía y el momento angular del meteorito respecto al Sol:

½ mvo2 – GmM/ro = ½ mvl

2 – GmM/rl (1)

y mvoro = mvlrl (2)

En ambas ecuaciones se cancela la masa del meteorito. Se calculan de ellas vl y rl. De (2):

vl = vo ro / rl. Sustituimos en (2) y arreglando se llega a:

(vo2 – 2GM/ro) rl

2 + 2GM rl – vo

2ro

2 = 0

Se resuelve la ecuación de segundo grado y se obtiene: rl = ro y rl = ro / [2GM/(rovo2) − 1]. La

segunda solución es la del punto más lejano.

c)Para buscar la distancia a la cual es máxima la velocidad de aproximación vr es conveniente

usar la ecuación de la energía con su energía cinética desdoblada en sus dos componente polares:

v

vr

r

θ ro

vo

Fig. 19.8

19.8-Usaremos coordenadas polares como en el ejercicio

anterior. En la figura se indican las coordenadas r,θ, así

como las componentes polares de la velocidad del me-

teorito.

a)La energía mecánica total del meteorito en cualquier

instante está dada por:

E = ½mv2 – GmM/r

vl rl

Page 201: problemas selectos

201

E = ½mvr2

+ ½mvθ2

– GmM/r donde E es una constante del movimiento, conocida a partir de

los datos en el punto más próximo al Sol. Por otro lado, la velocidad de barrido se puede expre-

sar en función de otra constante del movimiento, el momento angular, conocida también a partir

de los datos del punto más cercano. Así: L = mvθr, por lo que vθ = L / (mr). Sustituyendo ésta en

la ecuación de la energía se obtiene:

E = ½mvr2

+ L2/ (2mr

2) – GmM/r (3)

Podemos ahora analizar el problema como uno unidimensional, donde solamente hay energía

cinética radial K = ½mvr2, y una “energía potencial” U = ½ L

2/ (mr

2) – GmM/r. Como la energía

total se conserva, cuando disminuya la potencial deberá aumentar la cinética, y ésta será máxima

allí donde la potencial sea mínima. Y donde sea máxima la energía cinética, será máxima la

velocidad radial de aproximación. Debemos, pues, minimizar la función U:

dU/dr]min = −L2/(mr

3) + GMm/r

2 = 0

lo que conduce a: r = L2/(Gm

2M) = (mvoro)

2/(Gm

2M) = vo

2ro

2/(GM)

En cuanto a su máxima velocidad total en la órbita, será al pasar por el punto más cercano al Sol,

que es cuando menor energía potencial gravitatoria tiene, y por dato es vo.

d)El tiempo en aproximarse al Sol de nuevo representa su período de revolución, T, que puede

ser calculado por la tercera ley de Kepler, T2 = (4π

2/GM)A

3, donde M es la masa del centro de

atracción, el Sol, y A es la longitud del semieje mayor de la órbita, A = ½ (ro + rl), donde ya ro y

rl se conocen.

19.9-Llamemos λ a la densidad lineal de la cuerda (λ = dm/dl) que será constante a lo largo de la

cuerda pues es homogénea. Si M es la masa total de la cuerda, su densidad lineal será λ = M/l.

Cuando la cuerda empiece a resbalar, la fuerza que hala hacia abajo a la cuerda es la masa de la

parte que cuelga. Y la masa total que en cada instante se acelera es la masa que cuelga más la

que queda sobre la mesa, pero la que llegó al piso ya no se acelera más.

Tomemos un eje x con su origen en el borde de la mesa, positivo hacia afuera de la mesa. La

longitud de cuerda sobre la mesa es en cada instante −x, y la masa total que se acelera en cada

instante será λ (h – x). Entonces:

Fuerza que hala = (masa que se acelera) × (aceleración)

O sea: λh g = λ (h – x) a, donde a se puede medir por la aceleración del extremo de la cuerda en

el eje x: a = dv/dt. Entonces: λh g = λ (h – x) dv/dt (1)

Aquí se cancela λ. Además: dv/dt = (dv/dx)×(dx/dt), de acuerdo con la regla de la cadena en las

derivadas, y como dx/dt = v, entonces dv/dt = v dv/dx. Sustituimos en (1) y:

Page 202: problemas selectos

202

hg = (h – x) v dv/dx

Separamos variables para integrar: gh dx/(h – x) = v dv

Integramos:

=

Los límites de x van desde la posición inicial del extremo de la cuerda en la mesa, (h – l), hasta

su posición al saltar de la mesa, (0); y los límites de la velocidad van desde cero, (0), hasta la

velocidad que tenga la punta al saltar, (v). Integrando:

−gh ln (h – x) |h-l = v2/2|0

Evaluando y despejando: v = .

19.10-Los datos son: mA=250 kg, mB = 50 kg, h = RT, ΔE = −1.4×103 J por revolución, N =

20000 rev, además de los datos de la Tierra.

(b)La fuerza centrípeta es producida por la de gravitación: GMTmA/r2 = mAv

2/r, donde r = 2RT.

De aquí: v = = 5.58×103 m/s (se evaluó para r = 2RT)

Para el período: v = ωr, o sea: = 2π (2RT)/T, de donde se obtiene:

T = 2π = 1.44×10

4 s = 4.00 h

(c)La energía mecánica total es:

E = ½ mAv2 – GMTmA/r = ½ GMT mA /r – GMT mA /r = –½ GMT mA /r

Entonces, diferenciando: ΔE = ½ (GMT mA /r2)Δr = ½ (GMT mA /4RT

2)Δr = (mAg/8)Δr y de

aquí:

Δr = [8 / (mAg)] ΔE (1)

Ésta es la variación del radio orbital en función de la variación de energía. Si ΔE es la pérdida de

energía en una vuelta, el Δr será negativo y el radio disminuye en cada vuelta por el valor dado

por (1). En N vueltas disminución total del radio orbital será:

Δr =N [8 / (mAg)] ΔE, que para N = 20000 da: Δr = –91.4 km

La altura de vuelo no ha variado notablemente en 20000 vueltas. Es: h = 6400 – 91 = 6309 km.

d) El nuevo radio orbital será r’ = RT + h = 6400 + 6309 = 12709 km = 12.309×106 m.

La energía perdida en una vuelta es igual al trabajo realizado por la fuerza retardadora media:

0 v

Page 203: problemas selectos

203

ΔE = Fdcos180 = –F2πr = –F4πRT. De aquí: F = –ΔE/(4πRT) = 1.74×10-5

N

e)Necesitamos ahora la velocidad del satélite en la órbita circular al momento del choque,

v = = 5.60×103 m/s (ligeramente mayor que la velocidad original de 5.58×10

3 m/s)

Planteamos la conservación del momento lineal por componentes:

mBv = (mA + mB)v’cosθ y mAv = (mA + mB)v’senθ

y resolviendo se obtiene θ = 78.7o y v’=4.76 ×10

3 m/s.

Cuando los dos cuerpos unidos entren en su nueva órbita alrededor de la Tierra, se conservará su

momento angular durante cada revolución respecto al centro de la Tierra (solamente después de

cientos de vueltas es que la fuerza retardadora hará perceptible la variación de momento

angular).

En el punto del choque, el momento angular respecto a Tierra será:

Lo = (mA + mB) v’r’ sen(180 – θ) = (mA + mB) v’r’ senθ

donde r’ = 12709 km. Y en el punto más cercano a la Tierra, si no ha chocado antes con ella, el

ángulo entre los vectores rf y vf será de 90o, por lo que en ese punto de máximo acercamiento:

Lf = (mA + mB)vf rf

Igualando ambas expresiones: (mA + mB) v’r’senθ = (mA + mB) vf rf

O: v’r’ senθ = vf rf (2)

Aquí son incógnitas vf y rf. Pero podemos plantar también la conservación de la energía en una

revolución, por lo que:

½ (mA + mB) v’2 – GMT(mA + mB)/ r’ = ½ (mA + mB) vf

2 – GMT(mA + mB)/ rf

O: v’2 – 2GMT/ r’ = vf

2 – 2GMT/ rf (3)

mAv

mBv

(mA+mB) v’

θ

Hacia el centro

de la Tierra

En el choque se conservará el vector momento

lineal del sistema satélite-meteorito respecto al sis-

tema centrado en la Tierra:

(pA + pB)antes = (pA + pB)desp

Después del choque los dos cuerpos forman uno

solo de masa (mA + mB) y velocidad v’, que forma

un ángulo θ con la dirección radial (Fig.19.10S). Fig. 19.10S

Page 204: problemas selectos

204

Las ecuaciones (2) y (3) permiten el cálculo de rf y vf, pero estamos solamente interesados en el

valor de rf. De (2): vf = v’ (r’/rf) senθ . Sustituyendo en (3):

v’2 – 2GMT/ r’ = v’

2 (r’/rf)

2sen

2θ – 2GMT/ rf

De aquí sale una ecuación cuadrática en rf:: rf 2(v’

2 – 2GMT/r’) + rf (2GMT) – (v’ r’ senθ)

2 = 0

Sustituyendo valores: (–3.345×107) rf

2 + (7.977×10

14) rf – (4.5690×10

21) = 0

Resolviendo la ecuación da dos soluciones (una para el apogeo y otra para el perigeo):

rf = 14.3×106 m (apogeo, o extremo más lejano de la elipse) y rf = 9.56×10

6 m (perigeo, o

extremo más cercano de la elipse). Si la máxima aproximación al centro de la Tierra es de 9560

km y el radio de choque es de 6450 km (incluida la atmósfera), entonces el cuerpo compuesto

por el satélite y el meteorito siguen aún en una órbita sin tocar Tierra, con una altura mínima

sobre la Tierra de: 9560 – 6450 = 3110 km

Plantearemos las ecuaciones usando coordenadas cilíndricas. En este caso las coordenadas son r,

θ, z, donde r es la distancia (siempre positiva) de la partícula al eje z, θ es el ángulo que localiza

a la partícula, rotando de x hacia y, y la coordenada z es la distancia que separa a la partícula del

plano x,y (negativa hacia abajo). En coordenadas cilíndricas hay tres componentes de velocidad

mutuamente perpendiculares: vr, vθ, vz, donde vr mide la velocidad radial de alejamiento y

acercamiento al eje z (positiva en el alejamiento), vθ es la velocidad de “barrido” alrededor del

eje z (positiva del eje x hacia el y), y vz es la velocidad vertical de subida o bajada (positiva hacia

arriba en el sistema de la figura). Se cumplen las siguientes relaciones:

vr = dr/dt , vθ = r dθ/dt , vz = dz/dt

z

Fig. 19.11S

P vo 19.11-a)Si la pared es lisa (sin fricción) se va a conservar la

energía mecánica en este problema, pues de las dos fuerzas

presentes, la de gravedad es conservativa y la normal no

trabaja en esta trayectoria. Además, respecto al eje central del

cono, la fuerza normal y el peso crean torques en direcciones

horizontales, pero no producen componente vertical de torque

(analice el producto vectorial r×F para cada fuerza, con r

saliendo perpendicularmente del eje hacia la partícula) por lo

que se conserva la componente vertical del momento angular.

Contamos con dos ecuaciones de conservación: energía me-

cánica, E, y momento angular vertica, Lz, respecto al eje cen-

tral.

y

x

h

h+z

z<0

r

θ

Page 205: problemas selectos

205

(Notar que si el movimiento fuera circular en el plano x, y entonces vr = 0, vz = 0 y vθ = ωr,

donde ω = dθ/dt sería la velocidad angular).

La energía potencial gravitatoria será Ug = mgz. Entonces, la energía mecánica total en la altura

inicial z = 0, con velocidad de barrido vθ = vo, será solamente energía cinética, Eo = ½ mvo2, y en

una altura z cualquiera será E = ½ m(vr2 + vθ

2 + vz

2 ) + mgz. Y como Eo = E, entonces:

½ mvo2 = ½ m(vr

2 + vθ

2 + vz

2 ) + mgz (1)

El momento angular axial vertical en la altura inicial z = 0 es Lzo = mRvo, y a una altura cual-

quiera será Lz=mrvθ. Entonces, como se conserva este momento axial:

mRvo = mrvθ (2)

De (2): vθ = Rvo/r (3)

Por otra parte, por semejanza de triángulos (ver Fig. 19.10): r / (h + z) = R / h (donde z < 0)

De aquí: r = (h + z) R / h (4)

Y derivando respecto al tiempo: dr/dt = (R/h) dz/dt. Por tanto:

vr= (R/h)vz (5)

Sustituimos (3) y (5) en (1) y se obtiene:

½ mvo2 = ½ m [(R

2/h

2) vz

2 + (R

2/r

2) vo

2 + vz

2] + mgz (6)

Si sustituimos r por (4), cancelamos las masas y multiplicamos por 2, la ecuación (6) toma la

forma:

v02 = (R

2/h

2) vz

2 + [h

2/ (h + z)

2] vo

2 + vz

2 + 2gz = vz

2(1 + R

2/h

2) + vo

2 h

2/(h + z)

2 + 2gz

Despejamos vz =

b)Cuando la partícula baje la máxima distancia zmax, en ese momento vz = 0. Eso significa que en

el radical se cumplirá:

De aquí se puede despejar zmax. Después de algunas maniobras algebraicas se obtiene una ecua-

ción cúbica en la que una solución es zmax = 0, la cual corresponde a la posición inicial en la que

toda la velocidad es horizontal y vz = 0. Eliminada esta solución queda la ecuación cuadrática

siguiente:

2gzm2 + (4gh – vo

2) zm +2h (gh – vo

2) = 0

Es una ecuación cuadrática en zm que da:

Page 206: problemas selectos

206

zmax = [(vo2/4g) − h]

Arreglando: zmax =

]

Sólo tiene sentido la solución negativa, pues la partícula desciende por el cono:

zmax =

]

a)Si zmaz = 0, significaría que se le dio una velocidad a la partícula que no le da ni para subir ni

para bajar: se quedaría en órbita circular en el borde del cono, con la componente vertical de la

normal compensando su peso, y con la componente horizontal actuando como fuerza centrípeta

en esa circunferencia de radio R. Haciendo zmax= 0 y despejando vo se obtiene: vo = .

Este resultado está es obtenible también por la segunda ley de Newton aplicada a la partícula en

movimiento circular por el borde del cono. Si llamamos θ al semiángulo del cono (Fig. 19.11)

tendríamos:

Horizontal: N cosθ = mv2/R

Vertical: N senθ – mg = 0

Eliminando N: tanθ = gR/ v2

Pero, de la figura: tanθ = R / h

Entonces: v = , que es el resultado antes encontrado.

Si vo > la partícula se saldría del borde del cono hacia arriba. Y si vo < , la partícula

descenderá hasta la zmax calculada antes y volverá a subir hasta z = 0: la partícula se mantendrá

subiendo y bajando en espirales por el interior del cono.

19.12-Cuando un cuerpo de cierta simetría rota alrededor de un eje fijo que no coincide con el

Eje de simetría y’

que precesa for-

mando un ángulo

θ con ω

ω L

L precesa alrededor de

ω, formando un ángulo θ

con el eje y’de simetría

tal que θ<θ

θ y’ Lz’

Ly’ θ

ω L θ

y’ Lz=Ln Ly

θ

Fig. 19.12Sa

y Fig. 19.12Sb

y

Eje fijo

ω

y

x’

z’

y’

θ

Fig. 19.12

Page 207: problemas selectos

207

eje de simetría, el vector velocidad angular, ω, estará orientado según el eje fijo, pero el vector

momento angular del cuerpo, L, precesará alrededor de ese eje de rotación. Habrá una com-

ponente de L a lo largo del eje fijo que permanecerá constante, pero la componente perpendicular

al eje, Lz en la Fig.19.12Sa, rotará alrededor de dicho eje (en general, esta componente perpen-

dicular al eje de simetría tendrá componentes Lz, Ly). La variación de L se deberá al torque de un

par de fuerzas actuando sobre la caja de bolas del cuerpo.

Tomaremos unos ejes x, y, z tales que el eje y coincida con la dirección del eje de rotación fijo

(son ejes que no rotan). En estos ejes la velocidad angular tendrá componentes ωx = ωz = 0, y

ωy = ω. Sea otro sistema con ejes x’, y’, z’ tales que el eje de simetría del disco coincide con el

eje y’ y los ejes x’, z’ coinciden con dos diámetros del disco, y rotan con él. El eje y’ precesa

alrededor del y y los ejes x’, z’ se toman ligados al disco de tal manera que siempre ω se descom-

pone sobre los ejes y’ y z’, pero no sobre el x’: ωx’ = 0, ωy’ = ω cosθ y ωz’ = ω senθ, donde θ =

1o. El momento angular L tendrá también una componente sobre el eje z’ y otra sobre el eje y’,

pero no sobre x’ (el momento angular precesa entre el eje de simetría y el eje fijo, con la misma

frecuencia ω que precesa el eje de simetría y’): Lx’ = 0, Ly’ = Iy’ ωy’ y Lz’ = Iz’ ωz’ , donde Iy’ =

MR2/2 (momento de inercia de un disco respecto a su eje central) y Iz’ = MR

2/4 (momento de

inercia de un disco respecto a un diámetro). Podemos evaluar el momento angular del disco

mediante sus dos componentes:

L = [Ly’2 + Lz’

2]1/2

= [(Iy’ ωy’)2 + (Iz’ ωz’)

2]1/2

Pero ωz’ = ω senθ = ω sen1o = 0.01745ω y ωy’ = ω cosθ = ω cos1

o = 0.9998ω. Entonces:

L = {[(MR2/2) 0.9998ω]

2 + [(MR

2/4) 0.01745ω]

2}

1/2 = ½MR

2ω(0.9997 + 0.00008) ≈ ½MR

El vector L forma un ángulo θ con el eje fijo y alrededor del cual precesa, y un ángulo θ – θ con

el eje y’ (Fig. 19.12Sb). El ángulo θ entre L y ω puede calcularse a partir del triángulo con

ángulo θ – θ:

tan(θ – θ) = Lz’/Ly’ = Iz’ ωz’/ Iy’ ωy’ = (MR2/4)(ω senθ)/[( MR

2/2)(ω cosθ )] = ½ tanθ

Y como θ y θ son pequeños se puede aproximar la tangente al ángulo en radianes y queda:

θ – θ = ½ θ, de donde resulta θ = ½ θ = 0.5o.

Para calcular el torque sobre la caja de bolas del volante necesitamos conocer la rapidez con que

varía el momento angular del volante respecto al sistema fijo x, y, z: τ = dL/dt. La Fig. 19.12Sa

muestra la precesión del vector L alrededor del eje y: la componente Ly es constante, y la

componente normal al eje y, Ln, pivotea alrededor del eje y con la frecuencia ω. Significa que no

hay componente de torque en el eje y y éste sólo tiene componentes en el plano xy en que pivotea

la componente variable Ln.

Sabemos de cinemática que la rapidez con que cambia el vector de posición, dr/dt que localiza a

una partícula desde el centro de la circunferencia en que se mueve es |dr/dt| = ωr. De manera

Page 208: problemas selectos

208

análoga, la rapidez con que cambia el vector L cuando su componente Ln, pivotea alrededor del

eje será: |dL/dt| = ωLn. Pero de la Fig.19.12Sa: Ln = Lsenθ. Por consiguiente, |dL/dt| = ωLsenθ =

ω (½MR2ω) senυ = ½MR

2senθ. Y éste será el valor del torque sobre la caja de bolas. Susti-

tuimos los datos y obtenemos: η = 1.1 N m.

Nota:

El momento angular de un cuerpo que rota alrededor de un eje de simetría es L = Iω, donde I es

el momento de inercia respecto al eje; en este caso el momento angular y la velocidad angular

son paralelos, o colineales. Si la rotación no es alrededor de un eje de simetría, ya los vectores L

y ω no son colineales, aunque el eje pase por el centro de masas del cuerpo. Si se conocen los

momentos de inercia respecto a los ejes de simetría, Ixx, Iyy, Izz, y las componentes de la velocidad

angular sobre esos ejes, ωx, ωy, ωz, el momento angular puede calcularse en cualquier instante

por la expresión: L = Ixxωx i + Iyyωy j + Izzωz k. En este problema usamos tres ejes de simetría

para calcular L: el central del disco, y, y dos diametrales, x, z.

Si la rotación no es alrededor de un eje que pase por el centro de masas, el cálculo del momento

angular se hace más complejo y se requiere, en general, trabajar el momento de inercia como un

tensor (matriz simétrica de nueve componentes).

ña, en ambos lados de ésta hay una presión po y no habrá fuerza resultante sobre la cuña. Cuando

el frente de onda pase completo la cuña, a ambos lados de la misma habrá también la misma

sobrepresión 2po y ya no producirá fuerza resultante. El impulso se produce solamente mientras

el pulso vaya “subiendo” por la cuña. Esta sobrepresión sobre el lado inclinado tratará, en parte,

de enterrar a la cuña en el suelo, pero eso lo impedirá la fuerza normal con el suelo, que aumen-

tará lo que sea necesario. Solamente quedará una fuerza resultante horizontal sin compensar que

dará un “empujón” horizontal a la cuña. Esta fuerza resultante horizontal irá creciendo en la

medida que el pulso vaya cubriendo la cuña.

La velocidad u adquirida en el empujón puede calcularse por el impulso mecánico igual a la

variación del momento lineal:

∫0 Fx dt = Δp = mu – 0 = mu (1)

x x

p

po

2po

m

b

a

c

Fig. 19.13a Fig. 19.13b

19.13-Cuando el frente de onda

llegue a la cuña, el lado incli-

nado va a sufrir un aumento de

presión que va ir produciendo-

se de manera continua a medi-

da que el frente alcance puntos

cada vez más altos en la cuña.

Antes de llegar el pulso a la cu-

θ vt

Fx

F θ

t

Page 209: problemas selectos

209

Necesitamos calcular el impulso durante el tiempo que demora el paso del frente de onda sobre

la cuña.

La fuerza Fx será la componente horizontal de la fuerza de sobrepresión sobre la cuña, que

dependerá del área cubierta por el frente en cada instante:

Fx = Fsenθ = (2po – po) A senθ = po(c vt/cosθ) senθ = pocvt tanθ

donde c vt/cosθ es el área cubierta por el frente de onda después de un tiempo t avanzando por

la cuña. Por otra parte: tanθ = a/b, por lo que Fx = pocavt/b.

Sustituimos Fx en (1):

∫0 Fx dt =∫0 (pocavt/b) dt = mu

De aquí: (pocav/b) (½ t2) = mu, lo que nos permite calcular la velocidad u de la cuña al cabo de

un tiempo t de estar pasando el frente de onda. El tiempo de paso completo de la onda por la

cuña es t = b/v. Sustituyendo en la ecuación anterior:

(pocav/b) (½ b2/v

2) = mu

Despejando la velocidad de la cuña: u = (po/2mv) abc

19.14-La ley de enfriamiento de Newton establece que la velocidad de enfriamiento, dT/dt, de un

cuerpo en el ambiente es proporcional a la diferencia de temperaturas entre el cuerpo caliente, T,

y el ambiente, Ta: dT/dt = −H (T - Ta). La constante H depende del área de contacto entre el

sistema que se enfría y el ambiente, así como de la conductividad térmica del sistema. El signo

negativo es porque si T > Ta, la temperatura del cuerpo debe disminuir.

Si la esfera es muy densa, la superficie de contacto del agua con el recipiente es casi igual en

ambos casos, por lo que la constante H vale casi lo mismo para los dos sistemas. Además, los

recipientes son metálicos y ligeros, por lo que mrecip << magua y crecip<< cagua , y los recipientes

pueden despreciarse en el intercambio de calor.

Ahora: dQ = (maca + mece) dT para el recipiente con la esfera, y dQ = mcadT para el recipiente

con agua solamente. Entonces:

(maca + mece) dT = −H (T - Ta) dt (1) para el recipiente con esfera y

mca dT = −H (T - Ta) dt (2) para el recipiente sin esfera.

Separando variables en (1) e integrando en ambos miembros, desde T = 100oC hasta casi la

temperatura ambiente, Ta+, para la integral de la izquierda y desde t = 0 hasta t1 para la derecha:

∫[(maca + mece)/(T – Ta)] dT = ∫−Hdt , donde me = ma y t1 es el tiempo de enfriamiento:

t t

l

100

Ta+

0

t1

Page 210: problemas selectos

210

ma(ca + ce) ∫ dT/(T – Ta) = −Ht1 o ma(ca + ce) ln(T – Ta)| = −Ht1

O sea: ma(ca + ce) ln[(100 – Ta)/(Ta+

−Ta)] = −Ht1 (3)

Semejantemente, para el recipiente sin esfera: maca ln[(100 – Ta)/(Ta+

−Ta)] = −Ht2 (4)

Dividiendo (3) y (4) miembro a miembro: (ca+ce)/ ca = t1/t2 = k

Despejando: ce/ca = k – 1

9.15-Cuando el gas ideal se expande contra el vacío no realiza trabajo, por lo que su energía

interna no cambia y su temperatura al final es igual que al principio. Aunque este proceso es

irreversible y adiabático, para calcular la variación de entropía entre los estados inicial y final

deberemos suponer un proceso reversible que lleve del mismo estado inicial al mismo estado

final.

En la situación analizada se puede suponer un proceso isotérmico entre los estados inicial y final.

El cambio de entropía se calcula por: ΔS = ∫ dQ/T. Como la temperatura es constante, sale de la

integral y sólo hay que integrar el dQ, que da el calor intercambiado en la expansión isotérmica

del gas ideal, Q = nRT ln (V2/V1) = nRT ln 2 (pues V2 = 2V1). Entonces: ΔS = (nRT ln2)/ T =

nR ln 2, que da positivo, como corresponde al aumento de entropía. Como n = 1:

ΔS = R ln2 = 5.76 J/K.

Note que aunque en la expansión adiabática e irreversible no hay calor intercambiado, en el

proceso reversible escogido para calcular la variación de entropía sí hay calor intercambiado.

Cualquier otro proceso reversible que se considere entre los dos mismos estados dará el mismo

resultado. Puede intentarlo suponiendo un proceso que lleve el gas a presión constante al doble

de su volumen (con dQ=ncPdT), seguido de un enfriamiento a volumen constante que lleve el

gas a una presión que sea la mitad de la inicial (con dQ=ncVdT). Verá que el resultado es el

mismo que el calculado en un solo proceso isotérmico reversible.

19.16-La Fig.19.16Sa muestra la situación planteada. Analizaremos la situación asumiendo que

todo el flujo que sale de +q1 dentro de un cono de ángulo α entra en la carga –q2 dentro de otro

cono de ángulo β.

+q1

−q2

E

α β +q1

α ΔA

Fig. 19.16Sa Fig. 19.16Sb

100

Ta+

Ta+

100

Page 211: problemas selectos

211

En la figura de la derecha se ha representado la carga q1 en el centro de una esfera de radio r, y el

cono de ángulo α que sale a través del área oscurecida, ΔA. Ese cono de líneas entrará en –q2

dentro de un cono de ángulo β. El área oscurecida es la de un casco de esfera de radio r.

Necesitamos el área de este casco para calcular el flujo, ΦE = EΔA.

Un diferencial de área sobre la superficie de una esfera, en coordenadas esféricas, está dada por

dA = (r dθ) (r senθ dθ). Sumando estas áreas para el casco, haremos barrer θ desde 0 hasta α, y θ

barrerá de 0 a 2π alrededor de la línea de puntos en la figura derecha. El área del casco será

entonces:

ΔA = ∫2πr2senθ dθ = 2πr

2 (1 – cosα)

El flujo que sale por este casco es: ΦE1 = EΔA= (kq1/r2) 2πr

2 (1 – cosα) = 2πkq1 (1 – cosα)

Igualmente, la magnitud del flujo que llega a la carga q2 a través de una esfera de radio r en torno

a q2 satisface la relación:

ΦE2 = 2πkq2 (1 – cosβ) (Aquí q2 representa la magnitud, positiva, de la carga negativa)

Asumiendo que las líneas que salen bajo el ángulo α, o menor, llegan bajo el ángulo β, o menor,

podemos asegurar que ΦE1 = ΦE2.

O sea: 2πkq1 (1 – cosα) = 2πkq2 (1 – cosβ)

Por lo tanto: q1/q2 = (1 – cosβ) / (1 – cosα) = 2 sen2(β/2) / [2 sen

2(α/2)]

Entonces: sen(β/2) = (q1/q2)1/2

sen(α/2). O sea: β = 2 sen-1

[(q1/q2)1/2

sen(α/2)]

Si (q1/q2)1/2

sen(α/2) > 1, esa línea no entrará en –q2: la envolverá, pero seguirá de largo. Esto

pasará solamente si q1 > q2/ sen2(α/2)

Fig. 19.17S

x

d

+

+

+

+

+

+

S

+

+

+

+

+

+

d

x O

19.17-a)A medida que las cargas se separan, aumenta la carga

neta q a cada lado de la superficie S. Esta carga neta a cada lado

se puede calcular en cada instante multiplicando la densidad de

carga, n, por la carga del electrón e y por el volumen de la capa,

Ax, donde A es el área de la capa y x su espesor:

q = neAx (1)

Cuando el espesor de cada capa es d, la carga total en cada capa

será q = neAd, positiva a un lado y negativa al otro. Cada capa

puede considerarse el agregado de capas equidistantes del centro,

como las dos sombreadas en la Fig. 19.17S, y cada una de estas

parejas crea un campo entre ellas como el de un capacitor de pla-

α

0

Page 212: problemas selectos

212

cas paralelas, dE = dζ/εo, donde dζ es la densidad superficial de carga en cada capa, de espesor

diferencial, y εo es la permitividad del vacío. La intensidad del campo resultante en la superficie

S es la suma (integral) de todos estos campos:

E = ∫ dζ/εo, donde dζ = dq/A = (neA dx)/A = ne dx, por lo que: E = ∫ ne dx/ εo = nex/ εo

Y cuando el espesor de cada capa sea x = d, entonces E = ned/εo, que es la respuesta a la primera

pregunta.

b)Estas dos zonas de cargas opuestas se atraen. La fuerza de atracción de una zona sobre la otra

puede deducirse considerando que la carga de cada zona está concentrada a la mitad de cada

zona como las franjas sombreadas de la Fig. 19.17S (usamos el concepto de centro de carga

similar al de centro de masa, en este caso, a lo largo del eje x). Cada “placa” con la carga

concentrada en ella produce un campo E = ζ/2εo, y éste es el campo que siente la otra placa.

Aquí ζ = q/A, y la carga de la “placa” depende del espesor de la zona de carga, según la ecuación

(1): E = neAx/(A2εo) = (ne/2εo) x. La magnitud de la fuerza de atracción que sufre la zona

negativa hacia la positiva es entonces: F = qE = (Ne) (ne/2εo) x, donde Ne es la carga total

acumulada en la “placa” negativa en cada instante. Esa fuerza resultante sobre la placa actúa

sobre la masa total de la “placa”, M = Nm, donde m es la masa de un solo electrón.

Aplicando la segunda ley de Newton a la “placa” de electrones: Ma = −(Ne) (ne/2εo) x

O: Nma = −(Ne) (ne/2εo) x

donde el signo negativo indica que cuando la “placa” está en las x negativas sufre una fuerza de

atracción hacia las x positivas. Arreglando:

d2x/dt

2 + (e

2n/2mεo ) x = 0.

Se reconoce la ecuación del oscilador armónico, con frecuencia: ω =

Ésta es la “frecuencia del plasma”.

19.18-a)La Fig. 19.18S (próxima página) se ha extendido para mostrar un electrón que entra y

sale en la capa de carga positiva. Se han añadido ejes x,y con el origen O en el punto de entrada

en la zona de carga positiva, con x positiva en la dirección en que penetra el electrón y y positiva

hacia abajo. En el análisis se despreciará el peso del electrón comparado con las fuerzas eléc-

tricas que sufre. Nos interesa saber la máxima velocidad con que puede llegar el electrón al

punto A de entrada sin que llegue a penetrar en la zona de carga negativa, esto es, que el electrón

llegue hasta x = d y retorne hacia atrás, sin dejar de desplazarse paralelamente al eje y positivo.

El campo eléctrico en la zona de carga debe ser de líneas horizontales de la zona positiva hacia la

negativa, debido a que las capa son extensas comparadas con su espesor.

0

x

0

x

Page 213: problemas selectos

213

campo debido a todas las subcapas que hay entre P y la pared externa de la zona positiva

(sombreada con puntos), y en la zona negativa habrá otra región similar (sombreada también);

pero las cargas de la zona en blanco entre las líneas quebradas no colaboran al campo que siente

el electrón en el punto P.

En definitiva, el campo efectivo en cada punto x entre las placas es:

E = ∫ dζ/εo ,

donde dζ es la densidad superficial de cada subcapa δ’: dζ = dq/A = (ρA dx)/A = ρ dx, donde ρ

es la densidad volumétrica de carga, A es el área total de las subcapas y dq es la carga diferencial

total contenida en ellas.

Entonces: E = ∫ ρ dx /εo = ρx/εo. Se ve que E aumenta linealmente con x. El electrón sufrirá una

fuerza igual a:

F = −eE = −eρx/εo

donde el signo negativo indica que el electrón sufre una fuerza hacia las x negativas (hacia la

derecha en la Fig. 19.18S), cada vez mayor mientras más penetra la capa positiva, sin llegar a la

superficie central pues el campo empezaría a decaer en ese punto.

Entonces: ma = −eρx/εo y: d2x/dt

2 + (eρ/mεo) x = 0

d d

+ρ −ρ θ

-e, m, v

Fig. 19.18S

x

O

P

δ δ’

Esto significa que la fuerza eléctrica sobre el electrón

será toda horizontal, sin afectar la componente y de

velocidad.

Por otra parte, este campo no será uniforme pues según

se acerque el electrón a la superficie de separación entre

las capas positiva y negativa, más cargas de cada zona

colaboran a la intensidad de ese campo. Para explicarnos

mejor, vea que en cada zona pueden encontrarse dos fi-

nas subcapas δ y δ’ que equidisten de la superficie cen-

tral (Fig. 19.18S), las cuales se comportan como las pla-

cas paralelas de un capacitor, que entre ellas crean un

campo uniforme E = ζ/εo y fuera de ellas crean un cam-

po nulo (el campo existe sólo entre las placas).

Pero podemos dividir toda la zona de carga en estas

parejas de subcapas, y el campo total hacia el centro de

la zona crecerá de intensidad respecto a la de los puntos

lejanos al centro, pues más parejas de placas colaboran

con sus campos al campo total cerca del centro.

Por ejemplo, el electrón que llegue al punto P sentirá el

y

A

x

0

x

0

Page 214: problemas selectos

214

Esta ecuación representa un movimiento armónico simple a lo largo del eje x, por lo que el elec-

trón se mueve a lo largo del eje x como si hiciera medio ciclo de oscilación:

x = X senωt, con ω = durante medio período de oscilación.

Hasta dónde penetre dependerá de la componente de velocidad en el eje x con que entra en la

zona positiva: vx = v senθ. La máxima penetración que puede ocurrir sin llegar a la zona negati-

va ocurre cuando la amplitud de la oscilación sea X = d. Pero la amplitud de la oscilación y su

velocidad en el centro de la oscilación están relacionadas por vx = ωX = ωd, y ésta es la compo-

nente de velocidad en el eje x que tiene el electrón al entrar por O. Por tanto, la máxima veloci-

dad permitida se obtiene a partir de v senθ = .

Por tanto: v = / senθ

b)El tiempo que demore en salir de la zona cargada corresponderá a medio período de oscilación:

t = T/2 = π/ω = π

En ese tiempo descenderá por el eje y una distancia:

y = (v cosθ) t = { / senθ] cosθ} ( π ) = πd cotθ

Asumiendo que la carga Q es positiva, todo el aro se acelerará angularmente en la dirección de la

flecha circular.

Si llamamos E a la fem inducida en el aro, por la ley de Faraday:

E = -dΦB/dt = -A dB/dt = πR2 dB/dt (1)

También E = W/δq (trabajo por unidad de carga en el circuito cerrado), y en el circuito del aro el

trabajo será:

W = Fd = (Eδq) (2πR), por lo que E = 2πR E (2)

De (1) y (2): -πR2dB/dt = 2πR E, por lo que E = -(R/2) dB/dt

B

ω

Q

ΦB

19.19-Al aumentar el campo magnético B hacia abajo

se induce un campo eléctrico E de líneas circulares

superpuestas al aro de carga Q. Este campo aplicado a

lo largo del aro está dirigido en la dirección que indi-

ca la flecha circular de acuerdo con la ley de Lenz

(Fig. 19.19S). Fig. 19.19S

Page 215: problemas selectos

215

El signo negativo solamente refleja la dirección que debe tener el campo eléctrico inducido para

oponerse a la variación de flujo magnético, lo cual ya indicamos en la figura, por lo que dejamos

el signo y nos ocuparemos solamente de la magnitud: E = (R/2) dB/dt.

Bajo este campo eléctrico se acelera toda carga dq del aro con una fuerza dF = E dq. Esta fuer-

za, tangencial al aro en cada dq, crea un torque respecto al centro del aro dado por:

dη = R dF = R E dq

El torque total sobre todo el aro será:

η = ∫ RE dq = RE Q = (R2/2) Q dB/dt.

Este torque produce un impulso angular Iθ sobre todo el aro dado por:

Iθ = ∫ η dt = ∫ (R2/2) Q (dB/dt) dt = ∫ (R

2/2) Q dB = Bo QR

2/2.

Pero el impulso angular es igual a la variación del momento angular L:

Iθ = ΔL = Iω = mR2ω, donde I = mR

2 es el momento de inercia del aro.

Entonces: mR2ω = Bo QR

2/2.

Por tanto: ω = Bo Q/ (2m)

Pero otro par de fuerzas se opondrá a este giro hacia arriba: el del peso, mg, con la fuerza normal,

N, en el punto de contacto sobre el que se va a levantar el anillo (punto P en la Fig. 19.20Sa).

El torque de un par de fuerzas F iguales y opuestas separadas una distancia d es η = Fd respecto

a cualquier punto o eje. Al momento de empezar a levantarse el anillo, aún N = Mg, y la fuerza

de gravedad está aplicada al centro de masa, en el centro del anillo, por lo que d = R y el torque

del peso y la normal (“torque gravitatorio”) es:

ηgrav = MgR (1)

19.20-Cuando por el anillo circule una corriente

I como indica la Fig.19.20Sa, el campo magnéti-

co de líneas horizontales creará una fuerza verti-

cal hacia abajo en toda la mitad TPS del anillo, y

una fuerza igual hacia arriba en el lado SQT.

Este par de fuerzas producirá un torque alrede-

dor del eje ST que intentará levantar al anillo so-

bre el punto P.

B P

Q

S

B B

dF

dF

I

I

T

N

Mg

R

Fig. 19.20Sa

aro

0 0 0

t t Bo

Page 216: problemas selectos

216

la separación entre dos dl opuestos, K y L, sujetos a un par de fuerzas iguales y opuestos, es

2Rsenθ = 2y, por lo que el torque diferencial de cada par de fuerzas actuando sobre diferenciales

de corrientes es:

dηmagn = (IB dl senθ)(2R senθ) = (IB dx)(2y) = IB dA

donde dA = 2y dx es el área del listón sombreado. Entonces:

ηmagn =∫anillo IB dA = IB πR2 (2)

Justo cuando el anillo va a empezar a levantarse, con la fuerza normal concentrada en el punto P

se cumplirá que ηmagn = ηgrav , por lo que: IB πR2 = MgR.

De aquí: I = Mg / (πBR) = 0.30 A

K

L

θ

θ

B

R dx

x

y

dF

dF

y

Fig. 19.20Sb

El torque magnético habrá que calcularlo por integra-

ción, pues la fuerza magnética está distribuida por todo

el anillo: en cada segmento de longitud dl hay una

fuerza magnética dF = IB dl senθ, donde θ va desde 0

en el punto T hasta 180o en el punto S. Todos estos

diferenciales de fuerzas apuntas hacia abajo en el arco

TPS, y hacia arriba, en el arco SQT. La Fig. 19.20b

muestra una vista superior del anillo, con el ángulo θ

entre el dl de corriente y el campo B, que a su vez re-

sulta igual que el ángulo central θ.

De la Fig. 19.20Sb se aprecia que dl senθ = dx, y que

P

Q

T S

dl

dx

θ

Page 217: problemas selectos

217

Apéndice: Cantidades físicas fundamentales.

I-Cantidades físicas.

A continuación se presentan las magnitudes físicas más comunes de la Física Clásica. De cada

magnitud se da su nombre, la letra que suele representarla –cursiva (itálica) si es escalar y

negrita (bold) si es vector− su definición matemática (cuando se define a partir de otras mag-

nitudes), su significado con palabras, y su unidad en el S.I. Si es una unidad definida a partir de

otras, se muestra la relación con las otras unidades.

Algunas magnitudes se definen por la forma en que se miden, y suelen considerarse “magnitudes

fundamentales” (aunque éstas pueden ser diferentes de un sistema de unidades a otro); las magni-

tudes que se definen a partir de las magnitudes fundamentales se llaman “magnitudes derivadas”

y sus unidades son “unidades derivadas” a partir de las unidades fundamentales correspondien-

tes.

En las ecuaciones físicas, todos los sumandos a ambos lados de la igualdad deben tener las mis-

mas unidades (las mismas dimensiones físicas): si esto no se cumple la ecuación es errónea. Es

por ello importante conocer las dimensiones y unidades de las magnitudes derivadas.

Mecánica.

Intervalo de longitud (s o l: se define en relación con la unidad patrón de longitud): cantidad de

intervalos patrones de longitud que caben entre dos puntos a lo largo de una línea (metro: 1 m).

Intervalo de tiempo (t: se define en relación con la unidad patrón de tiempo): cantidad de inter-

valos patrones de tiempo que transcurren entre dos sucesos (segundo: 1 s).

Vector de posición (r = xi + yj + zk): vector que localiza cada punto del espacio respecto al

origen de un sistema coordenado (1 m).

Desplazamiento entre dos puntos (Δr = rf - ri): vector de longitud que va del punto inicial al

punto final (1 m).

Distancia recorrida (Δs = ∫׀dr׀): longitud recorrida sobre la trayectoria (1 m).

Velocidad media en un intervalo de tiempo Δt (vm = Δr/Δt): representa el desplazamiento por

unidad de tiempo que debe realizarse a ritmo constante en línea recta para cubrir el desplaza-

miento total en el tiempo total considerado (1 m/s).

Page 218: problemas selectos

218

Rapidez media en un intervalo de tiempo Δt (cm = Δs/Δt): representa la distancia por unidad de

tiempo que debe recorrerse a ritmo constante sobre la trayectoria para cubrir ésta en el tiempo

total considerado (1 m/s).

Velocidad instantánea (v = dr/dt): desplazamiento por unidad de tiempo en cada instante (1 m/s)

Rapidez instantánea (c =ds/dt): distancia recorrida por unidad de tiempo en cada instante (1 m/s).

[Aclaración: en valores instantáneos: c = |v |, pero en valores medios: cm ≥ |vm |].

Aceleración instantánea (a = dv/dt): cambio de velocidad por unidad de tiempo en cada instante

(1 m/s2). [Aclaración: a puede no ser paralelo a v].

Aceleración tangencial (at = dc/dt): cambio de rapidez por unidad de tiempo (1 m/s2). [Aclara-

ción: es la componente de la aceleración tangencial a la trayectoria].

Aceleración normal o centrípeta) (an = v2/R): cambio del vector velocidad por unidad de tiempo

perpendicularmente a la dirección del vector velocidad (1 m/s2). [Aclaración: es la componente

de la aceleración normal a la trayectoria, y apunta hacia su centro de curvatura instantáneo].

Fuerza (F = ma): mide la intensidad de la interacción de dos cuerpos, y es proporcional a la

deformación elástica que se produce en ellos (newton: 1 N = 1 kg m/s2)

Fuerza centrípeta (Fc = ∑Fni = man): componente de la fuerza resultante normal a la trayectoria

(1 N).

Fuerza resultante tangencial (Ft = ∑Fti = mat): componente de la fuerza resultante tangente a la

trayectoria (1 N).

Masa inercial (m: se define en relación con la unidad patrón de masa): medida de la oposición de

un cuerpo a ser acelerado (kilogramo: 1 kg).

Masa gravitatoria (m: se define por su relación con la masa inercial): medida de la capacidad de

cada cuerpo para atraer y ser atraído gravitatoriamente por otros (1 kg).

Peso (P = mg): fuerza con que un planeta, o astro, atrae a cada cuerpo cerca de su superficie

(1 N).

Fuerza de inercia en sistemas en traslación pura (Finercia = −mAsistema): efecto de la inercia que se

comporta como una fuerza cuando se observa un cuerpo desde un sistema acelerado respecto a

un sistema inercial; la “fuerza” aparece en sentido opuesto al de la aceleración del sistema (1 N).

Fuerza centrífuga de inercia (Fcentf = mω2r): efecto de la inercia sobre un cuerpo en un sistema en

rotación; es la fuerza aparente que sufre una partícula de masa m a una distancia r del eje de

rotación de un sistema que rota con velocidad angular ω; apunta en la dirección que se aleja del

eje (1 N)

Page 219: problemas selectos

219

Energía cinética (K = ½mv2, donde v = |v| = c): energía de movimiento de traslación de un

cuerpo (“vitalidad del movimiento”) (joule: 1 J = 1 kg m2

s-2

= 1 Nm).

Trabajo (W = ∫c F.dr): cantidad de energía que transfiere una fuerza al desplazar su punto de

aplicación. (1 J = 1 Nm = 1 kg m2

s-2

).

Energía potencial de un sistema en una configuración dada (U=-Wcons

): representa el trabajo que

realiza la fuerza conservativa del sistema cuando pasa de una configuración de referencia a la

configuración dada, con signo cambiado (1 J = 1 Nm = 1 kg m2

s-2

)

Energía mecánica (E = ∑Ki + ∑Ui): suma de las energías cinéticas y potenciales de un sistema

(1 J = 1 Nm). [Aclaración: se consideran solamente energías macroscópicas, no moleculares, ató-

micas o nucleares].

Energía de un sistema (suma de todos sus tipos de energías: mecánica, térmica, electromagnética,

nuclear, etc.): capacidad del sistema para realizar transformaciones sobre sí mismo o sobre su

entorno (1 J = 1 N m).

Potencia (P = dW/dt, o P = dQ/dt, o P = dU/dt): rapidez de intercambio de energía en

cualesquiera de sus formas: trabajo W, calor Q, energía U, etc. (watt: 1 W = 1 J/s).

Fuerza conservativa (F es conservativa si su trabajo por toda trayectoria cerrada es cero, y deriva

de un potencial: F = -dU/dx): fuerzas vinculadas con las energías potenciales (1 N).

Momento lineal de la partícula (p = mv): impulsión de traslación de un cuerpo (1 kg m/s).

Momento lineal del sistema de partículas (P = ∑ pi): suma vectorial de los momenta lineales de

todas sus partes (1 kg m/s). [Aclaración: P = MVcm donde M es la masa total del sistema y Vcm

es la velocidad de su centro de masas].

Impulso de una fuerza (I =

): variación de momento lineal que puede producir una fuer-

za actuando cierto intervalo de tiempo (impulsión que comunica al cuerpo) (1 N s = 1 kg m/s)

Coordenadas del centro de masas (Rcm = (1/M) ∫D r dm ): coordenadas del punto que promedia la

masa M del sistema en el espacio (1 m)

Velocidad del centro de masas (Vcm = (1/M) Σmivi: velocidad con que se desplaza el centro de

masas del sistema (m/s).

Energía cinética interna (Kint = ∑ ½ mi vi2, donde las velocidades de las partes del sistema se

miden respecto a un sistema de referencia fijo al centro de masas): energía de movimiento del

sistema respecto a su propio centro de masas (1 J).

Page 220: problemas selectos

220

Angulo en radianes (θ = s/r): medida de la apertura entre dos rectas, o entre dos direcciones

(longitud de arco s dividido por el radio r de la circunferencia utilizada para construir el arco).

(radián: 1 rad, sin dimensiones físicas).

Angulo de posición (θ: se define como en el punto anterior): ángulo entre una dirección de

referencia (un eje de referencia) y una recta incorporada al cuerpo que rota (1 rad).

Desplazamiento angular (Δθ = θf − θi): ángulo entre dos posiciones angulares de un cuerpo en

rotación alrededor de un eje fijo (1 rad).

Velocidad angular (ω = dθ/dt): desplazamiento angular por unidad de tiempo (1 rad/s).

[Aclaración: el vector ω es colineal con el eje instantáneo].

Aceleración angular (α = dω/dt): variación de velocidad angular por unidad de tiempo (1 rad/s2).

Torque de una fuerza respecto a un eje (τ = r × F, donde r sale perpendicular al eje y localiza el

punto de aplicación de la fuerza; τ = Fb, donde b es el brazo de fuerza): capacidad de una fuerza

para producir un cambio en el estado de rotación de un cuerpo respecto a un eje (1 Nm).

Momento de inercia (I =

): medida de la oposición de un cuerpo a acelerarse angular-

mente –rotacionalmente- alrededor de un eje específico (1 kg m2).

Energía cinética de rotación (Krot = ½Iω2): energía de movimiento de rotación de un cuerpo

alrededor de un eje dado (1 J).

Momento angular de la partícula respecto a un punto, o eje (L = r × p): impulsión de rotación de

la partícula alrededor de un punto, o eje (1 kg m2 /s).

Momento angular de un sistema de partículas respecto a un punto, o eje (L = ∑ri × pi): impulsión

de rotación del sistema de partículas alrededor de un punto, o eje (1 kg m2 /s).

Momento angular orbital (Lo = Rcm × Pcm): impulsión de rotación alrededor de un eje que no

pasa por el centro de masas (cm) del cuerpo; (1 kg m2 /s).

Momento angular de espín (Ls = Icm ω): impulsión de rotación alrededor de un eje que pasa por

el centro de masas (cm) del cuerpo (1 kg m2 /s).

Momento angular total de un cuerpo que rota y se traslada (L = Rcm × Pcm + Icm ω): impulsión

total de rotación de un cuerpo que rota y se traslada alrededor de un eje (1 kg m2/s).

Calor.

Presión (P = dFn /dA): fuerza distribuida que se ejerce por unidad de área, perpendicularmente a

la misma (pascal: 1 Pa = 1N / m2).

Page 221: problemas selectos

221

Densidad (volumétrica) (ρ = dm/dV): masa de una sustancia por unidad de volumen (kg/m3).

Gasto de masa (Gm = dm/dt): masa de fluido que pasa por una sección de un tubo de flujo en

cada unidad de tiempo (kg/s).

Gasto de volumen (cauce) (GV = dV/dt): volumen de fluido que pasa por una sección de un tubo

de flujo en cada unidad de tiempo (m3/s).

Temperatura absoluta (se define en relación con la temperatura del punto triple del agua): medida

de la energía cinética media de las moléculas de un sistema (K).

Temperatura (se define en relación con dos niveles de referencia –puntos termométricos−, T):

nivel de calentamiento de un sistema (derivado del SI: grado celsius: ºC).

Cantidad de calor (Q = mcΔT, o Q = mL): cantidad de energía que intercambia un sistema

macroscópico mediante interacciones microscópicas (1 J = 1 Nm = 1 kg m2

s-2

). [Aclaración: la

primera ecuación permite evaluar Q cuando hay cambios de temperatura ligadas al intercambio

de calor; la segunda ecuación permite evaluar Q cuando el sistema sufre cambios de estado –o de

fase− sin variaciones de temperatura]. (Unidad derivada: caloría: 1 cal = 4.186 J)

Energía interna termodinámica (U = ∑Ki + ½∑Uij): suma de las energías cinéticas y potenciales

internas microscópicas –moleculares, atómicas, nucleares− (1 J).

Velocidad media cuadrática de las moléculas (vrms = ): es la raíz del valor medio de

las velocidades al cuadrado de todas las partículas del sistema; se vincula con la temperatura

absoluta del sistema (m/s)

Oscilaciones y ondas.

Amplitud de una oscilación (A): máximo valor de la variable oscilatoria (su unidad depende de la

magnitud que oscila: posición, velocidad, presión, voltaje, etc.).

Frecuencia de una oscilación (f = ΔN/Δt): número de oscilaciones por unidad de tiempo (hertz:

1 Hz = 1 s-1

).

Período de una oscilación (T = Δt/ΔN): tiempo que dura una oscilación (1 s).

Frecuencia angular (ω = Δθ/Δt): variación del ángulo de fase por unidad de tiempo (1 rad/s).

[Aclaración: ω = 2π/T].

Longitud de onda (λ): distancia entre dos puntos de una onda con una diferencia de fase de 2π,

medida a lo largo de la línea de propagación de la onda (1 m).

Page 222: problemas selectos

222

Número de onda (k = Δθ/Δl): variación del ángulo de fase por unidad de longitud (1 rad/m).

[Aclaración: k = 2π/λ].

Ángulo de fase de una oscilación, o fase (θ = ωt +θo): ángulo que determina en cada instante en

qué parte del ciclo oscilatorio está el oscilador; θo es la fase inicial, cuando t = 0. (rad).

Intensidad de onda sonora (I = ΔU/AΔt): energía de la onda que incide sobre la unidad de área

por unidad de tiempo, sobre una superficie perpendicular a la dirección de la onda (W/m2)

Nivel sonoro (L = 10 log10 (I/Io), donde Io = 10-12

W/m2

representa el umbral sonoro). Es el nivel

de intensidad sonora que percibe el oído humano, relativa a la intensidad del umbral de audición,

apenas perceptible; tiene en cuenta la respuesta fisiológica sensorial del oído (1 dB: decibel).

Electromagnetismo.

Carga eléctrica (q= ±Ne: cantidad N de electrones –de carga e− en exceso o en defecto en un

cuerpo): propiedad fundamental de la materia que caracteriza su capacidad de interacción eléc-

trica y magnética. (coulomb: 1 C = 6.25 ×1018

e), donde e representa el valor absoluto de la

carga de un electrón).

Intensidad del campo eléctrico en un punto (E = F/q): fuerza que ejerce el campo sobre la unidad

de carga colocada en el punto (1 N/C, o 1 V/m, donde V es la unidad volt)

Potencial eléctrico en un punto del campo (V =U/q): representa la energía potencial eléctrica de

interacción del campo (o del sistema de cargas que lo produce) con cada unidad de carga que se

coloque en el punto. (volt: 1 V = 1 J/C).

Voltaje (ΔVab = ΔUab/q): es la diferencia de potencial entre dos puntos del campo eléctrico, o

entre dos puntos de un circuito. (1 V = 1 J/C).

Fuerza electromotriz (E = Wno conserv

/q): trabajo que realiza una fuerza no conservativa al mover

la unidad de carga a lo largo de una trayectoria cerrada. (1 V = 1 J/C).

Intensidad de corriente eléctrica (I =dq/dt): cantidad de carga que pasa por unidad de tiempo a

través de una sección dada de un circuito (ampere: 1 A = 1 C/s).

Inducción de campo magnético en un punto [|B| = |F| / (q |v| senθ); F = q v × B]: fuerza que

ejerce el campo magnético sobre la unidad de carga que pasa por el punto con la unidad de velo-

cidad, perpendicularmente a las líneas del campo B (tesla: 1 T = 1 N/(Cm/s) = 1 N/Am).

Momento dipolar eléctrico (pe = qd): mide la capacidad de un dipolo eléctrico para alinearse con

un campo eléctrico, bajo el torque del campo (Cm).

Page 223: problemas selectos

223

Momento dipolar magnético (pm = NIA): mide la capacidad de un dipolo magnético (o de N

espiras coplanares, de área A y corriente I) para alinearse con un campo magnético, bajo el tor-

que del campo. (1 Am2).

Flujo de campo eléctrico sobre una superficie (ФE = ∫S E . dA): simboliza la cantidad de líneas de

campo eléctrico que salen o entran en la superficie que se considera (1 Nm2 /C = 1 Vm).

Flujo de campo magnético sobre una superficie (ФB = ∫S B . dA): simboliza la cantidad de líneas

de campo magnético que salen o entran en la superficie que se considera (weber: 1 Wb = 1 Tm2).

Densidad de corriente eléctrica (J, tal que: dI = J.dA; J = dI/dA): es la cantidad de carga que

pasa a través de la unidad de área y por unidad de tiempo en un punto de un circuito. (1 A/m2).

Resistencia eléctrica de un dispositivo eléctrico (R = V/ I): representa el grado de oposición del

dispositivo al paso de una corriente eléctrica a través del mismo (ohm: 1 Ω = 1 V/A).

Capacitancia de un dispositivo eléctrico que conste de dos conductores aislados (C = q/V):

representa la capacidad del dispositivo de almacenar cargas opuestas en sus dos conductores por

cada unidad de voltaje que se aplique entre ellos (farad: 1 F = 1 C/V).

Coeficiente dieléctrico relativo (κe = Cdiel /Cvacío ): factor en que aumenta la capacitancia al llenar

el vacío del capacitor con dieléctrico (sin unidades, adimensional).

Inductancia de un dispositivo eléctrico (espiras y enrollados) [L =-E /(dI/dt) o L = N ФB /I]: re-

presenta la capacidad del dispositivo de generar una fuerza contraelectromotriz por cada unidad

de rapidez de cambio de corriente que se produzca en el dispositivo (henry: 1 H = 1 V/(A/s) =

1 Vs/A, o 1 H = 1 Wb/A)

Permeabilidad magnética relativa (κm = Lmagn /Lvacío): factor en que aumenta la inductancia al

llenar el vacío del inductor con material ferromagnético (sin unidades, adimensional).

Reactancia inductiva (XL = ωL): representa el grado de oposición del dispositivo al paso de una

corriente eléctrica alterna de frecuencia angular ω a través del mismo, en virtud de su capacidad

para generar un campo magnético interno (1Ω = 1 Hs-1

= V/A).

Reactancia capacitiva (XC = 1/ωC): representa el grado de oposición del dispositivo al paso de

una corriente eléctrica alterna de frecuencia angular ω a través del mismo, en virtud de su

capacidad para generar un campo eléctrico interno (1Ω = 1 F-1

s = V/A).

Valores eficaces de corriente y voltaje en corriente alterna (Ie = Im/ y Ve = Vm/ , donde Im y

Vm son los valores pico). Son los valores que registran los amperímetros y voltímetros de corrien-

te alterna y fijan el valor de la potencia media disipada en cada sección del circuito por la

ecuación P=IeVecosθ, donde θ es el desfasaje entre la corriente y el voltaje en la sección (1 A y

1 V).

Page 224: problemas selectos

224

Impedancia de un circuito (Z = Ee/Ie ; razón entre valores eficaces de la fem aplicada y de la

corriente en el circuito): representa el grado de oposición de un circuito completo al paso de una

corriente eléctrica alterna de frecuencia angular ω a través del mismo (1Ω = V/A).

Intensidad de una onda electromagnética (I = dP/dAn): potencia que transporta la onda a través

de la unidad de área perpendicularmente a su dirección de propagación (1 W/m2).

Densidad de energía en un campo (u = dU/dV): cantidad de energía (U) que se almacena por

cada unidad de volumen (V) del campo (1 J/m3).

Óptica.

Índice de refracción (n = c/v): relación entre la velocidad de la luz en el vacío y la velocidad de

la luz en un medio transparente (Sin dimensiones).

Distancia focal de un dispositivo óptico (f: distancia entre el dispositivo y su punto focal). El

punto focal es aquél en el que convergen todos los rayos que inciden sobre el dispositivo (lente o

espejo curvo) paralelos al eje óptico principal (O el punto a partir del cual divergen) (1 m)

Potencia dióptrica (PD = 1/f): es una medida del poder convergente o divergente de un sistema

óptico (dioptría: 1 D = 1 m-1

).

Aumento lineal de un dispositivo óptico (Ml = hi / ho): mide cuántas veces mayor es la longitud

de la imagen observada que la longitud del objeto (Sin dimensiones).

Aumento angular de un dispositivo óptico (Mθ = θi / θo): mide cuántas veces mayor es el ángulo

que abarca a la imagen observada bajo el dispositivo que el ángulo que abarca al objeto obser-

vado directamente sin el dispositivo (Sin dimensiones).

Poder de resolución de un dispositivo óptico (PR = λ/Δλ): mide la capacidad de un dispositivo

para permitir ver separadamente dos longitudes de onda en un espectro con la menor diferencia

posible Δλ entre ellas (Sin dimensiones)

Flujo luminoso (Φl: potencia luminosa visible al ojo humano): tiene en cuenta no solamente la

potencia en watts que llega al ojo, sino la capacidad del ojo para percibir luz; esta capacidad es

máxima para el ojo humano cuando λ = 555 nm (lumen: 1 lm = 1/683 de 1W a 555 nm)

Intensidad luminosa (Il = ΔΦl /Δθsolid): representa el flujo luminoso por unidad del ángulo sólido

por el que avanza el haz de luz (candela: 1 cd = 1 lm/sr).

Iluminancia (Jl = ΔΦl /ΔA): representa el flujo luminoso que incide por unidad de área sobre una

superficie y mide su iluminación (lux: 1 lux = 1 lm/m2)

Page 225: problemas selectos

225

Ángulo sólido (Ωsolid = ΔA/R2): entre las paredes de un cono, o de una pirámide, existe un ángulo

sólido, que es la razón entre el área de una esfera que atraviesa el cono, o la pirámide, (con su

vértice en el centro de la esfera), y el radio al cuadrado de esa esfera (esteroradián: 1 sr –es adi-

mensional−; una esfera completa tiene un ángulo de 4π sr).

II-Definiciones y leyes.

Las magnitudes físicas se definen por diversas necesidades, para describir un sistema físico

cuantitativamente. Estas definiciones son aplicables siempre, dentro del contexto en que se de-

finen (clásico, relativista, cuántico). Al pasar de un contexto a otro, algunas definiciones cambian

a expresiones diferentes. Pero dentro de su contexto de definición, se pueden aplicar a cualquier

problema.

Por otra parte, hay leyes físicas, que son las que establecen las relaciones que, de forma regular y

repetitiva, se establecen entre las magnitudes dentro de determinado fenómeno y bajo ciertas

circunstancias. Las ecuaciones de estas leyes son aplicables solamente bajo las circunstancias

para las cuales se dedujeron y, en tal sentido, son más limitadas que las definiciones.

A manera de ejemplo, la ley de la velocidad en el movimiento armónico simple es diferente a la

ley de velocidad en el movimiento rectilíneo uniforme, o a la ley de velocidad de movimientos

uniformemente acelerados, o a la ley de velocidad en movimientos bajo resistencia de un fluido

viscoso, etc. Sin embargo, la ecuación de definición de la velocidad es la misma en todos esos

casos (y es la misma en el contexto relativista y en el cuántico).

A continuación aparecen, en dos columnas, las definiciones y leyes de gran parte de la Física. El

estudiante debe aprender las definiciones con sus unidades, y las leyes con sus condiciones de

aplicación. Entre las dos columnas aparecen las unidades SI de las cantidades definidas en la

primera columna.

En la tabla se han expresado las ecuaciones en términos de incrementos finitos, Δx, no de

diferenciales, dx, En vez de derivadas aparecen cocientes de incrementos (del tipo Δx/Δt) y en

vez de integrales aparecen sumatorias (del tipo ΣviΔti , o simplemente ΣvΔt)

En algunos casos se emplean productos escalares y vectoriales que se definen del siguiente

modo:

Producto escalar: A • B = ABcosθAB = AxBx + AyBy + AzBz (cantidad escalar)

Producto vectorial: A × B = C (cantidad vectorial), cuya dirección está dada por la regla de la

mano derecha. El valor del vector es: C = ABsenθAB.

Page 226: problemas selectos

226

DEFINICIONES DE

CANTIDADES FÍSICAS

Unidad

SI

LEYES FÍSICAS (RELACIONES ENTRE

CANTIDADES FÍSICAS)

Cinemática de la partícula

Cinemática de la partícula

Vector de posición:

r(t) = xi + yj + zk

m a constante: r(t) = ro + vot + ½ at2

v(t) = vo + at

Vector desplazamiento:

Δr = r(t + Δt) – r(t)

m a constante: v2= vo

2 + 2a•Δr

vavg=(v + vo)/2

Distancia recorrida:

s = ΣΔsi

m Proyectiles: x = xo + vot cosθ

y = yo + vot senθ – ½gt2

Velocidad:

v = Δr/Δt

m/s Composición de movimientos:

v = v’ + V , a = a’ + A

Rapidez:

v = Δs/Δt

m/s

Aceleración:

a = Δv/Δt

m/s2

Aceleración tangencial:

at = Δv/Δt (v =|v|)

m/s2

Aceleración normal:

an= c2/R= v

2/R

m/s2

Dinámica de la partícula.

Dinámica de la partícula.

Masa inercial:

mx = mpatrón |apatrón| / |ax |

kg Leyes de Newton: a = 0 si Fres

= 0 1ª ley

Fres

= ma 2ª ley

Fab = − Fba 3ª ley

Fuerza dinámica:

F = ma

N Ft = mat F. tangencial

Fn = m v2/R F. normal

Fuerza estática:

|F| = kΔl

N Leyes de fuerza: F = GMm/r2 F. gravit.

w = mg

Fk = μkN Fricc. cinética

Fs ≤ μsN Fricc. estática

Fvisc= bv Fricc. viscosa

F = EAΔl / l F. elástica

F = qE F. eléctrica

F = q v×B F. magnética

Momento lineal de la partícula:

p = mv

kgm/s F = Δp/Δt 2ª ley partic.

Momento lineal del sistema de

partículas: P = Σ mivi

kgm/s Fext

= ΔP/Δt 2a ley al sist.

P = const. si ΣFext

= 0 Ley conserv. de P

Posición del centro de masa:

Rcm = (1/M) Σmiri

m P = MVcm

Fext

=MAcm

Trabajo y energía Trabajo y energía

Trabajo de una fuerza:

W = ΣFiΔsicosθi

J

Page 227: problemas selectos

227

DEFINICIONES DE

CANTIDADES FÍSICAS

Unidad

SI

LEYES FÍSICAS (RELACIONES ENTRE

CANTIDADES FÍSICAS)

Energía cinética de la partícula:

K = ½ mv2

J Wneto

= ΔK

Energía potencial:

ΔU = -Wconserv.

J Ug = mgh Gravit. peq. alturas

Ug = - GMm/r Gravit. toda altura

Ue = ½ kx2 Elástica

Energía mecánica:

E = ΣKi + ΣUi

J Ksist = ½ MVcm2 + Σ ½mivi’

2

Wno conserv

=ΔE

E = const. si Σ WNo Cons

= 0 Ley conserv. E

Potencia:

P = ΔW/Δt

W P = Fv cosθ Pot. mecánica

Movimiento circular.

Movimiento circular.

Ángulo en radianes:

θ = s / R

rad α: constante: θ(t) = θo + ωot + ½ αt2

ω(t) = ωo + αt

Velocidad angular:

ω = Δθ/Δt

rad/s α: constante: ω2 = ωo

2 + 2αΔθ

ωavg= (ωo+ ω)/2

Aceleración angular:

α= Δω/Δt

rad/s2

v = ωR

at = αR , an = ω2R

Momento angular de la partícula:

L = r × p

kgm2/s L = Δτ/Δt

Mom. ang. del sistema de partic.:

LS = Σ ri × pi

kgm2/s LS = Δτ

ext/Δt

Torque:

τ = r × F

Nm L = const. si Σ τiext

= 0 Ley conserv. de L

Dinámica del Sólido.

Dinámica del Sólido.

Torque axial:

ηax= r┴F

Nm ηax = Iα Ec. Fund. de Rotación

Momento de inercia:

I = Σri2Δmi

kg m

2 I = ½MR

2(cilindro) I = MR

2 (aro)

I = 2/5MR2 (esfera) I = 1/12 M l

2 (varilla)

Densidad:

ρ = Δm/ΔV

kg/m3 I = Icm + M d

2 Ejes paralelos

Energía cinética de rotación:

K = Σ½(Δmi)(ωri)2

J K = ½Iω2 Rotación pura

K = ½MVcm2+ ½Icmω

2 Rodadura

Mom. ang. axial del sist. partic.:

Lax = Σr┴i pi

kg m2/s Lax = Iω Rotación en eje fijo

Lax = Rcm┴MVcm + Icmω Rodadura

Lax = const. si ηax = 0 Ley conserv. Laxial

Condiciones de equilibrio estático:

Σ Fi = 0 y Στax.i = 0

Fluidos

Fluidos

Presión:

P = F┴/A

Pa P = Po + ρgh Presión y profundidad

Fboyante = ρf Vd g Arquímedes

Page 228: problemas selectos

228

DEFINICIONES DE

CANTIDADES FÍSICAS

Unidad

SI

LEYES FÍSICAS (RELACIONES ENTRE

CANTIDADES FÍSICAS)

Gasto de volumen (flujo):

GV = ΔV/Δt

m3/s GV =Av

A1v1 = A2v2 Ec. Continuidad

Gasto de masa:

Gm = Δm/Δt

kg/s P1+ ½ρv12+ ρgh1 = P2+ ½ρv2

2+ ρgh2

Ec. Bernoulli

Termodin. y Física Molecular Termodinámica y Física Molecular

Puntos termométricos Celsius:

Fusión hielo: 0oC

Ebullición agua: 100oC

oC TC /(TF – 32) = 5/9 Relación Celsius-Farenheit

Δl = loαΔT Expansión Lineal

Temperatura absoluta:

T(K) = T(oC) + 273

K Kavg=(3/2) kT(K) Energía Cin. Media

Cantidad de calor:

Q = mcΔT

J Q = L m Calor de cambio de estado

W = ΣPiΔVi Trabajo de un gas

Energía interna:

U = ΣKiint

+ ½ΣUij

J Q = ΔU + W 1ª Ley Termodinámica

Q=W=nRT ln(V2/V1) Gas ideal, Proc. Isoterm.

Cantidad de sustancia:

n =m/M = N/NAvog

NAvog = 6.02×1023

molec/mol

mol

PV = nRT = NkT Ec. Estado Gas Ideal

PV = const. Boyle (Isoterm.)

P/T = const. GayLussac (Isometr.)

V/T = const. Charles (Isobar.)

(γ=CP/CV) PVγ= const. Adiab. Revers.

P = ΣPi Dalton

U = (3/2)nRT = nCVT Gas ideal monoat.

Eficiencia térmica:

e =Wneto

/Qabsorb

Sin

unid.

eCarnot = 1- Tbaja

/Talta

ereal ≤ eCarnot 2ª Ley Termodinámica

Entropía:

ΔS = Σ(ΔQi)/ Ti(K)

J/K ΔS = k ln( Ω2/Ω1) Entropía y Probabilidad

Oscilaciones y Ondas Mecánic.

Oscilaciones y Ondas Mecánicas.

Período:

T = Δt/ΔN

s T = 1/f Relación frec.-período

ω = 2π/T Relación frec.ang.-período

Frecuencia cíclica:

f =ΔN/Δt

Hz ω = 2πf Relación frec.ang.-frec.cicl.

Frecuencia angular:

ω=Δθ/Δt

rad/s x =A sen(ωot + θo) Osc. Armon. Simple

d2x/dt

2+ Cx = 0 Ec. del Osc. Arm. Simple con

ωo = (C)1/2

v = Aω cos (ωot + θo)

a = − Aω2 sen (ωot + θo)

Fase instantánea:

θ= ωt + θo

rad ωo = (g/l )1/2

Pend. Simple

ωo = (mgd/I)1/2

Pend. Físico

ωo = (k/m)1/2

Masa-resorte

Fuerza armónica:

Faplicada= Fosenωaplt

N ωaplicada ≈ ωo Cond. Resonancia Mecánica

Page 229: problemas selectos

229

DEFINICIONES DE

CANTIDADES FÍSICAS

Unidad

SI

LEYES FÍSICAS (RELACIONES ENTRE

CANTIDADES FÍSICAS)

Longitud de onda:

λ = vT m v = (FT/μ)

1/2 Ondas en cuerda (μ=Δm/Δl)

v =λf =ω/k Ondas periódicas

Número de onda:

k =2π/λ

m-1

y = A sen(kx – ωt) Onda armónica

y = 2A senkx cosωt Onda estacionaria.

Δl = nλ Interf. constructiva

Δl =(n+1/2)λ Interf. destructiva

f = fo (1 vobs/c) / (1 vemis/c) Doppler

Intensidad de onda:

I= ΔU/AΔt

W/m2 I = P/4πr

2 Intens. fuente puntual

(P: potencia de la fuente)

Nivel sonoro: [con Io=10-12

W/m2]

β = 10 log (I/Io)

dB

Carga y campo eléctrico.

Carga y campo eléctrico.

Carga eléctrica de un cuerpo

(múltiplo de la carga electrónica):

q = Ne

C F = kqQ/r2 Ley Coulomb (k = 1/4πεo)

Intensidad de campo eléctrico en

un punto, E:

E = FE/q

N/C =

V/m

E = kQ/r2 Carga puntual

Eresult

= ΣEi Campo total en un punto

Momento dipolar eléctrico:

pe =qd

Cm U = pEcosθ Energía del dipolo en un E

τ = pe×E Torque sobre el dipolo

Flujo de campo eléctrico:

ΦE =ΣEi┴ΔAi

N/(Cm2) [ ΣEi┴ΔAi ]S.Cerrada = q/εo Ley Gauss

Potencial eléctrico en un punto:

V =U/q

V V = kQ/r Carga puntual

Vtotal

= ΣVi Potencial total en un punto

Voltaje (diferencia potenc. entre

dos puntos):

Vab =ΔUab/q

V Vab =Σ ΔVi Voltaje total entre dos

(de A a B) puntos

Densidad de carga:

Lineal: λ=Δq/Δl

Superficial: ζ=Δq/ΔA

Volumétrica: ρ=Δq/ΔV

C/m

C/m2

C/m3

E = ζ/εo Campo exterior a conductor

E = ζ/εo Campo entre placas paralelas

E = ζ/2εo Campo de una placa cargada

El= - ΔV/Δl Relación campo-potencial

Capacitancia:

C =q/V (V: voltaje)

F

C =κeεoA/d Capacitor placas paralelas

Coeficiente dieléctrico relativo:

κe = E/Eo= C/Co

Sin

unid.

Cp = ΣCi Capacit. en paralelo

1/Cs = Σ1/Ci Capacit. en serie

Densidad de energía eléctrica:

uE =UE/V (V: volumen)

J/m3

UE = ½CV2 Energía en el capacitor

uE = ½ εoE2 Dens. energ. del campo

Corriente y circuitos. Corriente y circuitos.

Intensidad de corriente:

I = Δq/Δt

A

Page 230: problemas selectos

230

DEFINICIONES DE

CANTIDADES FÍSICAS

Unidad

SI

LEYES FÍSICAS (RELACIONES ENTRE

CANTIDADES FÍSICAS)

Densidad de corriente:

J = I/A ; J.ΔA = ΔI

A/m2

J = nev Dens. Corr. y veloc. de deriva

Resistencia eléctrica:

R = V/I

Ω R = ρl/A Resistencia y resistividad

ρ =ρo(1 + αΔT) Resistividad y temperatura

Rs = ΣRi Resist. en serie.

1/Rp = Σ1/Ri Resist. en paralelo

Fuerza electromotriz (fem):

E = Wno cons

/ q

V E =IRtotal Ley Ohm para el circuito

r =(E – Vterminal)/I Resist. interna de batería

Potencia:

P = W/t = Q/t

W P = IV = I2R = V

2/R Potencia eléctrica

Tiempo característico RC:

η = RC

s q = E C(1 – e-t/η

) Proceso de carga

I = (E /R) e-t/η

Proceso de carga

ΣIi = 0 Kirchhoff (nodos)

ΣIi Rj + ΣEk = 0 Kirchhoff (mallas)

Corriente y campo magnético.

Corriente y campo magnético.

Inducción de campo magnético en

un punto, B:

F = q v × B; F=qvBsenθ

T F = I l × B ; F=IlBsenθ Fuerza sobre corriente

F = q v × B + q E Carga en campos E y B

Bresult

= ΣBi Campo total en un punto

Flujo de inducción magnética:

ΦE =ΣBi┴ΔAi

Wb [ ΣBi┴ΔAi ]S.Cerrada = 0 Ley Gauss

[ ΣBi║Δli ]Curva Cerrada = μoI Ley Ampere

Momento dipolar magnético de

una espira con corriente:

pm= IA

Am2

B = μoI/(2πr) B de corr. rectilínea

B = μonI (con n=N/l) B de solenoide vacío

τ = pm× B Torque sobre el dipolo

Inducción Electromagnética.

Inducción Electromagnética.

E = -NΔΦB/Δt Ley Faraday

[ ΣBi║Δli ]Curva Cerrada = μoI + μoεoΔΦE/Δt

Ley Ampere-Maxwell

[ ΣEi║Δli ]Curva Cerrada = -NΔΦB/Δt

Ley Faraday-Maxwell

E = Blv fem inducida en conductor móvil

Autoinductancia:

L = NΦB/I

H E = -LΔI/Δt Relación E y ΔI/Δt

Tiempo característico RL:

η = L/R

s I =(E /R)(1 – e-t/η

) Encendido del circuito

Coeficiente magnético relativo:

κm = B/Bo =Lmaterial /Lvacío

Sin

unid.

L = κμoN2A/l Inductancia de solenoide

(N/l =n)

Densidad de energía magnética:

uB =UB/V (V: volumen)

J/m3 uB = ½ B

2/μo Dens. energ. del campo

Page 231: problemas selectos

231

DEFINICIONES DE

CANTIDADES FÍSICAS

Unidad

SI

LEYES FÍSICAS (RELACIONES ENTRE

CANTIDADES FÍSICAS)

Corriente alterna. Corriente alterna.

Fem alterna:

E =Em senωaplic t

V VeR =IeR, VeC =IeXC , Ley de Ohm en C.A.

VeL =IeXL, VeCirc. =IeZ

Voltaje eficaz y corriente eficaz:

Ve = Vm/ , Ie = Im/

V , A

ωo = 1/(LC)1/2

Frecuencia propia circuito LC

Reactancia capacitiva:

XC = 1/ωC

Ω ωaplic. = ωo Condición de resonancia

Im = Em/Z = Em/[(ωL – 1/ωC)2 + R

2 ]

1/2

Reactancia inductiva:

XL = ωL

Ω Pavg= Ie Ee cosθ Potencia consumida en el

[tanθ = (XL – XC) /R] circuito

Impedancia:

Z = Ee/Ie = Em/Im

Ω

Z = [(XL – XC )2 + R

2]

1/2 Serie de R,L,C

Ondas electromagnéticas. Ondas electromagnéticas.

c =λf Velocidad ondas EM (luz)

c=1/(μoεo)1/2

(3.00×108m/s en vacío)

Intensidad de la onda:

I= ΔP/ΔA

W/m2

Iavg = BavgEavg/μo = Eavg2/(c μo) Intensidad

promedio en un ciclo

I = P/4πr2 Intens. fuente puntual

f = fo [(1 v/c) / (1 v/c)]1/2

Doppler

Óptica geométrica.

Óptica geométrica.

Índice de refracción:

n =c/v Sin

unid.

θincid = θreflex Ley de reflexión

(Ángulos se miden respecto a la normal)

Aumento lineal:

Ml = limag./ lobj.

Sin

unid.

n1senθ1 = n2senθ2 Ley de refracción

senθlim = n2/n1 Reflex. total interna

(Ángulos se miden respecto a la normal)

Aumento angular:

Mθ= θimag./ θobj.

Sin

unid.

1/f = 1/do +1/di Espejos cóncavos y lentes

delgadas

Potencia dióptrica de la lente:

PD = 1/f

D f = R/2 Espejo esférico

1/f = (n – 1)(1/R1 – 1/R2) Fabricante de lente

Flujo luminoso:

Фl=(1/683) watt de luz de 555 nm

lm

Intensidad luminosa:

Il = ΔΦl /Δθsolid

cd =

lm/sr

Iluminancia:

Jl = ΔΦl /ΔA

lux=

lm/m2

Ángulo sólido:

Ω = A/R2

sr

Óptica ondulatoria.

Óptica ondulatoria.

Longitud del camino óptico:

s = nd

m Io= I. cos2θ (con Io polarizada) Ley de Malus

tan θB = n2/n1 Ley de Brewster de Polariz.

Page 232: problemas selectos

232

DEFINICIONES DE

CANTIDADES FÍSICAS

Unidad

SI

LEYES FÍSICAS (RELACIONES ENTRE

CANTIDADES FÍSICAS)

Diferencia de fase entre ondas

coherentes:

Δθ = (s2 – s1)2π/λ

rad Δθ =2mπ, con m =0, 1, 2, 3, …, mmax

Cond. máximos interf. entre ondas coherentes

mλ =d senθm , m= 0, 1, 2, …, mmax

Cond. máximos interferencia dos rendijas

mλ =w senθm , m= 1, 2, …, mmax

Condición mínimos difracción en una rendija

Poder separador de una red:

R = λavg /Δλ

Sin

unid.

R = Nm Red lineal

Dispersión angular:

D=Δθ/Δλ

rad/m D = m/(d cosθ) Red lineal

Poder emisivo de un cuerpo que

radia ondas electromagnéticas a

cierta temperatura absoluta T:

ET = ΔP/ΔA

W/m2

ET = ζT4 Ley Steffan-Boltzman

Teoría de la Relatividad.

(c= 3.00×108m/s)

Teoría de la Relatividad.

Intervalo:

Δs2

= c2Δt

2 −Δx

2−Δy

2− Δz

2

m c = const. Invariante la vel. luz (vacío)

Δs =const. Invariante de espacio-tiempo

x’ = γ(x –Vt ), t’ = γ(t −xV/c2), y’ = y, z’ = z

con γ = 1/(1 – V2/c

2)1/2

Transf. Lorentz

Δt = Δη/(1 – V2/c

2 )

1/2 Dilatación de tiempo

Δl = Δlo (1 – V2/c

2 )

1/2 Contracción de long

vx’ = (vx−V) / (1−Vvx /c2) Suma de velocidades

vy’ = vy / [γ(1−Vvx /c2)]

Momento lineal:

p = mv /(1 – v2/c

2)1/2

kg m/s F = Δp/Δt Fuerza relativista

p = E/c Momento del fotón

Energía total:

E = mc2/(1 – v

2/c

2)1/2

J E2 – p

2c

2 = m

2c

4 Invariante de E-p

K = [mc2/(1 – v

2/c

2)1/2

] – mc2 Energía cinética

E = mc2 Energía de reposo

Física cuántica.

(h = 6.63×10-34

Js = 4.14×10-15

eVs)

Física cuántica.

Quantum de Energía (Planck):

E = hf

J p = h/λ Ondas de materia (de’Broglie)

nλ =2dsenθn , n =1, 2, …Max. difracc. de Bragg

Quantum de momento angular:

L =nh/2π

kg m2/s En = −13.6 (eV)/n

2 Átomo de hidrógeno (Bohr)

En−Em = hf Transición atómica

hf = Kmax. + Wextr. Ley del fotoefecto

Kmax = eVfrenado Voltaje de frenado

hfcorte= Wextr. Frecuencia de corte

Propiedades del núcleo:

Número másico: A = Nprot+Nneut

Número atómico: Z = Nprot

Sin

unid.

AXZ→

A-4YZ-2 +

4α2 Desintegración alfa

AXZ→

AYZ+1 +

0β-1 Desintegración beta

ΔE =(Δm)c2 Energía nuclear liberada

Actividad (radiactiva):

R = ΔN/Δt

(Unidad SI: 1 becquerel, Bq)

Bq

R = Roe-t/η

Ley de desintegración

1 Bq = 1 desintegración / segundo

Page 233: problemas selectos

233

Terminado de escribir el 26 de octubre del 2010, en Mayagüez, Puerto Rico